Download as docx, pdf, or txt
Download as docx, pdf, or txt
You are on page 1of 689

CA FOUNDATION COURSE MATHEMATICS TEXT BOOK

AUTHOR : CA. C GOVINDARAJ M.Com., M.B.A., M.Phil., F.C.A., A.C.S

CHARTERED ACCOUNTANT

Phone : 98431 28670

PUBLISHER :

CONFIDENT COACHING CENTRE

21, MELA MASI VEEDHI,

MADURAI—625 001

Phone : 98430 18359

Table of Contents :

Chapter title Page


Number
1 Interest
2 Arithmetic Progression and Geometric
Progression
3 Ratios, Proportions, indices and Logarithms
and Venn Diagrams
4 Functions
5 Permutations and Combinations
6 Equations
7 Differentiation and integration
8 Analytical Geometry

Interest

Interest means the amount resulting from PRINCIPAL AMOUNT.

PRINCIPAL AMOUNT means the basic amount which is deposited by a person normally into a
BANK.

Example 1:

Mr. Ram deposited `5,000 in Canara bank on 13-06-2013.

Canara bank is giving 8 % simple interest per annum. Mr Ram closes his deposit with canara
bank on 13-6-2014. How much interest will Mr. Ram get from Canara bank?

Solution:
Formula for computing the amount of simple interest from a deposit scheme:

I= (PNR) /100

I stands for the amount of interest

P stands for the Principal amount=`5,000

N stands for the number of periods for which the deposit was kept= 1 YEAR

R stands for the Rate of interest for the period =8% PER ANNUM

100 It is a constant.

Let us apply the appropriate data into the above INTEREST FORMULA :

I= PNR/100

I= (`5,000)(1 YEAR) (8)/ 100

I= (5,000)(1 ) (8)/100

I= `400

EXAMPLE 2:

Mr Kumar deposited `25,000 in Canara bank on

26-06-2013.

Canara bank is giving12 % simple interest per annum.

Mr Kumar closes his deposit with canara bank on 26-6-2015. How much interest Mr Kumar
would have earned from Canara bank?

Solution:

Formula for simple interest

I= (PNR) /100

I stands for the amount of interest

P stands for the Principal amount=`25,000


N stands for the number of periods for which the deposit was kept= 2 YEARS

R stands for the Rate of interest for the period =12% PER ANNUM

100 It is a constant.

Let us apply the appropriate data into the above INTEREST FORMULA

I= PNR/100

I= (`25,000)(2 YEARS) (12)/100

I= (25,000)(2 ) (12)/100

I= `6,000

Example [3]: Mr. Rajesh deposited a principal amount which has yielded a total simple
interest amount of `4,800 in 3 years at 8 % per annum. Derive the PRINCIPAL amount.

Solution:

Formula for simple interest

I= (PNR) /100

I stands for the amount of interest

P stands for the Principal amount=`?

N stands for the number of periods for which the deposit was kept= 3 YEARS

R stands for the Rate of interest for the period =8% PER ANNUM

100 It is a constant.
Let us apply the appropriate data into the above INTEREST FORMULA

I= PNR/100

Rs4,800= (`P)(3 YEARS) (8)/100

4,800= (P)(3 ) (0.08)

4,800

--------- =P

(3)(0.08)

20,000=P

SO PRINCIPAL =Rs20,000

Example [4] : Mr Balaji deposited a principal amount of `50,000 which has yielded a total
simple interest amount of `12,000 at 6 % per annum in the investment term. Derive the
period for which the deposit was kept.

Solution:

Formula for simple interest

I= (PNR) /100

I stands for the amount of interest

P stands for the Principal amount=`50,000

N stands for the number of periods for which the deposit was kept= N YEARS

R stands for the Rate of interest for the period =6% PER ANNUM
100 It is a constant.

Let us apply the appropriate data into the above INTEREST FORMULA

I= PNR/100

12,000= (`50,000)(N YEARS) (6)/100

12,000= (50,000)(N ) (0.06)

12,000

-------------------- =N

(50,000)(0.06)

4=N

SO PERIOD FOR WHICH THE DEPOSIT WAS KEPT ON SIMPLE INTEREST TERM=4
YEARS.

EXAMPLE [5]

A sum of money amounts to `9,800 after 5 years from deposit date and `12,005 after 8 years
from the same deposit date at the same rate of simple interest per annum. Find the rate of
interest per annum.

SOLUTION

9, 800 = P + 5 YEARS’ INTEREST

12,005 = P + 8 YEARS’ INTEREST


----------------------------------------------

2,205= 3 YEARS’ INTEREST

----------------------------------------------

1 YEAR’S INTEREST AMOUNT =2,205/3=Rs 735

----------------------------------------------------------------

9, 800 = P + 5 YEARS’ INTEREST

9, 800 = P + 5 [Rs735]

9, 800 = P + Rs 3,675

9,800—3,675=P

6,125=P

1 year’s interest amount

----------------------------------- x100 = R

Principal

Rs735

---------- x100 =12%

Rs6,125

Alternative solution:

LET THE PRINCIPAL BE “P”

IN 5 YEARS A= 9800
IN 8 YEARS A=12005

SINCE PRINIPAL IS THE SAME,

THE TOTAL INTEREST FOR 3 YEARS=12005-9800=2205

THE INTEREST AMOUNT FOR ONE YEAR=2205/3=735

Let us consider 5 year deposit

5 years‘ total interest=735 x5=3,675

A=P+TOTAL INTEREST

9800=P+3675

P=6125

6125 X R/100 =735

R=735/6125 X100

R=12%

EXERCISE [6]

A sum of money amounts to `5,500 after 2 years from a deposit date and `6,000 after 4 years
from the same deposit date at the same rate of simple interest per annum. Find the rate of
interest. Also find principal.

Solution:

LET THE PRINCIPAL BE “P”

IN 2 YEARS A= 5500

IN 4 YEARS A=6000
SINCE PRINIPAL IS THE SAME,

THE TOTAL INTEREST FOR 2 YEARS=6000-5500=500

THE INTEREST AMOUNT FOR ONE YEAR=500/2=250

Let us consider 2 year deposit

2 year ‘s total interest=250 x2=500

A=P+TI

5500=P+500

P=5000

Rate of interest

5000 X R/100 =250

R=250/5000 X100

R=5%

EXERCISE[7]

A PRINCIPAL amounts to `9,075 after 7 years from the deposit date and the same PRINCIPAL
amounts to `10,200 after 12 years from the same deposit date at the same rate of simple
interest per annum.

 * Find the rate of interest.


 * Also find the amount of principal.
Solution:

LET THE PRINCIPAL BE “P”

IN 7 YEARS A= 9075

IN 12 YEARS A=10200

SINCE PRINIPAL IS THE SAME,

THE TOTAL INTEREST FOR 5 YEARS=10200-9075=1125

THE INTEREST AMOUNT FOR ONE YEAR=1125/5=225

Let us consider 7 year deposit

7 year ‘s total interest=225 x7=1575

A=P+TI

9075=P+1575

P=7500

7500 X R/100 =225

R=225/7500 X100

R=3%

EXERCISE [8]

A sum of money amounts to `21,180 after 9 years from deposit date and `27,300 after 15 years
from the same deposit date at the same rate of simple interest per annum.
 Find the rate of interest.
 Also find principal.

27300—21180

-------------------

15 years—9 years

6120

--------

6 years

1020 is the 1 year’s interest

What is 9 years’ interest= 1020x9= 9180

Principal =21180—9180 =12000

Rate of interest per annum

1 year’s interest

----------------------- x 100

Principal amount

Rs 1,020

------------ x 100 = 8.5%

Rs12,000

Solution:
LET THE PRINCIPAL BE “P”

IN 9 YEARS A= 21180

IN 15 YEARS A=27300

SINCE PRINCIPAL IS THE SAME,

THE TOTAL INTEREST FOR 6 YEARS=27300-21180=6120

THE INTEREST AMOUNT FOR ONE YEAR=6120/6=1020

Let us consider 9 year deposit

9 year ‘s total interest=1020 x9=9180

A=P+TI

21180=P+9180

P=12000

12000 X R/100 =1020

R=[1020/12000] X100

R=8.5%

EXERCISE[9]

A sum of money amounts to `9,210 after 7 months from deposit date and `9,877.50 after 13
months from the same deposit date at the same rate of simple interest per annum. Find the
rate of interest per annum. Also find principal.
Solution :

Rs9,877.50—Rs 9,210

--------------------------------

13 months—7 months

Rs667.50

---------------

6 months

= Rs111.25 is the interest per month.

Let us consider 7 months’ deposit

What is 7 months’ interest ?

111.25 x 7 months = Rs778.75

We can now find the principal

Rs 9,210 ---[ 778.75]= Rs8,431.25 is the principal

What is 1 year’s interest = 111.25 x 12 months=Rs 1,335

We can find the rate of interest per annum

Interest per year

------------------------- x 100

Principal
Rs1,335

------------ x 100 = 15.83%

Rs8,431 .25

Solution:

LET THE PRINCIPAL BE “P”

IN 7 MONTHS A= 9210

IN 13 MONTHS A=9877.50

SINCE PRINCIPAL IS THE SAME,

THE TOTAL INTEREST FOR 6 MONTHS=9877.50--9210=667.5

THE INTEREST AMOUNT FOR ONE MONTH=667.5/6=111.25

Let us consider 7 MONTH deposit

7 MONTH’s total interest=111.25 x7=778.75

A=P+TI

9210=P+778.75

P=8431.25

8431.25 X R/100X7/12 =778.75

R=[778.75 /8431.25]X[100]X[12/7]

R=15.83%
EXERCISE[10]

A sum of money amounts to `4,050 after 3 months from a deposit date and ` 4,250after 15
months from the same deposit date at the same rate of simple interest per annum. Find the
rate of interest per annum. Also find principal.

Solution:

LET THE PRINCIPAL BE “P”

IN 3 MONTHS A= 4050

IN 15 MONTHS A=4250

SINCE PRINIPAL IS THE SAME,

THE TOTAL INTEREST FOR 12 MONTHS=4250--4050=200

THE INTEREST AMOUNT FOR ONE MONTH=200/12=16.6666666

Let us consider 3 MONTH deposit

3 MONTH’s total interest=16.666666 x3=50

A=P+TI

4050=P+50

P=4000

4000 X R/100X1/12 =16.6666666

R=[16.666666 /4000]X[100]X[12/1]

R=5%

EXERCISE[11]

A sum of money amounts to `8,960 after 8 months from deposit date and `9,320 after 11
months from deposit date at the same rate of simple interest per annum.

 * Find the rate of interest per annum.


 * Also find principal.

ANSWER

RATE OF SIMPLE INTEREST PER ANNUM=18%

PRINCIPAL AMOUNT=Rs8,000

COMPOUND INTEREST

COMPOUND INTEREST SCHEME IS

IMPLEMENTED MOSTLY IN THE FIXED DEPOSIT SCHEME OF BANK.

EXAMPLE 1:

WHEN A DEPOSITOR DEPOSITS SAY FOR

EXAMPLE Rs5,000 IN A FIXED DEPOSIT

SCHEME OF CANARA BANK AT 7 % INTEREST

PER ANNUM FOR A PERIOD OF 3 YEARS WHEN THE INTEREST COMPOUNDING REST IS ONE
YEAR, GIVE THE IMPORTANT COMPUTATIONS FOR THE FIXED DEPOSIT SCHEME.

Solution :
PARTICULARS RUPEES

PRINCIPAL AMOUNT 5,000


[On 01-04-2001 Mr. Arun Deposited Rs5,000 with Canara bank]
INTEREST INCOME OF YEAR 1 350
Interest income which is earned by the depositor from the bank on the FD
for the period 01-04-2001 to
31-03-2002 is
7/100 XRs5,000( in the compound interest scheme, the interest income
will get merged or attached or added or compounded to the principal
amount)
INTEREST INCOME OF YEAR 2 374.50
Interest income which is earned by the depositor from the bank on the FD
for the period 01-04-2002 to 31-03-2003
7/100 X[Rs5,000+Rs350]
=7/100[5,350]
=Rs374.50
( in the compound interest scheme, the interest income will get merged
or attached or added or compounded to the principal amount)
INTEREST INCOME OF YEAR 3
Interest income which is earned by the depositor from the bank on the FD 400.72
for the period 01-04-2003 to 31-03-2004
7/100 X[Rs5,000+Rs350+Rs374.50]
=7/100[5,724.50]
=Rs400.72
( in the compound interest scheme, the interest income will get merged
or attached or added or compounded to the principal amount)
AMOUNT OF MATURTY VALUE OF THE FIXED DEPOSIT SCHEME UNDER 6,125.22
COMPOUNDED INTEREST SCHEME WITH COMPOUNDING REST BEING
YEARLY INTERVAL.
Rs (5,000 +350 +374.50+400.72)

EXAMPLE 2:

WHEN A DEPOSITOR DEPOSITS SAY FOR EXAMPLE Rs5,000 IN A FIXED DEPOSIT SCHEME
OF A PRIVATE BANK AT 7 % INTEREST PER ANNUM FOR A PERIOD OF 3 YEARS WHEN THE
INTEREST COMPOUNDING REST IS HALF YEAR , GIVE THE IMPORTANT COMPUTATIONS FOR THE
FIXED DEPOSIT SCHEME.
Solution:

PARTICULARS RUPEES

PRINCIPAL AMOUNT 5,000


[On 01-04-2001 Mr. Arun Deposits Rs5,000 with Canara bank]
1st INTEREST INCOME 175
Interest income which is earned by the depositor from the bank on the FD
for the period 01-04-2001 to 30-09-2001
7/100 XRs5,000X6/12
Rs 175
( in the compound interest scheme, the interest income will get merged
or attached or added or compounded to the principal amount)
2nd INTEREST INCOME 181.125
Interest income which is earned by the depositor from the bank on the FD
for the period 01-10-2001 to 31-03-2002
7/100 X[Rs5,000+Rs175] X6/12
=Rs181.125
( in the compound interest scheme, the interest income will get merged
or attached or added or compounded to the principal amount)
3rd INTEREST INCOME 187.464375
Interest income which is earned by the depositor from the bank on the FD
for the period 01-04-2002 to 30-09-2002 :

7/100 X[Rs5,000+Rs175+Rs181.125]
X6/12
=Rs187.464375( in the compound interest scheme, the interest income
will get merged or attached or added or compounded to the principal
amount)
4th INTEREST INCOME 194.025628
Interest income which is earned by the depositor from the bank on the FD
for the period 01-10-2002 to 31-03-2003
7/100 X[Rs5,000+Rs175+Rs181.125+
187.464375]
X6/12
=Rs194.025628( in the compound interest scheme, the interest income
will get merged or attached or added or compounded to the principal
amount)
5th INTEREST INCOME 200.816525
Interest income which is earned by the depositor from the bank on the FD
for the period 01-04-2003 to 30-09-2003
7/100 X[Rs5,000+Rs175+Rs181.125+187.4643756 +194.025628]
X6/12
=Rs200.816525( in the compound interest scheme, the interest income
will get merged or attached or added or compounded to the principal
amount)
6th INTEREST INCOME 207.845103
Interest income which is earned by the depositor from the bank on the FD
for the period 01-10-2003 to 31-03-2004 =
7/100 X[Rs5,000+Rs175+Rs181.125+187.4643756
+194.025628+200.816525]
X6/12
=Rs207.845103( in the compound interest scheme, the interest income
will get merged or attached or added or compounded to the principal
amount)
AMOUNT OF MATURTY VALUE OF THE FIXED DEPOSIT SCHEME UNDER 6146.28
COMPOUNDED INTEREST SCHEME WITH COMPOUNDING REST BEING
HALF YEARLY INTERVAL.
Rs (5,000 +175 +181.125+ 187.464375+194.025628 +200.816525
+207.845103)

FORMULA FOR THE PURPOSE OF COMPUTING THE AMOUNT OF MATURITY VALUE OF A


PRINCIPAL AMOUNT OF DEPOSIT UNDER INTEREST COMPOUNDING SCHEME.

A= P(1 +i)n

A= Amount of maturity value of the principal under interest compounding scheme

P= Principal deposited

1= It is a constant

i = Interest percentage per compounding rest.

n= Number of compounding rests in the deposit scheme.


EXAMPLE 3:

WHEN A DEPOSITOR DEPOSITS SAY FOR

EXAMPLE Rs5,000 IN A FIXED DEPOSIT

SCHEME OF CANARA BANK AT 7 % INTEREST

PER ANNUM FOR A PERIOD OF 3 YEARS WHEN THE INTEREST COMPOUNDING REST IS ONE
YEAR, FIND THE AMOUNT OF MATURITY VALUE OF THE FIXED DEPOSIT SCHEME.

(USE SHORT CUT FORMULA)

SOLUTION :

A= P(1 +i)n

A= Amount of maturity value of the principal under interest compounding scheme

P= Principal deposited =Rs 5,000

1= It is a constant

i = Interest percentage per compounding rest=7/100 =0.07

n= Number of compounding periods in the deposit scheme =3

A= P(1 +i)n

A= 5,000(1 +0.07)3
A= 5,000(1 .07)3

A= 5,000 (1.225043)

A= 6,125.22

EXAMPLE 4. WHEN A DEPOSITOR DEPOSITS SAY FOR EXAMPLE Rs5,000 IN A FIXED


DEPOSIT

SCHEME OF A PRIVATE BANK AT 7 % INTEREST PER ANNUM FOR A PERIOD OF 3 YEARS WHEN
THE INTEREST COMPOUNDING REST IS HALF YEAR , FIND THE AMOUNT OF MATURITY VALUE OF
THE FIXED DEPOSIT SCHEME.(USE SHORT CUT FORMULA)

SOLUTION :

A= P(1 +i)n

A= Amount of maturity value of the principal under interest compounding scheme.

P= Principal deposited =Rs 5,000

1= It is a constant

i = Interest percentage per compounding rest.=3.5/100 =0.035

n= Number of compounding periods in the deposit scheme =6

A= P(1 +i)n

A= 5,000(1 +0.035)6

A= 5,000(1 .035)6
A= 5,000 (1.22925532633)

A= Rs.6,146.28

Exercise :

Arun deposited Rs6,000 for 2 ½ years at 10 % per annum, interest being compounded half
yearly.

 Find the amount of maturity.


 Also find the amount of total interest earned totally in 2 ½ years.
 Also find the effective rate of interest per annum.

Solution :

Answer to first question :

A= P(1 +i)n

A= Amount of maturity value of the principal under interest compounding scheme

P= Principal deposited =Rs 6,000

1= It is a constant

i = Interest percentage per compounding rest.=5/100 =0.05

n= Number of compounding periods in the deposit scheme. =5

A= P(1 +i)n

A= 6,000(1 +0.05)5

A= 6,000(1 .05)5
A= 6,000 (1.2762815625)

A= Rs7,657.69

Answer to second question :

Total interest earned in the term of 2 ½ years=

A –P =Rs7,657.69—Rs6,000 =Rs1,657.69

Answer to third question :

Effective rate of interest per annum:

[(Total interest earned in the term) /( Number of years)]

--------------------------------------------------------------------------------- x 100

Principal deposited

[(Rs1,657.69 /( 2.5)]

--------------------------------x100

Rs6,000

Rs663.076

---------------- x 100

Rs6,000
11.051266%

Exercise :

Swamyji deposited Rs16,000 for 1 ½ years at

12 % per annum, interest being compounded every 3 months [quarterly].

 Find the amount of maturity.


 Also find the amount of interest earned totally in 1 ½ years.
 Also find the effective rate of interest per annum.

Answer to first question :

A= P(1 +i)n

A= Amount of maturity value of the principal under interest compounding scheme

P= Principal deposited =Rs 16,000

1= It is a constant

i = Interest percentage per compounding rest.=[12/100]


------------- x3
12
=0.03
n= Number of compounding periods in the deposit scheme. =6

A= P(1 +i)n
A= 16,000(1 +0.03)6
A= 16,000(1 .03)6
A= 16,000 (1.19405229652)
A= Rs19,104.84

 Answer to second question :

Total interest earned in the term of 1 ½ years=


A –P =Rs19,104.84—Rs16,000 =Rs3,104.84

Answer to third question :


Effective rate of interest per annum:

[(Total interest earned in the term) /


( Number of years)]
-------------------------------------------------- x100
Principal deposited

[(Rs3,104.84 /( 1.5)]
--------------------------------x100
Rs16,000

Rs2069.89
---------------- x 100
Rs16,000

12.93%

CA EXAMINATION QUESTION:

In what time will Rs3,90,625 amount to Rs 4,56,976 at 8 % per annum , when the interest is
compounded semi –annually?

Solution :

A= P(1 +i)n

4,56,976= 3,90,625(1 +0.04)n

4,56,976= 3,90,625(1 .04)n

4,56,976/3,90,625= (1 .04)n

1.16985856= (1 .04)n
by trial and error method when we put n=4 we

get the formula values as fulfilled.

Hence the deposit period is 2 years.

CA EXAMINATION QUESTION:

How long will Rs 12,000 take to amount to Rs 14,000 at 5%p.a.compounded quarterly?

SOLUTION :

A= P(1 +i)n

HOW TO FIND i ?

5 % IS FOR 1 YEAR

IN THIS PROBLEM, THE COMPOUNDING REST IS 3 MONTHS( QUARTER OF AN YEAR)

THEREFORE,

i = 5% / 4

i = 1.25 % PER QUARTER

i = 1.25/100 PER QUARTER

i = 0.0125 PER QUARTER


14,000= 12,000(1 +0.0125)n

14,000= 12,000(1 .0125)n

14,000/12,000= (1 .0125)n

1.1666666= (1 .0125)n

By trial and error method when we put n=12 we get the formula values as fulfilled.

Hence the deposit period is 12 QUARTERS.

i.e 3 years.

CA EXAMINATION QUESTION:

THE DIFFERENCE BETWEEN THE SIMPLE AND COMPOUND INTEREST ON A CERTAIN SUM FOR 3
YEARS AT 5 % PER ANNUM IS Rs 228.75.

THE COMPOUND INTEREST ON THE SUM FOR 2 YEARS AT 5% PER ANNUM IS ?

SOLUTION :

WE UNDERSTAND THAT n=3 YEARS.


i=5% PER ANNUM

WE HAVE TO ASSUME THAT THE COMPOUNDING REST IS 1 YEAR.

IT IS MENTIONED IN THE PROBLEM THAT THE EXCESS IS RS 228.75 IN THE COMPOUNDING


INTEREST METHOD AS COMPARED TO THE SIMPLE INTEREST METHOD.

WE CAN FORMULATE THE FOLLOWING EQUATION

[ p(1 +i)n ] -- [ p + (pnr/100) ] =228.75

[ p(1 +i)n ] -- p [ 1 + (nr/100) ] =228.75

p {[ (1 +i)n ] -- [ 1 + (nr/100) ]} =228.75

{[ (1 +i)n ] -- [ 1 + (nr/100) ]} =228.75/p

{[ (1 +0.05)3 ] -- [ 1 + (3x5/100) ]} =228.75/p

{[ (1 .05)3 ] -- [ 1 + (15/100) ]} =228.75/p

1 .157625 -- [ 1 + 0.15] =228.75/p

1 .157625 -- 1 .15 =228.75/p

0.007625 =228.75/p

p= =228.75/0.007625

p= 30,000

question :

THE COMPOUND INTEREST ON THE SUM FOR 2 YEARS AT 5% PER ANNUM IS ?

p=Rs 30,000

n= 2 years
i = 5 % per annum

compounding rest = 1 year

Therefore i = 5/100 =0.05

Interest = A –p

A= p(1 +i)n

A= 30,000(1 +0.05)2

A= 30,000(1 .05)2

A= 30,000(1. 1025)

A= 33,075

Interest = A—p

Interest = 33,075—30,000

Interest = Rs3,075

CA EXAMINATION QUESTION:

IF Rs1,000 BE INVESTED AT AN INTEREST RATE OF 5 % PER ANNUM AND THE INTEREST BE


ADDED TO THE PRINCIPAL EVERY 10 YEARS. THEN THE NUMBER OF YEARS IN WHICH IT WILL
AMOUNT TO Rs 2,000 is ?

Solution :
Assumptions of the problem

 For the first 10 years, the bank is applying simple interest.


 We have to find the AMOUNT OF MATURITY of the PRINCIPAL RS1,000 AFTER 10
years by way of applying the concept of simple interest.
 Then the 11 th year beginning value is considered as fresh principal.
 We need to continue simple interest concept and find new n for that new principal
to become Rs2,000

Step 1 :

P=Rs1,000

r=5 % per annum

n= 10 years

A=?

A= P +( pnr /100)

A= 1,000 + [( 1,000)(10)(5) /100)]

A= 1,000 + [ 500]

A= 1, 500

Step 2 :

A= P + ( pnr /100)

2,000=1,500 + [( 1,500)(n)( 5) /100)]

2,000=1,500 + [( 7500n /100)]

2,000--1,500 = [( 7500n /100)]


500= 7500n /100

500x100= 7500n

50,000= 7500n

50,000/7500=n

6.67=n

Answer : n=16.67 years.

CA EXAMINATION QUESTION:

THE EFFECTIVE RATE EQUIVALENT TO NOMINAL RATE OF 6 % COMPOUNDED MONTHLY IS ?

SOLUTION :

LET Rs5,000 =P

AS PER COMPOUND INTEREST FORMULA WITH MONTHLY COMPOUNDING, THE AMOUNT OF


MATURITY AFTER 1 YEAR AT 6% INTEREST PER ANNUM WILL BE

A=P(1+i)n

A=5,000(1+0.005)n

A=5,000(1.005)n

A=5,000(1.005)12

A=5,000(1.06167781182)

A=5,308.38

INTEREST =A—P=5308.38—5000=308.38
EFFECTIVE RATE OF INTEREST PER ANNUM=

308.38

---------- X100=6.17%

5000

=6.17 %

Exercise :

Find the effective annual rate of interest if a debt investment scheme offers compounding once
in 2 months at a coupon rate of interest of 12% per annum.

SOLUTION :

THROUGH AN ILLUSTRATION WE CAN GET THE ANSWER…

LET P=1,000

i = 12 % PER ANNUM

COMPOUNDING REST =2 MONTHS

i = PER COMPOUNDING PERIOD=

[12 %/12 MONTHS] X 2 MONTHS= 2% PER 2 MONTHS

=2/100=0.02

A= p(1 +i)n

A= 1,000(1 +0.02)6
A= 1,000(1 .02)6

A= 1,000 (1.12616241926)

A= 1,126.16

TOTAL INTEREST = A—P

TOTAL INTEREST = 1,126.16—1,000

TOTAL INTEREST = 126.16

TOTAL INTEREST PER ANNUM =126.16 /1 =126.16

EFFECTIVE RATE OF INTEREST PER ANNUM

=(INTEREST PER ANNUM / P ) X100

=(Rs 126.16 /1,000) x100

=12.62 %

CA EXAMINATION QUESTION:

A PERSON DEPOSITED RS 5,000 IN A BANK. THE DEPOSIT WAS LEFT TO ACCUMULATE AT 6 %


PER ANNUM COMPOUNDED QUARTERLY FOR THE FIRST 5 YEARS AND AT 8 % PER ANNUM
COMPOUNDED SEMI ANNUALLY FOR THE NEXT 8 YEARS. THE COMPOUNDED AMOUNT AT THE
END OF 13 YEARS IS ?

SOLUTION:

FIRST STAGE TERMS OF THE BANK:

P=Rs5,000

i= 6 % per annum

here the compounding rest =3 months

therefore i for the 3 months =


(6% /12 months) x 3 months

=1.5 % for 3 months

=1.5/100

=0.015 for 3 months

Therefore strictly for formula i =0.015

P=Rs 5,000

n= 5years x4 quarters per year =20 quarters =20

i= 0.015

A = P(1 +i)n

A = 5,000 (1 +0.015)20

A = 5,000 (1 .015)20

A = 5,000 (1 .34685500644)

A = 6,734.28

SECOND STAGE TERMS OF THE BANK:

P=Rs6,734.28

i= 8 % per annum

here the compounding rest =6 months

therefore i for the 6 months =

(8% /12 months) x 6 months

=4 % for 6 months
=4/100

=0.04 for 6 months

Therefore strictly for formula i =0.04

P=Rs 6,734.28

n= 8years x 2 HALF YEARS per year =16 HALF YEARS =16

i= 0.04

A = P(1 +i)n

A = 6,734.28 (1 +0.04)16

A = 6,734.28 (1 .04)16

A = 6,734.28 (1 .87298124566)

A = 12,613.18

========lesson of 28.10.2017 completed=====

CA EXAMINATION QUESTION:

A PERSON DEPOSITED RS 5,000 IN A BANK. THE DEPOSIT WAS LEFT TO ACCUMULATE AT X %


PER ANNUM COMPOUNDED YEARLY FOR 2 YEARS AND THE AMOUNT OF MATURITY IS
Rs5512.50.FIND X.

SOLUTION :

A=P(1+i)n

5512.5=5000(1+i)2

5512.5/5000=(1+i)2

1.1025=(1+i)2
TAKING SQUARE ROOT ON BOTH SIDES

1.05=(1+i)

1.05=1+i

1.05—1=i

0.05=i

5%=i.

CA EXAMINATION QUESTION:

A PERSON DEPOSITED RS 18,000 IN A BANK. THE DEPOSIT WAS LEFT TO ACCUMULATE AT X %


PER ANNUM COMPOUNDED YEARLY FOR 2 YEARS AND THE AMOUNT OF MATURITY IS
Rs20,224.80.FIND X.

SOLUTION :

A=P(1+i)n

20,224.80=18000(1+i)2

20,224.80/18000=(1+i)2

1.1236=(1+i)2

TAKING SQUARE ROOT ON BOTH SIDES

1.06=(1+i)

1.06=1+i

1.06—1=i

0.06=i

6%=i.

=========================================
CA EXAMINATION QUESTION:

A PERSON WANTS TO DOUBLE A SUM IN 5 YEARS TIME AT SIMPLE INTEREST PER ANNUM. FIND
THE RATE OF SIMPLE INTEREST PER ANNUM.

SOLUTION:

LET P=Rs1000

A= P + PNR/100

2000=1000+PNR/100

1000=PNR/100

1000=1000(5)(R)/100

1000=5000R/100

1000=50R

1000/50=R

20=R

CA EXAMINATION QUESTION:

A PERSON WANTS TO TRIPLE A SUM IN 5 YEARS AT SIMPLE INTEREST PER ANNUM. FIND THE
RATE OF SIMPLE INTEREST PER ANNUM.
SOLUTION:

LET P=Rs1000

A= P + PNR/100

3000=1000+PNR/100

2000=PNR/100

2000=1000(5)(R)/100

2000=5000R/100

2000=50R

2000/50=R

40=R

CA EXAMINATION QUESTION:

A PERSON WANTS TO DOUBLE A SUM IN 2 YEARS AT COMPOUND INTEREST PER ANNUM. FIND
THE RATE OF coupon[nominal] COMPOUND INTEREST PER ANNUM.

SOLUTION :

A=P(1+i)n

10,000=5,000(1+i)2

10,000/5,000=(1+i)2

2=(1+i)2

TAKING SQUARE ROOT ON BOTH SIDES

1.41421356237=(1+i)

1. 41421356237=1+i

1. 4142135623706—1=i

0. 4142135623706=i

41.42%=i.
CA EXAMINATION QUESTION:

RAM DEPOSITS Rs500 ANNUALLY FOR 3 YEARS AT 6% COMPOUND INTEREST.FIND THE SUM.

SOLUTION :

DATES OF DEPOSIT

14-6-2015> A=P(1+i)2=500(1+0.06)2=500(1.06)2=561.8

14-6-2016> A=P(1+i)1=500(1+0.06)1=500(1.06)=530

14-6-2017> A=P(1+i)0=500(1+0.06)0=500(1)=500

-------------------------------------------------------------------

1591.8

-------------------------------------------------------------------

FORMULA FOR YEAR END ANNUITY:

A=[P/i] [(1+i)n ---1 ]

A=[500/0.06] [(1+0.06)3 ---1 ]

A=[8333.33333] [(1.06)3 ---1 ]

A=[8333.33333] [(1.191016) ---1 ]

A=1591.8

CA EXAMINATION QUESTION:

KUMAR DEPOSITS Rs10,000 ANNUALLY FOR 7 YEARS AT 12% COMPOUND INTEREST.FIND THE
SUM.
SOLUTION :

FORMULA FOR YEAR END ANNUITY:

A=[P/i] [(1+i)n ---1 ]

A=[10,000/0.12] [(1+0.12)7 ---1 ]

A=[83333.33333] [(1.12)7 ---1 ]

A=[83333.33333] [(2.2106814074) ---1 ]

A=[83333.33333] (1.2106814074)

A=1,00,890

CA EXAMINATION QUESTION:

BALAJI PAYS 4 ANNUAL PAYMENTS OF Rs4,000 EACH IN HIS DEPOSIT ACCOUNT THAT PAYS 10%
COMPOUND INTEREST PER YEAR.FIND OUT THE FUTURE VALUE OF ANNUITY AT THE END OF 4
YEARS.

ANSWER=Rs18,564

CA EXAMINATION QUESTION:

THE DIFFERENCE BETWEEN THE SIMPLE AND COMPOUND INTEREST ON A CERTAIN SUM FOR 3
YEARS AT 5 % PER ANNUM IS Rs228.75.THE COMPOUND INTEREST ON THE SUM FOR 2 YEARS
AT 5% PER ANNUM IS Rs X. FIND X.

SOLUTION :

COMPOUND INTEREST BASIS

A=P(1+i)n=P(1+0.05)3=P(1.05)3= P(1.157625)

SIMPLE INTEREST BASIS

A=P+ [PNR/100]=

P + (P)(3)(5)/100]=P+0.15P=1.15P

P(1.157625)--1.15P=228.75

1.157625P--1.15P=228.75
0.007625P=228.75

P=228.75/0.007625

P=30,000

COMPOUND INTEREST BASIS

A=P(1+i)n=30,000(1+0.05)2=30,000(1.05)2= 30,000(1.1025)=Rs33,075

I=A—P=33,075—30,000=3,075.

CA EXAMINATION QUESTION:

SIMPLE INTEREST ON RS2,000 FOR 5 MONTHS AT 16% PER ANNUM=?

SOLUTION:

[2,000][5/12][16/100]=133.33

CA EXAMINATION QUESTION:

THE SIMPLE INTEREST ON A SUM OF MONEY IS 4/9 OF PRINCIPAL AND THE NUMBER OF YEARS
IS EQUAL TO THE RATE OF INTEREST PER ANNUM.FIND THE RATE OF INTEREST PER ANNUM.

SOLUTION:

PNR/100 = [4/9]P

NR/100 = [4/9]

N=R

SO

RR/100 = [4/9]
R2 /100 = [4/9]

R2 = [4/9] X100

TAKING SQUARE ROOT ON BOTH SIDES

R = [2/3] X10

R = [20/3]

R=6.67

WE CAN SAY THAT R=6.67% PER ANNUM

CA EXAMINATION QUESTION:

NOMINAL RATE OF INTEREST IS 9.9% PER ANNUM. IF INTEREST IS COMPOUNDED MONTHLY,


FIND THE EFFECTIVE RATE OF INTEREST PER ANNUM.

SOLUTION:

A=P(1+i)n

A=P(1+0.00825)12

A=P(1.00825)12

A=P(1.10361798203)

A=(1.10361798203)P
LET US ASSUME P AS 5,000

A=(1.10361798203)(5000)

A=5518.09

SO INTEREST =5518.09—5000=518.09

EFFECTIVE RATE OF INTEREST PER ANNUM=[518.09/5000] X100=10.36%


CA EXAMINATION QUESTION:

NOMINAL RATE OF INTEREST IS 12% PER ANNUM. IF INTEREST IS COMPOUNDED ONCE IN 2


MONTHS, FIND THE EFFECTIVE RATE OF INTEREST PER ANNUM.

SOLUTION:

A=P(1+i)n

A=P(1+0.02)6

A=P(1.02)6

A=P(1.12616241926)

A=(1.12616241926)P
LET US ASSUME P AS 5,000

A=(1.12616241926)(5000)

A=5630.81

SO INTEREST =5630.81—5000=630.81

EFFECTIVE RATE OF INTERST PER ANNUM=[630.81/5000] X100=12.62%

CA EXAMINATION QUESTION:

IF THE SIMPLE INTEREST ON Rs1,400 FOR 3 YEARS IS LESS THAN THE SIMPLE INTEREST ON
Rs1,800 FOR THE SAME PERIOD BY Rs80,FIND THE RATE OF INTEREST PER ANNUM.
SOLUTION :

1800 X 3 X R/100(---)1400X3XR/100=80

54 R(---)42R=80

12R=80

R=80/12=6.67

SO THE RATE OF INTEREST PER ANNUM=6.67%

CA EXAMINATION QUESTION:

BY MISTAKE , A CLERK, CALCULATED THE SIMPLE INTEREST ON PRINCIPAL FOR 5 MONTHS AT


6.5% PER ANNUM INSTEAD OF 6 MONTHS AT 5.5% PER ANNUM. IF THE ERROR IN
CALCULATION WAS Rs25.40,FIND THE PRINCIPAL.

SOLUTION :

P X 5/12 X 6.5/100=0.027083333P

P X 5.5/12 X 6/100=0.0275P

0.0275P--0.027083333P=25.4

0.000416667P=25.4

P=25.4/0.000416667=60960

PROOF

PRINCIPAL= Rs60960

CORRECT CALCULATION

SIMPLE INTEREST=PNR/100

60960X6/12X5.5/100=1676.40
INCORRECT CALCULATION

SIMPLE INTEREST=PNR/100

60960X5/12X6.5/100=1651.00

DIFFERENCE =1676.40—1651.00=25.40

CA EXAMINATION QUESTION:

IF THE SIMPLE INTEREST ON A SUM OF MONEY AT 6% PER ANNUM FOR 7 YEARS IS EQUAL TO
TWICE OF SIMPLE INTEREST ON ANOTHER SUM FOR 9 YEARS AT 5 % PER ANNUM,FIND THE
RATIO IN BETWEEN TWO SUMS.

SOLUTION :

P(7)(0.06)=2(Q)(9)(0.05)

0.42P=0.9Q

P/Q= 0.90/0.42

P:Q = 0.90 : 0.42

P:Q = 90 : 42

P:Q = 30 : 14

P:Q = 15 : 7
CA EXAMINATION QUESTION:

IF THE DIFFERENCE OF SIMPLE INTEREST AND COMPOUND INTEREST IS Rs72 AT 12% FOR 2
YEARS,FIND PRINCIPAL.

SOLUTION:

P(1+0.12)2 –[P+P(2)(0.12)]=72

P(1.12)2 –[P+0.24P)]=72

P(1.2544) –[1.24P]=72

1.2544P –1.24P=72

0.0144P =72

P=72/0.0144

P=5,000

CA EXAMINATION QUESTION:

Mr RAM INVESTS “P” AMOUNT AT SIMPLE INTEREST RATE OF 10% PER ANNUM AND Mr
KUMAR INVESTS AMOUNT “Q” AT COMPOUND INTEREST OF 5 % PER ANNUM COMPOUNDED
ANNUALLY.AT THE END OF 2 YEARS OF DEPOSITING BOTH HAVE GOT THE SAME AMOUNT OF
INTEREST. P:Q =?

SOLUTION:

P(2)(0.10) = Q(1+0.05)2—Q

0.20P = Q(1.05)2—Q

0.20P = Q(1.1025)—Q
0.20P = (1.1025)Q—Q

0.20P = 0.1025Q

P 0.1025

--- = ----------

Q 0.2000

P 1025

--- = ----------

Q 2000

P:Q=1025:2000

P:Q=41:80

CA EXAMINATION QUESTION:

THE EFFECTIVE ANNUAL RATE OF INTEREST CORRESPONDING TO NOMINAL RATE 6% PER


ANNUM COMPOUNDED HALF YEARLY=?

SOLUTION :

LET P=Rs5,000

A AS PER COMPOUNDING INTEREST FORMULA

IS
A=P(1+i)n

A=5,000(1+0.03)2

A=5,000(1.03)2

A=5,000(1.0609)

A=5,304.50

INTEREST FOR 1 YEAR =5304.50—5000=304.5

EFFECTIVE RATE OF INTEREST PER ANNUM=[304.5/5000] X100=6.09%

CA EXAMINATION QUESTION:

THE FUTURE VALUE OF AN YEAR END ANNUITY OF Rs5,000 INVESTED ANNUALLY FOR 8 YEARS
AT AN INTEREST RATE OF 9 % COMPOUNDED ANNUALLY=?

SOLUTION :

FORMULA FOR YEAR END ANNUITY:

A =[P/i] [(1+i)n --1 ]

A= [5,000/0.09] [(1+0.09)8 ---1 ]

A= [5,000/0.09] [(1.09)8 ---1 ]

A= [5,000/0.09] [(1.99256264168) ---1 ]

A= [55,555.55555] [(1.99256264168) ---1 ]

A= [55,555.55555] (0.99256264168)

A= Rs 55,142.37

CA EXAMINATION QUESTION:
IF THE SIMPLE INTEREST ON A SUM OF MONEY AT 12% PER ANNUM FOR TWO YEARS IS
Rs3,600,FIND THE COMPOUND INTEREST ON THE SAME SUM FOR SAME PERIOD AT SAME
INTEREST RATE .

SOLUTION :

 SIMPLE INTEREST
PNR/100=3600

P(2)(0.12)=3600

P(0.24)=3600

P=3600/0.24

P=15,000

COMPOUND INTEREST

 A—P
P(1+i)n –P
15,000(1+0.12)2—15,000
15,000(1.12)2—15,000
15,000(1.2544)—15,000
18,816—15,000
3,816

CA EXAMINATION QUESTION:

FIND THE FUTURE VALUE OF AN ANNUITY OF Rs1,000 PAYABLE AT THE END OF EACH YEAR FOR
10 YEARS AT 6% PER ANNUM WITH YEARLY COMPOUNDING.

SOLUTION :

FORMULA FOR YEAR END ANNUITY:

A=[P/i] [(1+i)n ---1 ]

A= [1,000/0.06] [(1+0.06)10 ---1 ]

A= [1,000/0.06] [(1.06)10 ---1 ]


A= [1,000/0.06] [(1.79084769651) ---1 ]

A= [16,666.6666666] [0.79084769651)

A= 13,180.79

CA EXAMINATION QUESTION:

FIND THE PRESENT VALUE OF AN ANNUITY OF Rs1,000 PAYABLE AT THE END OF EACH YEAR
FOR 10 YEARS AT 6% PER ANNUM WITH YEARLY COMPOUNDING.

SOLUTION :

FORMULA FOR PRESENT VALUE YEAR END ANNUITY:

[P/i] [(1+i)n--1 ] X {100/(100+i)} n

= [1,000/0.06] [(1+0.06)10 ---1 ] {100/(100+6)}10

= [1,000/0.06] [(1.06)10 ---1 ] {100/(106)}10

= [1,000/0.06] [(1.79084769651) ---1 ]{0.5583934}

= [16,666.6666666] [0.79084769651){ {0.5583934}

= 13,180.79{{0.5583934}
=Rs 7,360.06

CA EXAMINATION QUESTION:

THE COMPOUND INTEREST FOR A CERTAIN SUM @ 5 % PER ANNUM FOR FIRST YEAR IS
Rs25.THE SIMPLE INTEREST FOR THE SAME PRINCIPAL @ 5% PER ANNUM FOR 2 YEARS = ?

SOLUTION :

COMPOUND INTEREST

 A—P
P(1+i)n –P=25
P(1+0.05)1—P=25
P(1.05)—P=25
P(1.05—1)=25
P(0.05)=25
P=25/0.05
P=500
 SIMPLE INTEREST
PNR/100=500(2)(0.05)

PNR=500(2)(0.05)

PNR=50

CA EXAMINATION QUESTION:

IN HOW MANY YEARS , A SUM OF Rs1,000 COMPOUNDED ANNUALLY @ 10% PER ANNUM WILL
AMOUNT TO Rs1,331?

SOLUTION :

COMPOUND INTEREST

A=P(1+i)n
1,331=1,000(1+0.10)n
1,331=1,000(1.10)n

1,331/1000=(1.10)n

1.331=(1.10)n

BY TRIAL AND ERROR METHOD, IF WE PUT 3, THE EQUATION SETTLES.

HENCE

n=3 YEARS.

CA EXAMINATION QUESTION:

IN HOW MANY YEARS , A SUM WILL DOUBLE ITSELF AT 5% PER ANNUM AT COMPOUND
INTEREST?

SOLUTION :

P[1.05]n =2P

[1.05]n =2

n LIES IN BETWEEN 14 AND 15.


SOLUTION:

COMPOUND INTEREST

A=P(1+i)n
2,000=1,000(1+0.05)n
2,000=1,000(1.05)n

2,000/1000=(1.05)n

2=(1.05)n

n lies in between 14 years and 15 years

ASSUME P AS Rs5,000

 at 14 th year end A=5,000(1.05)14=9,899.6579


====================================
Fresh principal= Rs9,899.6579
r=0.05
Interest=10,000—9899.6579=100.3420031

Pnr=100.6420031
9899.6579(n)(0.05)=100.6420031
494.982895(n)=100.6420031
n=100.6420031/494.982895
n=0.20332120396
so 14 +0.20332120396 years
so14.20332120396 years
so14 .3 years.
So 14 years and 2.4 months

CA EXAMINATION QUESTION:

IN HOW MANY YEARS , A SUM WILL DOUBLE ITSELF AT 8% INTEREST PER ANNUM ?

SOLUTION

100
---- =12.5 YEARS.
8
CA EXAMINATION QUESTION:

IN HOW MANY YEARS , A SUM WILL TRIPLE ITSELF AT 8% INTEREST PER ANNUM ?

SOLUTION

200
---- =25 YEARS.
8

CA EXAMINATION QUESTION:

IN HOW MANY YEARS , A SUM WILL QUADRUPLE ITSELF AT 25% INTEREST PER ANNUM ?

SOLUTION
300
---- =12 YEARS.
25

CA EXAMINATION PROBLEM ON SINKING FUND


Mr SWAMY ACQUIRED A MACHINERY AT A COST OF Rs2,50,000 ON 01-04-2005. ITS LIFE IS 3
YEARS.
Mr SWAMY CREATED A SINKING FUND ON 31-03-2006 AND INVESTED THE SINKING FUND IN THE
GOVERNMENT SECURITY WHICH FETCHES 9 % RATE OF INTEREST PER ANNUM. HE MADE
ADDITIONAL DEPOSITS TO SINKING FUND ON 31-03-2007 AND
31-03-2008.
FIND THE SINKING FUND DEPOSITS ON 31-3-2006, 31-3-2007 AND 31-3-2008.
ON 31-3-2008, Mr SWAMY HAD LIQUIDATED SINKING FUND INVESTMENT AND WITH THAT
CASH ,HE HAD BOUGHT A NEW MACHINE.

SOLUTION:

FORMULA :

A =P x { (1+i) n –1 }/ i

Rs2,50,000=P x { (1+0.09) 3 –1 }/ 0.09

Rs2,50,000=P x { (1.09)3 –1 }/ 0.09

Rs2,50,000=P x { (1.295029) –1 }/ 0.09


Rs2,50,000=P x ( 0.295029) / 0.09

Rs2,50,000=P x 3.2781

Rs2,50,000/3.2781 = P

Rs76,263.69 = P

Annual depreciation amount=Rs76,263.69

ANSWER: Rs76,263.69

CA EXAMINATION QUESTION:

Mr SWAMY WANTS Rs2,50,000 AT THE END OF 3YEARS FROM NOW. HOW MUCH IS THE YEAR
END ANNUITY AT 9% INTEREST PER ANNUM COMPOUNDED ANNUALLY?

ANSWER: Rs76,263.69

========================================

RAM GOT AN INTEREST AMOUNT OF Rs240 FROM HIS 2 MONTH DEPOSIT OF PRINCIPAL P.[ P
NOT GIVEN IN THE PROBLEM]. THE RATE OF INTEREST IS 18% PER ANNUM. ASSUMING SIMPLE
INTEREST BASIS, FIND OUT P.

SOLUTION

I=PNR/100

240=Px[2/12] x [18/100]

240= P x 0.03

240/0.03=P

8,000=P

ANSWER: Rs8,000.

=================================

EXERCISE :
RAM DEPOSITED Rs10,000 ON 1ST APRIL 2013. INTEREST RATE PER YEAR IS 8% . THE
COMPOUNDING REST IS HALF YEARLY BASIS.FIND “A” AS AT 31 ST MARCH 2014.

SOLUTION:

A= P[ 1+i]n

A= 10,000[ 1+4%]2

A= 10,000[ 1+(4/100)2

A= 10,000[ 1+0.04]2

A= 10,000[ 1.04]2

A= 10,000[ 1.0816]

A= 10,816

TI= A—P

TI= 10,816—10,000

TI= 816

EFFECTIVE RATE OF INTEREST PER YEAR

[TI/ NUMBER OF YEARS]

-------------------------------- X 100

816/1

----------- X100

10,000
816

--------- X100

10,000

8.16%

=========================================

EXERCISE :

KUMAR DEPOSITED Rs10,000 ON 1ST APRIL 2013. INTEREST RATE PER YEAR IS 12% . THE
COMPOUNDING REST IS QUARTERLY BASIS . FIND “A” AS AT 31 ST MARCH 2014.

SOLUTION:

A= P[ 1+i]N

A= 10,000[ 1+3%]4

A= 10,000[ 1+(3/100)4]

A= 10,000[ 1+0.03]4

A= 10,000[ 1.03]4

A= 10,000[ 1.12550881]

A= 11,255

TI= A—P

TI= 11,255—10,000
TI= 1255

EFFECTIVE RATE OF INTEREST PER YEAR

[TI/ NUMBER OF YEARS]

-------------------------------- X 100

1255/1

----------- X100

10,000

1255

--------- X100

10,000

12.55%

EXERCISE :

RAMESH DEPOSITED Rs10,000 ON 1 ST APRIL 2013. INTEREST RATE PER YEAR IS 12% . THE
COMPOUNDING REST IS HALF YEARLY BASIS. FIND “A” AS AT 31 ST MARCH 2015.

SOLUTION:

A= P[ 1+i]n

A= 10,000[ 1+6%]4

A= 10,000[ 1+(6/100)4]

A= 10,000[ 1+0.06]4
A= 10,000[ 1.06]4

A= 10,000[ 1.26247696]

A= 12,625

TI= A—P

TI= 12,625—10,000

TI= 2625

EFFECTIVE RATE OF INTEREST PER YEAR

[TI/ NUMBER OF YEARS]

-------------------------------- X 100

2625/2

----------- X100

10,000

1312.5

--------- X100

10,000
13.125%
CA EXAMINATION QUESTION:

BY MISTAKE , A CLERK, CALCULATED THE SIMPLE INTEREST ON PRINCIPAL FOR 5 MONTHS AT


6.5% PER ANNUM INSTEAD OF 6 MONTHS AT 5.5% PER ANNUM. IF THE ERROR IN
CALCULATION WAS Rs25.40,FIND THE PRINCIPAL.

Solution :

X x [5/12] x [6.5/100 ] =32.5X/1200=0.027083333x

X x [6/12] x [5.5/100 ] =33X/1200=0.0275x

0.0275x—0.027083333x =25.40

0.00041666x =25.40

X=60960

----------------------------------------------------------------

A= P{[1+i]n –1}

--------------

35,000= P{[1+0.08]4 –1}

-------------------

0.08

35,000 X.08 = P{[1.08]4 –1}

2,800 = P{[1.36048896] –1}

2,800 = P{[1.36048896] –1}

2,800 = 1.36048896P –P

2,800 = 0.36048896P
P= 2,800/0.36048896

P=7,767.23

---------------------------------------------------------

TOTAL DEPOSITS =7,767.23 X4 = Rs 31,068.92

7767.23 [1.08]3 =9,784.47

7767.23 [1.08]2 =9,059.69

7767.23 [1.08]1 =8,388.61

7767.23 [1.08]0 =7,767.23

---------------------------------------

35,000

---------------------------------------

Total interest earned= Rs3,931.08


Interest income

Financier

Lender

Investor

Deposit scheme

Debenture / bond fixed deposit

Annuity and interest

Mr. Ram

Rs4,00,000. Every 31st December.

[31.12.2016]

9 years

Rs25,00,000 [31-12-2025]
Interest 10 % per annum compounded annually.

31.12.16 xc

31.12.17

31.12.18

31.12.19

31.12.20

31.12.21

31.12.22

31.12.23

31.12.24

31.12.25

P= Principal deposited on 31.12.2016

Principal deposited on 31.12.2017

Principal deposited on 31.12.2018

Principal deposited on 31.12.2019

De Principal deposited on 31.12.2020

Principal deposited on 31.12.2021


Principal deposited on 31.12.2022

Principal deposited on 31.12.2023

Principal deposited on 31.12.2024

Principal deposited on 31.12.2025

r= rate of interest per annum , compounded annually

n=number of times the same principal is invested =10 years

A= 25,00,000

P=?

FORMULA FOR YEAR END ANNUITY:

A =[P/i] [(1+i)n --1 ]

A =[Rs5,000/0.09] [(1+0.09)8 --1 ]

A =[5,000/0.09] [(1+0.09)8 --1 ]

A =[55,555.56] [(1.09)8 --1 ]

A =[55,555.56] [(1.99256264) --1 ]

A =[55,555.56] [0.99256264) ]

A =Rs.55,142

============
PRESENT VALUE

6.8.2016 = Rs X

6.8.2018 = Rs X + --- = Rs1,00,000

RATE OF INTEREST PER ANNUM = 10%.

===================================

Rs1 X + [Rs 0.10X] +[Rs 0.11x] =Rs1,00,000

Rs 1.21 X = Rs1,00,000

X =1,00,000/1.21 =82,644.67

=========================================

On 16.4.2020 Mr Ram plans to sell AN ASSET for Rs3,000.

Applicable rate of interest [ bank rate]= 10% per annum.

On 16.4.2017, for what value will Mr Ram will BUY the ASSET ?

SOLUTION

3000 x [ 100/110]3 = BUYING PRICE

3000 x 0.751314 = 2253.94

=======================================

Example :

On 16.4.2020 Mr Suresh will get Rs3,000 from some person. [ Future Value]

The rate of interest is 10% per annum.

What is the PRESENT VALUE of Rs3,000 of 16.4.2020 on 16.4.2017 ?


Answer :

PRESENT VALUE OF A FUTURE VALUE=

[FUTURE VALUE]

---------------------------
n
[1+ i]

[Rs 3,000]

---------------------------
3
[1+ 0.10]

[Rs3,000]

---------------------------
3
[1.10]

[Rs3,000]

---------------------------
1.331

= Rs2,253.94

=====================================

HOME WORK OF 16.4.2017

[1] FIND THE FUTURE VALUE OF YEAR END ANNUITY OF Rs5,000 FOR 4 YEARS AT 6% PER
ANNUM WITH YEARLY COMPOUNDING ?

--------------------------------------------------------------
[2] FIND THE PRESENT VALUE OF YEAR END ANNUITY OF Rs5,000 FOR 4 YEARS AT 6% PER
ANNUM WITH YEARLY COMPOUNDING ?

--------------------------------------------------------------

16.4.2017

 SIMPLE INTEREST [Pnr/100]


 COMPOUND INTEREST A= P[1+i]n
 ANNUITY A =[P/i] [(1+i)n --1 ]
-----------------------------------------------------------------

EXAMPLE [1] ON ANNUITY

Mr. MUNEESH PLANS TO INVEST Rs5,000 AT THE END OF EVERY YEAR FOR THE NEXT 3 YEARS
16.4.18, 16.4.19 AND 16.4.20 IN AN ANNUITY OF BANK OF INDIA. THE INTEREST IS 9% PER
ANNUM COMPOUNDED ANNUALLY.

QUESTION : FIND THE SUM THAT Mr MUNEESH CAN DRAW FROM THE BANK ON 16.4.20.

 ANNUITY A =[P/i] [(1+i)n --1 ]


 ANNUITY A =[5,000/0.09] [(1+0.09)3 --1 ]
 ANNUITY A =[55,555.56] [(1.09)3 --1 ]
 ANNUITY A =[55,555.56] [1.295029 --1 ]
 ANNUITY A =[55,555.56] [0.295029 ]
 ANNUITY A =16,390.

THE INVESTOR Mr MUNEESH WILL ACCUMULATE AN AMOUNT OF Rs16,390 ON


16.4.2020.

===================================
WHAT IS THE TOTAL AMOUNT OF INTEREST THAT THE INVESTOR HAD EARNED
FROM THE ABOVE 3 YEAR ANNUITY ?

ANSWER = AMOUNT – SUM OF PRINCIPALS


TH
RsRs16,390 –[ Rs 5,000+ Rs5,000 + Rs5,000]= Rs 16,390—Rs15,000 = Rs1,390
=================================

What is the average annual interest which was earned by the investor from the
above annuity investment scheme ?

ANSWER :

TOTAL AMOUNT OF INTEREST EARNED


-----------------------------------------------------
NUMBER OF YEARS OF ANNUITY

RS1,390
-----------
3 YEARS
= Rs 463.33
========================================

Exercise :

Prabhat wants to accumulate Rs5,000 on 16.4.2021 to purchase a watch .

What is the year end annuity if he decides to create an year end annuity from now [16.4.17] i.e
the deposits will be made on 16.4.18 ,16.4.19 ,16.4.20 and 16.4.21 at 7% per annum at Indian
bank.

Find the principal to be invested each on these four dates.

Solutionsolution :

 ANNUITY A =[P/i] [(1+i)n --1 ]


 ANNUITY 5,000 =[P/0.07] [(1+0.07)4 --1 ]
5,000 =[P/0.07] [(1.07)4 --1 ]

5,000 =[P/0.07] [1.3107960 --1 ]

5,000 =[P/0.07] [0.3107960 ]

5,000/ [0.3107960 ]=[P/0.07]

16087.72=P/0.07

116087.72 X 0.07=P
1,126.14 = P

CA EXAMINATION QUESTION:

BY MISTAKE , A CLERK, CALCULATED THE SIMPLE INTEREST ON PRINCIPAL FOR 5 MONTHS AT


6.5% PER ANNUM INSTEAD OF 6 MONTHS AT 5.5% PER ANNUM. IF THE ERROR IN
CALCULATION WAS Rs25.40 ,FIND THE PRINCIPAL.

Solution

Correct interest = pni/100 = p (6/12)(5.5/100)

Correct interest = pni/100 = p (0.5)(0.55)

Correct interest = pni/100 = p (0.0275)= 0.0275 p

InCorrect interest = pni/100 = p (5/12)(6.5/100)

InCorrect interest = pni/100 = p (5/12)(0.065)

InCorrect interest= pni/100 = p (0.02708333)=0.027083333 p

0.0275 p --0.027083333p=25.40

0.0004166667p= 25.4

p=25.4/0.000416667

p= 60,960

CLASS WORK OF 30.10.2017

[1] IF Rs1,000 IS TO BE RECEIVED AFTER 2 YEARS FROM A SINGLE DEPOSIT TO BE MADE NOW AT
COMPOUNDED RATE OF INTEREST OF 8% PER ANNUM , COMPOUNDED SEMI ANNUALLY ,
DETERMINE THE AMOUNT OF DEPOSIT TO BE MADE.

ANSWER :Rs854.81

[2] IF Rs1,000 IS TO BE RECEIVED AFTER 2 YEARS FROM A SINGLE DEPOSIT TO BE MADE NOW AT
COMPOUNDED RATE OF INTEREST OF 8% PER ANNUM , COMPOUNDED ANNUALLY ,
DETERMINE THE AMOUNT OF DEPOSIT TO BE MADE.

ANSWER :Rs 857.33


[3] IF A PERSON WANTS TO ACCCUMULATE Rs1,000 BY WAY OF 3 YEAR END ANNUITY AT 10%
RATE OF INTEREST COMPOUNDED ANNUALLY, DETERMINE THE ANNUITY AMOUNT.

ANSWER :Rs 302.11

[4] IF A PERSON PLANS TO SELL A SHARE FOR Rs1,000 AFTER 2 YEARS FROM NOW
CONSIDERING 10% RATE OF INTEREST COMPOUNDED ANNUALLY, DETERMINE THE PURCHASE
AMOUNT OF SHARE THAT IS JUSTIFIED.

ANSWER :Rs 826.44

[5] FIND THE PRESENT VALUE OF 4 YEAR END ANNUITIES OF Rs1,000 EACH AT 10% RATE OF
INTEREST COMPOUNDED ANNUALLY.

ANSWER :Rs 3,168.40

[6] FIND THE FUTURE VALUE OF 4 YEAR END ANNUITIES OF Rs1,000 EACH AT 10% RATE OF
INTEREST COMPOUNDED ANNUALLY.

ANSWER :Rs 4,641

[7] FIND THE PRESENT VALUE OF Rs1,000 DUE IN THREE YEARS AT 6% PER ANNUM
COMPOUNDED QUARTERLY.

SOLUTION : 1,000 [ 100/101.5]12 =Rs 836.387

[8] FIND THE PRESENT VALUE OF Rs1,000 DUE IN THREE YEARS AT 6% PER ANNUM
COMPOUNDED six monthly .

SOLUTION : 1,000 [ 100/103]6 =Rs 837.48

[9] FIND THE PRESENT VALUE OF Rs1,000 DUE IN THREE YEARS AT 6% PER ANNUM
COMPOUNDED ANNUALLY .

SOLUTION : 1,000 [ 100/106]3 =Rs 839.61

[10] FIND THE PRESENT VALUE OF Rs1,000 DUE IN THREE YEARS AT 6% SIMPLE INTEREST PER
ANNUM .

SOLUTION :

A = P + TI

1,000 = P + PX3X6/100
1,000= P+18P/100

1000 = 118P/100

1000 X100 =118P

100000 =118 P

100000/118 =847.45

[11] WHAT IS THE PRESENT VALUE OF Rs500 DUE IN 4 YEARS AT 3% [ PER


ANNUM ]COMPOUNDED SEMI ANNUALLY ?

SOLUTION : 500 X [ 100/101.5]8 =444.04

[12] WHAT IS THE PRESENT VALUE OF Rs10,000 DUE IN 2 YEARS AT 8% [ PER ANNUM ]
COMPOUNDED SEMI ANNUALLY ?

SOLUTION : 10,000 X [ 100/104]4 =444.04

[13] FIND OUT HOW MUCH DOES A DEPOSIT OF Rs1,000 EVERY YEAR GROW AT THE END OF 6
YEARS, IF THE NOMINAL RATE OF INTEREST IS 12 % PER ANNUM AND THE FREQUENCY OF
COMPOUNDING IS 4 TIMES AN YEAR .

SOLUTION :

A= P { [ 1+i]n --1 } / { [ 1+i]n /YEARS --1 }

=1,000 { [ 1+0.03]24 --1 } / { [ 1+i]24/6YEARS --1 }

=1,000 { [ 1+0.03]24 --1 } / { [ 1+0.03]4 --1 }

=1,000 { 1.032794} / { [1.03] 4 – 1 }

=8,228.68
[14] FIND OUT HOW MUCH DOES A DEPOSIT OF Rs1,000 EVERY YEAR GROW AT THE END OF 1.5
YEARS, IF THE NOMINAL RATE OF INTEREST IS 24 % PER ANNUM AND THE FREQUENCY OF
COMPOUNDING IS 2 TIMES AN YEAR .

SOLUTION :

A= P { [ 1+i]n --1 } / { [ 1+i]n /YEARS --1 }

=1,000 { [ 1+0.12]3 --1 } / { [ 1+0.12]3/1.5YEARS --1 }

=1,000 { [ 1+0.12]3 --1 } / { [ 1+0.12]2 --1 }

=1,000 { 0.404928} / { [1.12] 4 – 1 }

=1,591.698

[15] FIND THE PRESENT VALUE OF AN ANNUITY OF Rs5,000 PER ANNUM FOR 12 YEARS, WITH
4% PER ANNUM, INTEREST BEING COMPOUNDED ANNUALLY.

ANSWER : Rs46,925

Home work[1] of 16.4.2017

An investor wants to accumulate Rs80,000 through a 3 year annuity [ year end] which fetches
interest at 8% per annum. Find the principal of each annuity. Also find the total principal.

Home work[2] of 20.8.2016

The simple interest for 9 months at 6% per annum on a principal is different as compared to
the simple interest for 6 months at 7.5% per annum for the same principal by Rs320. Find the
principal.

=1111

1111

ininn

I==========innnnnn
Interest chapter

Date : 20-4-2018

Question :

A company establishes a sinking fund to provide for the payment of Rs2,00,000 debt maturing
in 20 years. Contributions to the fund are to be made at the end of each year. Find the amount of
each annual deposit if interest is at 5% per annum.

Solution :

A =[P/i] [(1+i)n --1 ]

2,00,000 = [P/0.05] [(1+0.05)20 --1 ]

2,00,000 = [P/0.05] [(1.05) 20 --1 ]

2,00,000 = P/0.05 [2.65329770502—1]

2,00,000 = P/0.05 ( 1.65329770502)

2,00,000 x 0.05/ 1.65329770502 =P

6,048.50 =P

=============================================

Question :

A person borrows Rs5,000 for 2 years at 4% per annum at simple interest . He immediately lends
to another person at 6 ¼ % per annum for 2 years. Find the gain in the transaction per year.

Solution :

5,000 x 2.25 /100 x2


---------------------------- = 112.50

=============================================

Question :

A person deposited Rs5,000 in a bank. The deposit was left to accumulate at 6% PER ANNUM
compounded quarterly for the first five years and at 8% PER ANNUM compounded semi annually
for the next 8 years. The compound amount at the end of 13 years is ?.

Solution :

A = P ( 1+i )n

A = 5,000 ( 1+1.5/100 )20

A = 5,000 ( 1+0.015 )20

A = 5,000 ( 1.015 )20

A = 5,000 ( 1.34685500644 )

A = 6,734.2750

A = P ( 1+i )n

A = 6,734.2750 ( 1+4/100 )16

A = 6,734.2750 ( 1+0.04 )16

A = 6,734.2750 ( 1.04 )16

A = 6,734.2750 ( 1.87298124566 )

A = 12,613.17
=====================================================================

QUESTION :

Raja aged 40 years wishes his wife Rani to have Rs40,00,000 at his death. If his expectation of life
is another 30 years and he starts making equal annual investments commencing now at 3%
compound interest per annum. How much should he invest annually ?

Solution :
A =[P/i] [(1+i)n --1 ]

40,00,000 = [P/0.03] [(1+0.03)30 --1 ]

40,00,000 = [P/0.03] [(1.03)30 --1 ]

40,00,000 = P/0.03 [2.42726247103—1]

40,00,000 = P/0.03 (1.42726247103)

84,077.03 = P

=============================================
QUESTION :

Two equal sums of money were lent at simple interest at 11% per annum for 3 ½ years and 4 ½
years respectively. If the difference in interest for the two periods was Rs412.50 then such sum
is ?

Solution :

Difference in the simple interest earned

-----------------------------------------------------

Difference in years

= Annual simple interest

= Rs412.50

----------

= Rs412.50

Annual simple interest

--------------------------------
% of simple interest per annum
= Principal invested

Rs412.50

------------

11%

Rs412.50

------------

11/100

Rs412.50

------------

0.11

= Rs3,750
==================================================================

Question :

Anu’s father wishes to have Rs75,000 in a bank account when her first college expenses begin.
How much amount her father has to deposit now at 6.5% compounded annually if Anu has to
start college studies 8 years hence from now?

Solution :

A= P (1+i)n

75,000 = P ( 1+6.5/100)n

75,000 = P ( 1+ 0.065)8

75,000 = P ( 1.065)8

75,000 = P ( 1.65499567127)
75,000/
1.65499567127

=Rs45,317.33

============================================
QUESTION :

IN HOW MUCH TIME WOULD THE SIMPLE INTEREST ON A CERTAIN SUM BE 0.125 TIMES THE PRINCIPAL AT 10% PER ANNUM ?

SOLUTION
:

 LET THE PRINCIPAL BE 1,000.


 SIMPLE INTEREST PER YEAR=
Pnr/100 = [1000 X 1 X10]/100 =10000/100 =100

 TOTAL INTEREST = 0.125X1000=125


 INTEREST PER YEAR =100
BALANCE INTEREST=125—100=25.

 PRINCIPAL =1000
INTEREST=25

r=10%

n=?

pnr/100 =25

1000 (n) (10)/100 =25

100n=25

n=25/100=0.25

So answer = 1 year + 0.25 years =1.25 years

=========================Question : The difference between compound interest and simple


interest on a certain sum for 2 years @ 10%per annum is Rs10. Find the sum.

Solution :

 p(1+i)n --- p + {pnr/100} =10


2
 p(1+0.10) --- p + {p(2)10/100} =10
 p(1.10)2 --- p + {0.2p} =10
2
 p(1.10) --- p + {0.2p} =10
 p(1.21) --- 1.2p =10
 0.01 p =10
 p=10/0.01 =1000.
================================================================================
==

Question :
A SINKING FUND IS CREATED FOR REDEEMING DEBENTURES WORTH Rs 5 LAKHS AT THE END
OF 25 YEARS. HOW MUCH PROVISION NEEDS TO BE MADE OUT OF PROFITS EACH YEAR
PROVIDED SINKING FUND INVESTMENTS CAN EARN INTEREST AT 4% PER ANNUM ?

SOLUTION :

A =[P/i] [(1+i)n --1 ]

5,00,000 =[P/0.04] [(1+0.04)25 --1 ]

5,00,000 =[P/0.04] [(1.04)25 --1 ]

5,00,000=[P/0.04][(2.66583633134)--1 ]

5,00,000=[P/0.04](1.66583633134)

5,00,000 X 0.04 / 1.66583633134 = P

12,005.983 = P

============================================

QUESTION :

IF THE DIFFERENCE BETWEEN SIMPLE INTEREST AND COMPOUND INTEREST IS Rs 11 AT THE


RATE OF 10% PER ANNUM FOR 2 YEARS, FIND THE SUM.

SOLUTION :

 p(1+i)n --- p + {pnr/100} =11


 p(1+0.10)2 --- p + {p(2)10/100} =11
 p(1.10)2 --- p + {0.2p} =11
2
 p(1.10) --- p + {0.2p} =11
 p(1.21) --- 1.2p =11
 0.01 p =11
 p=11/0.01 =1100.
=============================================

QUESTION :

In how many years a sum will become double at 5% per annum compounded interest?

Solution :

We know that

A = P(1+i)n

It is given in the problem that A=2P


2P = P(1+0.05)n

2P = P(1.05)n

2 = (1.05)n

n lies in between 14 and 15

Put n=14

1.97993159938 = (1.05)14

1.97993159938 = (1.05)14

Difference = 2--1.97993159938 = 0.02006840062

NEXT STEP

TAKE THE DIFFERENCE AS SIMPLE INTEREST

p= 1.97993159938

r=5%

n=?

pnr/100 = 0.02006840062

(1.97993159938) (n) (5)/100 = 0.02006840062

n= 0.20339551253 YEARS

n= 0.20339551253 YEARS X 12 MONTHS =2.44 MONTHS

14.244 MONTHS

14.3 YEARS.

----------------------------------------------------------------------

QUESTION :

THE TIME BY WHICH A SUM OF MONEY IS 8 TIMES OF ITSELF IF IT DOUBLES ITSELF IN 15 YEARS
IF COMPOUNDED INTERST IS APPLICABLE

SOLUTION :

A= P(1+i)n
HERE A= 2P

n=15

A= P(1+i)n

2P = P(1+i)15

2= (1+i)15

Log 2 = log (1+i)15

0.301029995 = 15 log (1+i)15

0.301029995/15 = log (1+i)

0.301029995/15 = log (1+i)

0.020068666333 = log (1+i)

Taking anti log on both sides

1.047294123 = 1+i

i=0.047294123

rate of interest =4.7294123%

A= P(1+i)n
HERE A= 8P

n=?

A= P(1+i)n

8P = P(1+0.047294123)n

8= (1.0472941235)n

45= n.

----------------------------------------------------------------
22-4-2018

Question :

If an amount is kept a simple interest it earns an interest of Rs600 in first 2 years but when
kept at compound interest it earns an interest of Rs660 for the same period. Then the rate of
interest and the principal amount are ?

Solution

p(2)(i)/100 =600

p(2)(i) =60000

i= 60000 /2p

i= 30000/p

p=30000/i

p(1+[i/100])n—p =660

p(1+[i/100])2—p =660

30000/i (1+[i/100])2—30000/i =660

30000/i { (1+[i/100])2—1} =660

30000/i { (1+[0.01i])2—1} =660

30000/i{(1+0.01i+1) (1+0.01i—1)}=660

30000/i{(2+0.01i) (0.01i}=660

30000{(2+0.01i) (0.01}=660

{(2+0.01i) }=660/ { 30000 x 0.01}

2+0.01i= 660/300

2+0.01i=2.2

0.01i=0.2

i=20
so the rate of interest is 20% per annum.

Pni/100 =600

P(2)(20)/100=600

40p/100=600

40p=60000

p=1500

=======================================

Question :

THE SIMPLE INTEREST FOR A CERTAIN SUM FOR 2 YEARS AT 10% PER ANNUM IS Rs90. THE
CORRESPONDING COMPOUND INTEREST IS WHAT ?

SOLUTION :

pnr/100= 90

p(2)(10)/100 =90

0.2p=90

p= 90/0.2 =450

p(1+i)n—p

450(1+0.10)2—450

450(1.10)2—450

450x1.21---450

544.50—450 =94.50

===========================================
Question :

In how many years will a sum of money become 4 times at 12% per annum simple interest?

Solution :

p + pnr/100 =A

p + pnr/100 =4p

pnr/100 = 4p—p

pnr/100 = 3p

pn(12)/100 =3p

12n/100 =3

n= 3 x100 /12

n=25 years.

----------------------------------------------------------------------

Question :

Suppose you parent decides to open a PPF account in a bank towards your name with Rs 10,000
every year starting from today for next 15 years when you receive and get 8.5% per annum
interest rate compounded annually, what is present value of this annuity ?

Solution :

p=10,000

i=8.5%= 8.5/100=0.085

n=15

Present value of an annuity =

[P/i] [(1+i)n --1 ] x {1/ (1+i)}n

[10,000/0.085] [(1+0.085)15 --1 ] x {1/ (1+0.085)}15

[117647.058823] [(1.085)15 --1 ] x {1/ (1.085)}15

[117647.058823] [(3.39974287872) --1 ] x {1/ 3.39974287872


[117647.058823] [(2.39974287872) - x {0.29413989106=83,042.361

answer : 83042 .361 + today’s investment =83042.361+10,000


=93042.361

----------------------------------------------------

Question:

A sum of Rs44,000 is divided into three parts such that the corresponding interest earned after 2
years , 3 years and 6 years may be equal . If the rates of simple interest are 6% per annum, 8% per
annum and 6% per annum , then the smallest part of the sum will be ?

Solution :

X x 2 x 0.06 = y x 3 x 0.08 =z x 6 x 0.06

0.12 x = 0.24 y = 0.36 z

X+y+z= 44000
since 0.12 x=0.240 y

y= 0.12x/0.240 =0.5x

y= 0.5x-----------------[1]

since 0.12 x=0.36Z

Z= 0.12x/0.360 =0.33333333x

Z= 0.333333333x-----------------[2]

X+0.5X+0.3333333333X =44000

1.833333333333X= 44000

X=44000/1.83333333333=24000

Y=0.5X = 24000=12000

Z=0.33333333X=8000
---------------------------------------------------------------------------------

Question:
A certain sum of money was invested at simple rate of interest for 3 years. If the same has been invested at a rate that was 7% higher , the interest amount would have

been Rs 882 more. The amount of sum invested is ?

Solution :

882/3 = 294

294

------- x100 =4200

Question:
A certain sum of money invested at compound interest doubles itself in 4 years. It becomes 32 times of itself at the same rate of compound interest in how many years ?

Solution :

A= P(1+i)n

A= 2P

2P= P(1+i)4

2= (1+i)4

1.41421356237=(1+i)2

1.189207115=(1+i)

0.189207115 =i

A= P(1+i)n

A= 32P

32P= P(1+0.189207115)n

32= (1. 189207115)n


By trial and error method n=20 years.

-------------------------------------------------------------------------------

Question :

The future value of an annuity of RS1,000 made annually or 5 years at the interest of 14% compounded annually is ?

Solution :

A=
[P/i] [(1+i)n --1 ]
A=
[1,000/0.14] [(1+0.14)5 --1 ]
A= 6,610.

Question :

How much amount is required to be invested every year as to accumulate Rs6,00,000 at the end of 10 years if the interest is compounded annually at 10% rate of

interest ?

Solution:

A=
[P/i] [(1+i)n --1 ]
6,00,000=
[P/0.10] [(1+0.10)10 --1 ]
p=37,647.

Question :

If a sum triples in 15 years at simple rate of interest, the rate of interest per annum will be ?

Solution:

200/15=13.33%
Question :

The partners A and B together lent Rs3,903 at 4% per annum ,interest compounded annually. After a span of 7 years A gets the same amount as B gets after 9 years . The

share of A in the sum of Rs3,903 is ?

Solution:

A+B=3,903

A=3903—B

B=3903—A
P(1+i)n =(3903—B)(1+0.04)7
=
(3903—B)(1.31593177922)

=5136.08173429--1.31593177922B

P(1+i)n =(3903—A)(1+0.04)9

=(3903—A)(1.42331181239)

=5555.18600375--1.42331181239A

5136.08173429--1.31593177922B =5555.18600375--1.42331181239A

419.10426946 --1.42331181239A + 1.31593177922 B=0

A=3903—B

419.10426946 --1.42331181239(3903—B) + 1.31593177922 B=0

419.10426946 –5555.18600375—1.42331181239B + 1.31593177922 B=0

419.10426946 –5555.18600375—1.42331181239B + 1.31593177922 B=0

B=1874.995

A=2028.00411585.
Question :

What principal will amount to Rs370 in 6 years at 8% per annum at simple interest ?

solution :

A= p+pnr/100

370=p[1+nr/100]
370=p[1+6x8/100]

370=p[ 1+0.48]

370=1.48p

370/1.48=p

250=p

Question :

In what time will a sum of money doubles itself at 6.25% per annum simple interest ?

Solution :

100/6.25 = 16 years.

Question :

On what sum, difference between compound interest and simple interest for 2 years at 7% per
annum interest is Rs29.40

Solution :

 A=p(1+i)n
A=p(1+0.07)2

A=p(1.07)2

A=p(1.1449)

A=1.1449p

 A=p+pnr/100
 A=p+pnr/100
 A=p+p(2)(7)/100
 A=p+14p/100
 A=p[1+0.14]
 A=1.14p

1.1449p---1.14p=29.4

0.0049p=29.4

P=29.4/0.0049=6,000.
Question :

A sum of money compounded annually becomes Rs1,140 in 2 years and Rs1,710 in 3 years. Find
the rate of interest per annum.

Solution :

p(1+i/100)2 =1140

p(1+i/100)3 =1710

1+i/100=1.5

i/100=0.5

i=0.5x100=50%

Question :

The difference between compound and simple interest on a certain sum of money for 2 years at
4% per annum is Re1. The sum is ?

Solution:

 A=p(1+i)n
A=p(1+0.04)2

A=p(1.04)2

A=p(1.0816)

A=1.0816p

 A=p+pnr/100
 A=p+pnr/100
 A=p+p(2)(4)/100
 A=p+8p/100
 A=p[1+0.08]
 A=1.08p
1.0816p---1.08p=1

0.0016p=1

P=1/0.0016=625.

Question :

Mr X invested Rs90,500 in post office at 7.5% per annum simple interest . While calculating ,the
rate of interest was wrongly taken as 5.7% per annum. The difference in amounts at maturity is
Rs9,774. Find the period for which the sum was invested.

Solution :

90500 + 90500 (n) (0.075) –

90500 + 90500(n)(0.057) = 9774

6787.5 (n) – 5158.5(n) = 9774

6787.5 (n) – 5158.5(n) = 9774

1629n=9774

n=6.

Question :
How much investment is required to yield an annual income of Rs420 at 7% simple interest ?

Solution :

Rs420

----------- = Rs6,000

0.07

Question :

Simple interest on Rs2,000 for 5 months at 16% per annum is

Solution :

2,000 [5/12] [0.16] = Rs133.33

ILLUSTRATIONS FROM SEQUENCES AND SERIES

WHAT IS AN ARITHMETIC PROGRESSION?


ANSWER :

ARITHMETIC PROGRESSION IS A SERIES WHICH HAS THE FOLLOWING FEATURES

 The first term is called “a”


 The second term = a + d
 The third term is a+2d
 The fourth term is a+3d
 “n” denotes the number of terms in the AP.
 The last term of the Arithmetic Progression is a+(n—1)d or “l”
 The common difference is “d” which is the difference between two successive terms.

ILLUSTRATION 1

CONSIDER THE FOLLOWING ARITHMETIC PROGRESSION

4,9,14,19,24, ….49

Q.A . FIND THE COMMON DIFFERENCE.

ANSWER TO QUESTION A :

IN AN ARITHMETIC PROGRESSION, THE DIFFEERENCE BTWEEN TWO SUCCESSIVE TERMS IS


KNOWN AS COMMON DIFFERENCE “d”

IN THE ABOVE PROBLEM

d=9—4 =5

Q. B FIND THE SUM UPTO FIRST 4 TERMS OF THIS ARITHMETIC PROGRESSION USING THE
APPROPRIATE FORMULA

ANSWER TO QUESTION B :

THE FORMULA FOR SUM UPTO “n” TERMS OF AN ARITHMETIC PROGRESSION IS

Sn= n/2 { 2a+ [n—1]d}

S4= 4/2 {2(4) + [4—1]5}

S4= 2 {8 + [3]5}
S4= 2 {8 + 15}

S4= 2 {23}

S4= 46

PROOF:

SUM UPTO FIRST 4 TERMS OF THIS ARITHMETIC PROGRESSION=

4+9+14+19=46

Q. C FIND THE SUM UPTO FIRST 4 TERMS OF THIS ARITHMETIC PROGRESSION USING THE
APPROPRIATE FORMULA IF THE FOURTH TERM OF THIS ARITHMETIC PROGRESSION=19

ANSWER TO QUESTION C

Sn=n/2 [ a+l]

S4=4/2[4+19]

S4=2[23]

S4=46

PROOF :

SUM UPTO FIRST 4 TERMS OF THIS ARITHMETIC PROGRESSION=

4+9+14+19=46

Q.D HOW MANY TERMS ARE THERE IN THE ABOVE ARITHMETIC PROGRESSION ?

ANSWER TO QUESTION D

We know that the last term of an arithmetic progression =

l=a+[n—1]d

here l=49

a=4

d=5

let us substitute the relevant values in the above formula to derive the value of “n”

49=4+[n—1]5
49=4+[5n—5]

49=4+5n—5

49=5n—1

49+1=5n

50=5n

50/5=n

10=n

Therefore, there are 10 terms in this arithmetic progression.

PROOF :

4, 9, 14, 19, 24, 29, 34, 39, 44, 49.

===========================================

ILLUSTRATION 2:

FIND THE SUM OF THE SERIES

2+7+12+……297

SOLUTION:

HERE a=2

d=7—2=5

IN ANOTHER WAY ALSO

d=12—7=5

WE HAVE TO FIND AS HOW MANIETH TERM IS 297

WE KNOW THAT

l =a+[n—1]d

297=2+[n—1]5

297=2+5n—5
297=5n—3

300=5n

300/5=n

60=n

WE KNOW THAT

Sn = n/2 [ a+ l]

S60 =60/2 [2+297]

S60 =30 [299]

Sn =8970

Exercise 2:

FIND THE SUM OF THE SERIES

6+10+14+……98

SOLUTION:

HERE a=6

d=10—6=4

IN ANOTHER WAY ALSO

d=14—10=4

WE HAVE TO FIND AS HOW MANIETH TERM IS 98

WE KNOW THAT

l =a+[n—1]d
98=6+[n—1]4

98=6+4n—4

98=4n+2

96=4n

96/4=n

24=n

WE KNOW THAT

Sn = n/2 [ a+ l]

S24 =24/2 [6+98]

S24 =12 [104]

S24 =1248

ILLUSTRATION 3:

[X+1], 3X , [4X+2] ARE IN AP. FIND X

SOLUTION:

WE KNOW THAT IN ARITHMETIC PROGRESSION THE COMMON DIFFERENCE “d” IS THE


DIFFERENCE BETWEEN TWO SUCCESSIVE TERMS..

HERE,

d= 3X—[X+1]

ALSO

d= [4X+2]—3X

SO LET US EQUATE

3X—[X+1] = [4X+2]—3X

3X—X--1 = 4X+2—3X
2X--1 = X+2

2X--X = 2+1

X=3

ILLUSTRATION 4:

THE SUM OF THE THIRD AND NINTH TERM OF AN AP IS 8.FIND THE SUM OF THE FIRST 11
TERMS OF THE PROGRESSION.

SOLUTION:

WE KNOW THAT

n th TERM OF AN AP = a + [n—1]d

3rd TERM OF AN AP = a + [3—1]d= a + 2d

9th TERM OF AN AP = a + [9—1]d= a + 8d

LET US EQUATE [ a + 2d] + [a + 8d] = 8

[ a + 2d] + [a + 8d] = 8

a + 2d + a + 8d = 8

2a + 10d = 8

10d = 8—2a

d = [8—2a]/10

d = 8/10--2a/10

d = 0.80—0.2a

11TH TERM= a+[11—1][0.8—0.2a]

11TH TERM= a+[10][0.8—0.2a]

11TH TERM= a+8—2a

11TH TERM= 8—a


S11= n/2 [ a+l]

S11= 11/2 [ a+8--a]

S11= 11/2 [ 8]

S11= 44

EXERCISE 4:

THE SUM OF THE FOURTH AND SEVENTH TERM OF AN AP IS 42.FIND THE SUM OF THE FIRST
10 TERMS OF THE PROGRESSION.

SOLUTION:

WE KNOW THAT

n th TERM OF AN AP = a + [n—1]d

4TH TERM OF AN AP = a + [4—1]d= a + 3d

7th TERM OF AN AP = a + [7—1]d= a + 6d

LET US EQUATE [ a + 3d] + [a + 6d] =42

[ a + 3d] + [a + 6d] = 42

a + 3d + a + 6d = 42

2a + 9d = 42

9d =42—2a

9d = 2[21—a]

d = 2/9[21--a]

d = 2/9[21—a]

10TH TERM= a+[10—1] [2/9][21—a]

10TH TERM= a+[9][ 2/9][21—a]

10TH TERM= a+2[21—a]

10TH TERM=a+ 42—2a

10TH TERM=42--a
S10= 10/2 [ a+l]

S10= 5 [ a+(42--a)]

S10= 5 [ a+42--a]

S10= 5 [42]

S10= 210

EXERCISE 5:

THE SUM OF THE THIRD AND SEVENTH TERM OF AN AP IS --4. FIND THE SUM OF THE FIRST 12
TERMS OF THE PROGRESSION.

SOLUTION:

WE KNOW THAT

n th TERM OF AN AP = a + [n—1]d

3rd TERM OF AN AP = a + [3—1]d= a + 2d

7th TERM OF AN AP = a + [7—1]d= a + 6d

LET US EQUATE [ a + 2d] + [a + 6d] = --4

[ a + 2d] + [a + 6d] = --4

a + 2d + a + 6d = --4

2a + 8d = --4

8d = --4—2a

d = [--4—2a]/8

d = --4/8--2a/8

d = --0.5—0.25a

12TH TERM= a+[12—1][--0.5—0.25a]

12TH TERM= a+[11][--0.5—0.25a]

12TH TERM= a—5.5—2.75a

12TH TERM= --1.75a –5.5

S12= 12/2 [ a+l]


S12= 12/2 [ a—1.75a—5.5]

S12= 12/2 [ --0.75a—5.5]

S12= 6[--0.75a—5.5]

S12= --4.5a—33--------------eqn 1

S12= 2a + 11[--0.5—0.25a]

=2a –5.5—2.75a

=--0.75a –5.5-----------eqn 2

ILLUSTRATION 5:

IF EACH MONTH Rs100 INCREASES TO A SUM.FIND THE TOTAL SUM AFTER 10 MONTHS IF THE
AMOUNT OF THE FIRST MONTH IS Rs2,000

SOLUTION:

a=2,000

d= 100

WE KNOW THAT

Sn = n/2 [ 2a+(n—1)d]

S10 = 10/2 [ 2(2000)+(10—1)100]

S10 = 5 [ 4000+(9)100]

S10 = 5 [ 4000+900]

S10 = 5 [ 4900]

S10 = 24,500

CA EXAMINATION QUESTION ON ARITHMETIC PROGRESSION:

IF 8TH TERM OF AN ARITHMETIC PROGRESSION IS 15,FIND THE SUM OF ITS FIRST 15 TERMS.

SOLUTION :

WE KNOW THAT THE n th TERM OF AN ARITHMETIC PROGRESSION IS a + (n—1)d

SO
8TH TERM OF AN ARITHMETIC PROGRESSION IS = a + [8—1]d

8TH TERM OF AN ARITHMETIC PROGRESSION IS = a + 7d =15

WE FURTHER KNOW THAT THE SUM UPTO n TERMS OF AN ARITHMETIC PROGRESSION IS

Sn= n/2 [2a +(n—1)d]

S15=15/2 [2a +(15—1)d]

S15=15/2 [2a +14d]

S15=15/2 [2] [a +7d]

S15=15 [a +7d]

S15=15 [15]

S15=225

CA EXAMINATION QUESTION ON ARITHMETIC PROGRESSION:

THE FOURTH TERM OF AN ARITHMETIC PROGRESSION IS THREE TIMES THE FIRST TERM OF THE
ARITHMETIC PROGRESSION AND THE 7 TH TERM OF THE SAME ARITHMETIC PROGRESSION
EXCEEDS TWICE OF THE THIRD TERM OF THE SAME ARITHMETIC PROGRESSION BY 1. FIND a
AND d .

SOLUTION :

WE KNOW THAT THE n th TERM OF AN ARITHMETIC PROGRESSION IS a + (n—1)d

SO

4TH TERM OF AN ARITHMETIC PROGRESSION IS = a + [4—1]d

4TH TERM OF AN ARITHMETIC PROGRESSION IS = a + 3d

WE FURTHER KNOW THAT THE FIRST TERM OF AN ARITHMETIC PROGRESSION =a

IT IS MENTIONED IN THE PROBLEM THAT THE FOURTH TERM OF AN ARITHMETIC PROGRESSION


IS THREE TIMES THE FIRST TERM OF THE ARITHMETIC PROGRESSION

SO
a+3d=3a.

3d=2a

a=3d/2-----------------------equation 1

7TH TERM OF AN ARITHMETIC PROGRESSION IS = a + [7—1]d

7TH TERM OF AN ARITHMETIC PROGRESSION IS = a + 6d

3rd TERM OF AN ARITHMETIC PROGRESSION IS = a + [3—1]d

3RD TERM OF AN ARITHMETIC PROGRESSION IS = a + 2d

IT IS MENTIONED IN THE PROBLEM THAT

THE 7TH TERM OF THE SAME ARITHMETIC PROGRESSION EXCEEDS TWICE OF THE THIRD TERM
OF THE SAME ARITHMETIC PROGRESSION BY 1.

a+6d—2[a+2d]=1

a+6d—2a—4d=1

--a+2d=1----------------------equation 2

Let us substitute a= 3d/2 in the equation 2

--[3d/2]+2d=1

--3d+4d

---------- =1

--- =1

d=2

a= 3d/2 as per equation 1

a=3(2)/2

a=3
so

a=3

d=2

CA EXAMINATION QUESTION ON ARITHMETIC PROGRESSION:

THE FOURTH TERM OF AN ARITHMETIC PROGRESSION IS THREE TIMES THE FIRST TERM OF THE
ARITHMETIC PROGRESSION AND THE 7 TH TERM OF THE SAME ARITHMETIC PROGRESSION
EXCEEDS TWICE OF THE THIRD TERM OF THE SAME ARITHMETIC PROGRESSION BY 1. FIND a
AND d .

SOLUTION :

WE KNOW THAT THE n th TERM OF AN ARITHMETIC PROGRESSION IS a + (n—1)d

SO

4TH TERM OF AN ARITHMETIC PROGRESSION IS = a + [4—1]d

4TH TERM OF AN ARITHMETIC PROGRESSION IS = a + 3d

WE FURTHER KNOW THAT THE FIRST TERM OF AN ARITHMETIC PROGRESSION =a

IT IS MENTIONED IN THE PROBLEM THAT THE FOURTH TERM OF AN ARITHMETIC PROGRESSION


IS THREE TIMES THE FIRST TERM OF THE ARITHMETIC PROGRESSION

SO

a+3d=3a.

3d=2a

a=3d/2-----------------------equation 1

7TH TERM OF AN ARITHMETIC PROGRESSION IS = a + [7—1]d

7TH TERM OF AN ARITHMETIC PROGRESSION IS = a + 6d

3rd TERM OF AN ARITHMETIC PROGRESSION IS = a + [3—1]d

3RD TERM OF AN ARITHMETIC PROGRESSION IS = a + 2d

IT IS MENTIONED IN THE PROBLEM THAT


THE 7TH TERM OF THE SAME ARITHMETIC PROGRESSION EXCEEDS TWICE OF THE THIRD TERM
OF THE SAME ARITHMETIC PROGRESSION BY 1.

a+6d—2[a+2d]=1

a+6d—2a—4d=1

--a+2d=1----------------------equation 2

Let us substitute a= 3d/2 in the equation 2

--[3d/2]+2d=1

--3d+4d

---------- =1

--- =1

d=2

a= 3d/2 as per equation 1

a=3(2)/2

a=3

so

a=3

d=2
WHAT IS A GEOMETRIC PROGRESSION ?

ANSWER : A GEOMETRIC PROGRESSION IS A SERIES WHICH HAS THE FOLLOWING FEATURES…

n th term of a geometric progression= ar (n—1)


where a= first term of the geometric progression
r= common ratio of the geometric progression which is obtained by dividing
a term of the GP by its preceding term
n= number of terms in the geometric progression

ILLUSTRATION 6
consider the following geometric progression
5,15,45,135……3645.
USING FORMULA FIND THE FIFTH TERM OF THE ABOVE GEOMETRIC PROGRESSION

SOLUTION:
a=5

r= 15/5 OR 45/15 OR 135/45 = 3

t5 = ar (n—1)

t5 = [5][3] (5—1)

t5 = [5][3] (4)

t5 = [5][81]

t5 = [5][81]

t5 = 405

PROOF : WE KNOW THAT 4TH TERM OF THIS GP=135

5TH TERM OF THIS GP= 4TH TERM X r

5TH TERM OF THIS GP= 135X3

5TH TERM OF THIS GP= 405

5TH TERM OF THIS GP= 4TH TERM X3

CA EXAMINATION QUESTION ON GEOMETRIC PROGRESSION:


IF 5TH TERM OF A GEOMETRIC PROGRESSION IS 31/3 , FIND THE PRODUCT OF THE FIRST 9 TERMS
OF THIS GEOMETRIC PROGRESSION.

SOLUTION :

WE KNOW THAT THE FORMULA FOR n th TERM OF A GEOMETRIC PROGRESSION IS

a [rn—1]

IT IS MENTIONED IN THE PROBLEM THAT THE 5 TH TERM OF A GEOMETRIC PROGRESSION IS


cube root of 3 = 31/3

SO

t5= a [r5—1]

t5= a [r4]

31/3 = a[r4]

31/3 = ar4

PRODUCT OF THE FIRST 9 TERMS OF A GEOMETRIC PROGRESSION IS

[a] [ar][ar2] [ar3] [ar4][ar5] [ar6] [ar7] [ar8]=a9r36

a9r36 can be expressed as [ar4]9

let us substitute 31/3 = [ar4] in [ar4]9

so

[ar4]9 ={31/3}9

=33

y=27

CA EXAMINATION QUESTION ON GEOMETRIC PROGRESSION:

FIND THE NUMBERS WHOSE ARITHMETIC MEAN IS 12.5 AND GEOMETRIC MEAN IS 10.

SOLUTION

LET THE TWO NUMBERS BE X AND Y

IT IS MENTIONED IN THE PROBLEM THAT THE ARITHMETIC MEAN OF THE TWO NUMBERS =12.5

SO
X+Y

------ =12.5

X+Y=25-------------EQUATION 1

IT IS FURTHER MENTIONED IN THE PROBLEM THAT THE GEOMETRIC MEAN OF THE TWO
NUMBERS =10

SO

√ XY =10

XY=100-------------EQUATION 2

X= 100/Y

LET US SUBSTITUTE X=100/Y IN THE EQUATION 1

X+Y=25

100/Y + Y =25

100 + Y2 =25

-----------

100 + Y2 =25Y

Y2—25Y+100 =0

−b ± √b −4 ac
2
Y=
2a

25 ± √ 625−400
Y=
2a

25 ± √ 225
Y=
2(1)
25 ±15
Y=
2(1)

Y= 20

OR

Y=5

IF Y=20 X=5
IF Y=5 X=20

SO THE NUMBERS ARE 5 AND 20

CA EXAMINATION QUESTION ON GEOMETRIC PROGRESSION:

INSERT TWO ARITHMETIC MEANS BETWEEN 68 AND 260

SOLUTION

68 , 68+d, 68+2d ,260

68+d—68 = 260—(68+2d)

d= 260—68—2d

d=192—2d

3d=192

d=64

So the two arithmetic means between 68 and 260 are

68+64, 68+(2)(64)

132 , 196.

CA EXAMINATION QUESTION ON GEOMETRIC PROGRESSION:

IF THE SUM OF 3 ARITHMETIC MEANS BETWEEN “a” AND 22 IS 42, FIND “a”.
SOLUTION:

THE THREE ARITHMETIC MEANS BETWEEN “a” AND 42 ARE a+d, a+2d and a+3d.

IT IS MENTIONED IN THE PROBLEM THAT THE SUM OF 3 ARITHMETIC MEANS BETWEEN “a”
AND 22 IS 42.

SO

[a+d] + [a+2d] + [ a+3d]=42

3a+6d=42-----------equation 1

5TH TERM OF THIS ARITHMETIC PROGRESSION =22

WE KNOW THAT THE FORMULA FOR nth TERM OF AN ARITHMETIC PROGRESSION =a+(n—1)d

5th TERM OF AN ARITHMETIC PROGRESSION =a+(5—1)d=a+4d

a+4d=22-----------------equation 2

let us solve equations 1 and 2

3a+6d=42

3a+12d=66

--6d =--24

d=4

put d=4 in equation 2

a+4d=22

a+4(4)=22

a+16=22

a=6

CA EXAMINATION QUESTION ON GEOMETRIC PROGRESSION

IF EACH MONTH Rs100 INCREASES IN A SUM, FIND OUT THE TOTAL AMOUNT AFTER 10
MONTHS IF THE FIRST MONTH’S SUM IS Rs2,000.

SOLUTION.

a=2,000
d=100

n=10

we know that

Sn = n/2 { 2a+(n—1)d}

Sn = 10/2 { 2(2000)+(10—1)100}

Sn = 5 { 4,000+(9)100}

Sn = 5 { 4,000+900}

Sn = 5 { 4,900}

Sn = 24,500

----------------------------------------------------------------------

29.10.2015

Common features of an ARITHMETIC PROGRESSION…

a , a+d, a+d+d, a+d+d+d, a+d+d+d+d,…. L

a , a+d, a+2d, a+3d, a+4d,…. L

 The first term of an arithmetic progression is normally denoted as “a”


 The last term of an arithmetic progression is normally denoted as “L ”
 The common difference of an arithmetic progression is described as “d”

If we deduct a term of an AP from its next term, we will get “d”

 Formula for sum total up to n terms of an ARITHMETIC PROGRESSION is

n/2 x { 2 a+ [n—1] d }

Example [1] on Arithmetic Progression


Consider the AP 4,8,12,16,….
Find the sum up to first 2000 terms of this AP.

Solution :
Sn= n/2 x { 2 a+ [n—1] d }
S2000= 2000/2 x { 2 (4)+ [2000—1] 4 }
S2000= 1000 x { 8+ [1999] 4 }
S2000= 1000 x { 8+ 7996 }
S2000= 1000 x { 8004 }
S2000= 80,04,000
Example [2] on Arithmetic Progression
Consider the AP --16,--12,--8,--4….
Find the sum up to first 2000 terms of this AP.

Solution :
Sn= n/2 x { 2 a+ [n—1] d }
S2000= 2000/2 x { 2 (--16)+ [2000—1] 4 }
S2000= 1000 x { --32+ [1999] 4 }
S2000= 1000 x { --32+ 7996 }
S2000= 1000 x { 7964 }
S2000= 79,64,000

Example [3] on Arithmetic Progression


Consider the AP the first term of which is 12.
The sum up to first 50 terms of this AP is 12850.
Find the sum upto first 58 terms of this AP.
Solution :
Sn= n/2 x { 2 a+ [n—1] d }
S50= 50/2 x { 2 (12)+ [50—1] d }
12850= 25 x {24+ 49d }
12850/25= {24+ 49d }
514= {24+ 49d }
514--24 =49d
490=49d
d=10.

Sum up to first 58 terms of this AP


S58 =58/2 x { 2 (12)+ [58—1] 10 }
S58 =29 x { 24+ [58—1] 10 }
S58 =29 x { 24+ [57] 10 }
S58 =29 x { 24+ 570 }
S58 =29 x { 594 }
S58 =17226.
---------------------------------------------
Example [4]
The difference between the sum of the first 120 terms and the sum of the first 40
terms of an AP is19,400.

The difference between the sum of the 30 terms and the sum of the first 8 terms of
the same AP is1,309.

Find the sum up to first 84 terms of this AP.

Solution :

60 x{ 2a+119d} [---] 20x{2a+39d} =19400


120a+7140d—40a—780d=19400
80a+6360d=19400------equation [1]

15 x{ 2a+29d} [---]4x{2a+7d} =1309


30a+435d—8a—28d=1309
22a+407d=1309------equation [2]
Solving equations [1] and [2]
a=4 and d=3

S84= 84/2 x{ 2(4) + 83(3)}


S84= 42 x{ 8 + 249}
S84= 42 x{ 257}
S84=10794.
===================================
20— 4 —2018
Problems and solutions from Geometric progression :
Question :
Find n such that

an+1 + b n+1
-------------------------------

an + bn

may be the Geometric Mean between a and b.


solution :

 Geometric mean between the two terms a and b = whole square root of (ab)

 whole square root of (ab)


------------------------------------- =
1

an+1 + b n+1
-------------------------------

an + bn

 (ab)1/2
------------------------------------- =
1

an+1 + b n+1
-------------------------------

an + bn
 (a)1/2 (b)1/2
------------------------------------- =
1

an+1 + b n+1
-------------------------------

an + bn
 Let us cross multiply

* (a)1/2 (b)1/2 [ an + bn ] = an+1 + b n+1

* (a)1/2(b)1/2 (an) +(a)1/2(b)1/2 (bn)= an+1 + b n+1

* (a)1/2+n(b)1/2 +(a)1/2(b)1/2+n = an+1 + b n+1

* (a)1/2(b)1/2+n -- b n+1 = an+1--(a)1/2+n(b)1/2

* (a)1/2(b)1/2+n –bn+1/2+1/2= an+1/2+1/2--(a)1/2+n(b)1/2

*(a)1/2(b)1/2+n –(bn+1/2)(b)1/2= an+1/2 (a)1/2--(a)1/2+n(b)1/2

b
* n+1/2
{ a ½ --- b ½} = an+1/2 { a1/2 – b1/2}

* bn+1/2 = an+1/2

* bn+1/2
----------- = 1

an+1/2

 [b/a] n+1/2 =1
 [b/a] n+1/2 =[b/a]0
 n+1/2=0

 so n= --1/2

======================================

Question :

If the first term of a GP exceeds the second term by 2 and the sum to infinity is 50, find the
series.

Solution:

Let the GP be a , ar ….
a—2=ar

a—ar =2

a(1—r) =2----------------(1)

formula for sum to infinity of a GP

is

------

1—r

------ =50

1—r

a 50

----- = ------

1—r 1

1—r = a/50

Put 1—r = a/50 in (1)

a(1—r) =2

a(a/50) =2

a2/50 =2

a2 =100

a=10

put a= 10 in (1)
a(1—r) =2

10(1—r) =2

10—10r =2

8= 10r

r=8/10=4/5

so the GP is

10 , 10(4/5), …

10, 8 …

======================================

Question :

If a1/x = b1/y= c1/z

and a,b,c are in GP,

x,y,z are in ?

solution :

Let a1/x = b1/y= c1/z= k

a1/x = k

a= kx

b1/y= k

b= ky

c1/z= k
c= kz

KX , KY AND
K Z are in GP

[KY]2 = KX KZ

[K2Y] = KX+Z
2Y =X+Z

Y =[X+Z]/2

SO X,Y AND Z ARE IN AP.

QUESTION :

FIND THE SUM TO n TERMS OF THE SERIES

7+77+777+….n .

SOLUTION :

7+77+777+…. .n

Can be written as

7 [ 1+11+111+….n]

7/9 [ 9+99+999+…..n]

7/9[(10—1) + (100—1) + (1000—1)+….n]

7/9 [10 +100+1000+…n] – 7/9[1+1+1+…n]

7/9 [101 +102+103+…n] – 7/9[1+1+1+…n]

7/9 [10 (10n –1) 7/9[1+1+1+…n]

--------------- --

10—1

7/9 [10 (10n –1) {7/9}[n]

--------------- --
10—1

7/9 { [10 (10n –1)

--------------- ---n }

7/9 x 10 (10n –1) 7n

------------------------- --- -----

9 9

7/81 x 10 (10n –1) ---7n/9

---------------------------------------------------------

Question :

If the pth term of a GP is x and the qth term of the GP is y. Then find the nth term of this
GP.

Solution :

We know that the formula for nth term of a GP is a (r n—1)

So

p th term of a GP is a (rp—1) = x

So

q th term of a GP is a (rq—1) = y

x a (rp—1)

-- = -------------

Y a (rq—1)

x (rp—1)

-- = -------------
Y (rq—1)

x (rp—1) r1--q

-- =

x (rp—1+1--q)

-- =

x (rp--q)

-- =

(x/y)1/(p—q) = r

Let us substitute r = (x/y)1/(p—q) in

a (rp—1) = x

a {((x/y)1/(p—q)}p—1) = x

a {((x/y)(p--1/(p—q)} = x

a = x {((y/x)(p--1/(p—q)}

let us substitute a = x {((y/x)(p--1/(p—q)} and

(x/y)1/(p—q) = r in nth term of the GP.


nth term of the GP is a rn--1

x {((y/x)(p--1/(p—q)} {(x/y)1/(p—q)}n—1

x (y)(p--1/(p—q) {(x/y)1/(p—q)}n—1

------------------

x(p--1/(p—q)

x(p—q/p—q) (y)(p--1/(p—q) {(x/y)1/(p—q)}n—1

------------------

x(p--1/(p—q)
1/(p—q)

x(p—q) (y)(p—1) {(x/y)1/(p—q)}n—1

------------------

x(p--1)

1/(p—q)

x(p—q) (y)(p—1) {(x/y)n--1/(p—q)}

------------------

x(p--1)

1/(p—q)

x(p—q) (y)(p—1) x(n—1)/(p—q)

------------------ -------------

x(p--1) y(n—1)/(p—q)

1/(p—q)
x(p—q) (y)(p—1) x(n—1)/(p—q)

------------------ -------------

x(p--1) y(n—1)/(p—q)

answer : nth term of the GP is

xn—q 1/p--q

---------

Yn—p

QUESTION
:

A CERTAIN BALL WHEN DROPPED TO THE GROUND REBOUNCE TO THE 4/5 TH OF


THE HEIGHT FROM WHICH IT FALLS. IT IS DROPPED FROM A HEIGHT OF 100
METRES. FIND THE TOTAL DISTANCE OF ITS TRAVEL BEFORE FINALLY COMING
TO REST.
SOLUTION :

a=80

r= 4/5

the GP is a, ar, ar2 , …..

100 + 2{a + ar + ar2 , …..}

Sum of infinity of GP is

-----

1—r

The total distance travelled by the ball is…

100 + 2{80 + 80x4/5 + 80(4/5)2 , …..}

100 + 2 { 80/ 1—4/5}

100+ 2 { 80/1/5}
100+ 2 x 400

100+ 800

900 metres.

======================================

Question :

The sum of the series

0.5 + 0.55 + 0.555 +…..n terms is ?

SOLUTION :

0.5+0.55+0.555+…. .n

Can be written as

5 [ 0.1+0.11+0.111+….n]

5/9 [ 0.9+0.99+0.999+…..n]

5/9[(1—0.1) + (1—0.01) + (1—0.001)+….n]

5/9 [1 +1+1+…n] – 5/9[0.1+0.01+0.001+…n]

5/9 [n] – 5/9[0.1+.01+0.001+…n]

5/9 [n] --5/9[0.1(1—0.1n)

-----------------

1—0.1

Because : when r is less than 1

Sn = a (1—rn)

-----------

1—r

In this problem r= 0.1


0.01/0.1= 0.1

Because : when r is more than 1

Sn = a (rn--1)

-----------

r--1

5/9 [n] --5/9[0.1(1—0.1n)

-----------------

1—0.1

5n/9 --5/9[0.1(1—0.1n)

-----------------

0.9

5n/9 --5/9[0.1x10(1—0.1n)

-----------------

0.9x10

5n/9 --5/9[1(1—0.1n)

-----------------

5n/9 --5/9[1(1—0.1n)

-----------------

9
5n/9 --- 5/81 [(1—0.1n)]

------------------------------------------------------------------

Question : If x=1 +1/3 +1/32 +……infinity

y=1 +1/4 +1/42 +…… infinity

find x *y

solution :

let us consider the series

x=1 +1/3 +1/32 +……infinity

we know that the S upto infinity f a GP is

------

1—r

Here a= 1

r=1/3

sum upto infinity is

------

1—1/3

------

2/3

= 3/2

So x= 3/2

let us consider the series


y=1 +1/4 +1/42 +……infinity

we know that the Sum upto infinity of a GP is

------

1—r

Here a= 1

r=1/4

------

1—1/4

------

3/4

= 4/3

So y= 4/3

X *y = 3/2 x 4/3

=2.

-------------------------------------------------------------------

23—04—2018

Question :

The first , second and seventh term of an AP are in GP and the common difference is 2. The
second term of the AP is ?

Solution :

Tn= a+(n—1)d

T1= a+(1—1)2

T1= a
T2= a+(2—1)2

T2= a+2

T7= a+(7—1)2

T7= a+12

Therefore,

a, a+2 and a+12 are in GP.

(a+2)2 = a(a+12)

a2 + 4a +4=a2+12a

4a+4=12a

4=8a

4/8= a

½=a

The second term of AP is

a+2

½ + 2 = 5/2.

Question :

If a, b, c are in AP and x, y and z are in GP, then the value of x b—c yc—a z a—b =?

Solution

a,b,c are in AP.

a, a+d, a+2d are in AP.

x,y,z are in GP

x,xr , xr2 are in GP

a=a

b= a+d
c=a+2d

x=x

y=xr

z=xr2

xb—c yc—a z a—b

b—c=a+d—a—2d= --d

c—a=a+2d—a =2d

a—b=a—a—d = --d

xb—c yc—a z a—b

x—d (xr)2d (xr2) —d

x—d x2d r2d x —d r —2d

x—d+2d—d r2d—2d

x0 r 0
1x1=1.

----------------------------------------------------------------------------------------------------------------question
:

The sum of how many terms of the sequence

256 , 128 , 64, … is 511 ?

Solution :

A=256

r= ½

Sn= 511

Sn= a (1—rn)

------------

1—r
511 = 256 ( 1—(1/2)n)

------------------

1—1/2

511 = 256 ( 1—(1/2)n)

------------------

1/2

511 = 512 ( 1—(1/2)n)

511 / 512 = 1—(1/2)n

(1/2)n =1—511/512

(1/2)n =(512—511)/512

(1/2)n =1/512

(1/2)n =1/29

(1/2)n =(1/2)9
so n=9

----------------------------------------------------------------------------------------------------------------

Question :

The sum of terms of an infinite GP is 15. The sum of the squares of the term is 45. Find the
common ratio.

Solution:

Formula for sum of terms of an infinite GP

= a

------

1—r

so
15= a

------

1—r

15(1—r) =a

15—15r=a

Sum of the squares of the GP up to infinity =

a2
--------

1—r2

45 = a2

--------

1—r2

45(1—r2) = a2

45—45r2 = a2

a2 +45r2—45 =0

put a=15—15r in the above equation

(15—15r)2 +45r2—45 =0

225—450r+ 225r2 + 45r2 –45 =0

270r2 –450 r +180 =0

3r2 –5 r +2 =0

r=2/3 or r=1

r=1 is not feasible so r= 2/3.

-------------------------------------------------------------------

Question :

If G be the geometric mean between two numbers a and b ,then the value of

1 1
------------- + ------------- = ?

(G2 --- a2 ) (G2 --- b2 )

solution :

G is the geometric mean between a and b

So

G = (ab)1/2

1 1

------------- + ------------- =?

({(ab)1/2}2 --- a2 ) ({(ab)1/2}2 --- b2 )

1 1

------------- + ------------- = ?

(ab) --- a2 ) (ab) --- b2 )

1 1

------------- + ------------- = ?

(ab) --- a2 ) (ab) --- b2 )

1 1

------------- + ------------- = ?

a(b --- a ) b(a --- b )

1 1

------------- + ------------- = ?

a(b --- a ) -b(b --- a )


1 1

------------- -- ------------- = ?

a(b --- a ) b(b --- a )

b--a

-----------

ab(b --- a )

1/ab

{1/(ab)1/2 }2

{1/G}2.

-----------------------------------------------------------------

Question :

Find the product of 243, 2431/6, 2431/36 …to infinity.

Solution :

Let us ignore the base initially and let us consider the power only.

a=1

r=1/6

Sum to infinity of a GP = a

---------

1—r

Sum to infinity of this GP = 1

---------

1—1/6
=

----- = 6/5.

5/6

[35]6/5 = 36=729.

-------------------------------------------------------------------

Question :

Geometric mean of p, p2, p3…..pn =?

Solution :

[p x p2 x p3x…..pn ] 1/n

Ignore p initially and consider power only.

1+2+3+…n

Sum of n terms of an AP

= n(n+1)/2

Now

{ Pn(n+1)/2 }1/n

P(n+1)/2
================================================================question
:

If 5th term of a GP is 3√3, [ i.e cube root of 3] then the product of first 9 terms of the GP is ?

Solution :

Tn= a rn—1

T5= a r5—1

T5= a r4

a r4= 3√3

product of first 9 terms of the GP


a x ar x ar2x ar3 x ar4x ar5x ar6x ar7x ar8

a9 x r36

a9 x r9x4

a9 x {r9}4

a9 x {r4}9

a9 x {(3√3)/a}9

{(3√3)}9

{(31/3}9

33=27.

----------------------------------------------------------

Question :

The first term of a GP where second term is 2 and sum of infinite terms is 8 will be ?

Solution :

Tn=a rn—1

T2=a r2—1

T2=a r

ar=2

sum of the terms of the GP upto infinity =

----

1--r

---- =8

1--r

a= 8—8 r
ar=2

(8—8 r)r = 2

8r—8r2 =2

--8r2+8r –2=0

8r2--8r +2=0

r= 1/2

Tn=a rn—1

T2=a r2—1

2=a r

2=a [1/2]

4=a

---------------------------------------------------------------

Question :

In a GP the 6th term is 729 and the common ratio is 3, then the first term of the GP is?

Solution :

Tn =a rn—1

T6 =a r6—1

T6 =a r5

T6 =a r5=729

a r5=729

a [3]5=729

a [243]=729

a= 729/243 =3
----------------------------------------------------------------------------------------------------------------
question :

If geometric mean of a,b,c,d is 3, then GM of 1/a, 1/b, 1/c, 1/d is what ?

Solution:

Geometric mean of a,b,c,d=

4th root of [ abcd]

4th root of [ abcd]=3

abcd = 34

abcd = 81

Geometric mean of 1/a , 1/b, 1/c, 1/d is

4th root of [1/abcd]

4th root of [1/abcd] = 4th root of [abcd]—1

4th root of [abcd]—1= { [abcd]—1 }1/4

{ [abcd]—1 }1/4= [abcd]—1/4 = [81]—1/4

{34}—1/4
=

3—1 = 1/3
----------------------------------------------------------------------------------------------------------------

Question :

The sum to m terms of the series

1+11+111+……m terms =

Solution :

1+11+111+……m terms

1{1+11+111+….m}

1/9{9+99+999+….m}

1/9{(10—1)+(100—1)+(1000-1)+….m}

1/9{(10+100+1000…m)—(1+1+1….m}
1/9{(10+100+1000…m)} — 1/9(1+1+1….m}

1/9{(10+100+1000…m)} — 1/9(1+1+1….m}

1/9{10(10m—1 ) --- m/9

----------------

10—1

1/9{10(10m—1 ) --- m

---------------- ----------

9 9

1/81{10(10m—1 ) --- m

----------

{10(10m—1 ) --- m

----------------- ----------

81 9

10(10m—1 )—9m
---------------------------------------------------------

81

10* 10m—10 —9m


---------------------------------------------------------

81

10m+1 --9m—10

----------------------

81

======================================
QUESTION

IF X, Y , Z ARE THE TERMS IN GP, THEN THE TERMS X 2 + Y2, XY+ YZ, Y2+Z2
ARE IN ?

SOLUTION :

SINCE X,Y AND Z ARE IN GP

a=X

ar= Y

ar2=Z

Let us assume a=1 and r=2

1=X

1(2)= 2 =Y

1(2)2=4=Z

1,2,4

12 + 22 , 1x2 + 2x4, 2 2 + 42
5, 10 , 20

it is a GP.

===============================================================

question :

A GP CONSIST OF 2n TERMS. IF THE SUM OF THE TERMS OCCUPYING THE ODD


PLACES IS S1 AND THAT OF THE TERMS IN THE EVEN PLACES IS S2, THE
COMMON RATIO OF THE PROGRESSION IS ?

Solution :

a1,a2,a3….2n terms

a1+a3+a5+….n= s1

a+ar2+ar4+…..n=s1

a{1+r2 +r4+ ….n}=s1

1+r2 +r4+ ….n is a GP


Where first term =1

Common ratio = r2

we know that the formula for n terms of a GP

a (rn --1 )
------------------------- where r is greater than 1.

r—1

here first term a=1

here common ratio =r2

a{ 1 (r2 )n –1

-----------------

r2 ---1

a{ 1 (r2 )n –1

-----------------

r2 ---1

a{ 1 (r2n ) –1

----------------- =s1

r2 ---1

a2+a4+a6+….n= s2

ar+ar3+ar5+…..n=s2

ar{1+r2 +r4+ ….n}=s2

1+r2 +r4+ ….n is a GP


Where first term =1

Common ratio = r2

we know that the formula for n terms of a GP

a (rn --1 )
------------------------- where r is greater than 1.

r—1

here first term a=1

here common ratio =r2

ar{ 1 (r2 )n –1

-----------------

r2 ---1

ar{ 1 (r2 )n –1

-----------------

r2 ---1

ar{ 1 (r2n ) –1

----------------- =s2

r2 ---1

s2

----- = common ratio of the entire GP.

S1

ar{ 1 (r2n ) –1

-----------------

r2 ---1
------------------------

a{ 1 (r2n ) –1

-----------------

r2 ---1

=r

-------------------------------------------------------------------
question :

if 1 1 1

----- , ------ , -------- are in AP

b+c c+a a+b

then a2, b2, c2 ARE IN ?

SOLUTION .

1 1 1 1

------ --- ------ = -------- --- --------

c+a b+c a+b c+a

b+c—c—a c+a—a—b

------------------ = ----------------------

(c+a) (b+c) (a+b) (c+a)

b—a c—b

------------------ = ----------------------

(c+a) (b+c) (a+b) (c+a)

b—a c—b

------------------ = ----------------------
(b+c) (a+b)

(a+b) (b—a) = (c—b) (b+c)

ab—a2+ b2—ab =bc+c2—b2—bc

—a2+ b2 =c2—b2

b2+ b2 =a2+ c2

2b2 =a2+ c2

Therefore , a2, b2 and c2 are in AP.

=====================================

QUESTION :

IF Sn=n2p and Sm=m2p is the sum of an AP then Sp =? { m not equal to n}

Solution :

Sn= n/2 x { 2a + (n—1)d}= n2p

n2p = n/2 x { 2a + (n—1)d}

2n2p/n = { 2a + (n—1)d}

2np = { 2a + (n—1)d}

Sm= m/2 x { 2a + (m—1)d}= m2p

m2p = m/2 x { 2a + (m—1)d}

2m2p/m = { 2a + (m—1)d}

2mp = { 2a + (m—1)d}

2np = { 2a + (n—1)d}

2np = { 2a + nd—d}
2mp = { 2a + (m—1)d}

2mp = { 2a + md—d}

2np = { 2a + nd—d}

2mp = { 2a + md—d}

2(np—mp) = d(n—m)

2p(n—m) = d(n—m)

2p=d

d=2p

2np = { 2a + nd—d}

2np = { 2a + n(2p)—2p}

0 = { 2a —2p}

2p=2a

a=p

Sp= p/2 {2a+ (p—1)d}

Sp= p/2 {2p+ (p—1)2p}

Sp= p/2 {2p+ 2p2--2p}

Sp= p/2 { 2p2}

Sp= p3

======================================

Om sriramajayam

RATIO AND PROPORTION

What is a ratio ?
Ratio is a mathematical expression of same variable out of two or more observations.

Example : Ram’s weight is 1.2 times of the weight of Kumar.

If Kumar’s weight is 45 kgs what is Ram’s weight?

Solution:

(Ram’s Weight ) : ( Kumar’s Weight ) :: (1.2) :(1)

(1.2) :(1) :: (X) :(45)

THEOREM:

PRODUCT OF EXTREMS=PRODUCT OF MEANS

HERE EXTREMES ARE 1.2 & 45

HERE MEANS ARE 1 AND X

(1.2 X45) =(1xX)

54=X

THEREFORE WEIGHT OF RAM=54 KGS.

EXERCISE:

(40:X) :: (1/4 : 3/9)

FIND X.

SOLUTION :

40 x [3/9] = X x [1/4]

120 X

----- = ----
9 4

120 X4 X

----- =

480 X

------ =

53.33

Answer =53.33

Example

Find the duplicate ratio of 3:2

Answer :

Duplicate ratio of 3:2

=32 : 22
=
9 :4

Exercise :

Find the duplicate ratio of 5:6

Answer 25:36
Example

Find the triplicate ratio of 3:2

Answer :

Triplicate ratio of 3:2

=33 : 23
=
27 :8

Exercise :

Find the triplicate ratio of 5:6

Answer 125:216

Example :

Find the sub duplicate ratio of 16:36

Answer:

= sq root of 16 : sq root of 36

=4:6

Exercise
Find Sub duplicate ratio of 25 :4

Answer 5:2

Exercise

Find the sub triplicate ratio of 27:125

SOLUTION :

√3 27 : √3 125

Answer :3:5

Example :

Find the compounded ratio of

2/3 , 1/4 and 3/8

Answer: Compounded Ratio of 2/3 ,1/4 and 3/8

=2/3 x ¼ x3/8

=(2x1x3)/(3x4x8)
=6/96

=1/16

=1:16

Exercise :

Find the compounded ratio of

2/3 , 3/4, 5/6 and 7/8.

Answer :105:288

35:96

Example :

Find the inverse to the ratio 6:5


Answer:

The inverse to the ratio 6:5

=5:6

Example :

Find the inverse to the ratio 6/5

Answer:

The inverse to the ratio 6/5

=5/6

Example :

What will happen if a ratio is compounded with its inverse ratio ?

Answer: If a ratio is compounded with its inverse ratio, the result will be

UNITY i.e 1.

For example 5/6 and 6/5 are inverse to each other and if we compound

both these ratios

(compounding means multiplication)we will get unity.

i.e

5/6 x 6/5 =1.

What is INVERTENDO ?
------------------------------

If two ratios are equal, then their reciprocals are also equal. This is

called invertendo.

For Example

8/4 = 20/10

2 =2

4/8 =10/20

0.5 =0.5

What is ALTERNENDO ?

------------------------------

If two ratios are equal, then the ratio of their antecedents is

equal to the ratio of their SUBSEQUENTS.

What is antecedent ?

Consider 5/9 as a ratio under discussion.

5 is antecedent.

What is subsequent ?

Consider 5/9 as a ratio under discussion.

9 is subsequent.

=============lesson of 25—4—2018 is over====================

Example :

5/9 =10/18

5 :9 = 10:18
5 is the antecedent of first ratio

9 is the subsequent of first ratio

10 is the antecedent of second ratio

18 is the subsequent of the second ratio

5 :10 = 9 :18

5/10 =9/18

Example

5: x =20:40

What is x?

5/20 = x/40

0.25 =x/40

0.25x40 =x

10=x

X=10.

========================================================

Exercise 1

Find the duplicate ratio of X : Y

Answer

X 2 : Y2

Exercise 2

Find the duplicate ratio of 9 : 15


Answer

81:225

Exercise 3

Find the duplicate ratio of 7 : 11

Answer

49:121

Exercise 4

Find the sub-duplicate ratio of a:b

Answer

Square root of a : Square root of b

Exercise 5

Find the sub-duplicate ratio of 25:64

Answer

5:8

Exercise 6

Find the sub-duplicate ratio of 100:400

Answer

10:20

Exercise 7

Find the triplicate ratio of a:b

Answer

a3: b3
Exercise 8

Find the triplicate ratio of 3:4

Answer

27:64

Exercise 9

Find the triplicate ratio of 5:6

Answer

125:216

Exercise 10

Find the sub-triplicate ratio of a:b

Answer

Cube root of a : Cube root of b

Exercise 11

Find the sub-triplicate ratio of 64:125

Answer

4:5

Exercise 12

Find the sub-triplicate ratio of 27:216

Answer

3:6

Exercise 13

A sum of money is divided between two persons in the ratio 3:5

If the share of one person is `20 less than that of the other,

find the sum.

SOLUTION:
SHARE OF FIRST PERSON=3X

SHARE OF SECOND PERSON=5X

DIFFERENCE =5X—3X=2X=20

X=10

SO

SHARE OF FIRST PERSON=3X=30

SHARE OF SECOND PERSON=5X=50

SUM=80

EXERCISE:

A SUM IS DIVIDED IN 1:3 RATIO .

ONE PERSON GETS `5 LESS THAN OTHER PERSON. FIND THE SUM.

Answer

Rs10.

Exercise :

Monthly income of Anbu and Babu are in the ratio 5:6 and their

expenses are in on the ratio 4:5.

If each save `200 per month, find their monthly income.

Solution:

Anbu’s income=5x

Babu’s income=6x

Anbu’s savings=Rs200
Babu’s savings=Rs200

Anbu’s expense=5x--200

Babu’s expense=6x—200

(5x—200) : (6x—200) :: 4:5

Product of extremes= (5x—200) X 5=25x—1000

Product of means=(6x—200)x4=24x—800

Product of extremes=product of means

25x—1000=24x—800

X=200

Anbu’s income=5x=5x200=1,000

Babu’s income=6x=6x200=1,200

Exercise:

Vanathi and Malathi earned in 5:4 ratio. They spent in 2:1 ratio.

Vanathi saved `90 and Malathy saved `90.

 Find income and expense of Vanathi .


 Find income and expense of Malathi.

Solution :

INCOME RATIO OF VANATHI AND MALATHI 5x : 4x

SAVINGS OF VANATHI =Rs90 AND SAVINGS OF MALATHY=Rs90

WE KNOW THAT

INCOME—SAVINGS =EXPENSE

SO

5X—90 IS THE EXPENSE RATIO OF VANATHI


4X—90 IS THE EXPENSE RATIO OF MALATHI

5X—90 : 4X—90 :: 2:1

[5X—90]X1 =[ 4X—90] X2

5X—90 =8X—180

90 =3X

X=30

SO INCOME OF VANATHI= 5X=5X30=150

SO INCOME OF MALATHI=4X=4X30=120

SO SAVINGS OF VANATHI=90

SO SAVINGS OF MALATHI=90

SO EXPENSE OF VANATHI=150—90=60

SO EXPENSE OF MALATHI=120—90=30

EXERCISE:

If ` 8520 be divided among Viji, Suba and Rama so that suba gets

twice like Viji and Rama gets thrice like Viji. Find the share of each.

Solution :

V+S+R=8520

S=2V

R=3V

V+2V+3V=8520

6V=8520

V=1420
V=1420=VIJI

S=2V=2840=SUBA

R=3V=4260=RAMA

Solution

Viji :suba:rama =1:2:3

X:2x:3x

X+2x+3x=8520

6x=8520

X=8520/6=1420

X:2x:3x=1420 :2840:4260

Exercise :

Sheela, Neela and Urmila are the daughters of Kala.

Kala has 4,000 gold coins with her. Neela has to get twice like sheela

but half of Urmila.

You can break coins if needed. DIVIDE COINS among Neela, Sheela and

Urmila.

SOLUTION :

N=2S

N=0.50U
S=N/2

N=N

U=2N

N/2 + N + 2N= 4000

[N+2N+4N] /2 =4000

7N/2 = 4000

7N= 8000

N= 8000/7

 S=[8000/7 ] /2 =8000/14 =571.42


 N=8000/7=1142.80
 U=2N= 2 [8000/7] =16000/7 =2285.71

SOLUTION :

S+N+U=4000

N=2 S

N=0.5U

0.5U=2S

U=2S/0.5=4S

S+N+U=4000

S+2S+4S=4000

7S=4000

S=4000/7 =571.42

N= 8000/7=1142.85

U=16000/7=2285.71
Solution

Sheela =x

Neela=2x

Urmila=4x

7x=4000

X=[4000/7]

Sheela =x=4000/7=571.43

Neela=2x=8000/7=1142.86

Urmila=4x=16000/7=2285.71

Exercise:

Divide ` 71,540 among Tarika, Sarika, Boomika and Nika.

Tarika and Sarika share in 2:5 ratio.

Sarika and Boomika share in 4:7 ratio.

Boomika and Nika share in 9:13 ratio.

Give the share amount of each party.

Solution :
Tarika =2X

Sarika = 5X

Boomika= [5X / 4 ] x 7 =35X/4

Nika = [35X/4 ] /9 x 13 =455 X/36

2X + 5X + 35X + 455 X 71540

------- --------- =

4 36

72 X +180 X + 315X +455 X 71540

--------------------------------------- = ----------

36 1

1022 X 71540

--------- = ---------

36 1

1022 X =2575440

X=2575440/1022=2520

 Tarika =2X=2 x 2520=5040


 Sarika = 5X= 5 x 2520=12600
 Boomika= 35X/4 = [35 x 2520] /4 =22050
 Nika = 455 X/36 = [455 x 2520 ] /36=31850

Solution :

Tarika : sarika =2:5


T:S=2:5

Sarika:boomika=4:7

S:B=4:7

Boomika: nika =9:13

B:N=9:13

T:S=2:5

S:B=4:7

B:N=9:13

 T:S=2:5
 IF T=200 S= 500

 S:B=4:7
 IF S=500 B= 500X[7/4]=875

 B:N=9:13
 IF B=875 N= 875 X[13/9]=1263.89

T=200

S=500

B=875

N=1263.89

---------------

2838.89

-----------------
 T’S SHARE IN 71540= [200/2838.89]X71540=5040
 S’S SHARE IN 71540= [500/2838.89]X71540=12,600
 B’S SHARE IN 71540= [875/2838.89]X71540=22050
 N’S SHARE IN 71540= [1263.89/2838.89]X71540=31850
----------------------------------------------------------------------------------
71540
---------------------------------------------------------------------------------------

Assume that tarika’s share= 200 points then sarika’s points=500

If sarika’s points are 500 then boomika’s points=[500/4]x7=875 points

If boomika’s points are 875 then nika’s points are[ 875/9] x13=1264

So overall ratio of tarika :sarika:boomika:nika

=200 :500:875:1264

Total points=200+500+875+1264=2839

Tarika’s share= [200/2839] x71540=Rs5,040

sarika’s share= [500/2839] x71540=Rs12,600

Boomika’s share= [875/2839] x71540=Rs22,050

Nika’s share= [1264/2839] x71540=Rs31,850

Exercise :

A DIAMOND WORTH `1,56,000 IS BROKEN INTO THREE

PARTS IN THE RATIO 2:3:5 IN WEIGHT.

THE OVERALL VALUE OF EACH PIECE IS

PROPORTIONATE TO THE TRIPLICATE RATIO OF WEIGHT.

FIND THE VALUE OF EACH PIECE OF DIAMOND.


8 : 27: 125

156000 X [8/160]=7800

156000X[27/160]=26325

156000X[125/160]=121875

------------------------------------------

156000

------------------------------------------

ANSWER :

Rs 7,800, Rs 26,325, Rs1,21,875

EXERCISE :

The ratio of the prices of two houses was 16 :23.

Two years later, the price of the first house

increased by 10% and the price of the second

house increased by `4,77,000. The ratio of the

prices is 11:20 after the price increase. Find the original old price

of both houses.

Solution:

Let the original price of two houses be (16x : 23x)

After two years, the price of first house=16x X 110/100= 17.6 x


At this time , the price of second house=23x + 4,77,000

Now the ratio of NEW price Of two houses=

(17.6 x ) :(23x+4,77,000)=(11:20)

We know that the product of means=product of extremes.

(17.6x)(20)= (23x+4,77,000)(11)

(352x)= 253x+52,47,000

352x—253x= 52,47,000

99x= 52,47,000

X= 53,000

Original price of first house=16x=16x53,000=`8,48,000

Original price of second house =23x=23x53,000=`12,19,000

Exercise:

The monthly salaries of two persons are in the ratio 3:5.

If each receives an increase of `20 in the monthly salary the ratio is altered to 13:21.

Questions

1) Find the old salary of two persons.


2) Find the new salary of two persons.

Solution :

Let the old salaries be 3x: 5x

Let the new salaries be

(3x+20 : 5x+20)

Now the ratio of new salaries are in the ratio 13:21


(3x+20 : 5x+20) =(13:21)

WE KNOW THAT THE PRODUCT OF EXTREMES=PRODUCT OF MEANS

(3x+20 )(21) =(5X+20)(13)

63x +420 =65X +260

420—260 =65X --63X

160=2X

X=80

OLD SALARY OF FIRST PERSON=3X=3X80=240

OLD SALARY OF SECOND PERSON=5X=5X80=400

NEW SALARY OF FIRST PERSON=240+20 =260

NEW SALARY OF SECOND PERSON=400+20 =420

Exercise :

Yuva’s marks and Aish’s marks before coaching was in the ratio 3:2.

After coaching , each have lost 10 marks. The new ratio of marks is 2:1.

Questions:

1) Find the old marks of Yuva and Aish.


2) Find the new marks of Yuva and Aish.

Answer :

The old marks of Yuva and Aish are 30 and 20.


The new marks of Yuva and Aish are 20 and 10.

Exercise:

If 2:7 = 21:X
and

X:Y = 1:2

find Y.

Answer

Y= 147

Exercise:

If a:b = 2:3,

b:c = 4:5 ,

c:d = 1:2 find a:b:c:d

SOLUTION :

a= 2X

b= 3X

c= [3X/4] x 5 =15X/4

d= [15X/4]/1 x 2= 30 X/4

a : b : c :d = 2X : 3X : 15X/4 : 30X /4
8X : 12 X : 15X : 30 X

----------------------------------------------

8:12:15:30

SOLUTION :

PUT A= 200 POINTS

SO B=300 POINTS

PUT B=300 POINTS

C= [300 POINTS/4]=375 POINTS

PUT C= 375 POINTS

D= [375 POINTS/1] X2=750 POINTS

SO POINTS ARE

A=200

B=300

C=375

D=750

SO RATIO

200:300:375 :750
8:12:15:30

8 12 15 30

Exercise:

If a:b:c = 1:2:3 and b:c:d = 4:6:5 find a:d

Solution :

a= 1 X

b= 2X

c=3X

d= [3X/6] x 5 = 15X/6

a:d= 1X : 15X/6

6 X : 15X

= ---------------------

= 6X :15X

= 6:15

= 2:5

SOLUTION

PUT A=100 POINTS

SO B=200 POINTS

SO C=300 POINTS

IF C= 300 POINTS

D=[300 POINTS/6] X5=250 POINTS

A:D =100:250
10:25

2:5

2 4 6 5

Exercise:

If a:b = 5:3 , b:c = 7:4 find a:b:c.

SOLUTION

PUT A= 500 POINTS

SO B=300 POINTS

PUT B= 300 POINTS

C= [300/7] X4 =1200/7

A:B:C =500:300:1200/7

3500:2100:1200

35:21:12

35 21 12

Exercise:

Find the mean proportion of 9 and 4.

Answer:

9 :X = X:4
X2 = 36

X=6

Exercise:

Find the mean proportion of 7 and 11.

Answer:

7:X = X : 11

X2 = 77

X = 8.7

Exercise :

What is the third proportion of 4 and 6 ?

Answer:

4:6 = 6:X

4X = 36

X= 9

Exercise:

What is the third proportion of 7 and 5?

Answer:

7:5 = 5:X

7X = 25

X = 3.57
Exercise:

Find the fourth proportional to 10 , 25 and 40

Answer:

10:25 = 40:X

1000 = 10X

X=100

Exercise:

Find the fourth proportional to 8, 7 and 11.

Answer:

8:7 = 11:X

8X = 77

X = 9.625

EXERCISE PROBLEMS IN RATIOS AND PROPORTIONS:

[1] IF FOUR NUMBERS ½, 1/3, 1/5, 1/X ARE PROPORTIONAL THEN

X IS

(A) 6/5 (B) 5/6 (C) 15/2 (D) NONE OF THESE

Answer : [C]

[2] THE MEAN PROPORTIONAL BETWEEN 12X2 AND 27Y2 IS

(A)18XY (B) 81 XY (C) 8XY (D) NONE OF THESE

Answer : [ A]
12X2 : K :: K: 27Y2
2 2 2
[12X ] x[ 27Y ] =K
324X2Y2 = K2
18XY=K

[3]THE NUMBER WHICH HAS THE SAME RATIO TO 26 THAT 6 HAS TO 13 IS


(A)11 (B) 10 (C) 21 (D) NONE OF THESE

Answer : [ D ]
X:26 :: 6:13

13X=156

X=12

[4]IF A= B/2=C/5 THEN A:B:C IS

(A)3:5:2 (B) 2:5:3 (C) 2:3:5 (D) NONE OF THESE

SOLUTION :

A=0.50 B

B=B

C=5B/2= 2.5B

A:B:C = 0.50 B : B : 2.5B

A:B:C = 0.50 : 1 : 2.5

A:B:C =1 : 2 :5

LET THERE BE 1000 APPLES

A+B+C=1000

A=0.5B

B=2A

A=0.2C

=5A

A+2A+5A=1000
8A=1000

A=125

B=250

C=625

125 :250 :625

1:2:5

ANSWER [D]

[5] TWO NUMBERS ARE IN THE RATIO 3:4. IF 6 BE ADDED TO EACH

TERM OF THE RATIO ,THEN THE NEW RATIO WILL BE 4:5. THE NUMBERS

ARE

(A)14,20 (B)17,19 (C) 18,24 (D) NONE OF THESE

ANSWER =[C]

[6]DIVISION OF `7,500 INTO THREE PARTS IN THE RATIO 4:5:6 IS

(A)(2000,2500,3000) (B) (2500,2500,2500) (C) (3500,2500,1500)

(D) NONE OF THESE

ANSWER =[A]
[7] A MIXTURE CONTAINS MILK AND WATER IN THE RATIO 5:1.

ON ADDING 5 LITRES OF WATER, THE RATIO OF MILK AND WATER BECOMES

5:2. THE QUANTITY OF MILK IN THE ORIGINAL MIXTURE IS

(A)16 LITRES (B) 25 LITRES (C) 22.75 LITRES (D) 32.5 LITRES

Solution :

Milk= 5x litres

Water =1x litre

Ratio= 5x :1x

 Now we add 5 litres of water

Milk = 5x litres

Water =[1x+5] litres

 Now ratio is 5x : [1x+5]

5x : [1x+5] = 5:2

10x=5x+25

5x=25

X=5

Answer =25

ANSWER :[B]

[8].THE SUM OF THE AGES OF 3 PERSONS IS 150 YEARS. 10

YEARS AGO, THEIR AGES WERE IN THE RATIO 7:8:9. THEIR PRESENT AGES ARE

(A)(45,50,55) (B)(40,60,50) (C)(35,45,70) (D) NONE OF THESE


Solution :

Let the age of the three persons 10 years ago

be

7x , 8x and 9x

Now their ages will be

7x+10, 8x+10 and 9x+10

 7x+10 +8x+10+9x+10=150
 24x+30=150
 24x=120
 X=5

45,50,55

ANSWER :[A]

[9]. IF AN ALLOY CONTAINS COPPER AND SILVER IN THE

RATIO 3:7,THEN THE PERCENTAGE OF SILVER IN THE ALLOY IS

(A)90% (B) 70% (C) 60% (D) NONE OF THESE

ANSWER :[ B]

[10]WHAT SHOULD BE ADDED TO EACH TERM 3,15,38 AND 134 SO THAT THEY SHOULD BECOME
PROPORTIONATE TO EACH OTHER ?

(A)1 (B) 2 (C)3 (D) 4

[3+X ],[ 15+X], [38+X] AND [134+X] ARE IN PROPORTION


[3+x] :[15+x] :: [38+x]:[134+x]
[3+X][134+X] =[15+X][38+X]
402+3X+134X+X2 = 570+15X+38X+X2
137X +402 =53X+570

84 X=168

X=2

ANSWER =OPTION [B]


ADDITIONAL PROBLEMS AND SOLUTIONS IN RATIOS & PROPORTION

[1]Two numbers are in the ratio 2:3 and the difference of their

squares is 320.Find the two numbers.

Solution:-

Let the two numbers be 2x and 3x.

2x and 3x are in the ratio 2:3.

Square of first number=(2x)2=4x2

Square of second number=(3x)2 =9x2

Difference of the squares of the two numbers =9x2 -- 4x2= 5x2

It is given that the difference of the squares of the two numbers =320

Therefore,

5x2 = 320

x2 = 320 /5

x2 =64

x= square root of 64

x= 8.

Therefore , the two numbers are

2x= 2x8= 16

and

3x=3x8=24.

Therefore the two numbers are

16 and 24.

[2]Two numbers are in the ratio 5:2 and the difference of their

squares is 1701.Find the two numbers.


ANSWER : 45 AND 18

[3] Two numbers are in the ratio 7:8. If 3 is added to each of

them,their ratio becomes 8:9.

Find the two numbers.

ANSWER : 21 AND 24

Solution :

Let the first number be 7x

and let the second number be 8x.

Now we find that the numbers 7x and 8x are in the ratio 7:8.

We are going to add 3 to 7x

Now the new first number is (7x+3)

We are going to add 3 to 8x

Now the new second number is (8x+3).

It is given in the problem that the new numbers follow the ratio 8:9

Therefore,

(7x+3) :(8x+3) =8:9

In ratios there is a rule which is given below…

“ product of extremes”

“ product of means”
“ product of extremes”

(7x+3)(9) =(8x+3)(8)

63x+27 =64 x+24

27—24 =64x—63x

3 =x

Therefore, the two numbers are

7(3) & 8(3)

21 & 24.

[4]. Two numbers are in the ratio 2:3.

If 4 is subtracted from each, they are in the ratio 3:5.

Find the two numbers.

SOLUTION :

LET THE TWO NUMBERS BE 2X AND 3X

(2X—4) : (3X—4) = 3 :5

(2X—4) x 5 = (3X—4) x 3

10X—20 =9X—12

X=8

16 AND 24
[5]The ages of two persons are in the ratio 5:7. Eighteen

years ago their ages were in the ratio 8:13.

Find the present ages of two persons.

Solution :

Let the present age in years of first person=5x.

Let the present age in years of second person=7x.

18 years ago(before), the age in years of the first person=(5x—18)

18 years ago(before), the age in years of the second person=(7x—18)

It is given in the problem that the ratio of ages before 18 years=8:13

Therefore,

(5x—18) : (7x—18) =8:13

In ratios there is a rule which is given below…

“ product of extremes”

“ product of means”

(5x--18)(13) =(7x--18)(8)

65x--234 =56 x--144

65 x—56 x =--144+234

9x =90

x= 90/9
x= 10

present age in years of first person

= 5 x =5x10=50

present age in years of second person

= 7 x =7x10=70

Proof:

Age of first person 18 years ago=50—18=32

Age of second person 18 years ago=70—18=52

Ratio of age of two persons before 18 years=

32:52

16:26

8:13

[7]In a film shooting ,A and B received money

in a certain ratio and B and C also received the money in the same ratio. If A gets `1,60,000 and C gets
`2,50,000, find the amount received by B.

Solution:

A : B =B : C

`1,60,000 : B =B : `2,50,000

Let us apply the principle

PRODUCT OF MEANS =PRODUCT OF EXTREMES

1,60,000 X2,50,000=B2

TAKING SQUARE ROOT ON BOTH SIDES

400 X500= B
`2,00,000=B

[8] X and Y earn in the same ratio of earnings of Y and Z.The earnings

of X and Z are `400 and `625 respectively.Find the earnings of Y.

Answer:Rs500

[9] In what ratio should tea worth`10 per kg be mixed with tea worth `14 per kg,so that the average
worth price of the mixture may be `11 per kg ?

SOLUTION:

Let the mixture be 50 kgs.

Let X kgs be the cheap variety.

So [50—x ]kgs be the costly variety

[X x Rs 10] +[50—X]Rs 14

-------------------------------------- =Rs11

50kg

[10X ] +[700—14X]

--------------------------- =11

50kg

[10X ] +[700—14X] 11

--------------------------- = ----

50kg 1
--4x+ 700 11

--------------------------- = ----

50kg 1

--4x+ 700 = 550

--4x= --150

X=37.5

37.5 : 12.5

3:1

[10]Prasad & co produces paint.Cheap paint costs Rs200 per litre

and costly paint costs `300 per litre.

Prasad &Co plans to market a new grade paint mixing cheap

and costly paints and the cost per litre of

blended paint is planned at `250 per litre.

In what ratio, the cheap and costly paints have to be blended?

SOLUTION:

Let the mixture be 50 litres.

Let X kgs be the cheap variety.

So [50—x ]kgs be the costly variety

[X x200] +[50—X]300
--------------------------- =250

50litres

[200X ] +[15000—300X]

---------------------------------- =250

50litres

[200X ] +[15000—300X] 250

-------------------------------- = ----

50litres 1

--100x+ 15000 250

--------------------------- = ----

50litres 1

--100x+ 15000 = 12500

--100x= --2500

X=25

25 : 25

1:1

[10] Eight persons are planning to share equally the fixed cost of a

rental car for a trip .If one person withdraws from the arrangement
and the others share equally entire cost of car rental,then the share

of each of the remaining persons increases by what proportion ?

a) 1/9 b)1/8 c)1/7 d)7/8

solution :

original cost share of one traveller = 1/8

Revised cost share of one traveller =1/7

Increase in cost per traveller=

1/7-- 1/8

=( 8—7) /56

=1/56

(1/56 ) / (1/8)

=1/56 x8/1

=8/56

=1/7

Proof :

Let us assume that BALA TRAVELS

Charges `2,100 in innova car for 8 persons for a PICNIC.

So , the share of one traveller=2100/8=`262.50

Now there are only 7 travellers.

So ,the share of one traveler=`2100/7=`300

Increase per one traveller=300-262.50=37.5.

Increased proportion

=increase /original cost

37.5/262.5

1/7
[11] 6 FRESH CAs have got jobs at Chennai. They have hired a

common house for living.

7 th CA joined them. By what proportion will their cost come down ?

Solution :

Assume that the room rent is Rs4,200

Original share per inmate=Rs4,200/6=Rs700

Revised share per inmate=Rs4,200/7=Rs600

Reduction in rent per inmate=Rs100

Proportion of rent reduction= Rs100/Rs700 =1/7

[12] An alloy is to contain copper and zinc in the ratio 9 :4.

The zinc required to melt with 24 kg of copper is

a) 10 2/3 kg ={(3x10)+2}/3=32/3kg
b) 10 1/3kg={(3x10)+1}/3=31/3kg
c) 9 2/3 kg={(3x9)+2}/3=29/3kg
d) 9 kg

Solution :

Copper : zinc = 9:4

24 :z =9:4

Using the principle

Product of extremes=product of means

24 x4 =9z

96=9z

Z= 96/9

Z=32 /3

[13] Gold and copper has to be in13:2 ratio to make an ornament.


A gold smith has 2100 grams of copper.How many grams of

gold has to be melted with it to make a single ornament ?

solution:

2100

------ x 13 = 13650 grams.

Answer :13650 gms

2100

-------- x13 =13650

Solution :

gold : copper = 13:2

g : 2100 =13:2

Using the principle

Product of extremes=product of means

2100 x13 =2g

27300=2g

g= 27300/2

g=13650

[14] What must be added to each term of the ratio 49:68 so that it

becomes 3:4 after addition ?

SOLUTION :
In the problem we have been presented with a ratio which is

49 :68.

Now let us ADD x to the antecedent 49. Now the new antecedent is (49+x).

Now let us ADD x to the subsequent 68. Now the new subsequent is (68+x).

Now , the new ratio=

(49+x) : (68+x)

It is given in the problem that

(49+x) : (68+x) = 3:4

Our aim is to find the value of x.

(49+x) : (68+x) = 3:4

Let us use the principle that “ THE PRODUCT OF EXTREMES

= THE PRODUCT OF MEANS “

Therefore,

(49+x) (4) = (68+x) (3)

196 + 4x =204 +3x

4x—3x =204—196

X =8

PROOF :

ORIGINAL RATIO was 49 :68

NEW RATIO is (49+8) :(68+8)

57:76

3:4

15.Two numbers are in the ratio 2:3. What should


be subtracted from each term of the above ratio so

that the ratio becomes 3:5 ?

SOLUTION:

Let the numbers be 2 and 3

In the problem we have been presented with a ratio which is 2:3

[2—x] :[3—x] =3:5

[2—x] [5] = [3—x] [3]

10—5x =9—3x

1=2x

X=1/2 =0.50

16. The ratio between two numbers is 3:4. If their l.c.m is 180,

find the numbers.

SOLUTION :

Let the two required numbers be 3x and 4x.

Now we observe that

3x : 4x =3:4

The two required numbers are 3x and 4x.

l.c.m =least common multiple

of 3x and 4x =12x

12x =180

X =180/12

X =15
therefore , the two numbers are

45 and 60

3x and 4x

=3(15) and 4 (15)

=45 and 60

Proof :

The two numbers are 45 and 60

l.c.m=180..

17. If the ratio of (5x—3y) and (5y—3x) is 3:4,what is the value of x:y ?

SOLUTION:

It is given in the problem that

(5x—3y) : (5y—3x) = 3:4

(5x—3y) : (5y—3x) :: 3:4

Let us apply the principle that

“the product of extremes =the product of means”

(5x—3y) (4)= (5y—3x)(3)

20x—12y =15y—9x

20x+9x =15y+12y

29x =27y

x/y =27/29

x:y =27:29

18. Mr. Ramkumar went to a bank with a `1,000 currency note .He

requested the bank officer to give him `5 and `10 notes only in return. Mr Ramkumar got 175 currency
notes in total.Find how many `5 currency
notes and how many `10 currency notes did Mr Ramkumar receive from

bank officer?

SOLUTION

Let x represent the number of `5 currency notes.

Let y represent the number of `10 currency notes.

It is given in the problem that the total number of currency notes

which was received by Mr Ramkumar from the bank officer=175

So we can express this business situation in the form of the following

Equation

x+y=175-------------eqn 1

We observe that the total amount is `1,000 which is represented

by `5 currency notes as well as `10 currency notes.

This business situation can also be expressed as an equation in the

following manner

(`5) (x) + (`10)(y) =`1,000

5x+10y=1000

X+2y=200------------------eqn(2)

Let us solve eqn 1 and eqn 2 so that we derive the values of x and y.

X + y =175 ---------(eqn1)

X + 2y =200----------(eqn2)
Eqn 2—Eqn 1

Results in y=25

Therefore x=175—25=150

So

X=150 and

Y= 25.

PROOF:

`5(150) + `10(25)

=`750 +`250

=`1,000

19. A BOX CONTAINS `56 IN THE FORM OF COINS OF ONE RUPEE,

50PAISE AND 25 PAISE.THE NUMBER OF 50 PAISE COINS IS

DOUBLE THE NUMBER OF 25 PAISE COINS AND FOUR TIMES THE

NUMBER OF ONE RUPEE COINS.

 HOW MANY 50 PAISE COINS ARE THERE INSIDE THE BOX?


 HOW MANY 25 PAISE COINS ARE THERE INSIDE THE BOX?
 HOW MANY 1 RUPEE COINS ARE THERE INSIDE THE BOX?

SOLUTION:

Let x represent the number of 100 paisa coins

Let y represent the number of 50 paisa coins

Let z represent the number of 25 paisa coins

Equation 1 :

100x + 50 y + 25 z= 5600

THE NUMBER OF 50 PAISE COINS IS


DOUBLE THE NUMBER OF 25 PAISE COINS AND FOUR TIMES THE

NUMBER OF ONE RUPEE COINS.

y=2z

y=4x

x= y/4 = 0.25 y

So x=0.25 y

y=y

z= 0.50 y

so we expresss

x= 0.25y , y=y and z= 0.50 y

100x + 50 y + 25 z= 5600

100[0.25y] + 50y +25[0.50y] =5600

25y +50y+12.5y=5600

87.5y=5600

Y= 5600/87.5=64

So y= 64

x= 0.25y , y=y and z= 0.50 y

x= 0.25 [64] =16

so x=16

z=0.50[64] =32
 THERE ARE 5,600 PAISE INSIDE THE BOX

5,600 paise is the TOTAL AMOUNT in the bag.

This 5,600 paise is represented by


 50 PAISE COINS
 25 PAISE COINS
 100 PAISE COINS

A clue is given in the problem. The number of 50paise coins is twice as that of 25 paise coins. It is a ratio.

Another clue is given in the problem. The number of 50paise coins is four times as that of 100 paise
coins. It is also a ratio.

Let X be the number of 50 paise coins in the bag.


Therefore 0.50 X will be the number of 25 paise coins in the bag.
Also 0.25 X will be the number of 100 paise coins in the bag.

 50 PAISE COINS ARE THERE


 (X numbers)
 25 PAISE COINS ARE THERE
(0.5X numbers)
 100 PAISE COINS ARE THERE
(0.25X numbers)

(50paise )X + (25paise)(0.5X) + (100paise)(0.25X)


=5600 paise
Here, X will represent the number of 50 paise coins in the box.

0.5 X will represent the number of 25 paise coins in the box.

0.25 X will represent the number of 100 paise coins in the box.

50X+12.5X +25X=5600
87.5X=5600
X=5600/87.5
X=64
50 PAISE COINS=64
25 PAISE COINS=32
100PAISE COINS=16
PROOF:
50 PAISE COINSX64=`32
25 PAISE COINSX32=`08
100PAISE COINSX16=`16
TOTAL =`56.

20. The students of two classes are in the ratio 5:7.

If 10 students have left from each class, the remaining

students are in the ratio 4:6. How many students continue in each class ?

SOLUTION:

LET THE NUMBER OF STUDENTS IN I CLASS BE 5X.

LET THE NUMBER OF STUDENTS IN II CLASS BE 7X.

NOW 5X:7X =5:7

5X:7X ::5:7

WE HAVE TO DEDUCT 10 FROM ANTECEDENT 5X WHICH BECOMES (5X—10)

WE HAVE TO DEDUCT 10 FROM CONSEQUENT 7X WHICH BECOMES (7X—10)

NOW WE HAVE NEW RATIO WITH US

i.e

(5x—10) : (7x—10) = 4 :6

Product of extremes

=
product of means

(5x—10) (6) = (7x—10)(4)

30X—60 =28X—40

30X—28X=--40 +60

2X=20

X=20/2

X=10

THEREFORE

ORIGINAL CLASS STRENGTH

I CLASS 5X =5(10)=50

II CLASS 7X=7(10)=70

REDUCED CLASS STRENGTH

I CLASS 5X =5(10)=50—10=40

II CLASS 7X=7(10)=70—10=60

21. RATIO OF EARNINGS OF ARUN AND BALAJI IS 4:7.

IF THE EARNINGS OF ARUN INCREASES BY 50 %


AND THE EARNINGS OF BALAJI DECREASES BY 25%,

THE NEW RATIO OF THEIR EARNINGS EQUALS TO 8:7

 FIND THE ORIGINAL EARNINGS OF ARUN


 FIND THE ORIGINAL EARNINGS OF BALAJI
 FIND THE REVISED EARNINGS OF ARUN
 FIND THE REVISED EARNINGS OF BALAJI
Solution :

Original earnings ratio is 4x : 7x


Revised earnings ratio 6x :5.25x
6x:5.25x :: 8:7
42x =42x

 SOLUTION

LET THE ORIGINAL EARNINGS OF ARUN BE 4X

LET THE ORIGINAL EARNINGS OF BALAJI BE 7X

LET THE REVISED EARNINGS OF ARUN BE (4X) + 50/100(4X)

4X+2X=6X

OR

4X X 150/100 =6X

LET THE REVISED EARNINGS OF BALAJI BE (7X) -- 25/100(7X)

7X—1.75X=5.25X

OR

7X X 75/100 =5.25X

THE REVISED RATIO

6X : 5.25X = 8:7

PRODUCT OF EXTREMES=PRODUCT OF MEANS


6X X 7 = 5.25 X 8

42X =42X

SO THE SOLUTION CAN NOT BE FOUND AS THE GIVEN DATA IS INADEQUATE.

 FIND THE ORIGINAL EARNINGS OF ARUN


 FIND THE ORIGINAL EARNINGS OF BALAJI
 FIND THE REVISED EARNINGS OF ARUN
 FIND THE REVISED EARNINGS OF BALAJI

22. THE EARNINGS OF AMAR AND PREM ARE IN THE RATIO 3:2.

THE EXPENSES OF AMAR AND PREM ARE IN THE RATIO 5:3

AMAR SAVES `1,500 AND PREM ALSO SAVES `,1500.

 FIND AMAR’S EARNINGS


 FIND PREM’S EARNINGS
 FIND AMAR’S EXPENSES
 FIND PREM’S EXPENSES

SOLUTION

WE HAVE TO UNDERSTAND THE BASIC ECONOMIC EQUATION

{EARNINGS}—{EXPENSES}={SAVINGS}

EARNINGS OF AMAR AND PREM ARE IN THE RATIO

3:2

OR 3X:2X

EXPENSES OF AMAR AND PREM ARE IN THE RATIO

5:3
OR 5Y:3Y

3X—5Y=1500----------(1)

2X—3Y=1500-----------(2)

(1)=(2)

3X—5Y=2X—3Y

3X—2X=--3Y+5Y

X=2Y IN (1)

LET US SUBSTITUTE X=2Y IN

3(2Y)—5Y=1500----------(1)

6Y—5Y=1500

Y=1500

X=2Y

X=2(1500)

X=3000

 AMAR’S EARNINGS
3X=3(3000)=`9,000
 PREM’S ERNINGS
2X=2(3000)=`6,000

 AMAR’S EXPENSES
5Y=5(1500)=`7,500

 PREM’S EXPENSES
3Y=3(1500)=`4,500

23. A BAG CONTAINS `187 IN THE FORM OF 1 RUPEE, 50 PAISE AND

10 PAISE COINS IN THE RATIO 3:4:5.FIND THE NUMBER OF EACH TYPE


OF COIN.

SOLUTION:

LET US CONVERT `187 INTO PAISE

`187 X 100 PAISE=18,700 PAISE

18,700 PAISE IS REPRESENTED BY

 100 PAISE COINS


 50 PAISE COINS
 10 PAISE COINS

LET 3X REPRESENT THE NUMBER OF 100 PAISE COINS.


LET 4X REPRESENT THE NUMBER OF 50 PAISE COINS.
LET 5X REPRESENT THE NUMBER OF 10 PAISE COINS.

NOW WE CAN FORM THE FOLLOWING EQUATION


100 PAISE(3X) +50PAISE(4X) +10 PAISE(5X)=18,700 PAISE

300 X+200X+50X=18,700

550X=18,700

X =18,700/550

X = 34

NUMBER OF 100 PAISE COINS=3X=3X34=102

NUMBER OF 50 PAISE COINS=4X=4X34=136

NUMBER OF 10 PAISE COINS=5X=5X34=170

PROOF:

(102 X100) + (136X50) + ( 170 X10)=

=10,200 PAISE +6,800 PAISE+1,700 PAISE =18,700 PAISE.


24. A,B AND C ARE THREE CITIES. THE RATIO OF

AVERAGE TEMPERATURE BETWEEN A AND B IS 11:12 AND

THAT BETWEEN A AND C IS 9:8. WHAT IS THE RATIO OF

AVERAGE TEMPERATURE BETWEEN B AND C?

SOLUTION :

A:B= 11:12

A:C=9:8

LET THE TEMPERATURE OF CITY A BE 864 UNITS

 THEN THE TEMPERATURE OF CITY B WILL BE =


[864/11]X12 = [10368/11] UNITS

 THEN THE TEMPERATURE OF CITY C WILL BE=[864/9]X8 = [6912/9] UNITS

B:C= 10368 6912

--------- : -------

11 9

10368X9 : 6912X11

93312 :76032

46656 :38016

23328 :19008

11664 :9504

5832 :4752

2916 :2376
1458:1188

729:594

243:198

81:66

27:22

SOLUTION :

A:B=11:12

A/B = 11/12

[12/11]A=B

A:C =9:8

A/C =9/8

[8/9]A=C

A= A

B=[12/11]A

C=[8/9]A

B:C = 12/11 : 8/9

[ 108 :88]/99

108:88
54:44

27:22

GIVEN THAT A:B=11:12

ALSO GIVEN THAT A:C=09:08

B:C =?

A B C
11 12 **LOOK
UP
WARDS
108/11
BECAME
12.

8 WILL
BECOME
WHAT ?
12/
(108/11)

X 8
=

12 X11 X8
------------
108
=1056
--------
108
=264
------
27
=88
----
9

09 * LOOK DOWN 08
WARDS
11
BECAME
9.
12 WILL
BECOME
WHAT ?

(9/11 )X12=
108/11
A=11 PONTS B=12 POINTS C=88/9
POINTS
MULTIPLY MULTIPLY MULTIPLY
BY9 BY9 BY9
A=99 B=108 C=88

B:C=108:88

B:C=54: 44

B:C=27:22

25. IN 40 LITRES MIXTURE OF GLYCERINE AND WATER, THE

RATIO OF GLYSERINE AND WATER IS 3:1.

THE QUANTITY OF WATER TO BE ADDED TO THE MIXTURE

IN ORDER TO MAKE THE RATIO AS 2:1 =?

SOLUTION:

PRESENTLY, THE MIXTURE HAS 40 LITRES.

GLYCERINE CONTENT IN THE ABOVE 40 LITRES= 3/(3+1) X40

¾ X 40=30 LITRES.
WATER CONTENT IN THE ABOVE 40 LITRES= 1/(3+1) X40

¼ X 40=10 LITRES

GLYCERINE CONTENT IS 30 LITRES WHICH IS NOT CHANGED.

THE NEW RATIO OF GLYCERINE AND WATER IS 2:1

30 : X = 2:1

30X1 =2X

30=2X

15=X

THEREFORE, THE REVISED WATER CONTENT IN THE MIXTURE=15 LITRES.

ORIGINAL WATER CONTENT=10 LITRES.

ADDED WATER =5 LITRES.

26. ARUN, BALAJI AND CHANDRAN STARTED A BUSINESS INVESTING `1,26,000 , `84,000 AND `2,10,000.
AT THE END OF THE YEAR,THE PROFIT OF THEIR JOINT BUSINESS IS `2,42,000. THEY HAVE AGREED TO
SHARE THE TOTAL PROFIT OF THEIR JOINT BUSINESS IN THE RATIO OF THEIR ORIGINAL CAPITAL
CONTRIBUTION.

* WHAT IS ARUN’S PROFIT SHARE?

* WHAT IS BALAJI’S PROFIT SHARE?

* WHAT IS CHANDRAN’S PROFIT SHARE?

SOLUTION:

LET US SIMPLIFY IF POSSIBLE THE CAPITAL CONTRIBUTION RATIO OF EACH PARTNER…

(`1,26,000):(`84,000):(`2,10,000)

(`126):(`84):(`210)

(`63):(`42):(`105)
(`9):(`6):(`15)

(3):(2):(5)

WE HAVE TO APPORTION `2,42,000 AMONG THE PARTNERS ARUN , BALAJI AND CHANDRAN IN THIS
RATIO

3:2:5

ARUN’S PROFIT SHARE

`2,42,000 X{3/(3+2+5)}

=`2,42,000 X 3/10

=`72,600

BALAJI’S PROFIT SHARE

`2,42,000 X{2/(3+2+5)}

=`2,42,000 X 2/10

=`48,400

CHANDRAN’S PROFIT SHARE

`2,42,000 X{5/(3+2+5)}

=`2,42,000 X 5/10

=`1,21,000

PROOF

`(72,600+48,400+1,21,000)

=`2,42,000

ARUN’S PROFIT SHARE

`2,42,000 X
{1,26,000/(1,26,000+84,000+2,10,000)}

=`2,42,000 X

1,26,000/4,20,000

=`72,600

BALAJI’S PROFIT SHARE

`2,42,000 X

{84,000/(1,26,000+84,000+2,10,000)}

=`2,42,000 X

84,000/4,20,000

=`48,400

CHANDRAN’S PROFIT SHARE

`2,42,000 X

{2,10,000/(1,26,000+84,000+2,10,000)}

=`2,42,000 X

2,10,000/4,20,000

=`1,21,000

PROOF

`(72,600+48,400+1,21,000)

=`2,42,000

27. HOME WORK:

PRADHEEP ,SANDHEEP AND SUDHIR ARE THREE PARTNERS OF M/s PSS & CO EACH CONTRIBUTING
`50,000 , 75,000 AND `25,000 AS CAPITAL
RESPECTIVELY. THE PROFIT FOR THE ACCOUNTING YEAR 1-4-12 TO 31.3.13 OF PSS & CO WAS `15,000.
YOU ARE THE CA OF THE FIRM. AS PER THEIR AGREEMENT, THE PROFIT HAS TO BE DIVIDED IN THE RATIO
OF ORIGINAL CAPITAL CONTRIBUTION. FIND THE RESPECTIVE SHARE OF EACH PARTNER.

Answer :

5,000

7,500 &

2,500

28. ` 407 HAS TO BE DIVIDED AMONG ARUN, BALAJI AND CHANDRAN IN THE RATIO 1/4 : 1/5 :
1/6.

FIND THE SHARE AMOUNT OF EACH PARTNER.

SOLUTION:

THE FOLLOWING RATIO IS PRESENTED IN THE PROBLEM

1/4 : 1/5 : 1/6.

WE HAVE TO REARRANGE THE RATIO INTO A USEFUL FORM.

IF ALL RATIOS ARE MULTIPLIED OR DIVIDED BY A COMMON QUANTITY, THE RATIO WILL REMAIN
UNALTERED.

LET US TAKE L.C.M(LEAST COMMON MULTIPLE) OF THE DENOMINATORS 4,5 AND 6

WHICH IS 60. LET US MULTIPLY THE FRACTIONAL RATIOS WITH 60 IN THE NUMERATOR.

THE FOLLOWING IS THE OUTCOME OF THIS MATHEMATICAL OPERATION.


60 (1/4) :60(1/5) : 60( 1/6)

15 :12 :10

ARUN’S SHARE

=`407X{15/(15+12+10)}

=`407 X15 /37

= `165.

BALAJI’S SHARE

=`407X{12/(15+12+10)}

=`407 X12 /37

= `132.

CHANDRAN’S SHARE

=`407X{10/(15+12+10)}

=`407 X10 /37

= `110.

PROOF :

`(165 +132+110)=`407.

29. DIVIDE `1,020

IN 2/7 : 3/4 : 1/2 RATIO.


8:21:14

-------------------------

28

1020 x [8/43] = 189.77

1020 x [21/43]= 498.14

1020 x [14/43] = 332.09

30.

IF A:B=3:4 AND B: C=8:9

WHAT IS A:C ?

SOLUTION

A B C
3 4 LOOK
UPWARDS
.
IN B COLUMN
8 BECAME
4.
9 WILL
BECOME
WHAT?
8 9

4
--- X9
8

=9
--
2
3 4 9
---
2
MULTIPLY MULTIPLY MULTIPLY
BY BY BY
2 2 2
WE GET WE GET WE GET
6 8 9

RATIO IN BETWEEN A AND C

= 6:9

OR

2:3

31.

IF X:Y =7:9, Y: Z=5:4, WHAT IS X:Y:Z ?

Solution

X:Y = 7:9

9X= 7Y

Y= [ 9/7 ] X

Y= 9X/7

Y:Z= 5:4

[9X/7] : Z = 5:4

36X/7 =5Z

[36X/7 ] /5 =Z

36X/35 =Z

Z= 36X/35

X:Y:Z = 1X : 9X/7 : 36X/35

X:Y:Z = 35 X : 45X : 36X


-------------------------------

35

X:Y:Z =35 : 45 : 36

32.

IF A:B =2:3, B: C=4:5 AND C:D=6:7 WHAT IS

A:B:C:D?

ANSWER : 16:24:30:35

33.

IF A:B =3:2, B: C=3:5 ,

WHAT IS A:B:C?

ANSWER : 9:6:10

34.

IF `782 BE DIVIDED INTO THREE PARTS, PROPORTIONAL TO

1 2 3

--- : --- :----


2 3 4,

THEN

 WHAT IS FIRST PART?


 WHAT IS SECOND PART?
 WHAT IS THIRD PART?
ANSWER : Rs 204 , Rs272 and Rs306

35 .

IN A BAG, THERE ARE COINS OF 25 PAISE,10 PAISE AND 5 PAISE IN THE RATIO OF 1:2:3. IF THERE ARE
`30 IN ALL , HOW MANY 5 PAISE COINS ARE THERE?

ANSWER : 150.

What is the duplicate ratio of a ratio ?

Answer : The square the terms of a ratio is known as the duplicate ratio of the original ratio.

Example : Let A : B be a ratio .

The duplicate ratio of A:B

A2 : B 2

Question

What is the triplicate ratio of a ratio ?

Answer : The cube of the terms of a ratio is known as the triplicate ratio of the original ratio.

Example : Let A : B be a ratio .

The triplicate ratio of A:B


=

A3 : B 3

Question

What is the quadruplicate ratio of a ratio ?

Answer : The quadruple

(4 th power ) of the terms of a ratio is known as the quadruplicate ratio of the original ratio.

Example : Let A : B be a ratio .

The quadruplicate ratio of A:B

A4 : B 4

question

What is the sub -duplicate ratio of a ratio ?

Answer : The square root of the terms of a ratio is known as the sub duplicate ratio of the original ratio.

Example : Let A : B be a ratio .

The sub duplicate ratio of A:B

=√
2
A : √2 B
Question :

What is the sub triplicate ratio of a ratio ?

Answer : The cube root of the terms of a ratio is known as the sub triplicate ratio of the original ratio.

Example : Let A : B be a ratio .

The sub triplicate ratio of A:B

√3 A : √3 B
Question

What is the sub quadruplicate ratio of a ratio ?

Answer : The fourth root

of the terms of a ratio is known as the sub quadruplicate ratio of the original ratio.

Example : Let A : B be a ratio .

The sub quadruplicate ratio of A:B

√4 A : √4 B

Example 36

If p : q is the sub duplicate ratio of (p—x 2) : (q-- x2 ),

what is x2 ?

(a) pq/ (p+q)


(b) q/(p+q)
(c) qp/(p—q)
(d) none of the above.

SOLUTION
 It is given in the problem

that p : q is the SUB DUPLICATE

ratio of (p—x2) : (q-- x2 )


2 ( p — x 2 ) : √2 (q-- x2 )
¿

p:q

squaring both sides


(p—x2) : (q-- x2 ) =p2 : q2

 Using the principle product of means=product of extremes

q2 (p—x2) = p2 (q-- x2 )

q2p-- q2 x2 = p2 q-- p2 x2

p2 x2 -- q2 x2 = p2 q-- p q2

x2(p2-- q2) =pq(p—q)

x2(p+ q) (p- q) =pq(p—q)

x2(p+ q) =pq

x2 =pq/(p+ q)

37.find the triplicate ratio of 3:2

Answer :27:8

38.The salaries of Mr A, Mr B and Mr C are in the ratio2:3:5.

If the increments of 15%, 10%and 20 % are allowed respectively in their salaries, then what will be the
new ratio of their salaries ?

SOLUTION:

It is given in the problem that the ratio of unrevised salaries before increment=
2:3:5

After increment, the ratio of revised salaries will be

(2 x115/100 ) : ( 3x110/100):

(5x120/100)

(230/100):(330/100):( 600/100)

230:330:600

23:33:60

***********************

2+ (15/100 x2)

2+0.30

2.30

*********************

1 + (10/100 x3)

3+ 0.30

3.30

************************

5+ (20/100 x5)

5+1

****************

New ratio

2.3 : 3.3 :6
23:33:60

39.Ram , Robert and Rahim had weights in the ratio 4:5:2.

They have done exercise for certain months. Their weights got reduced by 5% ,20% and 10 %
respectively.

Find the ratio of new weights of the three patients.

ANSWER : 19: 20: 9

40.

One kilogram of rice was priced at `30 in the year 2005. In the year 2013, the price got increased by
15%. What is the price per kilogram of rice in the year 2013 ?

Solution

It is given in the problem that the price of one kilogram of rice in the year 2005 was `30.

In the year 2013, the percentage increase in the price is 15%

Therefore, price per kilogram of rice in the year 2013

=`30 x 115/100

=`34.50

Alternative method of answer

(Old price) + (percentage increase xold price)

=`30 + ( 15/100 x`30)

=`30+ `4.50

`34.50

41.

The price of one gram of gold on first day of a week was `2,950.

On the last day of the week, the price fell by 4 %. Find the price of one gram of gold on the last day of the
week in the commodity market.
ANSWER : 2950 X [96/100]= 2832.

42.

Mr X’s profit is to Mr Y’s profit is 4:5.

Mr Y’s profit is to Mr Z’s profit is 2:3.

Mr X’s profit is `8,000.

Find the profit of Mr Z.

Solution:

X:Y =4:5

Y:Z= 2:3

GIVEN THAT X=8,OOO

8000:Y=4:5

PRODUCT OF MEANS =PRODUCT OF EXTREMES

8,000X5=4Y

40,000=4Y

Y=40,000/4

Y=10,000.

10,000 :Z=2:3

PRODUCT OF MEANS =PRODUCT OF EXTREMES

10,000 X3=2Z

30,000=2Z

Z=30,000/2=15,000

43.
If

a/3 = b/4=c/7

find the value of (a+b+c)/c

SOLUTION:

Let

a/3 = b/4=c/7 =k

a/3 =k

a=3k

b/4=k

b=4k

c/7=k

c=7k

(a+b+c)/c

(3k+4k+7k)/7k

14k/7k

=2

ALTERNATIVE METHOD:

a/3 = b/4= c/7

[a+b+c]/c =

Let us express “a” in terms of c

a/3 =c/7

a= 3c/7
Let us express “b” in terms of c

b/4 =c/7

b= 4c/7

[3c/7] + [4c/7] + c

----------------------------

3c +4c+7c

-----------------

------------

=14c

---------

7c

=2

****************************

44.

If 2A=3B and

4B=5C

WHAT IS A:C ?
SOLUTION:

Given that 2A=3B

A/B = 3/2

A:B=3:2

Also given that

4B=5C

B/C =5/4

B:C=5:4

A B C
3 2 *5
BECAME2
,4
BECOMES
WHAT ?
(2/5 ) X4
=8/5.
5 4
3 2 8/5
3X5 2X5 8/5 X5
=15 =10 =8

A:C =15:8

****************************

45.

4 3.5 : 2 5 = ?

(2 2) 3.5 : 2 5

2 (2X3.5) : 2 5

27 :25

27/25

2 (7—5)

22
=

ANS

OR

4:1

46.

IF

(1/5 : 1/X) = (1/X : 100/125)

WHAT IS X ?

SOLUTON :

GIVEN THAT

(1/5 : 1/X) = (1/X : 100/125)

WE KNOW THAT THE PRODUCT OF MEANS =PRODUCT OF EXTREMES

(1/5 : 1/X) = (1/X : 100/125)

1/5 x 100/125 =1/X x 1/X

100/625 =1/ (X 2 )

100 (X 2 )=625

X 2 =625/100

X 2 =6.25

X=2.5
47.

FIND IN WHAT RATIO WILL THE TOTAL WAGES OF THE WORKERS OF A FACTORY BE INCREASED OR
DECREASED IF THERE BE A REDUCTION IN THE NUMBER OF WORKERS IN THE RATIO 15:11 AND AN
INCREMENT IN THEIR WAGES BE IN THE RATIO 22:25?

Answer :

11 25

---- x ------

15 22

275

-----

330

55

----

66

----

A)5:6

B)2:3

C)6:5
D)4:7

48. THE RATIO OF THE PRICES OF TWO TYPES OF CARS WAS 16 :23.

TWO YEARS LATER, WHEN THE PRICE OF FIRST CAR HAS INCREASED BY 10% AND THAT OF THE SECOND
CAR INCREASED BY `477, THE RATIO OF THE PRICES BECOMES 11 :20. FIND THE ORIGINAL PRICES OF THE
TWO CARS.

SOLUTION:

LET THE ORIGINAL PRICES OF THE TWO CARS TWO YEARS AGO BE 16 X : 23 X

NOW THE NEW PRICES WILL BE

(16 X x110/100) :(23X +`477)

17.6 X : (23X + 477)

{17.6 X : (23X + 477)} =11:20

PRODUCT OF MEANS=PRODUCT OF EXTREMES

(17.6 X)(20)= (23X + 477)(11)

352 X = 253 X + 5247

352 X -- 253X = 5247

99 X = 5247

X =5247/99

X=53

ORIGINAL PRICE OF FIRST CAR=

16X53=`848

ORIGINAL PRICE OF SECOND CAR=

23X53=`1,219

49.The inverse ratio of 11:17 = ?


Answer:

The inverse ratio of 11:17

=17:11

50.

If (2x +3) : (5x—38) be the duplicate ratio of √


2
5 : √2 6

,then find the value of x ?

Solution :

We know that the duplicate ratio of a:b = a 2 :b2 .

It is given in the problem that the

duplicate ratio of

√2 5 : √2 6 = (2x +3) : (5x—38)

We have to square LHS

5:6 =(2x +3) : (5x—38).

We know that the PRODUCT OF MEANS = PRODUCT OF EXTREMES


(5)( 5x—38) =(6) (2x +3)

25x—190 =12x + 18

25x—12x = 18 +190

13x=208

X =208/13

X=16

51. Find the sub –triplicate ratio of 8:27

Answer:

We know that the sub triplicate ratio of a : b = √


3
a : √3 b
Similarly, the sub triplicate ratio of 8 : 27 = √
3
8 : √3 27
= 2 : 3

52. Find the COMPOUNDED RATIO of the following three ratios.


(X:Y) , ( 3A : 5B) and ( 6C:5D)

SOLUTION:

HERE WE HAVE THREE RATIOS WHICH ARE

(X:Y) , ( 3A : 5B) and ( 6C:5D).

WE ARE ASKED TO FIND THE COMPOUNDED RATIO.

IF WE MULTIPLY THE THREE ANTECEDENTS WE GET THE ANTECEDENT OF THE COMPOUNDED RATIO

= (X)(3A)(6C)=18ACX.

SIMILARLY WHEN WE MULTIPLY THE THREE SUBSEQUENTS WE GET THE CONSEQUENT OF THE
COMPOUNDED RATIO

= (Y)(5B)(5D)=25BDY.

THEREFORE, THE COMPOUNDED RATIO=

18ACX :25BDY

OR

18ACX / 25BDY

53.FIND THE COMPOUNDED RATIO OF THE FOLLOWING TWO RATIOS.

(a+b) : (a—b) and (a2 --b2) : (a+b)2

SOLUTION

HERE WE HAVE BEEN GIVEN TWO RATIOS

WHICH ARE

(a+b) : (a—b) and (a2 --b2) : (a+b)2

.WE HAVE BEEN ASKED TO COMPOUND THE ABOVE TWO RATIOS.


THE FIRST TERM (ANTECEDENT) OF THE COMPOUNDED RATIO IS THE MULTIPLICATION OF THE
ANTECEDENTS OF THE GIVEN TWO RATIOS.

(a+b) (a2 --b2)

=(a+b) (a+b)(a—b)

=(a+b)2 (a—b)

THE SECOND TERM

( SUBSEQUENT )OF THE COMPOUNDED RATIO IS THE

MULTIPLICATION OF THE SUBSEQUENTS OF THE GIVEN TWO RATIOS.

(a--b) (a +b)2

THE COMPOUNDED RATIO =

(a+b)2 (a—b) : (a--b) (a +b)2


1 :1

54.THE RATIO COMPOUNDED OF 4:9 AND THE DUPLICATE RATIO OF 3:4 =?

SOLUTION

FIRST OF ALL, LET US COMPUTE THE DUPLICATE RATIO OF 3:4=

32 : 4 2

=9 :16

NOW WE HAVE TO COMPOUND THE FOLLOWING TWO RATIOS

4:9 AND 9:16

(4/9) X (9 /16 )

4 :16

1:4

***************************

55.
IF X:Y=2:3,

FIND THE VALUE OF

(6X—Y) : (3X+2Y)

SOLUTION

IT IS GIVEN IN THE PROBLEM THAT

X:Y=2:3

PRODUCT OF MEANS =PRODUCT OF EXTREMES

THEREFORE,

3X=2Y

LET US SUBSTITUTE

3X=2Y IN

(6X—Y) : (3X+2Y)

SINCE 3X=2Y,

WE CAN EXPRESS 6X AS 4Y

ALSO 3X=2Y,

IN THE PLACE OF 3X WE CAN PUT 2Y IN THE SUBSEQUENT

(4Y—Y) : (2Y+2Y)

3Y : 4Y

3:4

56.

THE RATIO COMPOUNDED OF 4:9, THE DUPLICATE RATIO OF 3:4 ,THE TRIPLICATE RATIO OF 2:3 AND 9:7
=?

SOLUTION:
WE HAVE TO COMPOUND THE FOLLOWING FOUR RATIOS

4:9 , 9:16, 8:27 AND 9 :7

(4X9X8X9) : (9X16X27X7)

(4X8X9) : (16X27X7)

(8X9) : (4X27X7)

(2X9) : (27X7)

2: (3X7)

2:21

57. THE DUPLICATE RATIO OF

3√ X : 4 √ Y IS

(A) 9X:16Y
(B) 3X:4Y
(C) 16Y:9X
(D) NONE OF THESE

SOLUTION :

THE ORIGINAL RATIO AS GIVEN IN THE PROBLEM IS

3√ X : 4 √ Y .

WE HAVE TO DUPLICATE THE ABOVE RATIO.

{(3√ X )( 3√ X ¿ ¿ } :{(4 √ Y ¿¿ 4 √ Y ¿}¿

9X : 16 Y

58.
THE SUB DUPLICATE RATIO OF 289 :441 IS ?

SOLUTION:

WE KNOW THAT THE SUB DUPLICATE RATIO OF A:B

=√
2
A : √2 B
THEREFORE , THE SUB DUPLICATE RATIO OF 289 :441

=√
2
289 : √2 441
=17 : 21

59. The ratio compounded of

2:3, 9 :4 , 5:6 and 8:10 =?

SOLUTION

(2X9X5X8) : (3X4X6X10)

(9X8) : (3X4X6)

(3X8) : (4X6)

(3X2) : (6)

6:6

1:1

60. The ratio of the quantities is 5:7. If the subsequent of its inverse ratio is 5,the antecedent =?

SOLUTION

Original ratio= 5:7

Inverse ratio = 7:5

ANTECEDENT OF INVERSE RATIO=7

61. If p:q=2:3 and x:y=4:5, then the value of (5px +3qy):(10px+4qy) =?

SOLUTION:
It is given in the problem that

p:q=2:3

3p=2q

p= 2q/3

Also given that

x:y=4:5

5x=4y

x= 4y/5

(5px +3qy) :(10px+4qy)

(5(2q/3)(4y/5) +3qy) : (10(2q/3)(4y/5)+4qy)

(2q/3)(4y) +3qy) : (2(2q/3)(4y)+4qy)

(2q/3)(4y) +3qy) : (2(2q/3)(4y)+4qy)

(8qy/3 +3qy) : (16qy/3 +4qy)

(8qy/3 +3qy) : (16qy/3 +4qy)

Multiply all terms with 3

(8qy+9qy) : (16qy +12 qy)

17qy :28qy

17 :28

62.Arun earns `80 in 7 hours and varun earns `90 in 12 hours. Find the ratio of their earning efficiency ?

SOLUTION:

ARUN’S HOURLY EARNING =`80/7

VARUN’S HOURLY EARNING =`90/12

RATIO OF EARNING EFFICIENY OF ARUN AND VARUN =

80 /7 : 90/12

960/84 : 630 /84


960: 630

32:21

63. SALARIES OF RAVI AND SUMEET ARE IN THE RATIO 2:3.IF THE SALARY OF EACH IS INCREASED BY
`4,000, THE RATIO OF SALARY BECOMES 40:57 . WHAT IS SUMEET’S PRESENT SALARY ?

SOLUTION:

LET THE PRESENT SALARY OF RAVI BE 2X

AND THE PRESENT SALARY OF SUMEET BE 3X

IN FUTURE THE SALARY OF RAVI WILL BE 2X+4000

AND IN FUTURE THE SALARY OF SUMEET WILL BE 3X+4000

NEW SALARY RATIO =

(2X+4OOO) : (3X+4000) =40 :57

(2X+4OOO) (57)=(3X+4000)(40)

114X+2,28,000 =120X +1,60,000

2,28,000-1,60,000 =120X—114X

`68,000 =6X

X=`11,333.333333

SUMEET’S PRESENT SALARY(BEFORE INCREMENT)=

3X11,333.333333=`34,000

64.

A CERTAIN AMOUNT WAS DIVIDED BETWEEN ARUN AND BALAJI IN THE RATIO 4:3. IF BALAJI’S SHARE
IS `4,800,FIND THE TOTAL AMOUNT.

SOLUTION:
GIVEN THAT BALAJI IS SHARING 3 RATIO POINTS OUT OF THE TOTAL RATIO POINTS OF 7

If 4,800 =3 points

7 points = ?

(`4,800/ 3 points ) x 7 points

(4800/3) x7 =`11,200

65.

In an express train the number of passengers travelling in I class, II class and III class are in the ratio 1:2:7
and the rate for each class is in the ratio 5:4:2. If the total income from the train is `54,000, then find the
income from I class.

Solution:

Number of passengers ratio 1:2:7

Rate per passenger ratio 5:4:2

Total collection ratio 5:8:14

First class collection=

`54,000 x5/(5+8+14)

`54,000 x5/27

=`10,000

Alternative solution

First class collection is [1/10 ] x [5/11 ] = 5/110

Second class collection is [2/10]x[


66. In a shop , 3 types of products are sold which are A, B and C.The price of product A is thrice of
product B and twice of product C.

The ratio of quantity of sales of product A is ¼ of the quantity of sales of product B and 1/5 of the
quantity of sales of product C.

If the total sales value is `99,000, find the sales value from product B .

Solution:

Let the price per product A be P.

The price per product B will be P/3

The price per product C will be P/2

Let the quantity of product A be Q

The quantity of product B will be 4Q

The quantity of product C will be 5Q

The total sales value = Rs99,000

PQ + (P/3) x 4Q + (P/2) x 5Q = 99,000

PQ + 4PQ/3 + 5PQ/ 2 =99,000

6PQ + 8PQ +15PQ = 99000 X6

29PQ=594000

PQ = 594000/29

B’s SALES= 4PQ/3

B’s SALES = 4 (594000/29)3

B’s SALES = 4(594000/87)

B’s SALES = 2376000/87


B’S SALES =27310.34

ALTERNATIVE METHOD

Number of products sold ratio

The ratio of quantity of sales of product A is ¼ of the quantity of sales of product B and 1/5 of the
quantity of sales of product C.

A B C
1 4 BOTTOM
TO UP
1>1
5>?

(1/1 )X5
=5
1 5

RATIO
1 4 5

SALES QUANTITY

A B C
3 1 BOTTOM
UP
2>3
1>?
(3/2)X1
=3/2

2 1

3 1 3/2
MULTIPLY
THROUGH OUT
BY 2

6 2 3
SALES VALUE RATIO=

(SALES QUANTITY RATIO)X(SELLING PRICE RATIO)

(1:4:5) X(6:2:3)

6:8:15

B’S TOTAL SALES=

8/( 6+8+15) X99000

8/29 X99000

27310.34

67.

1230 BASKETS OF MANGOES WERE LOADED IN THREE TRUCKS. WHEN UNLOADED, IT WAS FOUND THAT
5, 10 AND 15 BASKETS WERE ROTTEN IN THE TRUCKS RESPECTIVELY, BUT THE REMAINING BASKETS
WERE IN THE RATIO 3:4:5. HOW MANY BASKETS WERE LOADED INITIALLY IN EACH TRUCK?

SOLUTION:

IT IS GIVEN IN THE PROBLEM THAT THE NUMBER OF GOOD BASKETS AFTER UNLOADING ARE IN THE
RATIO 3:4:5.

THEREFORE , WE CAN FORM AN EQUATION HERE

(NUMBER OF BASKETS LOADED) – ( NUMBER OF BASKETS ROTTEN)

=( NUMBER OF GOOD BASKETS)

WE CAN DERIVE ANOTHER EQUATION

( NUMBER OF GOOD BASKETS) + ( NUMBER OF BASKETS ROTTEN)


= (NUMBER OF BASKETS LOADED)

TOTAL NUMBER OF GOOD BASKETS=

(TOTAL NUMBER OF BASKETS LOADED)—(TOTAL NUMBER OF BOXES ROTTEN)

TOTAL NUMBER OF GOOD BASKETS=

(1230)—( 5+10+15)

TOTAL NUMBER OF GOOD BASKETS=

(1230)—(30)

TOTAL NUMBER OF GOOD BASKETS=

1200

IT IS GIVEN IN THE PROBLEM THAT THE NUMBER OF GOOD BASKETS AFTER UNLOADING

ARE IN THE RATIO 3:4:5.

NUMBER OF GOOD BASKETS IN THE FIRST TRUCK= 1200 X3/(3+4+5)

=1200 X3/12=300

NUMBER OF GOOD BASKETS IN THE SECOND TRUCK= 1200 X4/(3+4+5)

=1200 X4/12=400

NUMBER OF GOOD BASKETS IN THE THIRD TRUCK= 1200 X5/(3+4+5)

=1200 X5/12=500
(NUMBER OF BASKETS LOADED)=

( NUMBER OF GOOD BASKETS) + ( NUMBER OF BASKETS ROTTEN)

I TRUCK = 300 BASKETS + 5 BASKETS =305 BASKETS

II TRUCK = 400 BASKETS + 10 BASKETS =410 BASKETS

III TRUCK = 500 BASKETS + 15 BASKETS =515 BASKETS

TOTAL :1200 BASKETS +30BASKETS =1230 BASKETS

68.

][[\

14 . 2160 STUDENTS APPEARED FOR CPT AT MADURAI, TRICHY AND SALEM EXAMINATION CENTRES
IN TOTAL.

TOTAL SUCCESSFUL CANDIDATES WERE IN THE RATIO 6:3:1

100 CANDIDATES, 50 CANDIDATES AND 10 CANDIDATES HAVE FAILED IN THESE CENTRES RESPECTIVELY.

FIND THE NUMBER OF EXAM TAKERS IN EACH CENTRE.

SOLUTION :

PASSED TOTAL CANDIDATES =2160—(100+50+10) =2160—160=2000

MADURAI PASSED CANDIDATES =2000 X [6/10]=1200

TRICHY PASSED CANDIDATES =2000X [3/10] =600

SALEM PASSED CANDIDATES =2000 X[1/10] = 200

APPEARED CANDIDATES—FAILED CANDIDATES = PASSED CANDIDATES

MADURAI APPEARED CANDIDATES—100 =1200


MADURAI APPEARED CANDIDATES =1200 + 100 =1300

TRICHY APPEARED CANDIDATES—50 =600

TRICHY APPEARED CANDIDATES =600 + 50 =650

SALEM APPEARED CANDIDATES—10 =200

SALEM APPEARED CANDIDATES =200 + 10 =210

PROPORTION:

(2 : 6) ::( 5:15 )

2 IS TO 6 AS 5 TO 15

2:6 IS IN PROPORTION TO 5:15

A:B :: C:D

A IS TO B AS C TO D

A/B = C/D

69 . FIND THE FOURTH PROPORTIONAL TO 2A , 3B AND 4C

SOLUTION

2A:3B :: 4C :X

PRODUCT OF MEANS = PRODUCT OF EXTREMES


2AX =12BC

X= 12 BC/2A

X=6BC/A

70 . Find the third proportional to 5 and 10

SOLUTION :

LET 5 ,10 ,C BE IN PROPORTION .

THEN 5:10 ::10:C

THEN 5:10 =10:C

5C =100

C= 100/5

C=20

71. FIND THE MEAN PROPORTIONAL BETWEEN 25 AND 81.

SOLUTION

MEAN PROPORTIONAL MEANS THE VALUE THAT LIES IN BETWEEN.

LET 25 ,B AND 81 BE IN PROPORTION

THEN

25:B :: B:81

THEN

25:B=B:81

(25X81)=(BXB)

2025 =B2

√ 2025=B
B=45

72.FIND TWO NUMBERS SUCH THAT THEIR MEAN PROPORTIONAL IS 24 AND THEIR THIRD
PROPORTIONAL IS 1536.
SOLUTION :

LET THE NUMBERS BE X AND Y.

IT IS GIVEN IN THE PROBLEM THAT THE MEAN PROPORTIONAL OF X AND Y =24

THEN WE CAN WRITE THAT

X , 24 AND Y ARE IN PROPORTION

THEN

X:24 ::24:Y

THEN

X:24 =24:Y

XY =(24X24)

XY =576-------------------EQUATION (1)

FURTHER IT IS GIVEN IN THE PROBLEM THAT THE THIRD PROPORTIONAL TO X,Y IS 1536.

WE CAN WRITE THAT

X,Y,1536 ARE IN PROPORTON

THEN

X:Y :: Y:1536

THEN

1536 X =Y 2

WE CAN WRITE X= Y 2 / 1536------------------EQUATION (2)

LET US SUBSTITUTE X= Y 2 / 1536 IN EQUATION -----(1)

XY =576-------------------EQUATION (1)

(Y 2 / 1536)Y =576

(Y 3 / 1536) =576

Y 3 =576 X 1536

Y 3 =884736
Y =√
3
884736

Y =√
3
27 X 32768

Y =√
3
27 X 8 X 4096

Y =√
3
27 X 8 X 8 X 512

Y =√
3
27 X 8 X 8 X 8 X 64

Y =√
3
27 X 8 X 8 X 8 X 8 X 8

Y= 3X2X2X2X2X2

Y=96

LET US SUBSTITUTE Y=96 IN THE EQUATION (1)

XY =576-------------------EQUATION (1)

X x 96= 576

X=576/96

X=6.

PROOF:

6,96,1536 ARE IN PROPORTION

6:96 =96:1536

6X1536 = 96 X96

9216 =9216

WHAT IS INVERTENDO ?

ANSWER:

IF A:B =C:D

THEN

B:A=D:C
WHAT IS THE INVERSE RATIO OF A:B ?

ANSWER

B:A

EXAMPLE

2:3 = 20: 30

THEN

3:2=30:20

EXAMPLE

100:20 = 5:1

100/20 =5/1

5=5

20:100 =1:5

20/100 =1/5

0.20 =0.20

WHAT IS THE MEANING OF ALTERNENDO ?

ANSWER:

LET X:Y = M:N

THEN

X:M = Y:N

PROOF

10:5 = 30:15

THEN

10:30 = 5:15
THIS RELATIONSHIP IS KNOWN AS ALTERNENDO.

73. A PROPORTION IS GIVEN TO YOU.

D: H :: G:T

REWRITE THE ABOVE PROPORTION USING THE PRINCIPLE OF ALTERNENDO

ANSWER:

ORIGINAL PROPORTION AS GIVEN IN THE PROBLEM IS

D: H :: G:T

NOW WE HAVE TO APPLY THE PRINCIPLE OF ALTERNENDO. THEN THE PROPORTION

D: H :: G:T

BECOMES

D: G :: H:T

WHAT IS THE MEANING OF COMPONENDO ?

ANSWER:

LET US ASSUME A PROPORTION

X:Y :: M:N

OR

X:Y = M:N

COMPONENDO MEANS ADDITION.

IF WE APPLY THE PRINCIPLE OF COMPONENDO THEN

X :Y = M :N

BECOMES
(X+Y):Y =(M+N):N

PROOF:

ORIGINAL PROPORTION

10:5 = 30:15

IF WE APPLY THE PRINCIPLE OF COMPONENDO THEN

10:5 = 30:15

BECOMES

(10+5):5 =(30+15):15

15: 5 = 45 :15

74. REWRITE THE FOLLOWING PROPORTION APPLYING THE PRINCIPLE OF COMPONENDO

10:3 :: 50:15

ANSWER:

THE ORIGINAL PROPORTION IS

10:3 :: 50:15

OR

10:3 = 50:15.

NOW LET US APPLY THE PRINCIPLE OF COMPONENDO

(10+3) : 3 =(50+15) :15

13 : 3 =65 :15

WHAT IS THE MEANING OF DIVIDENDO?

ANSWER:

LET US ASSUME A PROPORTION


X:Y :: M:N

OR

X:Y = M:N

IF WE APPLY THE PRINCIPLE OF DIVIDENDO, THE PROPORTION BECOMES

(X—Y):Y = ( M—N):N

PROOF:

ORIGINAL PROPORTION

10:5 = 30:15

IF WE APPLY THE PRINCIPLE OF DIVIDENDO THEN

(10—5):5 = (30—15):15

5 :5 = 15 : 15

75. REWRITE THE FOLLOWING PROPORTION APPLYING THE PRINCIPLE OF DIVIDENDO

10:3 :: 50:15

ANSWER:

THE ORIGINAL PROPORTION IS

10:3 :: 50:15

OR

10:3 = 50:15.

NOW LET US APPLY THE PRINCIPLE OF DIVIDENDO

(10-3) : 3 =(50-15) :15

7 : 3 = 35 :15
WHAT IS THE MEANING OF “COMPONENDO AND DIVIDENDO” ?

ANSWER:

LET US ASSUME A PROPORTION

X:Y :: M:N

OR

X:Y = M:N

(X+Y) : (X—Y) =(M+N) :(M—N)

PROOF:

ORIGINAL PROPORTION

10:5 = 30:15

IF WE APPLY THE PRINCIPLE OF “COMPONENDO AND DIVIDENDO” THEN

(10+5):(10—5)=( 30+15) : (30—15)

15 :5 =45 : 15

76. REWRITE THE PROPORTION

8:3::24:9 APPLYING THE PRINCIPLE OF “COMPONENDO AND DIVIDENDO”

SOLUTION:

THE ORIGINAL PROPORTION IS

8:3 :: 24:9

OR

8:3 = 24:9

NOW LET US APPLY THE PRINCIPLE OF “COMPONENDO AND DIVIDENDO”

(8+3) :(8—3) =(24+9):(24—9)


11 :5 =33 :15

---------------------------------copy-----------------------------------------------------------------

77.

IF

(4A+7B)/(4C+7D)=(4A—7B)/(4C—7D)

WHAT IS A/B ?

SOLUTION:

IT IS GIVEN IN THE PROBLEM THAT

(4A+7B)/(4C+7D)=(4A—7B)/(4C—7D)

OR

(4A+7B) : (4C+7D) = (4A—7B):(4C—7D)

LET US APPLY THE PRINCIPLE OF ALTERNENDO. THEN

(4A+7B) : (4C+7D) = (4A—7B):(4C—7D)

WILL BECOME

(4A+7B) : (4A—7B) = (4C+7D) : (4C—7D)

LET US APPLY THE PRINCIPLE OF “COMPONENDO AND DIVIDENDO”

(4A+7B) : (4A—7B) = (4C+7D) : (4C—7D)

WILL BECOME

(4A+7B+4A—7B):(4A+7B—4A+7B) = (4C+7D+4C—7D):(4C+7D—4C+7D)

THAT IS

(8A):(14B) = (8C):(14D)
THAT IS

(8A)/(14B) =(8C)/(14D)

A/B =C/D

--------------copy------------------

78.

SOLVE :√ A + X +√ A− X 1

______________ = ___

√ A + X --√ A− X B

SOLUTION

IT IS GIVEN IN THE PROBLEM THAT

√ A + X +√ A− X 1

______________ = ___

√ A + X --√ A− X B

First , let us consider LHS and let us write down the LHS

√ A + X +√ A− X
______________

√ A + X --√ A− X
LET US APPLY THE PRINCIPLE OF “COMPONENDO AND DIVIDENDO” ON LHS

(EXPLANATION

M:N CAN BE WRITTEN AS M/N

M:N CAN BE WRITTEN AS

(M+N) :(M—N) {COMPONENDO AND DIVIDENDO})

√ A + X +√ A− X +√ A + X --√ A− X
__________________________

√ A + X +√ A− X -- √ A + X +√ A− X
NOW LHS BECOMES

2√ A + X

--------------

2√ A− X

THAT IS

√ A +X
-------------- =LHS.

√ A− X

SQUARE LHS

A+ X

-------------- =NEW LHS.

A--X

WE DID TWO OPERATIONS IN LHS


 COMPONENDO AND
DIVIDENDO OPERATION
 SQUARING OPERATION
OPERATED LHS IS
A+ X

-----------------------NEW LHS

A--X

LET US WRITE DOWN THE RHS

---

LET US APPLY THE OPERATIONS AS WE DID FOR LHS IN THE SAME SEQUENCE…

LET US APPLY THE PRINCIPLE OF “COMPONENDO AND DIVIDENDO” IN THE RHS

1+B

-----

1--B

LET US SQUARE THE ABOVE

(1+B)2

--------

(1—B)2
{ PLEASE RECALL THAT

(M+N)2 = M2 +2MN+N2

(M--N)2 = M2 --2MN+N2

1 +2B+B2
--------------------------------------NEW RHS

1—2B+B2

NOW LET US EQUATE THE NEW LHS AND NEW RHS

A+ X

------------- =

A--X

1+B2 +2B
------------------

1+B2—2B

LET US APPLY THE PRINCIPLE OF “COMPONENDO AND DIVIDENDO”

A+ X +( A—X)

----------------- =

A+X—( A—X)
1+B2 +2B +( 1+B2—2B)

----------------------------------------------

1+B2 +2B -( 1+B2 — 2B)


THAT IS

A+ X + A—X

----------------- =

A+X— A+X

1+B2 +2B + 1+B2—2B

------------------------------------------

1+B2 +2B - 1--B2 +2B

A+ A
----------------- =

X+X

1+B2 + 1+B2

------------------------------------------

2B +2B

2A

----------------- =

2X

2+2B2

------------------------------------------

4B

----------------- =
X

2(1+B2 )

------------------------------------------

4B

----------------- =

(1+B2 )
------------------------------------------

2B

----------------- =

A
2B

------------------------------------------

(1+B2 )

2AB

------------------------------------------

(1+B2 )

--------------copy------------------------

79.

IF X= 4AB / (A+B),

SIMPLIFY

{(X+2A)/(X—2A)} + (X+2B)/(X—2B)
SOLUTION:

IT IS GIVEN IN THE PROBLEM THAT

X=4AB/(A+B)

DIVIDING BOTH SIDES BY 2A

X/2A= 4AB/(A+B)/2A

X/2A= 4AB/(A+B)(2A)

X/2A= 2AB/(A+B)(A)

X/2A= 2B/(A+B)

APPLY “ COMPONENDO AND DIVIDENDO” ON THE ABOVE

X/2A= 2B/(A+B)

(X+2A)/(X—2A)=

2B+(A+B)/{2B-(A+B)}

(X+2A)/(X—2A)=2B+A+B/(2B-A-B)

(X+2A)/(X—2A)=3B+A/(B-A)---(1)

IT IS GIVEN IN THE PROBLEM THAT

X=4AB/(A+B)

DIVIDING BOTH SIDES BY 2B

X/2B= 4AB/(A+B)/2B

X/2B= 4AB/(A+B)(2B)

X/2B= 2AB/(A+B)(B)

X/2B= 2A/(A+B)

APPLY “ COMPONENDO AND DIVIDENDO” ON THE ABOVE


X/2B= 2A/(A+B)

(X+2B)/(X—2B)=2A+(A+B)/{2A-(A+B)}

(X+2B)/(X—2B)=2A+A+B/(2A-A-B)

(X+2B)/(X—2B)=3A+B/(A-B)---(2)

WE HAVE TO SIMPLIFY

{(X+2A)/(X—2A)} +

(X+2B)/(X—2B)

THAT IS

RHS OF EQN(1) + RHS OF EQN(2)

3B+A/(B-A) + 3A+B/(A-B)

{ EXPLANATION

8/4 CAN BE WRITTEN AS (–8/--4)

SIMILARLY

(3A+B)/(A-B) CAN BE WRITTEN AS --(3A+B)/ --(A-B)

--(3A+B)/ --(A-B) CAN BE WRITTEN AS

--(3A+B) /(--A+B)

--(3A+B) /(--A+B) CAN BE WRITTEN AS

--(3A+B) /(B--A)

******************************

NOW

3B+A/(B-A) + 3A+B/(A-B)

3B+A/(B-A) -- ( 3A+B)/(B-A)

COMMON DENOMINATOR IS

(B—A)

{ 3B+A—(3A+B) } /(B—A)
(3B+A—3A--B) /(B—A)

(2B—2A) /(B—A)

2(B—A) /(B—A)

2.

----------------copy----------------------------------------

80.

SOLVE THE EQUATION

{ √ 3 X + √ ( 2 X +1 ) }

{ √ 3 X -- √ ( 2 X +1 ) }

=5

SOLUTION :

LET US WRITE DOWN THE LHS

{ √ 3 X + √ ( 2 X +1 ) }

{ √ 3 X -- √ ( 2 X +1 ) }

LET US APPLY THE PRINCIPLE OF COMPONENDO AND DIVIDENDO

{ √ 3 X + √ ( 2 X +1 ) }

{ √ 3 X _ √ ( 2 X +1 ) }

{ √ 3 X + √ ( 2 X +1 ) }

--

{ √ 3 X _ √ ( 2 X +1 ) }
2 √3 X / 2 √ (2 X +1 )

√3 X / √ ( 2 X +1 )
LET US SQUARE

3X / (2X+1) ----------EQN 1

LET US WRITE DOWN RHS

5 CAN BE WRITTEN AS 5/1

LET US APPLY THE PRINCIPLE OF “COMPONENDO AND DIVIDENDO” FOR 5/1


(5+1) /(5—1)
6/4
3/2
NOW LET US SQUARE IT
9/4---------EQN2

WE CAN EQUATE 1 & 2

3X / (2X+1) = 9/4

12X =18X + 9

12X—18X=9

--6X=9

X=9/(--6)

X= --1.50

----------------copy------------------------------------

81.

SOLVE THE EQUATION


{ √ X +5 + √ ( X−16 ) }

/ { √ X +5 -- √ ( X−16 ) }

=7/3

Solution

X=20

82.

A SUM OF `53 IS DIVIDED AMONG MrA , Mr B AND Mr C IN SUCH A WAY THAT Mr A GETS `7 MORE
THAN WHAT Mr B GETS AND Mr B GETS `8 MORE THAN WHAT Mr C GETS. FIND THE RATIO OF THEIR
SHARES.

SOLUTION

TOTAL AMOUNT WHICH HAS TO BE DIVIDED AMONG Mr A , MrB AND Mr C=`53.

A + B + C =`53

(C+8+7) + (C+8) + C=53

(C+15) + (C+8) + C=53

3C+23=53

3C=53—23

3C=30

C=10

THEREFORE

A=10+15=25

B=10+8=18

C= 10

TOTAL=53

RATIO OF SHARE = 25:18:10


ALSO THE AMOUNT OF SHARE

Mr A=`25

Mr B=`18

Mr C=`10

83. A SUM OF MONEY IS TO BE DISTRIBUTED AMONG A,B,C AND D IN THE PROPORTION 5:2:4:3. IF C
GETS `1,000 MORE THAN D, WHAT IS B’S SHARE?

SOLUTION

LET THE AMOUNTS BE

5X ,2X, 4X AND 3X

4X—3X=1000

X=1000.

THE SHARES ARE

`5,000 , `2,000 , `4,000 AND `3,000

84.

IF X= 2AB/(A+B)

SIMPLIFY

(X+A) / (X—A) + (X+B) /(X—B)

SOLUTION

GIVEN THAT

X= 2AB/(A+B)

LET US DIVIDE BOTH SIDES BY A

WE GET

X/A = 2AB/(A+B)/A

X/A = 2AB/(A+B)(A)
X/A = 2B/(A+B)

LET US APPLY “COMPONENDO AND DIVIDENDO”

(X+A) / (X—A) =2B+(A+B) /2B—(A+B)

(X+A) / (X—A) =2B+A+B /2B—A--B)

(X+A) / (X—A) =3B+A /(B—A)----------EQN 1

GIVEN THAT

X= 2AB/(A+B)

LET US DIVIDE BOTH SIDES BY B

WE GET

X/B = 2AB/(A+B)/B

X/B = 2AB/(A+B)(B)

X/B = 2A/(A+B)

LET US APPLY “COMPONENDO AND DIVIDENDO”

(X+B) / (X—B) =2A+(A+B) /2A—(A+B)

(X+B) / (X—B) =2A+A+B /2A—A--B)

(X+B) / (X—B) =3A+B /(A—B)----------EQN 2

WE WANT TO

SIMPLIFY

(X+A) / (X—A) + (X+B) /(X—B)

NOW LET US EQUATE RHS OF EQN 1 AND RHS OF EQN 2

{3B+A /(B—A) } +{ 3A+B /(A—B)}

LET US MULTIPLY { 3A+B /(A—B)} WITH -1 IN THE NUMERATOR AS WELL AS THE DENOMONATOR SO
THAT THE VALUE REMAINS UNALTERED BUT THE DENOMINATOR BECOMES (B—A)
{(-1)( 3A+B) /(--1) (A—B)}

{(-- 3A--B) /(--A+B)}

{--( 3A+B) /(B--A)}

{3B+A /(B—A) } +{--( 3A+B) /(B--A)}

{3B+A /(B—A) } --{( 3A+B) /(B--A)}

{3B+A--( 3A+B)} /(B—A)

{3B+A-- 3A-B)} /(B—A)

{2B-- 2A} /(B—A)

2{B-- A} /(B—A)

2 ANSWER

85.

IN WHAT PROPORTION MUST A NUMBER BE DIVIDED SO THAT ¼ OF FIRST PART AND 2/3 RD OF THE
SECOND PART ARE TOGETHER EQUAL TO ½ OF THE ORIGINAL NUMBER ?

SOLUTION :

1000

0.25X + 0.666666666666Y = 0.50(1000)

0.25X +0.6666666666666Y= 500

X+Y=1000

Y=1000—X

0.25X +0.6666666666666[1000—X]= 500

0.25X +666.666666666--0.66666666X= 500

166.66666666666666=0.41666666X

X=400
SOLUTION:

LET 100 BE THE ORIGINAL NUMBER

LET X: Y BE THE RATIO IN WHICH 100 IS DIVIDED

{EXPLANATION

IF 100 IS DIVIDED FOR EXAMPLE IN 5:7 RATIO,

FIRST PART

= 5/(5+7) X100

SECOND PART

= 7/(5+7) X100

SIMILARLY

X/(X+Y) X100 BE THE FIRST PART

Y/(X+Y) X100 BE THE SECOND PART

¼ { (X/(X+Y) X100} +

2/3{ Y/(X+Y) X100}

THAT IS

25 X/(X+Y) + 66.66666 Y/(X+Y) ==50

25X+66.6666666Y =50X+50Y

16.66666666Y =25X

25X=16.66666666Y

X/Y =16.66666666/25

X:Y =16.66666666:25

X:Y=2:3

0.25{ X/(X+Y) X100 } + 0.6666666{ Y/(X+Y) X100}


50

PROOF

(100 X2/5 X1/4) +(100X3/5 X2/3)

10 +40=50 .

86.

A FRACTION BEARS THE SAME RATIO TO 1/27 AS 3/7 DOES TO 5/9.

FIND THE FRACTION.

SOLUTION:

LET THE UNKNOWN FRACTION TO BE FOUND BE

X: (1/27) :: 3/7 : 5/9

OR

X: (1/27) = 3/7 : 5/9

PRODUCT OF EXTREMES = PRODUCT OF MEANS

( X ) (5/9) = (1/27) (3/7)

5X/9 =3/(27X7)

5X =(3X9)/(27X7)

5X =1/7

X= =1/35
87.

IF P/Q=7

WHAT IS (P+Q)/(P—Q) ?

SOLUTION :

P/Q = 7 / 1

THEN P=7Q

SO ,

(7Q+Q) / (7Q—Q)

8Q / 6Q

4Q / 3Q

=4/3

88.

WHICH OF THE FOLLOWING NUMBERS ARE NOT IN PROPORTION ?

(A) 6,8,5,7
(B) 7,3,14,6
(C) 18,27,12,18
(D) 8,6,12,9

SOLUTION

LET US CHECK WHETHER 6,8,5,7 ARE IN PROPRTION

6:8 ::5:7

OR

6:8=5:7

WE KNOW THAT THE PRODUCT OF MEANS=THE PRODUCT OF EXTREMES

(8X5)=(6X7)

40 IS NOT EQUAL TO42

HENCE , 6,8,5,7 ARE NOT IN PROPRTION


LET US CHECK WHETHER 7,3,14,6 ARE IN PROPORTION

7:3 ::14:6

OR

7:3=14:6

WE KNOW THAT THE PRODUCT OF MEANS=THE PRODUCT OF EXTREMES

(3X14)=(7X6)

42 =42 TRUE

HENCE 7,3,14,6 ARE IN PROPORTION

LET US CHECK WHETHER 18,27,12,18 ARE IN PROPORTION

18:27 ::12:18

OR

18:27=12:18

WE KNOW THAT THE PRODUCT OF MEANS=THE PRODUCT OF EXTREMES

(27X12)=(18X18)

324 =324 TRUE

HENCE 18,27,12,18 ARE IN PROPORTION

LET US CHECK WHETHER 8,6,12,9 ARE IN PROPORTION

8:6 ::12:9

OR

8:6=12:9

WE KNOW THAT THE PRODUCT OF MEANS=THE PRODUCT OF EXTREMES

(6X12)=(8X9)

72 =72 TRUE

HENCE 6,12,8,9 ARE IN PROPORTION


89.

FIND THE TWO NUMBERS SUCH THAT THE MEAN PROPORTIONAL BETWEEN THEM IS 18 AND THE THIRD
PROPORTIONAL OF THEM IS 144.

SOLUTION :

X ,18,Y---------PATTERN

X,Y,144-------PATTERN

SOLUTION:

LET THE TWO NUMBERS WHICH WE HAVE TO FIND OUT BE X & Y.

IT IS GIVEN IN THE PROBLEM THAT THE MEAN PROPORTIONAL BETWEEN X & Y =18

X,18 & Y ARE IN PROPORTION

X:18=18:Y

USING THE PRINCIPLE THAT THE PRODUCT OF MEANS = THE PRODUCT OF EXTREMES, WE CAN FORM
THE FOLLOWING EQUATION

XY=18 X18

XY=324------------EQUATION (1)

IT IS FURTHER GIVEN IN THE PROBLEM THAT THE THIRD PROPORTIONAL OF X & Y =144

X, Y & 144 ARE IN PROPORTION

X:Y=Y:144

USING THE PRINCIPLE THAT THE PRODUCT OF MEANS = THE PRODUCT OF EXTREMES, WE CAN FORM
THE FOLLOWING EQUATION

Y2=144 X---------------EQUATION (2)

LET US REWRITE THE EQUATIONS 1 AND 2

XY=324------------EQUATION (1)
Y2=144 X---------------EQUATION (2)

FROM EQUATION (1)

X=324/Y

NOW LET US SUBSTITUTE X=324/Y IN THE EQUATION (2)

Y2=144 X

Y2=144 (324 /Y)

Y3=144 (324 )

Y3=46656

Y3=8X5832

Y3=8X8X729

Y3=8X8X27X27

Y3=8X8X27X27

TAKING CUBE ROOT ON BOTH SIDES

Y=2X2X3X3

Y=36

LET US SUBSTITUTE Y=36 IN THE EQUATION (1)

XY=324------------EQUATION (1)

X(36) =324

36X=324

X=324/36

X=9

THEREFORE, THE TWO NUMBERS ARE

9 AND 36

PROOF:
X,18,Y

9,18,36

9:18::18:36

9:18=18:36

(9X36)=(18X18)

324=324

.PROVED

X,Y,144

9,36,144

9:36::36:144

9:36=36:144

(9X144)=(36X36)

1296 =1296

PROVED.

90. The third proportional between ( a2-b2) and (a+b)2 is

(A) a+b
-----
a-b
(B) a-b
-----
a+b
(C) (a-b)2
-----
a+b

(D) (a+b)3
-----
a-b

SOLUTION :
The third proportional between ( a2-b2) and (a+b)2

( a2-b2),(a+b)2 , X

(a+b)(a—b) , (a+b)(a+b) ,X

(a+b)(a—b) : (a+b)(a+b) :: (a+b)(a+b):X

(a+b)(a—b) : (a+b)(a+b) = (a+b)(a+b):X

We know that the product of means =product of extremes

(a+b)(a—b)(X) = (a+b)(a+b) (a+b)(a+b)

(a+b)(a—b)(X) = (a+b)(a+b) (a+b)(a+b)

X= (a+b)(a+b) (a+b)(a+b)

------------------------------

(a+b)(a—b)

X= (a+b) (a+b)(a+b)

------------------------------

(a—b)

X= (a+b)3

------------------------------

(a—b)

91.

IF `3,910 IS TO BE DIVIDED INTO THREE PARTS,PROPORTIONAL TO ½ : 2/3 : ¾, FIND THE SHARES .

SOLUTION

THE PROPORTION IS ½ : 2/3 : ¾

12 IS THE LEAST COMMON MULTIPLE FOR THE THREE DENOMINATORS


LET US MULTIPLY THROUGHOUT BY 12

12/2 : 24/3 : 36/4

WHICH IS

6:8:9

FIRST PART = 6/(6+8+9) X 3,910

=(6/23) X3,910=`1,020

SECOND PART = 8/(6+8+9) X 3,910

=(8/23) X3,910=`1,360

THIRD PART = 9/(6+8+9) X 3,910

=(9/23) X3,910=`1,530

PROOF: FIRST PART +SECOND PART +THIRD PART=`3,910

`1,020 + `1,360 + `1,530=`3,910

92. M/s RAMCO CEMENTS LIMITED has 3 machines. The horse power of 2 nd machine is thrice like 1st
machine and half of 3rd machine.

1st machine worked for 12 hours, 2nd machine worked for 5 hours and the 3rd machine had worked for 2
hours. The total electricity bill came for `1,500. Find --the electricity cost of each machine.

Solution :

12 [ H] + 5[3H] + 2[6H] =12H+15H+12H= 39H

39H=1500

H=1500/39=38.4615384615
12H= 12X38.46153846=461.538461=1ST

15H= 15X38.46153846=576.9230769=2ND

12H= 12X38.46153846=461.538461=3RD

SOLUTION:

STEP 1:

LET US SIMPLIFY THE RATIO OF HORSE POWER OF EACH MACHINE.

A B C
1 3 6

STEP 2:

LET US WRITE DOWN THE WORKING HOURS OF EACH MACHINE

A B C
12 HOURS 5 HOURS 2 HOURS

STEP 3

LET US WRITE DOWN THE COMPOSITION OF STEP 1 AND STEP 2

A B C
1HPX12HOURS 3HPX5HOURS 6HPX2HOURS
=12 =15 =12

STEP 4

LET US WRITE DOWN THE COST APPORTIONMENT

A B C
12/(12+15+12) 15/(12+15+12) 12/(12+15+12)
X`1,500 X`1,500 X`1,500
=12/39 X =15/39 X =12/39 X
`1,500 `1,500 `1,500
`461.54 `576.92 `461.54

STEP 5

LET US FIND OUT THE ELECTRICITY COST PER WORKING HOUR FOR EACH MACHINE.

A B C
ELECTRICITY ` ` `
COST 461.54 576.92 461.54
WORKING 12 5 2
HOURS
ELECTRICITY ` ` `
COST 38.46 115.38 230.77
PER HOUR

PROOF:
LET US ASSUME THAT THE COST OF SINGLE UNIT OF ELECTRICITY IS `4.

93. If

X 2
---- = ---
Y 3

Simplify

3x+5y
--------
5x+6y

Solution:

3x+5y
-------- can be written as
5x+6y

3(x/y) + 5(y/y)
----------------- we have divided nr and dr by y.
5(x/y) + 6(y/y)
So, value remains the same.

3(x/y) + 5
--------------
5(x/y) + 6

We know that x/y =2/3. Let us substitute


x/y=2/3 in
3(x/y) + 5
--------------
5(x/y) + 6

3(2/3) + 5
--------------
5(2/3) + 6

2 + 5
--------------
10/3 + 6

7
--------------
(10 + 18)/3

21
--------------
28

3
--------------
4

ALTERNATIVE SOLUTION

IT IS GIVEN IN THE PROBLEM THAT


X 2
---- = ---
Y 3

WHEN WE CROSS MULTIPLY


3X=2Y
&
X= 2Y/3

LET US SUBSTITUTE 3X=2Y AND X= 2Y/3


IN

3x+5y
--------
5x+6y

2y+5y
--------
5(2y/3)+6y

7y
--------
10y/3+6y

7
--------
10/3+6

7
--------
(10+18)/3

21
--------
28

¾.

94.
The ratio of the number of boys to the number of girls in a school of 720 students is 3:5. If 18
new girls are admitted in the school, find how many new boys have to be admitted so that
the ratio of the number of boys to the number of girls may change to 2:3.

SOLUTION:

270:450
[270+X] :468 :: 2:3

It is given in the problem that now there are 720 total students in the school.
Now , the number of boys in the school
Is 3/(3+5) x720 =(3/8) x720=270

Now , the number of girls in the school


Is 5/(3+5) x720 =(5/8) x720=450

In future , 18 new girls join the school.


In future, the number of girls in the school will be 450 +18=468.

In future the ratio of boys to girls is 2:3

2:3:: X:468
2:3=x:468
Using the principle that the product of means= product of extremes
(2 x 468) =3x
936 =3x
X=936/3
X=312

Therefore, in future , the number of boys=312

how many new boys have to be admitted


312—270=42.
95.
If
X:Y =2:3
Simplify

6X--Y
--------
3X+2Y

Solution:

6X--Y
-------- can be written as
3X+2Y

6(X/Y) --(Y/Y)
------------------------ we have divided nr and dr by y.
3(X/Y) + 2(Y/Y)

So, value remains the same.


6(X/Y) --1
--------------
3(X/Y) + 2

We know that X/Y =2/3. Let us substitute


X/Y=2/3 in
6(X/Y) --1
--------------
3(X/Y) + 2

6(2/3) --1
--------------
3(2/3) + 2

4 --1
--------------
2 + 2

3
---
4

96.
THE RATIO BETWEEN THE SPEEDS OF TWO TRAINS IS 7:8. IF THE SECOND TRAIN COVERS 400
KMS IN 5 HOURS, WHAT IS THE SPEED OF THE FIRST TRAIN?

SOLUTION

7:8 = X:80
7X80=8X
560=8X
X=70

A ` ` `
ELECTRICITY 38.46 115.38 230.77
COST
PER HOUR
B Cost per unit `4 `4 `4
of electricity
C Units of 9.615 28.825 57.6925
Electricity
Consumed
Per hour
A/B
PROPORTION 1 3 6

HOME WORK PROBLEMS IN RATIOS AND PROPORTIONS:

1. IF FOUR NUMBERS ½, 1/3,1/5,1/X ARE PROPORTIONAL THEN X IS


(A) 6/5 (B) 5/6 (C) 15/2 (D) NONE OF THESE

SOLUTION :

½ : 1/3 :: 1/5 : 1/X

PRODUCT OF MEANS = PRODUCT OF EXTREAMES

½ X 1/X = 1/3 X 1/5

1/2X = 1/15

X = 15 / 2 .

2 THE MEAN PROPORTIONAL BETWEEN


12X2 AND 27Y2 IS
(A)18XY (B) 81 XY (C) 8XY (D) NONE OF THESE

12 X2 , [M ], 27 Y2

M 27 Y2
------ = --------
12 X2 M

M2 = 324 X2 Y2
M=18XY

3.THE NUMBER WHICH HAS THE SAME RATIO TO 26 THAT 6 HAS TO 13 IS

(A)11 (B) 10 (C) 21 (D) NONE OF THESE


X:26::6:13

4.IF A= B/2=C/5 THEN A:B:C IS

(A)3:5:2 (B) 2:5:3 (C) 2:3:5 (D) NONE OF THESE


A = B/2

LET A =250

250 = B/2

250 X2 =B =500

A=250

B= 500

500/2 = C/5

250 =C/5

C= 1250

250 : 500:1250

1:2:5

5. TWO NUMBERS ARE IN THE RATIO 3:4. IF 6 BE ADDED TO EACH TERM OF THE RATIO ,THEN THE NEW
RATIO WILL BE 4:5. THE NUMBERS ARE

(A)14,20 (B)17,19 (C) 18,24 (D) NONE OF THESE

6.DIVISION OF `7,500 INTO THREE PARTS IN THE RATIO 4:5:6 IS

(A)(2000,2500,3000) (B) (2500,2500,2500) (C) (3500,2500,1500) (D) NONE OF THESE

7. A MIXTURE CONTAINS MILK AND WATER IN THE RATIO 5:1. ON ADDING 5 LITRES OF WATER, THE
RATIO OF MILK AND WATER BECOMES 5:2. THE QUANTITY OF MILK IN THE ORIGINAL MIXTURE IS

(A)16 LITRES (B) 25 LITRES (C) 22.75 LITRES (D) 32.5 LITRES

5X : [1X+5]

5X : (X+5):: 5:2

8.THE SUM OF THE AGES OF 3 PERSONS IS 150 YEARS. 10 YEARS AGO, THEIR AGES WERE IN THE RATIO
7:8:9. THEIR PRESENT AGES ARE
(A)(45,50,55) (B)(40,60,50) (C)(35,45,70) (D) NONE OF THESE

9. IF AN ALLOY CONTAINS COPPER AND SILVER IN THE RATIO 3:7,THEN THE PERCENTAGE OF SILVER IN
THE ALLOY IS

(A)90% (B) 70% (C) 60% (D) NONE OF THESE

10. WHAT SHOULD BE ADDED TO EACH TERM 3,15,38 AND 134 SO THAT THEY SHOULD BECOME
PROPORTIONATE TO EACH OTHER ?

(A)1 (B) 2 (C)3 (D) 4

A:B :: C:D
3:15 :: 38:134
4:16::39:135
5:17::40:136

Exercise:

Divide ` 71,540 among Tarika, Sarika, Boomika and Nika. Tarika and Sarika share in 2:5 ratio.

Sarika and Boomika share in 4:7 ratio. Boomika and Nika share in 9:13 ratio.

Give the share amount of each party.

Solution

TARIKA SARIKA BOOMIKA NIKA


2 5

4 7

9 13

BOOMIKA : NIKA :: 9:13

===============================================================
KANMANI HAS 4 LEGAL HEIRS. MANI, GANI,SONY AND GONY.

KANMANI HAS DECIDED TO DISTRIBUTE HIS 1000 GOLD COINS AMONG THE FOUR LEGAL HEIRS
SUCH THAT THE RATIO BETWEEN MANI AND GANI IS 7:3.

THE RATIO BETWEEN GANI AND SONY IS 4:1

THE RATIO BETWEEN SONY AND GONY IS 5:2

================================================================================

M 7

---- = -----

G 3

M = [7/3] GANI----------------------[1]

G 4

---- = ------*

S 1

G 4S

---- = -----

1 1

G = 4S
S= [1/4] GANI --------------------------------[2]

S 5

--- = ------

GONY 2

S= [5/2] GONY

[1/4] GANI = [5/2] GONY

GONY = [1/4] GANI X[2/5]

GONY= [1/10] GANI---------------------------------[3]

7/3 : 1/1 : ¼ : /1/10

Answer : 140 : 60 : 15 : 6

1. 1000 x 140 / 221 = 633.48


2. 1000 x 60 / 221 = 271.49
3. 1000 x 15 / 221 =67.87
4. 1000 x 6 / 221 = 27.15

PROOF : 633.48 + 271.49 + 67.87 + 27.15 = 1000

MEENA DECIDED TO DISTRIBUTE HER WEALTH Rs80,000 TO VEENA, GUNA, SENA AND JANA IN
SUCH A WAY THAT THE FOLLOWING ARE THE SHARING RATIOS..

VEENA AND GUNA IN 3:2 RATIO

GUNA AND SENA IN 2:3 RATIO AND

SENA AND JANA IN 6:1 RATIO

QUESTION: FIND THE SHARE OF EACH PARTY.


Solution :

-------------

veena guna sena jana


3 2
3 2 3
6 4 6 1
28235.29 18823.52 28235.29 4705.88

Exercise[1]

The monthly salaries of two persons are in the ratio 3:5. If each receives an increase of `20 in the
monthly salary the ratio is altered to 13:21.

Questions

1. Find the old salary of two persons.


2. Find the new salary of two persons.

SOLUTION :

 LET THE OLD MONTHLY SALARY OF THE PERSONS BE 3X AND 5X


 THE INCREMENT IN THE MONTHLY SALARY = Rs 20 PER PERSON.
 THE NEW MONTHLY SALARY OF THE PERSONS WILL BE 3X +20
AND 5X+20
 IT IS MENTIONED IN THE PROBLEM THAT THE NEW MONTHLY
SALARY OF THE PERSONS ARE IN THE RATIO OF 13:21.

 SO (3X +20) : (5X+20):: 13:21

USING THE PRINCIPLE OF PRODUCT OF EXTREMES = PRODUCT OF MEANS

WE CAN GET

(3X +20) x 21 = (5X+20) x 13

63X + 420 = 65 X +260

420—260 = 65X—63X
160 = 2X

80=X

OLD SALARY OF FIRST PERSON = 3X= 3X80 =Rs240

OLD SALARY OF SECOND PERSON = 5X= 5X80 =Rs400

NEW SALARY OF FIRST PERSON = 3X+20= [3X80]+20 =Rs260

OLD SALARY OF SECOND PERSON = 5X+20=[ 5X80] +20=Rs420

EXERCISE [2]

The ratio of the prices of two houses was 16 :23. Two years later, the price of the first house increased
by 10% and the price of the second house increased by `4,77,000. The ratio of the prices is 11:20 after
the price rise. Find the original old price of both houses.

SOLUTION :

 LET THE OLD PRICE OF THE FIRST HOUSE BE 16X


 THE INCREMENT IN THE PRICE OF THE FIRST HOUSE = 1.6X.
 LET THE OLD PRICE OF THE SECOND HOUSE BE 23X
 THE INCREMENT IN THE PRICE OF THE SECOND HOUSE =
Rs4,77,000
 THE NEW PRICE OF THE FIRST HOUSE WILL BE 16X +1.6X=17.6X
 THE NEW PRICE OF THE SECOND HOUSE WILL BE 23X +4,77,000
 IT IS MENTIONED IN THE PROBLEM THAT THE NEW PRICE OF
THE HOUSES ARE IN THE RATIO OF 11:20.

 SO (17.6X) : (23X+4,77,000):: 11:20

USING THE PRINCIPLE OF PRODUCT OF EXTREMES = PRODUCT OF MEANS

WE CAN GET

(17.6X) x 20 = (23X+4,77,000) x 11
352X = 253 X +52,47,000

352X—253X = 52,47,000

99X = 52,47,000

X=53,000

ORIGINAL OLD PRICE OF FIRST HOUSE= 16X= 16x53,000=8,48,000

ORIGINAL OLD PRICE OF SECOND HOUSE= 23X= 23x53,000=12,19000

--------------------------------------------------------------------------------------

12.10.2015

10 :20 :: 4:8

10 :20 :: 50:100

10 50

----- = ------

20 100

1000=1000

EXRCISE [1]

FIND X IF

25: X : : 40:160

SOLUTION :

25 40

---- = --------

X 160

40 X= 4000
X= 100

EXERCSE[2]

FIND Y IF

Y:10 :: 5: 50

SOLUTION

Y 5

---- = ----

10 50

50Y= 50

Y= 1

----------------------------------------------------------------------

IN A PROPORTION, WE CAN SAY THAT

PRODUCT OF EXTREMES = PRODUCT OF MEANS

5 : 15 :: 25:75

EXTREMES ARE 5 AND 75

MEANS ARE 15 AND 25

PRODUCT OF EXTREMES = PRODUCT OF MEANS

5X75 = 15X25

375 =375.

EXERCISE

IF

X: Y :: 10:50

[A] 50 Y= 10X
[B] 50X= 50Y

[C] 50X=10Y

[D] 10X= 10Y

-----------------------------------------

Ram’s income is 2.5 times of Kumar’s income.

Ram’s income=2.5x

Kumar’s income= 1x

Ram’s expense is equal to Kumar’s expense.

Ram’s expense=1 y

Kumar’s expense= 1y

 We know that

Income—Expense = Saving

It is mentioned in the problem that

Ram’s savings = 3 times of Kumar’s savings

[2.5x—y]—[1x—1y]

2.5x—y --x +y =270

1.5x= 270

X= 270/1.5 =180

So

Ram’s income =2.5 X 180 =450

Kumar’s income =1 X 180 =180

[450—y] :[180—y] = 3:1


450—y= 540—3y

450—540 =--3y+y

--90 =--2y

Y= 45

-------------------------------------------

Ram’s income=450

Kumar’s income=180

Ram’s expense=45

Kumar’s expense=45

Ram’s saving= 405

Kumar’s saving=135

----------------------------------------------

Arun’s income=3x

Kumar’s income= 1x

Arun’s expense is equal to Balaji’s expense.

Arun’s expense=1 y

Balaji’s expense= 1y

 We know that

Income—Expense = Saving

It is mentioned in the problem that

Balaji’s savings = 4 times of Arun’s savings

[5x—y]—[3x—1y]

5x—y --3x +y =250

2x= 250
X= 125

So

Arun’s income =3 X 125 =375

Balaji’s income =5 X 125 =625

[375—y] :[625—y] = 1:4

1500—4y= 625—y

1500—625 =3y

875 =3y

Y= 291.66666666

-------------------------------------------

13.10.2015

EXERCISE:

If ` 8520 be divided among Viji, Suba and Rama so that suba gets twice like Viji and Rama gets thrice
like Viji. Find the share of each.

Solution :`

V + S+R=8520

S= 2V

R=3V

V + 2V+3V=8520

6V=8520

V=8520/6=1420

S= 2V=2X1420=2840

R=3V=3X1420=4260
Exercise :

Sheela, Neela and Urmila are the daughters of Kala.

Kala has 4,000 gold coins with her.Neela has to get twice like sheela but half of Urmila.

You can break coins if needed. DIVIDE COINS among Neela, Sheela and Urmila.

SOLUTION :

S+N+U=4000

N=2S

N=0.5U

SINCE N=2S

S=N/2=0.5N

SINCE N=0.5U

U=N/0.5=2N

S+N+U=4000

0.5N+ N+ 2N =4000

3.5N=4000

N= 4000/3.5 =1142.85714285 U=2N=2X1142.85714285=

2285.7142857

S=0.5N=0.5X 1142.85714285=

571.428571425

Exercise:

Divide ` 71,540 among Tarika, Sarika, Boomika and Nika. Tarika and Sarika share in 2:5 ratio.

Sarika and Boomika share in 4:7 ratio. Boomika and Nika share in 9:13 ratio.

Give the share amount of each party.


SOLUTION :

T +S +B+N =71540

T 2

--- = ----

S 5

T = 2S/5 =0.40 S

S 4

--- = ----

B 7

S= 4B/7=0.57145857142 B

B 9

--- = ----

N 13

B= 9N/13 =0.6923076923N

T+S+B+N

T = 0.40 S

S= 0.57145857142 B

B= 0.6923076923N

-------------------------------------------------------------------------

R :K =4:1
K:M=5:2

M:D=7:4

TOTAL=4,00,000

ASSUME R AS 400 POINTS

THEN K= 100 POINTS

IF K= 100 THEN M= [100/5] X2= 40

IF M= 40 THEN D= [40/7] X4= 22.8571428571

R:K:M:D

400 : 100: 40: 22.8571428571

============================================================

A:B =2:1

B:C=4:5

C:D=1:2

A:B:C:D =?

SOLUTION

LET US ASSIME A AS 80 POINTS

B=40 POINTS

C= 50 POINTS

D=100 POINTS

80:40:50:100

8:4:5:10
EXERCISE :

The ratio of the prices of two houses was 16 :23. Two years later, the price of the first house increased
by 10% and the price of the second house increased by `4,77,000. The ratio of the NEW prices is 11:20
after the price rise. Find the original old price of both houses.

SOLUTION :

16X : 23X- PRICE

TWO YEARS LATER

[16X X110/100] : [23X+477000] = 11:20

[16X X110/100] [20]= [23X+477000] [11]

352X = 253X +5247000

99X=5247000

X=53000

OLD PRICE

16X : 23X- PRICE

16X53000 , 23 X53000

848000, 1219000

-------------------------------------------------------------------------------------------------------

Exercise:

If 2:7 = 21:X

and

X:Y = 1:2

find Y.

2:7 = 21:X

2X= 147
X=73.5

73.5 :Y =1:2

Y=147

Exercise:

If a:b:c = 1:2:3 and b:c:d = 4:6:5 find a:d

LET a= 720 points

b=1440

c=2160

d= [2160/6] x5 =1800

720:1440:2160:1800

720:1800

2:5

26.8.16

PRABHAT 1 MEANS MUNISH 2/3

PRABHAT AND MUNISH SHARE FRUITS IN 1 : 2/3 RATIO

1 : 2/3

MULTIPLY THROUGH BY 3

3:2

1
½

1/12

1 ½ ¼ 1/12

12 6 3 1

=================================

45 : 5

9 :1

======================

45:10

9:2

===================================

170 CMS : 165 CMS :: 160 CMS : N

=======================================================

170 : 165 :: 160 : N

160

------ X 165 =155


170

===================================================================

Rs 200 : Rs X :: Rs40 : Rs90

18,000 = 40X

X= 18,000/40

X= Rs450

================================================================

(2/3) Y : 900 :: 10 : 30

FIND Y

===============================================================

(2Y/3) * 30 =900*10

60Y =9000

----

20Y=9000

Y= 9000/20

Y=450.

========================================

IF Z : 15 :: 1 : 250
WHAT IS Z ?

SOLUTION :

250 Z = 15

Z= 15/250

Z= 3/50

28.8.2016

50 LITRES

MILK 45 LITRES WATER 5 LITRES

50+ X LITRES LITRES

MILK [45+X] LITRES WATER 5 LITRES

[45+X] : 5 :: 11: 1
45+X =55

X=55—45=10

50+ 10 LITRES LITRES

MILK [45+10] LITRES WATER 5 LITRES

60 LITRES

MILK 55 LITRES WATER 5 LITRES

HOME WORK:

AN ALLOY CONTAINS GOLD AND COPPER IN 6:1 RATIO. THE ALLOY WEIGHED 35 KGS. THE
ORNAMENT MAKER PLANS TO KEEP THE GOLD AND COPPER RATIO AS 10:3. HOW MANY
KILOGRAMS OF copper HAS TO BE MELTED AND BLENDED NEWLY WITH THE PRESENT ALLOY ?

Answer :

Gold =30 kgs

Copper=5kgs

30kg

---- x3 = 9 kg

10

Answer : 4 kg.

===========================

10:40 = 100:400
10: 100 = 40:400

=====================================================

EXERCISE

PRABHAT AND MUNISH SHARED A PIZZA CIRCLE IN 3:1 RATIO. MUNISH ATE

6 PIZZA PIECES LESS THAN PRABHAT.

HOW MANY PIZZA CUTS WERE THERE IN THE CIRCLE PIZZA ?

ANSWER :

3X : 1X

3X—1X =6

2X= 6

X=3

SO PIECES EATEN BY PRABHAT = 3x3= 9 PIECES

SO PIECES EATEN BY MUNISH = 3X1 =3 PIECES

SO THE CONSUMPTION RATIO = 9:3 =3:1

SO TOTAL PIECES IN THE CIRCLE PIZZA= 12

Exercise :

Monthly income of Anbu and Babu are in the ratio 5:6 and their

expenses are in on the ratio 4:5.

If each save `200 per month, find their monthly income.

Solution :
Let the income of Anbu be 5X

Then what is the income of Babu ? 6X

Let the expense of Anbu be 4Y

Then what is the expense of Babu ? 5Y

Remember that

INCOME—EXPENSE = SAVING

WHAT IS ANBU’S SAVING PER MONTH ?

ANBU’S INCOME—ANBU’S EXPENSE

5X—4Y =200

WHAT IS BABU’S SAVING PER MONTH ?

BABU’S INCOME—BABU’S EXPENSE

6X—5Y= 200

SO NOW WE HAVE TWO EQUATIONS BEFORE US…

5X—4Y= 200 ----------- EQUATION [1]

6X—5Y= 200------------EQUATION [2]

--------------------------------------------------------------

LET US MULTIPLY EQUATION [1] WITH 6

WE GET
30X—24Y=1200 ------------ EQUATION 3

LET US MULTIPLY EQUATION [2] WITH 5

WE GET

30X—25Y= 1000---------------EQUATION 4

LET US DEDUCT EQUTION 4 FROM EQUTION 3

1 Y = 200

Y= 200

LET US PUT Y= 200 IN ANY ONE EQUTION

5X—4Y= 200 ----------- EQUATION [1]

5X—4[200]= 200 ----------- EQUATION [1]

5X—800 =200

5X= 1000

X= 200

SO THE INCOMES OF ANBU AND BABU ARE

5X: 6X

5[200] : 6[200]

1000 :1200

SO THE EXPENSES OF ANBU AND BABU ARE

4Y:5Y

4[200] : 5[200]
800 :1000

SO THE SAVINGS OF ANBU AND BABU ARE

1000—800=200

1200—1000=200

HOME WORK :

RAM AND KUMAR HAVE EARNED IN 3:4 RATIO

RAM AND KUMAR SPENT IN 5:2 RATIO

RAM SAVED Rs20 AND KUMAR SAVED Rs50.

FIND

 RAM’S EARNINS
 KUMAR’S EARNINGS
 RAM’S EXPENSES
 KUMAR’S EXPENSES.

SOLUTION HINT:

3X—5Y=20

4X—2Y=50

-------------------------------------------------------------------------------------

30.8.16

EXERCISE:

If ` 8520 be divided among Viji, Suba and Rama so that suba gets

twice like Viji and Rama gets thrice like Viji. Find the share of each.
Solution :

V = share of Viji=V

2V = share of Suba= S

3V= share of Rama=R

V+ S+R= 8520

V+2V+3V= 8520

6V= 8520

V= 8520/6 =1420

VIJI GETS Rs1420

SUBA GETSRs2840

RAMA GETSRs4260

---------------------------------------------------------------------------------------------

Exercise :

Sheela, Neela and Urmila are the daughters of Kala.

Kala has 4,000 gold coins with her. Neela has to get

twice like sheela but half of Urmila.

You can break coins if needed. DIVIDE COINS among Neela,

Sheela and Urmila.

SOLUTION :

S= SHEELA

N= NEELA= 2S

URMILA= 2N =4S
SHEELA + NEELA + URMILA=4000

S + 2S+4S =4000

7S= 4000

S= [4000/7]

N= 2[4000/7] =8000/7

U=4[4000/7] =16000/7

ANSWER

S=571.42

N=1142.85

U=2285.71

-------------------------------------------------------------------------

Exercise:

Divide ` 71,540 among Tarika, Sarika, Boomika and Nika.

Tarika and Sarika share in 2:5 ratio.

Sarika and Boomika share in 4:7 ratio.

Boomika and Nika share in 9:13 ratio.

Give the share amount of each party.

SOLUTION :

TARIKA =T

SARIKA =S= T x [ 5/2] = 5T/2


BOOMIKA= [5T/2]X [7/4] = 35T /8

NIKA = [35T/8]X [13/9] =455 T/72

T + 5T/2 + 35T /8 + 455 T/72 = 71540

MULTIPLY WITH 72 BOTH SIDES

72T + 180 T + 315T + 455T= 5150880

1022 T =5150880

T= 5150880/1022 =5040

TARIKA GETS Rs5040

SARIKA GETS 5T/2 = [5X5040]/2 = Rs 12,600

BOOMIKA GETS 35T /8= [35 X5040] /8=22050

NIKA GETS 455 T/72= [455 X5040]/72 =31850

EXERCISE :

The ratio of the prices of two houses was 16 :23.

Two years later, the price of the first house

increased by 10% and the price of the second

house increased by `4,77,000. The ratio of the

prices is 11:20 after the price increase. Find the original old price

of both houses.

Solution :
Let the old price of the two houses be 16 X and 23 X

Now after 2 years, the price of the first house had increased by 10%. Therefore, the new price of
the first house = 16 X x[110/100] = 17.6 X

Now after 2 years, the price of the second house had increased by Rs4,77,000.

Therefore, the new price of the second house = 23X +4,77,000

The new price ratio of the two houses is 11 :20

So we can express the following proportion

17.6 X : [ 23X+477000] = 11 : 20

Product of extremes = Product of means

17.6 X x 20 = [ 23X + 477000] x 11

352 X = 253 X + 52 47 000

352 X –253 X =52 47 000

99X = 5247 000

X = 5247 000/99 =53000

Old price of first house =16 X = 16 x53000 = Rs848000

Old price of second house = 23 X= 23x53000= Rs1219000

New price of first house = 17.6 X =17.6 x 53000 = Rs932800

New price of second house = 23 X + 477000 = 1219000+477000= Rs1696000


------------------------------------------------------------------------------------------

Exercise:

If a:b = 2:3,

b:c = 4:5 ,

c:d = 1:2 find a:b:c:d

solution :

a= a

b= [3a/2]

c= [3a/2] x[5/4] =15a/8

d= [15a/8] x [2/1] =30a/8

a : 3a/2 : 15a/8 : 30a/8

8 : 12a :15a :30a

8 :12:15:30

-----------------------------------------

Exercise:

Find the mean proportion of 9 and 4.

9,X,4

9:X :: X :4

36 = X2

X= 6
So the mean proportion of 9 and 4 = 6

-------------------------------------------------------------------------------

Exercise :

What is the third proportion of 4 and 6 ?

Solution :

4,6,X

4:6 : : 6:X

4X = 36

X= 9

---------------------------------------------------------------------------------------

Exercise:

Find the fourth proportional to 10 , 25 and 40

Solution :

10 :25 :: 40: X

10X =1000

X= 100

--------------------------------------------------------------------------------------

If a:b:c = 1:2:3 and b:c:d = 4:6:5 find a:d

a b c d
1 2 3
4 4 6 5
---x 1= 2
2
If a:b = 5:3 , b:c = 7:4 find a:b:c.

a b c
5 3
7 7 4
---- x5 = 35/3
3
35 21 12
==================================================

400 kg

Xkg x Rs10 + (400—x) Rs14

-------------------------------------------- = Rs11

400

10x + 5600—14x =4400

--4x =--1200

4x=1200

X=300

300:100

3:1

==========================================
NEW INDICES

8 can be written as 23

Here 2 is the base and

3 is the index

The another name for index is power.

The another name for index is exponential.

*******************************************

23 can be expressed as 2 x 2 x 2

2 x 2 x 2 can be expressed as 21 x 21 x 21

21 x 21 x 21 can be expressed as 2(1+1+1) = 23

*********************************************

Exercise : index 6 to the base 2=?

Answer =26= 64

Exercise : index 2 to the base 6=?

Answer =62=36.

***************************************************

----

26

Can be expressed as 2—6


***************************************************************************

50 =?
50 =1

510

---- =?

58

510

---- can be expressed as

58

510 x 5—8

510 x 5—8 can be expressed as

510—8
=

52 =25
Exercise

Express 2—4 without using (--) sign

Answer :

----

24

*********************************************************

Express

----

23
without using
denominator.

Answer :

[4] [2—3]

=22 X 2—3

=2 [2—3]

=2--1

*********************************************************

√4 can be expressed as [4] ½

*********************************************************

√3 125 can be expressed as [125]1/3

*****************************************************************************************

Express 4 3/2

Solution :

4 3/2 can be expressed as 4 1 x 41/2

4 1 x 41/2 can be expressed as 4 x2 =8

*********************************************************

Express 8 2/3
solution :

8 2/3 can be expressed as

[8 2]1/3

[8 2]1/3 can be expressed as

641/3

641/3 can be written as √3 64


√3 64 = 4

***********************************************************
Simplify

2n + 2 n—1
---------------------

2n+1 – 2n
solution :

2n + 2 n—1
---------------------

2n+1 – 2n

2n + (2 n )(2—1)

-------------------

(2n )(21)—2n
=

2n [1+(2—1)]

-------------------

(2n )[(21)—1]

[1+2—1]

-------------------

[21—1]

[1+2—1]

-------------------

[2—1]
=
[1+2—1]

-------------------

[1]
=

[1+2—1]

1+ 1

------

2
=

3/2

--------------------------------copy-----------------------------------------------------------------------------------------------------------------

Exercise:

Simplify

√ 45 -- √ 180 +√ 125

Solution

√ 45 -- √ 180 +√ 125
Can be written as

√ 9 x 5 --√ 9 x 20 + √ 25 x 5

√ 9 x 5 --√ 9 x 4 x 5 + √ 25 x 5
Can be written as

3[√ 5 ¿ -- (3)(2) [√ 5 ¿ + 5[√ 5 ¿


3[√ 5 ¿ -- (3)(2) [√ 5 ¿ + 5[√ 5 ¿

Can be written as

[√ 5 ¿ { 3—6+5}

[√ 5 ¿ { 3—6+5} can be written as

[√ 5 ¿ { 2}

===============================================================

(23)2 can be expressed as 2(3x2) =26


===================================================================================================

Simplify :

{ (5—1 )(72) / (52 )(7--4) } 7/2


{ (5—2 )(73)/ (53 )(7--5 }—5/2

(5—1 )(72) (5—2 )(73)

{------------ }7/2 X { --------------} —5/2

(52 )(7--4) (53 )(7--5 )

Solution:

{ (5—1 )(72) (5--2 )(74)}7/2 { (5—2 )(73) (5--3 )(75 }—5/2


=

{ (5—3 )(76) }7/2 { (5—5)(78) }—5/2


=

{ (5—21/2 )(742/2) } { (525/2)(7—40/2) }


=

{ (5(-21/2)+(25/2) )(7(42/2)—(40/2)) }

{ (52 )(71 }
=25x7

=175

=============================================================

Example :

If

ax=b,

by=c and

cz=a

what is [xyz] ?

solution :

ax=b,

PUTTING b=a x in the place of b

by=c CAN BE WRITTEN AS [ax]y =c

[a x]y =c CAN BE WRITTEN AS [a]xy

PUTTING c= [a] xy in the place of c

cz=a CAN BE WRITTEN AS [axy]z =a

[a xy]z =a CAN BE WRITTEN AS [axyz] =a

[a xyz] =a CAN BE WRITTEN AS [axyz] =a1

So xyz=1

==================================================================

Simplify

{ xb/xc }a { xc/xa }b { xa/xb }c


solution :
{ xb/xc }a x { xc/xa }b x { xa/xb }c
=

{ xb x--c }a { xc x--a }b { xa x--b }c

{ xab x--ac } { xbc x--ab } { xac x--bc }

{ xab—ac+ bc –ab+ac --bc }

{ x0 }

Simplify:

[x]2/7 [y]1/2 [x]2/5[y]—2/3 [x] –9/7[y]—1/3[y]1/2[x]3/5

Answer : 1

Simplify:

[x]4/7 [x]3/5 [x]3/7 [x]3/8 [y]--5/5[x]--8/5 [y]2[x]--3/8

Answer :y
-------------------------------------------------------------------------------------------------------------------------------------------------------------

8.6.2016

EXERCISE [1]

[243/32]—4/5 IS

[A] 18/16 [B] 16/81 [C] 4/9 [D]9/4


EXERCISE [2]

(10)150 / (10)146 IS

[A] 1000 [B] 10000 [C] 100000 [D]10 6

EXERCISE [3]

(10)150 / (10)146 IS

[A] 1000 [B] 10000 [C] 100000 [D]10 6

=================================================

3.9.16

82/3

[23]2/3

2 [3X2/3]

22

4
Venn diagram and logarithms

EXAMPLE 1

IN A CLASS THERE ARE 11 STUDENTS. 8 STUDENTS KNOW FRENCH AND 5 STUDENTS KNOW
GERMAN. 2 STUDENTS KNOW BOTH THE LANGUAGES.

 HOW MANY STUDENTS KNOW FRENCH ALONE ? 6


 HOW MANY STUDENTS KNOW GERMAN ALONE ? 3

EXAMPLE 2

IN A CLASS THERE ARE 11 STUDENTS. 3 STUDENTS KNOW FRENCH ALONE AND 5 STUDENTS
KNOW GERMAN ALONE .

 HOW MANY STUDENTS KNOW FRENCH AS WELL AS GERMAN ?

ANSWER :

 HOW MANY STUDENTS KNOW FRENCH AS WELL AS GERMAN ? 3

EXAMPLE 3

IN A CLASS THERE ARE 20 STUDENTS. 8 STUDENTS KNOW FRENCH ALONE AND 6 STUDENTS
KNOW GERMAN ALONE . 2 STUDENTS KNOW NEITHER FRENCH NOR GERMAN.

 HOW MANY STUDENTS KNOW FRENCH AS WELL AS GERMAN ? 4

EXAMPLE 4

IN A CLASS THERE ARE 25 STUDENTS. 7 STUDENTS KNOW FRENCH ALONE AND 6 STUDENTS
KNOW NEITHER FRENCH NOR GERMAN . 2 STUDENTS KNOW BOTH FRENCH AND GERMAN.

 HOW MANY STUDENTS KNOW GERMAN ALONE ? 10


EXAMPLE 5

IN A CLASS THERE ARE 37 STUDENTS. 21 STUDENTS KNOW FRENCH.14 STUDENTS KNOW


GERMAN .16 STUDENTS KNOW GREEK. 3 KNOW ALL 3 LANGUAGES.

6 KNOW NONE OF THE LANGUAGES.

6 KNOW FRENCH AS WELL AS GERMAN BUT NOT GREEK.

1 KNOWS GERMAN AND GREEK BUT NOT FRENCH .7 KNOWS FRENCH AND GREEK BUT NOT
GERMAN.

 HOW MANY KNOW FRENCH ALONE ? 5


 HOW MANY KNOW GERMAN ALONE ?4
 HOW MANY KNOW GREEK ALONE ?5

EXAMPLE 6

IN A CLASS THERE ARE 60 STUDENTS. 42 STUDENTS KNOW FRENCH.

12 KNOW FRENCH AS WELL AS GERMAN BUT NOT GREEK.

6 KNOW FRENCH, GERMAN AS WELL AS GREEK.

10 KNOW FRENCH ALONE .

 HOW MANY KNOW FRENCH AND GREEK BUT NOT GERMAN ? 14

EXAMPLE 7

IN A CLASS 56 STUDENTS KNOW GERMAN .

4 KNOW GERMAN AS WELL AS GREEK BUT NOT FRENCH .

12 KNOW FRENCH, GERMAN AS WELL AS GREEK.

16 KNOW GERMAN ALONE

 HOW MANY KNOW GERMAN AND FRENCH BUT NOT GREEK ? 24

EXAMPLE 8.

If X has 32 elements, Y has 42 elements and

X U Y has 62 elements, the number of elements in X∩Y is ?


[a] 74 [b] 12 [c] 15 [d] 40

EXAMPLE 9.

A place has a total population of 50,000 people. Out of it 28,000 people read Hindu news
paper and 20,000 people read Dhinamalar newspaper while 4,000 people read both
newspapers. The number of persons not reading Hindu and Dhinamalar both is

[a] 3,000 [b] 2,500 [c] 6,000 [d] 5,000

EXAMPLE 10.

In a group of 20 persons, 8 like tea but not coffee and 13 like tea. The number of persons
who like coffee but not tea is ?

[a] 1 [b] 6 [c] 7 [d] 33

EXAMPLE 11.

At a function of 100 people ,there are 29 Indian women and 23 Indian men. Out of these Indian
people , 4 are engineers and 24 are either men or engineers . There are no foreign engineers.
The number of women engineers attending the function is ?

[a] 1 [b] 2 [c] 4 [d] 48

EXAMPLE 12.

Out of 2,000 students of a hostel,48% preferred coffee, 54% preferred tea and 64% preferred
milk,28%preferred coffee and tea, 32% preferred tea and milk and 30% preferred coffee and
milk. Only 6% did not prefer all the three. The number preferring all the three is

[a] 300 [b] 360 [c] 380 [d] 1,000

A+B+D+E=960-A+D=400 , A+B=360, D—B=40

B+C+E+F=1080----B+C=440B=160

D+E+F+G=1280-- D+G=640

B+E=560

E+F=640

D+E=600

H=120
A+B+C+D+E+F+G+H=2000

C=280

EXAMPLE 13.

Out of a group of 20 students of an institute, 10 like Maths, 9 like Physics and 7 like Chemistry.4
like Maths and Physics but no one likes both Maths and Chemistry.

How many like Chemistry and Physics? 2

How many like only Physics ? 3

SOLUTION :

A+B+D+E=10A+B=10, A+D=6,B—D=4

B+C+E+F=9, C+F=5

D+E+F+G=7-F+G=7

D+E=0

B+E=4

A+B+C+D+E+F+G+H=20

EXAMPLE 14.

Out of a group of 20 persons , 8 like tea but not coffee and 13 like tea. How many like coffee but
not tea?

Answer :7

EXAMPLE 15.

If X has 32 elements , Y has 42 elements and X U Y has 62 elements, find the number of
elements in X ∩ Y.

Answer :12

EXAMPLE 16.
Out of 60 persons , 25 failed in paper A, 24 failed in paper B, 32 failed in paper C, 9 failed in
paper A alone , 6 failed in paper B alone, 5 failed in papers B & C and 3 failed in papers A & B.

 How many persons have failed in all the three papers ? 10


 How many persons have passed in all the three papers ? 10

Example 16 :
On a survey of 100 students, it was found that 50 used white shirt, 40 used red shirt, 30
used blue shirt, 20 used both white and red shirt, 15 used both red and blue shirt and 10
used both blue and white shirt.
 How many used all shirts ? 25

Example 17 :
Out of 1000 students, 658 failed in the aggregate, 166 failed in the aggregate and in
group 1, 434 failed in aggregate and in group 2, 372 failed in group 1, 590 failed in
group 2 and 126 failed in both the groups.

 How many students have failed in all the three ? 106

Example 18 :
Out of 1000 persons , the following were the data collected ….
X type X and Z Z type X type X type Z and Y None
alone types but not Y type
type
180 80 480 230 360 80 140
 How many buy Y grade ? 280

Example 19 :

After qualifying out of 400 professionals, 112 joined industry, 120 started practice and 160
joined as paid assistants. There were 32 who were in practice as well as industry, there were 40
in both practice and paid assistantship, there were 20 in both industry and paid assistantship.
There were 12 who did all the three. Find how many could not get any of these. 88

Example 20 :

Out of 150 students, 45 passed in Accounts, 50 passed in Maths and 30 passed in Costing, 30
passed in both Accounts and Maths, 32 passed in both Maths and Costing ,35 passed in both
Accounts and Costing, 25 passed in all the three subjects. Find the number of students who
have passed in atleast any one of the subjects.

Answer :53
WHAT IS THE MEANING OF LOGARITHM?

ANSWER:
LOGARITHM IS AN INDIRECT REPRESENTATION OF A NUMBER. SAY FOR EXAMPLE FOR
THE PURPOSE OF REPRESENTING THE NUMBER 10 WE USE 1. HERE 1 MEANS 10.

SO WE CAN SAY THAT THE LOGARITHMIC VALUE OF 10 IS 1.

HOW SHOULD IT BE REPRESENTED ?

LOG 10 = 1
10

HOW TO PRONOUNCE THE ABOVE ?

LOGARITHMIC VALUE OF 10 TO THE BASE 10 IS EQUAL TO 1.

SAY FOR ANOTHER EXAMPLE FOR THE PURPOSE OF REPRESENTING THE NUMBER 100
WE USE 2. HERE 2 MEANS 100.

SO WE CAN SAY THAT THE LOGARITHMIC VALUE OF 100 IS 2.

HOW SHOULD IT BE REPRESENTED ?

LOG 100 = 2
10

HOW TO PRONOUNCE THE ABOVE ?

LOGARITHMIC VALUE OF 100 TO THE BASE 10 IS EQUAL TO 2.

SAY FOR ANOTHER EXAMPLE FOR THE PURPOSE OF REPRESENTING THE NUMBER 1000
WE USE 3. HERE 3 MEANS 1000.

SO WE CAN SAY THAT THE LOGARITHMIC VALUE OF 1000 IS 3.

HOW SHOULD IT BE REPRESENTED ?

LOG 1000 = 3
10

HOW TO PRONOUNCE THE ABOVE ?

LOGARITHMIC VALUE OF 1000 TO THE BASE 10 IS EQUAL TO 3.


THE NATURAL LOGARITHMIC BASE IS 10 .UNLESS OTHERWISE STATED IS TO BE TAKEN
AS 10.

QUESTION: EXPRESS THE LOGARITHMIC VALUE OF10,000.

ANSWER :

LOG 10000 =4
10

=====================================

101 =10 IS AN EXPONENTIAL EXPRESSION

LOG 10 = 1 IS A LOGARITHMIC EXPRESSION


10
=====================================
102 =100 IS AN EXPONENTIAL EXPRESSION

LOG 100 = 2 IS A LOGARITHMIC EXPRESSION


10
=====================================
103 =1000 IS AN EXPONENTIAL EXPRESSION

LOG 1000 = 3 IS A LOGARITHMIC EXPRESSION


10
=====================================

THE NATURAL LOGARITHMIC BASE =10.


HOWEVER, WE CAN HAVE ANY NUMBER AS PER OUR CONVENIENCE AS A LOGARITHMIC
BASE.

SAY FOR EXAMPLE , WE CAN EXPRESS THE NUMBER 125 INDIRECTLY BY WAY OF
HAVING THE BASE AS 5

THEN WE CAN SAY THAT

LOG 125 =3
5

EXPONENTIAL EXPRESSION

53 =125.

QUESTION :
EXPRESS THE NUMBER 625 LOGARITHMICALLY BY WAY TAKING 5 AS BASE.
ANSWER :
LOG 625 =4
5

QUESTION

LOG 243 =?
3

ANSWER =5

QUESTION

LOG 1331 =?
11

ANSWER =3

====================================

ADDITION RULE OF LOGARITHM

LOG 1000 CAN BE WRITTEN AS


LOG[100X10]

LOG[100X10] CAN BE WRITTEN AS


LOG 100 + LOG 10

LOG 100 + LOG 10 = 2 +1=3

SO THE ADDITION RULE OF LOGARITHM IS

LOG [A x B] = LOG A + LOGB


=====================================

SUBTRACTION RULE OF LOGARITHM

LOG 1000 CAN BE WRITTEN AS


LOG[10000/10]

LOG[10000/10] CAN BE WRITTEN AS


LOG 10000 -- LOG 10

LOG 10000 -- LOG 10 = 4 --1=3

SO THE SUBTRACTION RULE OF LOGARITHM IS

LOG[A/B] = LOG A -- LOGB


=====================================
EXPONENTIAL RULE OF LOGARITHM

LOG 10000 CAN BE WRITTEN AS LOG 1002.

LOG 1002 CAN BE WRITTEN AS 2 [LOG 100]

2 [LOG 100] CAN BE WRITTEN AS


2X2=4

SO EXPONENTIAL RULE OF LOGARITHM


=
LOG[ XA] = A[ LOG X]
=====================================

LOG M = LOG M x LOG B


A B A

LOG 625 = 4
5

LOG 625 = LOG 625 X LOG 25


5 25 5

4 =2 X 2
SO PROVED

LOG 625 = 2
25

LOG 625 X LOG 5


5 25
4 X ½ =2

SO PROVED.

---------------------------------------------------------
LOG 625 = 1
625

LOG 625 X LOG 25


25 625

2 X ½ =1
SO PROVED

LOG 125 =3
5
LOG 125 X LOG 125
125 5
1 X 3=3
SO PROVED.

LOG 10000 =4
10

LOG 10000 X LOG 100


100 10
2 X 2=4
SO PROVED.
====================================

CPT QUESTION ON LOGARITHM

LOG 85 = LOG 85 x LOG W


2 4 V

FIND W AND V

SOLUTION :
W=4
V=2

CPT QUESTION ON LOGARITHM

LOG 85 = LOG W x LOG 2


3 2 V
FIND W AND V
SOLUTION :
W=85
V=3

CPT QUESTION ON LOGARITHM

LOG Y = LOG 94 x LOG 4


3 4 3

FIND Y

SOLUTION :

Y=94

CPT QUESTION ON LOGARITHM

[LOG 7 ] = [ LOG X ] x [ LOG 2]


Y 2 Y
FIND X
SOLUTION
X=7
===================================
LOG 1=?
10

LOG 1=X
10

10X =1

X=0
===================================== LOG 1=?
170

LOG 1=X
170

170X =1
X=0
=====================================
CONVERT THE EXPRESSION 34 =81 INTO LOGARITHMIC EXPRESSION
SOLUTION
LOG 81 = 4
3
=====================================
CONVERT THE EXPRESSION 6251/4 =5 INTO LOGARITHMIC EXPRESSION
SOLUTION
LOG 5 = ¼
625

CONVERT THE EXPRESSION 100001/2 =100 INTO LOGARITHMIC EXPRESSION


SOLUTION
LOG 100 = ½
10000
=====================================
CONVERT THE EXPRESSION 3431/3 =7 INTO LOGARITHMIC EXPRESSION
SOLUTION
LOG 7 = 1/3
343
=====================================
CPT QUESTION

LOG X =2
100

FIND X

SOLUTION =10,000
=====================================
GIVE THE EXPONENTIAL EXPRESSION FOR THE FOLLOWING LOGARITHMIC
EXPRESSION…

LOG 625 =4
5

ANSWER

54=625
=====================================
GIVE THE EXPONENTIAL EXPRESSION FOR THE FOLLOWING LOGARITHMIC
EXPRESSION…

LOG 216 =3
6
ANSWER

63=216
=====================================

SIMPLIFY
LOG [25/7]2
SOLUTION :
LOG [25/7]2 =
2 LOG [25/7]=
2{ LOG 25—LOG 7}=
2{ LOG 52 –LOG 7}=
2{2(LOG 5)—LOG 7=
4LOG5—2 LOG7
==================================
SIMPLIFY
LOG 11 LOG 11
9 3
------------ ------------- = ?
LOG 13 LOG 13
5 5

SOLUTION
LOG 11
9
------------
LOG 11
3
=
LOG 11 X LOG 3
3 9
-----------------------------
LOG 11
3
= LOG 3
9
=1/2
==================================
SIMPLIFY
LOG [A2/BC] + LOG [B2/CA] + LOG [C2/AB]
SOLUTION:
[LOGA2—LOG BC]+
[LOGB2—LOG CA]+
[LOGC2—LOG AB]
[LOGA2—(LOG B+LOG C)]+
[LOGB2—(LOG C+LOG A)]+
[LOGC2—(LOG A+LOG B)]
[LOGA2—LOG B--LOG C]+
[LOGB2—LOG C--LOG A]+
[LOGC2—LOG A--LOG B]
LOGA2—LOG B--LOG C+LOGB2—LOG C--LOG A + LOGC2—LOG A--LOG B
2LOGA—LOG B--LOG C+2LOGB—LOG C--LOG A + 2LOGC—LOG A--LOG B
2LOGA—LOG B--LOG C+2LOGB—LOG C--LOG A + 2LOGC—LOG A--LOG B =0
SOLUTION =0
SIMPLIFY
7LOG [16/15] + 5LOG [25/24] + 3LOG [81/80]
SOLUTION:
=
7LOG [16/15]
=7[ LOG16—LOG15]
=7LOG16—7LOG15
=7LOG42—7LOG[5X3]
=14LOG4—7{LOG 5+LOG3}
=14LOG4—7LOG5—7LOG3
= 14LOG22—7LOG5—7LOG3
=28LOG2—7LOG5—7LOG3
5LOG [25/24]
=5{LOG25—LOG24}
=5{LOG52—LOG(8X3)}
=5{2LOG5—[LOG8+LOG3]}
=5{2LOG5—[LOG23+LOG3]}
=5{2LOG5—[3LOG2+LOG3]}
=5{2LOG5—3LOG2—LOG3}
=10LOG5—15LOG2—5LOG3

3LOG [81/80]
=3LOG81—3LOG80
=3LOG34—3[LOG(16X5)]
=12LOG 3—3[LOG16+LOG5]
=12LOG3—3LOG16—3LOG5
=12LOG3—3LOG24—3LOG5
=12LOG3—12LOG2—3LOG5

28LOG2—7LOG5—7LOG3+10LOG5—5LOG3—15LOG2+12LOG3—12LOG2—3LOG5
=LOG 2.

=====================================
FIND X IF
LOG X + LOG X + LOG X =21/4
2 4 16
SOLUTION :
LOG X + LOG X x LOG 2 +
2 2 4

LOG X x LOG 2
2 16
= LOG X + LOG X x {1/2} +
2 2

LOG X x {1/4} = 21/4


2
LOG X { 1+ ½ + ¼ } = 21/4
2

LOG X { 7/4} = 21/4


2

LOG X = [21/4]
2 ---------
[7/ 4]

LOG X = [21]
2 ---------
[7 ]

LOG X =3
2

X= 8
=====================================
SIMPLIFY

LOG√ 27+ log 8+ log √ 1000


=LOG [3X9]1/2 + LOG 23 + LOG 10001/2
=½ LOG[3X9] + 3 LOG 2 + ½ LOG 1000
=½ LOG 3 + ½ LOG32 + 3 LOG 2 + ½ [3]
=½ LOG 3 + LOG3 +3 LOG 2+ ½ [3]
=3/2 [LOG3 ] + 3LOG 2 +3/2
=3 [ LOG3 1
------- + LOG2 + --- ]
2 2
===================================
SIMPLIFY

LOG√ 27+ log √ 8−log √ 125


------------------------------------------------
3/2(LOG 6—LOG 5)
=====================================
Exercise [1]
Express 10,000 in natural logarithmic form.
Solution :
10,000= 104
logarithmic expression of 10,000 = 4

log [10,000] =4

10
Exercise [2] Express 625 in logarithmic form to
base 5.
Solution :
625= 54
logarithmic expression of 625 = 4

log [625] =4

Exercise [3]
Express 6561 in logarithmic form to base 9.
Solution :
6561= 94
logarithmic expression of 6967 = 4

log [6561] =4

Exercise [4]
Express 5 in logarithmic form to the base 25.

Solution :

5= 251/2
Log [5] = 1/2
25

Exercise [5]
Express 3 in logarithmic form to the base 27.
Solution :
3= 271/3
Log [3] = 1/3
27

---------------------------------------------
Exercise
If Log [100] = 10
x
find x
Answer :
X10 =100
[X10]1/2 =[100]1/2
X5 =10
X = 101/5
Exercise
If Log [9] = 81
x
find x
Answer :
X81 =9
X =[9]1/81
X =[32]1/81
X =[3]2/81
Exercise
If Log [100] = 2
x
find x

Answer :
X2 =100
X =[100]1/2
X= 10

Exercise
If Log [x] = 3
7
find x

Answer :
73 =x
343=x
X= 343

Exercise
If Log [25,60,000] = x
40
find x

Answer :
40x =25,60,000
4=x
X= 4

----------------------------------------------------
Addition law of logarithm
Log[1,000] can be written as
Log[(100 x10)]
Log[(100 x10)] can be written as
Log 100 + Log 10
2 + 1 =3
We know that log 1000=3
So we can say that
Log [AB] can be written as Log A + Log B
Log[AB] = Log [A] + Log [B]
-------------------------------------------------------
Division law or quotient law of logarithm
Log[1,000] can be written as
Log[(10000 /10)]
Log[(10000 /10)] can be written as
Log 10000 -- Log 10
4 -- 1 =3
We know that log 1000=3
So we can say that
Log [A/B] can be written as Log A -- Log B
Log[A/B] = Log [A] -- Log [B]

23.10.2015

EXERCISE [1]

IF LOG 10000 = LOG X

10 100

FIND X.

4= LOG X
100
X= 1004

CHANGE OF BASE LAW.

LOG M = LOG M x LOG B


A B A
-----------------------------------------------------
EXERCISE
FIND X IF
LOG X + LOG X + LOG X =21/4
2 4 16

SOLUTION :

LOG X + LOG X + LOG X =21/4


2 4 16

1 1 1
--------- + ----------- + ----------- =21/4
LOG 2 LOG 4 LOG 16
X X X

1 1 1
--------- + ----------- + ----------- =21/4
LOG 2 LOG 22 LOG 2 4
X X X

1 1 1
--------- + ----------- + ----------- =21/4
LOG 2 2LOG 2 4LOG 2
X X X

1 1 1
--------- + ----------- + ----------- =21/4
LOG 2 2LOG 2 4LOG 2
X X X

4 +2 +1
----------------------------------------------- = 21/4
4 LOG 2
X

7 21
----------------------------------------------- = -----
4 LOG 2 4
X

1 3
----------------------------------------------- = ------
4 LOG 2 4
X

1 3
----------------------------------------------- = ------
4 LOG 2 4
X

1
----------------------------------------------- = 3
LOG 2
X

3 LOG 2 =1
X

LOG 23 =1
X
LOG 8 =1
X

X=8
-------------------------------------------------------

LOG X * LOG 4 +
4 2

LOG X +
4

LOG X * LOG 4 = 21/4


4 16

LOG X {2+1+1/2} =21/4


4
LOG X {7/2} =21/4
4
LOG X =[21/4] [2/7]
4
LOG X =[42/28]
4

LOG X =[3/2]
4

43/2 =X
4 [1/2 +1] =X
4 [1/2] * 41 =X
2 X4 =X

LOG {3.5X} =21/4


4

4[21/4] =3.5 X
421 --44 =3.5 X
------------------------------------------------------------
Previous year problems and solutions from ratios, proportions, indices and logarithms
-----------------------------------------------------------

Question [1]
Value of [a1/8 + a—1/8] [a1/8 -- a—1/8]
[a1/4 + a—1/4] [a1/2 + a—1/2]
Solution :
[a1/8]2 ---[a--1/8]2 [a1/4 + a—1/4] [a1/2 + a—1/2]
=[a2/8] ---[a--2/8] [a1/4 + a—1/4] [a1/2 + a—1/2]
=[a1/4] ---[a--1/4] [a1/4 + a—1/4] [a1/2 + a—1/2]
=[a1/4]2 ---[a--1/4]2 [a1/2 + a—1/2]
=[a2/4] ---[a--2/4] [a1/2 + a—1/2]
=[a1/2] ---[a--1/2] [a1/2 + a—1/2]
=[a1/2]2 ---[a--1/2]2
=[a2/2] ---[a--2/2]
=[a1] ---[a--1]
= a --- 1
----
a

Question [2]
Find the Value of the expression
[alog a [b] log b [c] log c [d] log d [t]]
Solution :
log [b] log [c] log [d] log [t]

a b c d
log b log c log d log t

------- x --------x -------x ---------

Log a log b log c log d

= log t

-------

Log a

= log t

= log t

= t.
proof:
for example :
log 100
10

Log 100 2
=----------- = ----
Log 10 1

Question [3]
If log x = --1/4 then x is given by
10000
Solution :

Log x = --1/4
10000

10000—1/4 =x

1
--------- =x
100001/4
1
------------=x
[104]1/4

1
------------=x
[104/4]
1
------------=x
[101]
1
------------=x
10

Question [4]
Simplification of

[Xm+3n] [X4m—9n]
-----------------------
[X6m—6n]
Solution :
=x m+3n+4m—9n
--------------------------

X6m—6n
=x 5m—6n
--------------------------

X6m—6n

=[x 5m—6n] [x --6m+6n]


=

[x 5m—6n—6m+6n]
=
x--m

Question [5]
If log [ 2a—3b] = log a—logb then a= ?
Solution :
log [ 2a—3b] = log a—log b
can be written as
log [ 2a—3b] = log [a/b]
[ 2a—3b] = [a/b]
Cross multiply
2ab—3b2 =a
2ab—a =3b2
a[2b—1] =3b2
a=3b2/ [2b—1]
Question [6]
On simplification
1
--------------- +
1+za—b+za--c

1
--------------- +
1+zb--c+zb--a

1
--------------- +
1+zc—a+zc--b

reduces to

Solution :
1
--------------- +
1+za—b+za--c

Can be written as
1
-----------------------
1+[za/zb]+ [za/zc]
1
--------------- +
1+zb—c+zb--a

Can be written as
1
-----------------------
1+[zb/zc]+ [zb/za]

1
--------------- +
1+zc—a+zc--b

Can be written as
1
-----------------------
1+[zc/za]+ [zc/zb]

1
----------------------- +
1+[za/zb]+ [za/zc]

1
----------------------- +
1+[zb/zc]+ [zb/za]

1
-----------------------
1+[zc/za]+ [zc/zb]
=
1
----------------------- +
1+[z--b/z--a]+ [z--c/z--a]

1
----------------------- +
1+[z--c/z--b]+ [z--a/z--b]

1
----------------------- +
1+[z--a/z--c]+ [z--b/z--c]
=
1
-------------------------
[z—a] +[z—b ]+ [z—c]
---------------------------
z—a

z—a
------------------------------------ + similarly

[z—a] +[z—b ]+ [z—c]

z—b
------------------------------------ + similarly

[z—a] +[z—b ]+ [z—c]


z—c
------------------------------------ + similarly

[z—a] +[z—b ]+ [z—c]


[z—a]+ [z—b ]+ [z—c]
------------------------------------ =1

[z—a] +[z—b ]+ [z—c]


-------------------------------------------------------------------------------------

Question [7]
1 1 1
----------------- + ------------+ ------------------- =?
LOG abc LOG abc LOG abc
ab bc ca
solution :
1 1 1
----------------- + ------------+ -------------------
LOG abc LOG abc LOG abc
---------- --------------- --------------
LOG ab LOG bc LOG ca

LOG ab LOG bc LOG ca


---------- + ---------- + -------------
LOG abc LOG abc LOG abc

LOG ab + LOG bc + LOG ca


-------------------------------------------
LOG abc

LOG a+LOG b + LOG b + LOG c+lOGc+lOGa


-----------------------------------------------------------
LOG abc

2LOGa+ 2 log b+ 2 log c

------------------------------------

Log abc
2 (log a+log b+logc)

----------------------------

Log abc

2 log abc

-------------

Log abc

=2

Question [8]
Number of digits to the numeral 264 is

Solution :

264
Taking logarithm

Log 264
=

64 log 2= 64 [0.30103=19.26

20 decimals.

Question [8]
If 4x=5y=20z

Then Z=?

Solution :

4x=5y=20z=k

4x=k

4= k[1/x]

5y=k

5= k[1/y]
20z=k

20= k[1/z]

k[1/x] k[1/y] = k[1/z]

k[1/x]+[1/y] = k[1/z]

1/x + 1/y = 1/z

Y+x 1

------- = ------

Xy z

Z= xy/ x+y

Question 9:

[(Root 3) /9]5/2 [9/3(root 3)]7/2 [9] =?

Solution :

31/2

{-------}5/2 {3/( 3)1/2 }7/2 [3]2

32
=

{31/2 3—2}5/2 {31( 3)--1/2 }7/2 [3]2


=

{31/2 3—2}5/2 {31( 3)--1/2 }7/2 [3]2

{3—3/2}5/2 { 31/2}7/2 [3]2

3—15/4 37/4 3 2

3 (–15/4)+( 7/4)+ (8/4)


3 (–15/4)+( 7/4)+ (8/4)

3(—15+7+8/4)

30=1

--------------------------------------------------------------

CA EXAMINATION QUESTION 1 ON INDICES AND LOGARITHMS:

IF 2x—2x—1 = 4 then, xx =?

Solution :

2x—2x—1 = 4 can be written as

2x—2x 2--1 = 4

2x—2x 2--1 = 4 can be written as

2x—2x = 4

----

21

2x—2x = 4 can be written as

----

21

2x—2x = 4 can be written as

----

2x [ 1—1/2] =4

2x [ 1/2] =4

2x =8

2x =23

X=3

Xx=33=27.
CA EXAMINATION QUESTION 2 ON INDICES AND LOGARITHMS:

IF x= en-- e—n
----------------- then the value of n is ?

en + e—n
SOLUTION:

X = en-- e—n
-----------------

en + e—n

1
en + e—n
------- = --------------------

X en-- e—n
1+X
en + e—n + en-- e—n
------- = --------------------------------------------------

1--X en+ e—n -- en+ e—n

1+X
en + en
------- = --------------------------------------------------

1--X e—n + e—n

1+X
2en
------- = --------------------------------------------------

1--X 2e—n
1+X
en
------- = --------------------------------------------------

1--X e—n
1+X
en en
------- =

1--X
1+X
e2n
------- =

1--X

LOG { (1+X)/(1—X)} = LOG e2n

LOG { (1+X)/(1—X)} = 2nLOG e

LOG { (1+X)/(1—X)} = 2n

2n= LOG { (1+X)/(1—X)}

n= LOG { (1+X)/(1—X)}/ 2

CA EXAMINATION QUESTION 3 ON INDICES AND LOGARITHMS:

IF X= Ya

Y= Zb

Z= xc
then abc is ?

solution :

X= Ya

X= [zb] a

X= [(xc] b] a

X= [(xabc]

abc =1.

CA EXAMINATION QUESTION 4 ON INDICES AND LOGARITHMS:


IF LOG ( LOG (LOG X ) ) = 1 FIND X.

2 3 2

SOLUTION :

LOG ( LOG (LOG X ) ) = 1

2 3 2

LOG (LOG X ) ) = 21

3 2

LOG (LOG X ) ) = 2

3 2

(LOG X ) ) = 32

(LOG X ) = 9

29 = X

X = 512.

CA EXAMINATION QUESTION 5 ON INDICES AND LOGARITHMS:

If log ( a+b)/4 = ½ (log a + log b) then the value of a/b + b/a is ?

Solution :

log ( a+b)/4 = ½ (log a + log b)

log ( a+b)/4 = ½ (log ab )

log ( a+b)/4 = log (ab )1/2

( a+b)/4 = (ab )1/2

( a+b)/4 = √ab

( a+b) = 4√ab
squaring on both sides
( a+b)2 = 16ab

( a2+2ab+b2) = 16ab

( a2+b2) = 16ab—2ab

( a2+b2) = 14ab

( a2+b2) = 14

-----------

ab

a2 b2

----- + ------ = 14

ab ab

a b

----- + ------ = 14.

b a

CA EXAMINATION QUESTION 6 ON INDICES AND LOGARITHMS :

If p/q = --2/3 then the value of 2p+q / 2p—q is


solution :

here p=--2 and q= 3 , so substitute in


2p+q / 2p—q

2(--2)+(3) / 2(--2)—3

(--4)+(3) / (--4)—3
-- 1 / --7

= 1/7.

CA EXAMINATION QUESTION 7 ON INDICES AND LOGARITHMS :

If x = 31/3 + 3—1/3 then the value of 3x3 –9x ?


solution :
x = 31/3 + 3—1/3 ---------- eqn 1

substitute in 3x3 –9x

cube on both sides

x3 = (31/3 + 3—1/3)3

(a+b)3 = a3 + b3 + 3ab(a+b)

(x)3 = (31/3 )3 + (3—1/3)3 + 3(31/3)( 3—1/3) (31/3 + 3—1/3)

x3 = (31 ) + (3—1) + 3(31/3)(1) (31/3 + 3—1/3)

----

3 1/3

x3 = (31 ) + (3—1) + 3 (31/3 + 3—1/3)

x3 =3 + 1/3 + 3 (31/3 + 3—1/3)

here x = 31/3 + 3—1/3 so

x3 =3 + 1/3 + 3x

x3—3x =3 + 1/3

x3—3x =9 + 1

-------

3(x3—3x) =10

3x3—9x = 10.

CA EXAMINATION QUESTION 8 ON INDICES AND LOGARITHMS :

Find the value of [1—{1—(1—x2)—1}—1]—1/2

Solution :

[1—{1—(1—x2)—1}—1]—1/2

[1—{1—( 1 )}—1]—1/2
--------

1—x2

[1—{1—x2—1}—1]—1/2

------------

1—x2

[1—{—x2}—1]—1/2

-------

1—x2

[1—{1—x2}]—1/2

-------

--x2

[1+{1—x2}]—1/2

-------

x2

[x2+{1—x2}]—1/2

----------------

x2

[--------]—1/2

x2

(x2)1/2

= x1 = x.

CA EXAMINATION QUESTION 9 ON INDICES AND LOGARITHMS :


If log (m+n) = log m+ log n , then m is

Solution :

log (m+n) = log m+ log n

log (m+n) = log (mn)

(m+n) = mn

n=mn—m

n = m(n—1)

m(n—1) =n

m = n/ (n—1)

CA EXAMINATION QUESTION 10 ON INDICES AND LOGARITHMS :

Log ( x2+x) – log (x+1) = 2 find x .

4 4

Solution :

Log ( x2+x) – log (x+1) = 2 .

4 4

Log ( x2+x) / x+1 = 2

( x2+x) / x+1 = 42

( x2+x) / x+1 = 16

X(x+1)/ x+1 = 16

X =16.

CA EXAMINATION QUESTION 11 ON INDICES AND LOGARITHMS :

If 2x x 3y x 5z = 360 , find x,y,z .

Solution :
2x x 3y x 5z = 360

Factors of 360 = 2x2x2x3x3x5

Factors of 360 = 23 x 32 x 5
So x = 3, y = 2 , z = 1 .

CA EXAMINATION QUESTION 12 ON INDICES AND LOGARITHMS :

Find the value of{ log √25 – log 23 + log 42}x

10 10 10

Solution :

{log √25 – log 23 + log 42}x

10 10 10

{log 5 – 3log 2+ 2log 4}x

10 10 10

{log 5 – 3log 2+ 2log 22}x

10 10 10

{log 5 – 3log 2+ 4log 2}x

10 10 10

{log 5 + log 2}x

10 10

{log (5 x 2)}x

10

{log 10}x

10

= 1x = 1.

CA EXAMINATION QUESTION 12 ON INDICES AND LOGARITHMS :

If log b + log c = 0 , then

a a
solution :

log b + log c = 0

a a

log bc = 0.

a0 = bc

1 = bc .

b = 1 /c

so b and c are reciprocals.

CA EXAMINATION QUESTION 13 ON INDICES AND LOGARITHMS :

The value of 2 log x + 2 log x2 + 2 log x3 + ……+ 2log xn will be ?

Solution :

2 log x + 2 log x2 + 2 log x3 + ……+ 2log xn


2 { log x + log x2 + log x3 + ……+log xn}
2 { log x + 2 log x +3 log x + ……+ n log x}
2 log x{ 1 +2 + 3+ …….+ n}
2 log x{ n(n+1)/2}
= log x{ n(n+1).

CA EXAMINATION QUESTION 14 ON INDICES AND LOGARITHMS :

Solve [ log10x -- 3 ] + [11 -- log10x ]

--------------- ---------------- = 2

2 3

Solution :

[ log10x -- 3 ] + [11 -- log 10x ]

--------------- ---------------- = 2

2 3
3[ log10x -- 3 ] + 2 [11 -- log10x ]

---------------------------------------------- = 2

[3 log10x -- 9 + 22 -- 2log10x ]

---------------------------------------------- = 2

[ log10x + 13] = 12

[ log10x ] = 12 – 13

[ log10x ] =--1

So x = 10—1

CA EXAMINATION QUESTION 15 ON INDICES AND LOGARITHMS :

If n = m! where “m” positive integer ≥ 2 then value of 1 1 1 1

-------- + ------- + -------- + ……….. + --------

Log2n log3n log4n log mn

Solution :

1 1 1 1

-------- + ------- + -------- + ……….. + --------

Log2n log3n log4n log mn

By Reciprocal ,

Logn2 + logn3 + logn4 +……. + lognm

Logn (2 x3 x4 x …… x m)

Logn(m!)

Lognn
= 1.

CA EXAMINATION QUESTION 16 ON INDICES AND LOGARITHMS :

The ratio compounded of 4:5 and sub duplicate of a:9 is 8:15 . then a is

Solution :

sub duplicate of a:9 is √a:√9

compound 4/5 x √a/√9 = 8/15

4/5 x √a/3 = 8/15

4√a/ 15 = 8/15

4√a =8

√a = 2

a =4.

CA EXAMINATION QUESTION 17 ON INDICES AND LOGARITHMS :

If x varies inversely as square of y and given that y = 2 for x = 1 then value of x ,if y = 6?

Solution :

x∞1/y2

x= k (1/y2)

x= (k/y2)

x =1 , y = 2

1= (k/22)
1= (k/4)
k = 4.

x =? , y = 6, k=4

x= (4/62)
x =4/36
x= 1/9.

CA EXAMINATION QUESTION 18 ON INDICES AND LOGARITHMS :

If log2x + log4x = 6 find x.

Solution :

log2x + log4x = 6

log x log x

------- + ---------- = 6

Log 2 log 4

log x log x

------- + ---------- = 6

Log 2 log 22

log x log x

------- + ---------- = 6

Log 2 2log 2

log x

------- {1 + ½} = 6

Log 2

log x

------- {2 + 1}/2 = 6

Log 2

log x

------- {3/2} = 6

Log 2

log x
------- {3/2} = 6

Log 2

log x

------- = 6 x 2/3

Log 2

log x

------- = 4

Log 2

Log2x = 4

X = 24

X = 16 .

CA EXAMINATION QUESTION 19 ON INDICES AND LOGARITHMS :

The value of 3n+1 + 3n is equal to

------------

3n+3 -- 3n+1

Solution :

3n+1 + 3n

------------

3n+3 -- 3n+1

3nx 31 + 3n

------------

3n x 33 -- 3n x 31

3n ( 31 + 1)

-----------------
3n ( 33 -- 31)

( 31 + 1)

-----------------

( 33 -- 31)

( 3 + 1)

-----------------

( 27 -- 3)

4 1

------ = ------ .

24 6

CA EXAMINATION QUESTION 20 ON INDICES AND LOGARITHMS :

If logxy = 100 and log2x = 10, then the value of y

Is?

Solution:

logxy = 100 ----- eqn 1

log2x = 10 ------ eqn 2

multiply 1 and 2 ,

logxy x log2x = 100 x 10

log y log x

------- x ------- = 1000

Log x log 2

log y

------- = 1000
Log 2

log2y = 1000

y = 21000 .

CA EXAMINATION QUESTION 21 ON INDICES AND LOGARITHMS :

Find x , if x (x)1/3 = (x1/3) x .

Solution :

x (x)1/3 = (x1/3) x

x1 (x)1/3 = x1/3 x

x1+1/3 = xx/3

x4/3 = xx/3

4/3 = x/3

X=4.

CA EXAMINATION QUESTION 22 ON INDICES AND LOGARITHMS :

Which of the following is true ?

1 1 1 1

---- + ----- + ------ = ---------

ab bc ca abc

solution :

1 1 1 1

---- + ----- + ------ = ---------

ab bc ca abc

c+a+b 1

--------------------- = ---------

abc abc
a+b+c=1

by taking log ,

log(a + b + c) = log 1

log(a + b + c) = 0 .

CA EXAMINATION QUESTION 23 ON INDICES AND LOGARITHMS :

For what value of x , the equation (log√x2 )2 = logx2 is true .

Solution :

(log√x2 )2 = logx2

Log 2 log 2

{---------}2 = ---------

Log √x log x

Log 2 log 2

{---------}2 = ---------

Log x1/2 log x

Log 2 log 2

{---------}2 = ---------

1/2Log x log x

2 Log 2 log 2

{---------}2 = ---------

Log x log x

Log 2 log 2

4 {---------}2 = ---------

Log x log x

Log 2
4 --------- = 1

Log x

4 log 2 = log x

Log 24 = log x

Log 16 = log x

X = 16.

CA EXAMINATION QUESTION 24 ON INDICES AND LOGARITHMS :

If cube root of a + cube root of b + cube root of c = 0 , then the value of{ (a + b + c )/ 3 } 3 is

Solution :

cube root of a + cube root of b + cube root of c = 0

a1/3 + b1/3 + c1/3 = 0

a1/3 + b1/3 = -- c1/3 ----- eqn 1

cube on both sides

(a1/3 + b1/3 )3= (-- c1/3 )3

(a+b)3 = a3 + b3 + 3ab (a+b)

(a1/3)3 +(b1/3)3 + 3 a1/3 b1/3 (a1/3 + b1/3) = (-- c1/3 )3

a + b + 3 a1/3 b1/3 (-- c1/3 ) = -- c

a + b -- 3 a1/3 b1/3 c1/3 = -- c

a + b + c = 3 a1/3 b1/3 c1/3

(a + b + c) / 3 = a1/3 b1/3 c1/3

Cube on both sides

{(a + b + c) / 3}3 = ( a1/3 b1/3 c1/3)3

{(a + b + c) / 3}3 = abc .

CA EXAMINATION QUESTION 25 ON INDICES AND LOGARITHMS :


The value of log49 x log32 is

Solution :

log49 x log32

log 9 log 2

------- x --------

Log 4 log 3

log 32 log 2

------- x --------

Log 22 log 3

2 log 3 log 2

---------- x --------

2 Log 2 log 3

log 3 log 2

---------- x --------

Log 2 log 3

=1.

CA EXAMINATION QUESTION 26 ON INDICES AND LOGARITHMS :

The value of ( logyx . logzy . logxz )3 is

Solution :

( logyx . logzy . logxz )3

Log x log y log z

{---------- x --------- x --------- } 3

Log y log z log x

= (1)3 = 1 .
CA EXAMINATION QUESTION 27 ON INDICES AND LOGARITHMS :

Divide 80 into two parts so that their product is maximum . then the numbers are ?

Solution :

Sum of two numbers = 80

Let first no be x

Second no ( 80 – x )

product of two numbers = x ( 80 – x )

= 80x – x2

= 80x – x2

dp/dx = 80 -- 2x

For maximum ,

80 – 2x = 0

80 = 2x

So x = 40 .

first no is 40

Second no ( 80 – x ) = ( 80 – 40 ) = 40 .

CA EXAMINATION QUESTION 28 ON INDICES AND LOGARITHMS :

If ( 25 )150 = ( 25x )50 find x .

Solution :

( 25 )150 = ( 25x )50

( 25 )150 = (25)50 (x)50

( 25 )150 / (25)50 = (x)50

( 25 )100 = (x)50

(52)100 = (x)50
(52 )50 x 2 = (x)50

(54 )50 = (x)50

So x = 54 .

CA EXAMINATION QUESTION 29 ON INDICES AND LOGARITHMS :

The value of ( ya/yb)a2+ab+b2 x (yb/yc)b2+bc+c2 x (yc/ya)c2+ca+a2 is ?

Solution :

( ya/yb)a2+ab+b2 x (yb/yc)b2+bc+c2 x (yc/ya)c2+ca+a2

( ya--b)a2+ab+b2 x (yb--c)b2+bc+c2 x (yc--a)c2+ca+a2

Y0 = 1.

CA EXAMINATION QUESTION 30 ON INDICES AND LOGARITHMS :

If the salary of P is 25% lower than that of Q and the salary of R is 20% higher than that of Q ,
then the ratio of salary of R and P will be?

Solution :

Let the salary of Q = 100

the salary of P = 100 – 25% = 75

the salary of R = 100 + 20% = 120

RATIO OF SALARY OF R AND P = 120: 75 = 8:5 .

CA EXAMINATION QUESTION 31 ON INDICES AND LOGARITHMS :

IF x2 + y2 = 7xy , then log 1/3 (x + y) = ?

Solution :

x2 + y2 = 7xy

add 2xy on both sides

x2 + y2 + 2xy = 7xy + 2xy

( x + y )2 = 9xy
Taking log on both sides

log( x + y )2 =log 9xy

2 log (x + y ) = log 9 + log x + log y

2 log (x + y ) = log 32 + log x + log y

2 log (x + y ) = 2log 3 + log x + log y

2 log (x + y ) – 2log 3 = log x + log y

2 [log (x + y ) – log 3] = log x + log y

2 [log{ (x + y ) / 3} ] = log x + log y

log{ (x + y ) / 3} = 1/2(log x + log y) .

CA EXAMINATION QUESTION 32 ON INDICES AND LOGARITHMS :

If x = log2412 , y = log3624 , z = log4836 then xyz + 1 = ?

Solution :

x = log2412 , y = log3624 , z = log4836

substitute in xyz +1

(log2412)x (log3624)x (log4836) + 1

(log12/log24)x (log24/log 36) x (log 36/ log 48)+1

(log12/log48)+1

{(log12+log48)/log48 }

{log(12x48)/log48}

Log576/log48

Log 242/log48

2log24/log48

Here log24/log48 can be written as (log24/log36)x(log36/log48)

So 2 x (log24/log36)x(log36/log48)
= 2yz .

CA EXAMINATION QUESTION 33 ON INDICES AND LOGARITHMS :

If log x = a + b , log y = a—b , then the value of log (10x / y 2 ) is ?

Solution :

log (10x / y2 ) = log 10x – log y2

= log10 + logx – 2logy

Here log x = a + b , log y = a—b and log10 = 1

So ,

1+(a+b)—2(a--b)

1+(a+b)—2a+2b

1—a+3b .

CA EXAMINATION QUESTION 34 ON INDICES AND LOGARITHMS :

If x= 1+ logpqr , y = 1+ logqrp , z = 1+ logrpq , then the value of (1/x) +(1/y)+(1/z) is ?

Solution :

x= 1+ logpqr

x = 1+(log qr / logp)

x = (logp+log qr / logp)

x = log pqr / log p

1/x = logp/logpqr -----eqn 1

y = 1+ logqrp

y = 1+(log rp / logq)

y = (logq+log rp / logq)

y = log pqr / log q

1/y = log q/log pqr -----eqn 2


z = 1+ logrpq

z = 1+(log pq / logr)

z = (logr+log pq / logr)

z = log pqr / log r

1/z = logr/logpqr -----eqn 3

So (1/x) +(1/y)+(1/z) =

(log p/log pqr) + (log q/log pqr) + (logr/logpqr)

(log p +log q +log r/log pqr)

(log pqr/ logpqr ) = 1 .

CA EXAMINATION QUESTION 35 ON INDICES AND LOGARITHMS :

The value of (1/log360) +(1/log460) + (1/log560) is?

Solution :

(1/log360) +(1/log460) + (1/log560)

Log603 + log604 + log605

Log60(3 x 4 x 5 )

Log6060

= 1.

CA EXAMINATION QUESTION 36 ON INDICES AND LOGARITHMS :

If px=q , qy=r , rz=p6 then the value of xyz is?

Solution :

px=q , qy=r , rz=p6

substitute qy=r in rz=p6

(qy )z=p6

qyz = p6
substitute px=q in qyz = p6

(px )yz = p6

pxyz = p6

so xyz = 6 .

CA EXAMINATION QUESTION 37 ON INDICES AND LOGARITHMS :

If 15(2p2—q2) = 7pq where p and q are positive . then p:q will be

Solution :

15(2p2—q2) = 7pq

(30p2—15q2) = 7pq

30p2—7pq -- 15q2 = 0

30p2—25pq +18pq -- 15q2 = 0

5p(6p—5q)+3q(6p—5q)=0

(6p—5q)(5p+3q)=0

If (6p—5q)=0

6p=5q

p/q=5/6 = 5:6

if (5p+3q)=0

5p=--3q

p/q=--3/5 =--3:5 (not possible).

CA EXAMINATION QUESTION 37 ON INDICES AND LOGARITHMS:

The value of log53 x log34 x log25 is


Solution :

log53 x log34 x log25

(log3/log5) x (log4/log3) x (log5/log 2)

Log4/log2

Log22/log2

2log2/log2

= 2.

CA EXAMINATION QUESTION 38 ON INDICES AND LOGARITHMS:

The value of x2—(y—z)2 y2—(x--z)2 z2—(x--y)2

---------------- + -------------- + -------------

(x+z) 2—y2 (x+y)2—z2 ( y+z)2 –x2

Solution :

x2—(y—z)2 y2—(x--z)2 z2—(x--y)2

---------------- + -------------- + -------------

(x+z)2—y2 (x+y)2—z2 ( y+z)2 –x2

(x+y—z) (x—y+z) (y+x--z)(y—x+z)

----------------------- + ------------------------ +

(x+y+z)(x—y+z) (y+x--z)(y+x+z)

(z+x--y)(z—x+y)

----------------------
(z+x+y)(z—x+y)

(x+y—z) (y—x+z) (z+x--y)

----------- + ------------- + -----------

(x+y+z) (y+x+z) (z+x+y)

(x+y—z+y—x+z+z+x--y)

----------------------------------

(x+y+z)

X+y+z/x+y+z

= 1.

Function:

Function is also known as mapping that takes place from SET A to SET B.

Consider SET X

Consider SET Y

Consider FUNCTION f

f : XY

f : XY has to be read as “f MAPS X IN TO Y”

Domain of the function f is SET X

Co domain of the function f is SET Y


Range of the function f is the image of the element of SET X got under the mapping.

Range of function f of SET X is described as f(X)

Example : SET X ={ OLD STATES OF SOUTH INDIA}=SET X={TAMIL NADU, ANDHRA


PREDESH,KARNATAKA,KERALA}

SET Y ={ CAPITAL CITIES OF OLD STATES OF SOUTH INDIA}=SET Y={CHENNAI,


HYDERABAD,BENGALURU,TRIVANDUM}

LET f assigns to each south Indian state its capital city.

CPT EXAMINATION QUESTION 1 ON FUNCTION

CONSIDER SET X ={1,2,3}

CONSIDER SET Y={3,5,7}

DEFINE f:XY.

SOLUTION:

f by observation is 2x+1

So y=f(x)=2x+1

CPT EXAMINATION QUESTION 2 ON FUNCTION

What is an INTO FUNCTION OR OBJECTIVE MAPPING ?

ANSWER :

f(x) is the subset of set Y

for example

X={ 1,2,3}

Y={0,2,4,6,8}

f(X)=y=2X={2,4,6}

SO f IS AN INTO FUNCTION

IN OTHER WORDS, CONSIDER SET X={S,M,B} =DOMAIN

SET Y={S,M,B,F}=CO DOMAIN

Function =students passing from set X=range={S,M,B} =RANGE


DEFINITION OF INTO FUNCTION = range is a subset of co domain.

CPT EXAMINATION QUESTION 3 ON FUNCTION

Consider set X={5,6,8}

Consider set Y={set of all natural numbers}

Whether f(x)=2x is an INTO FUNCTION or not if X is domain set and Y is co domain set ?

Solution :

Domain set function Range set co domain set


y=2x 1,2,3,4,5,6,7,8,9,10,11,12,13,14,15,16,17,18…
5 10
6 12
8 16

Solution :

X= {5,6,8}=DOMAIN SET

f(x)=2x

f(5)=10

f(6)=12

f(8)=16

range set={10,12,16}=RANGE SET

CODOMAIN SET={1,2,3,4,5,6,7,8,9,10,11,12,13,14,15,16,17,…….}

RANGE SET IS A SUBSET OF CO DOMAIN SET

SO

f(X) IS AN INTO FUNCTION.


CPT EXAMINATION 4 ON FUNCTION

Consider set X={5,6,8}

Consider set Y={set of all natural numbers exceeding 15 }

Whether f(x)=2x is an INTO FUNCTION or not if X is domain set and Y is co domain set ?

Solution :

X= {5,6,8}=DOMAIN SET

f(x)=2x

f(5)=10

f(6)=12

f(8)=16

range set={1`0,12,16}=RANGE SET

CODOMAIN SET={16,17,…….}

RANGE SET IS not A SUBSET OF CO DOMAIN SET

SO

f(X) IS not AN INTO FUNCTION.

EXAMPLE:

f(x) is the subset of set Y

for example

X={ 1,2,3}-DOMAIN SET

Y={0,2,4,6,8}-CO DOMAIN SET

f(X)=y=2X={2,4,6}RANGE SET

SO f (X) IS AN INTO FUNCTION


IN OTHER WORDS, CONSIDER SET X={S,M,B} =DOMAIN

SET Y={S,M,B,F}=CO DOMAIN

Function =students passing from set X=range={S,M,B} =RANGE

DEFINITION OF INTO FUNCTION = range is a subset of co domain.

CPT EXAMINATION 5 ON FUNCTION

What is an ONTO FUNCTION OR SUBJECTIVE MAPPING?

ANSWER :

f(x) is the SAME OF set Y

for example

X={ 1,2,3}

Y={2,4,6}

f(X)=y=2X={2,4,6}

SO f (X) IS AN ONTO FUNCTION

IN OTHER WORDS, CONSIDER SET X={S,M,B} =DOMAIN

SET Y={S,M,B}=CO DOMAIN

Function =students passing from set X=range={S,M,B} =RANGE

DEFINITION OF ONTO FUNCTION = range SET is THE co domain SET.IN OTHER WORDS, RANGE
SET AND CO DOMAIN SET ARE EQUAL SETS.

CPT EXAMINATION 6 ON FUNCTION

Consider set X={5,6,8}

Consider set Y={10,12,16}

Whether f(x)=2x is an ONTO FUNCTION or not if X is domain set and Y is co domain set ?

Solution :
X= {5,6,8}=DOMAIN SET

f(x)=2x

f(5)=10

f(6)=12

f(8)=16

range set={10,12,16}=RANGE SET

CO DOMAIN SET={10,12,16}

RANGE SET AND CO DOMAIN SET ARE EQUAL SETS.

SO

f(X) IS AN ONTO FUNCTION.

CPT EXAMINATION 7 ON FUNCTION

What is an INJECTIVE FUNCTION OR ONE ONE MAPPING?

ANSWER :

CONSIDER

SET X={ --1, 1,2,3,--3}=DOMAIN SET

Y={ SET OF ALL REAL NUMBERS}=CO DOMAIN SET

f(X)=y= X3

DEFINITION OF INTO FUNCTION = range is a subset of co domain.

RANGE SET=R={--1,1,8,27,--27}

RANGE SET HAS NO REPEATING ELEMENTS.

HENCE IT IS A ONE ONE MAPPING OR INJECTIVE FUNCTION

CPT EXAMINATION 7 ON FUNCTION


Consider set X={--5,5,6,8}=DOMAIN

Consider set Y={SET OF ALL NUMBERS}=CODOMAIN

Whether f(x)= X 2 is an injective FUNCTION or not if X is domain set and Y is co domain set ?

Solution :

The image of –5=25

The image of 5=25

So it is not a one one mapping. It is not an injective mapping.

CPT EXAMINATION 9 ON FUNCTION

What is a BIJECTIVE FUNCTION OR ONE ONE ONTO MAPPING?

ANSWER :

IF RANGE SET HAS NO REPEATING ELEMENTS AND THE RANGE SET AND CO DOMAIN SET ARE
EQUIAVALENT SETS, THEN THE FUNCTION IS BIJECTIVE FUNCTION OR ONE ONE ONTO
FUNCTION.

CPT EXAMINATION 10 ON FUNCTION

CONSIDER SET X= {1,--1,2}=DOMAIN SET.

CONSIDER SEY Y={1,--1,32}=CO DOMAIN SET

CONSIDER THE FUNCTION f(X)=Y=X5

IS THE FUNCTION BIJECTIVE ?

SOLUTION:

RANGE SET=SET R={15, --15, 25} ={1,—1,32}=EQUAL SET OF CODOMAIN SET.ALSO IN THE RANGE
SET, THERE ARE NO REPEATING ELEMENTS.

THEREFORE, THE FUNCTION IS ONE ONE ONTO FUNCTION OR BIJECTIVE FUNCTION.

CPT EXAMINATION 11 ON FUNCTION

CONSIDER SET X= {1,--1,2}=DOMAIN SET.

CONSIDER SEY Y={1,--1,16}=CO DOMAIN SET


CONSIDER THE FUNCTION f(X)=Y=X4

IF THE FUNCTION BIJECTIVE ?

SOLUTION:

RANGE SET=SET R={14, --14, 24} ={1,1,16}=EQUAL SET OF CODOMAIN SET.

THEREFORE, THE FUNCTION IS NOT A ONE ONE ONTO FUNCTION OR BIJECTIVE FUNCTION.

CPT EXAMINATION 12 ON FUNCTION

WHAT IS A MANY ONE FUNCTION ?

IF TWO OR MORE ELEMENTS OF DOMAIN SET CAN HAVE SAME RANGE ELEMENT, THEN THE
FUNCTION IS CALLED MANY ONE FUNCTION.

FOR EXAMPLE CONSIDER DOMAIN SET X={2,-2,3,4}

CONSIDER THE FUNCTION X2

THE RANGE SET IS SET R={4,4,9,16}

SINCE 4 WHICH IS APPEARING IN THE RANGE SET CAN HAVE 2 AS WELL AS –2 AS A VALID PRE
IMAGE

EXPLANATION

DOMAIN SET RANGE SET


2 4
--2
3 9
4 16

CPT EXAMINATION 13 ON FUNCTION

WHAT IS AN IDENTITY FUNCTION OR IDENTITY MAPPING ?

IF THE DOMAIN SET ELEMENTS AND THE RANGE SET ELEMTENTS ARE THE SAME THEN THE
FUNCTION IS KNOWN AS AN IDENTTY MAPPING.

FOR EXAMPLE CONSIDER DOMAIN SET X={2,-2,3,4}

CONSIDER THE FUNCTION f(X)=X

THE RANGE SET IS SET R={2,--2,3,4}


CPT EXAMINATION 14 ON FUNCTION

IF THE DOMAIN SET ELEMENTS AND THE RANGE SET ELEMNTS ARE EQUAL SETS, WHAT TYPE OF
A FUNCTION WAS ADMINISTERED ON THE DOMAIN SET.

ANSWER: IDENTITY FUNCTION OR IDENTITY MAPPING.

CPT EXAMINATION 15 ON FUNCTION

WHAT IS A CONSTANT FUNCTION OR CONSTANT MAPPING ?

IF THE DOMAIN SET ELEMENTS HAVE NO SAY AND THE RANGE SET ELEMENTS SINCE THE
RANGE SET ELEMENTS DO NOT HAVE A CHEMISTRY WITH DOMAIN ELEMENTS AND ALL THE
RANGE SET ELEMENTS ARE THE SAME AND INDEPENDENT OF THE DOMAIN SET ELEMENTS.

FOR EXAMPLE CONSIDER DOMAIN SET X={2,-2,3,4}

CONSIDER THE FUNCTION f(X)=10

THE RANGE SET IS SET R={10,10,10,10}

CPT EXAMINATION 16 ON FUNCTION

IF THE DOMAIN SET X={5,15,25,35) AND THE RANGE SET ELEMENTS ARE R={10,10,10,10}

WHAT IS THE NATURE OF FUNCTION INVOLVED IN IT ?

ANSWER:CONSTANT FUNCTION.

CPT EXAMINATION 17 ON FUNCTION

WHAT IS AN INVERSE FUNCTION ?

CONSIDER SET X ={2,4,6} =DOMAIN SET

CONSIDER THE FUNCTION f(X)=Y=3X

RANGE SET={6,12,18}

IF WE CONSIDER {6,12,18} AS DOMAIN SET

AND IF WE CONSIDER {2,4,6}AS RANGE SET, DEFINE THE FUNCTION


f(Y)=X= 1/3{Y]

SO WE CAN EASILY FIND OUT THE INVERSE FUNCTON OF AN ORIGINAL FUNCTION

LET THE ORIGINAL FUNCTION BE

f(X)=Y=3X

Y= 3X

THE INVERSE FUNCTION IS SEPARATING X

Y/3 =X

X= [1/3] Y

SYMBOLICALLY THE INVERSE FUNCTION OF f(X) IS = f`(X)

CPT EXAMINATION QUESTION 18 ON FUNCTION

IF Y=2X3 , WHAT IS ITS INVERSE FUNCTION?

ANSWER

[Y/2] = X3

[Y/2]1/3 =X IS THE INVERSE FUNCTION.

CPT EXAMINATION QUESTION 19 ON FUNCTION

IF Y=2X3 , AND IF THE RANGE SET IS {2,16} FIND THE DOMAIN SET OF THIS FUNCTION.

ANSWER

IF Y=2X3 , WHAT IS ITS INVERSE FUNCTION?

ANSWER

[Y/2] = X3

[Y/2]1/3 =X IS THE INVERSE FUNCTION.


[2/2]1/3 IS THE DOMAIN ELEMENT OF RANGE ELEMENT 1=1

[16/2]1/3 IS THE DOMAIN ELEMENT OF RANGE ELEMENT 16=2

SO THE DOMAIN SET IS {1,2}

ORIGINAL FUNCTION IS Y=2X3

CPT EXAMINATION QUESTION 20 ON FUNCTION.

WHAT IS A COMPOSITE FUNCTION ?

IF f(X)=2X

FIND f{f(X)}

Solution

f{f(X)}= fof

f(x)=2x

fof(x)=2(2x)=4x

CPT EXAMINATION QUESTION 21 ON FUNCTION.

IF f(X)=X+3

and

g(X)=X2

FIND g{f(X)}

Solution

g{f(X)}= gof

(x+3)2

gof=(x+3)2
CPT EXAMINATION QUESTION 21 ON FUNCTION.

IF f(X)=X+3

and

g(X)=X2

FIND f{g(X)}

Solution

fog= x2 +3

CPT EXAMINATION QUESTION 22 ON FUNCTION.

IF f(X)=X+3

and

g(X)=X2

FIND f{f(X)}

Solution

fof= [x +3] + 3=x+6

CPT EXAMINATION QUESTION 23 ON FUNCTION.

IF f(X)=X+3

and

g(X)=X2

FIND g{g(X)}

Solution
gog= [x2]2 =x4

CPT EXAMINATION QUESTION 24 ON FUNCTION.

f(x) =x+5

g(x)=x—5

find fof

solution

fof=(x+5) +5=x+10

CPT EXAMINATION QUESTION 25 ON FUNCTION.

f(x) =x+5

g(x)=x—5

find fog

solution

fog=(x--5) +5=x

CPT EXAMINATION QUESTION 26 ON FUNCTION.

f(x) =x+5

g(x)=x—5

find gog

solution

gog=(x-5)—5=x—10

CPT EXAMINATION QUESTION 27 ON FUNCTION.

f(x) =x+5
g(x)=x—5

find gof

solution

gof=(x+5)—5=x

CPT EXAMINATION QUESTION 28 ON FUNCTION.

What is an even function ?

If f(x) = f(--x) then we can say that f(x) is an even function.

CPT EXAMINATION QUESTION 29 ON FUNCTION.

Is f(x) = x3 an even function ?

Solution

f(x) = x3

put some value as per your imagination for x say x=4

f(x) = f(4)= x3 =43=64

f(--x) = f(--4)= x3 =(-4)3=--64

64 ≠--64

So the function is not an even function.

CPT EXAMINATION QUESTION 30 ON FUNCTION.

Is f(x)= x2 an even function ?

Solution
f(x) = y= x2

put some value as per your imagination for x say x=4

f(x) = f(4)= x2 =42=16

f(--x) = f(--4)= (--4)2 =(-4)2=16

16=16

So the function is an even function.

CPT EXAMINATION QUESTION 32 ON FUNCTION.

What is an odd function ?

If f(--x) = (--) f(x) then we can say that f(x) is an odd function.

CPT EXAMINATION QUESTION 32 ON FUNCTION.

Is f(x) = y= x3 an odd function for x=4.

Solution

f(x) = y= x3

put some value as per your imagination for x say x=4

f(--x) = f(--4)= x3 =[--4]3=--64

--f(x) =-- f(4)= --(x)3 =--(4)3=--64

--64=--64

So the function is an odd function.

CPT EXAMINATION QUESTION 33 ON FUNCTION.

If f(x) = y= x2 . Is the function an odd function ?

Solution :

f(x) = y= x2
put some value as per your imagination for x say x=4

f(--x) = f(--4)= x2 =(--4)2=16

--f(x) =-- f(4)= --(x)2 =--(-4)2=--16

16 ≠--16

So the function is not odd function.

CPT EXAMINATION QUESTION 34 ON FUNCTION.

What is a propositional function ?

Solution :

Let set M ={1,2,3}

Let set N={4,5,}

The Cartesian product M x N = { (1,4), (1,5), (2,4), (2,5) (3,4), (3,5)}

P(x,y) SUCH THAT THE TOTAL OF X AND Y=6

SO THE PAIRS FULFILLING THIS FUNCTION= { (1,5) ,(2,4)}

CPT EXAMINATION QUESTION 35 ON RELATION

Give an introduction to the concept of “ RELATION”

Solution :

Assume that there is set A

Assume that there is set B

Obtain the Cartesian product AxB and the universal set containing all the ordered pairs be
named as set C.

Assume that there is an open sentence. The open sentence should fulfill one or more ordered
pairs of the Cartesian product.

R is the relation set which is a sub set of C.

CPT EXAMINATION QUESTION 36 ON RELATION

Set A={1,2,3}
Set B={1,6}

Relation P(x,y)=x less than y.

Derive relation set P.

Solution :

 The set containing the Cartesian product of sets A and B=

{(1,1) ,(1,6), (2,1) ,(2,6), (3,1) ,(3,6)}.

 The relation is defined as “ x less than y” from and out of the Cartesian pairs.
 Relation set R = {(1,6), (2,6), (3,6)}.

CPT EXAMINATION QUESTION 37 ON RELATION

Set A={2,3,4}

Set B={3,4,5,6}

Relation P(x,y)= x divides y into an integer.

Derive the relation set P.

Solution :

 The set containing the Cartesian product of sets A and B=

{(2,3) ,(2,4), (2,5) ,(2,6), (3,3) ,(3,4), (3,5) ,(3,6), (4,3) ,(4,4), (4,5) ,(4,6)}

 The relation is defined as “ x divides y” from and out of the Cartesian pairs.
 Relation set R = {(2,4), (2,6), (3,3) , (3,6), (4,4)}

CPT EXAMINATION QUESTION 37 ON RELATION

The cartesian product set C of a set M having m number of elements and another set N
having n number of elements will have how many number of ordered pairs. Give an illustration.

Solution :m x n

Illustration

Set M={f,d,l}

Set N={l,k}

Cartesian product MXN={(f,l), (f,k), (d,l), (d,k), (l,l), (l,k)}

CPT EXAMINATION QUESTION 37 ON RELATION


The cartesian product set C of a set M having m number of elements and another set N having
n number of elements will have how many number of relations. Give an illustration.

Solution : 2 (mxn)

Illustration

Set M={f,d,l}

Set N={l,k}

Cartesian product MXN={(f,l), (f,k), (d,l), (d,k), (l,l), (l,k)}

The maximum number of relation statements between M and N=2 6 =64

CPT EXAMINATION QUESTION 38 ON RELATION

“Every relation is a function but every function need not be a relation”

Say true/false.

Answer :

True.

CPT EXAMINATION QUESTION 39 ON RELATION

“Every function is a relation”. Say true/false

Answer : false

CPT EXAMINATION QUESTION 40 ON RELATION

“Every relation has to be function ”. Say true/false

Answer : true

CPT EXAMINATION QUESTION 41 ON RELATION

Write a note on inverse relation

Consider set M={ 1,2}

Consider set N={4,5,6}

R=MxN={{(1,4), (1,5), (1,6), (2,4), (2,5), (2,6)}


Inverse raltion of R=R—1= NXM={{(4,1), (4,2), (5,1), (5,2), (6,1), (6,2)}

CPT EXAMINATION QUESTION 41 ON RELATION

Write a note on IDENTITY relation.

Consider set M={ 1,2}

Consider set N={1,5,6}

R=MxN={{(1,1), (1,5), (1,6), (2,1), (2,5), (2,6)}

IDENTITY relaltion of R ={1,1)}

CPT EXAMINATION QUESTION 41 ON RELATION

Write a note on void relation.

Consider set M={ 1,2}

Consider set N={1,5,6}

R=MxN={{(1,1), (1,5), (1,6), (2,1), (2,5), (2,6)}

Void relation of R =total of x and y is 150 ={null set}

CPT EXAMINATION QUESTION 43 ON RELATION

Write a note on reflexive relation.

Consider set M={ 1,2}

Consider set N={1,5,6}

R=MxN={{(1,1), (1,5), (1,6), (2,1), (2,5), (2,6)}

Reflexive relation comes where the same element is related to itself.

Reflexive relation of R = pairs where sum of x and y are less than 3

={(1,1)}
CPT EXAMINATION QUESTION 44 ON RELATION

Write a note on symmetric relation.

Consider set M={ son 1,son 2}

Consider set N={daughter 1,daughter2}

R=MxN={{(s1,d1), (s1,d2), (s2,d1), (s2,d2)}

Relation: persons having same parents

Relation set =all elements of MxN as well as all elements of NxM.

CPT EXAMINATION QUESTION 43 ON RELATION

Write a note on anti symmetric relation.

Consider set M={ student1 ,student 2}

Consider set N={teacher 1,}

R=MxN={{(s1,t1), (s2,t1)}

N XM={(T1,S1),(T1,S2)}

Relation: persons commanding others where x=commander and y =subordinate

{(T1,S1),(T1,S2)}

CPT EXAMINATION QUESTION 43 ON RELATION

Write a note on TRANSITIVE relation.

Consider set M={ CPT 1 ,CPT 2}

Consider set N={ IPC 1 ,IPC 2}

Consider set T={ FINAL 1 ,FINAL 2}

RELATION CA STUDENT.

{CPT1, IPC1), (IPC1 ,FINAL 2) (CPT1,FINAL2)

CPT EXAMINATION QUESTION 43 ON RELATION

Write a note on EQUIVALENCE RELATION.


ANSWER:

A RELATION WHICH HAS ALL THE THREE CHARACTERS

 REFLEXIVE
 SYMMETRIC
 TRANSITIVE
++++++++++++++++++++++++++++++++++++++++++++++++++++++
=================================================
FUNCTIONS AND RELATIONS

CA CPT QUESTION 1 ON FUNCTIONS AND RELATIONS:


CONSIDER SET A={ 2,7,9}
CONSIDER SET B={ 4,5}
FIND AXB
SOLUTION :
AXB={ (2,4), (2,5),(7,4),(7,5),(9,4),(9,5)}
THIS IS NAMED AS CARTESIAN PRODUCT.

CA CPT QUESTION 2 ON FUNCTIONS AND RELATIONS:


CONSIDER SET A={ 2,7,9}
CONSIDER SET B={ 4,5}
WITHOUT FINDING AXB ,TELL AS TO HOW MANY NUMBER OF PAIRS (a,b) SHALL BE
AVAILABLE IN THE CARTESIAN PRODUCT.

(mXn)
ANSWER :
(3X2)

(mXn) =6

================================================================================

NEW QUESTION 1 ON RELATION:


Let Set A={ 1,2,3}
AXA= { (1,1),(1,2),(1,3), (2,1),(2,2),(2,3), (3,1),(3,2),(3,3)}
RELATION 1: { (1,1),(2,2),(3,3)}
QUESTION : WHAT IS THE CHARACTER OF THE ABOVE RELATION SET?
SOLUTION
SINCE , ALL THE REFLEXIVE PAIRS FROM THE CARTESIAN PRODUCT IS FOUND INSIDE THIS
RELATION SET,WE CAN SAY THAT THE ABOVE RELATION SET IS A REFLEXIVE RELATION SET.

TEST FOR SYMMETRIC RELATIONSHIP

RELATION’S FOCUS

ORIGINAL DOMAIN ORIGINAL RANGE


1 1
2 2
3 3
1 2

RELATION’S REFOCUS

NEW DOMAIN = ORIGINAL NEW RANGE = ORIGINAL


RANGE DOMAIN
1 1
2 2
3 3
2 1

(2,1) IS NOT AVAILABLE IN THE RELATION SET. HENCE THE ABOVE RELATION IS NOT
SYMMETRIC.

===================

{1,2,3}

AXA=(1,1),(1,2),(1,3),( (2,1),(2,2),(2,3), (3,1),(3,2),(3,3)

RELATION SET=(1,1),(1,2),(2,2),(2,1)

DOMAIN SET OF THE RELATION RANGE SET OF THE RELATION


SET SET
1 1
2 2

======================

2—12—2016

Question [1]
Find the inverse of f(x) = x2

Solution :

f(x) = x2

Write it as

y = x2

y1/2 = x

x= y1/2

so the inverse function is x1/2

Question [2]

Find the inverse of f(x) = x3 --2

Solution :

f(x) = x3 --2

Write it as

y = x3 --2

y+2 = x3

[y+2]1/3 =x

so the inverse function is [x+2]1/3

--------------------------------------------------------------------------------------------------------------

CA QUESTION 1 ON FUNCTIONS AND RELATIONS:

OUT OF 20 MEMBERS IN A FAMILY , 11 LIKE TO TAKE TEA AND 14 LIKE COFFEE. ASSUME THAT
EACH ONE LIKES ATLEAST ONE OF THE TWO DRINKS . FIND HOW MANY LIKE BOTH COFFEE AND
TEA :

SOLUTION :

n(T)=11 n(C)=14 n(T U C)=20 n(T∩C)=?

n(T∩C)= n(T) + n(C) -- n(T U C)


n(T∩C)= 11 + 14 –20

n(T∩C)=5.

CA QUESTION 2 ON FUNCTIONS AND RELATIONS:

IN A GROUP OF 70 PEOPLE , 45 SPEAK HINDI , 33 SPEAK ENGLISH AND 10 SPEAK NEITHER HINDI
NOR ENGLISH . FIND HOW MANY CAN SPEAK BOTH ENGLISH AS WELL AS HINDI :

SOLUTION :

n(H)=45 n(E)=33 n(E U H)=70—10 = 60 n(E∩H)=?

n(E∩H)= n(E) + n(H) -- n(E U H)

n(E∩H)=33+45--60

n(T∩C)=18.

CA QUESTION 3 ON FUNCTIONS AND RELATIONS:

LET R IS THE SET OF REAL NUMBERS , SUCH THAT THE FUNCTION f : R  R AND

g : R  R ARE DEFINED BY f(x) = x2 + 3x +1 and g(x) = 2x –3 .FIND (fog) .

solution :

(fog)x=f(g(x))

f(g(x)) = f(2x –3)

so f(2x –3) =(2x—3)2 + 3(2x—3) +1

= 4x2 – 12x + 9 + 6x – 9 + 1

= 4x2 – 6x + 1

CA QUESTION 4 ON FUNCTIONS AND RELATIONS:

In a survey of 300 companies , the number of companies using different media – newspaper
(N) , radio (R) and television (T) are as follows :

n(N)=200 , n(R)=100 , n(T)=40 , n(N∩R)=50 , n(R∩T)=20 , n(N∩T)=25 , n(N∩R∩T)=5 .

FIND THE NO OF COMPANIES USING NONE OF THESE MEDIA


SOLUTION :

n(NURUT)= n(N) + n(R) + n(T) -- n(N∩R) -- n(R∩T) -- n(N∩T) + n(N∩R∩T)

n(NURUT)= 200+100+40—50—20—25+5

n(NURUT)=250.

THE NO OF COMPANIES USING NONE OF THESE MEDIA

= 300 -- n(NURUT)

=300 – 250

= 50 .

CA QUESTION 5 ON FUNCTIONS AND RELATIONS:

IF R IS THE SET OF REAL NUMBERS SUCH THAT THE FUNCTION f : R R IS DEFINED BY f(x) = (x +1
)2 , THEN FIND (fof).

Solution :

(fof) = f(f(x))

f(f(x)) = f((x +1 )2)

f(x) = (x +1 )2

(fof) =((x +1 )2 + 1)2

CA QUESTION 6 ON FUNCTIONS AND RELATIONS:

LET f : RR BE SUCH THAT f(x) = 2X , THEN f( x + y) EQUALS :

SOLUTION :

f(x) = 2X

f(x+y) = 2x+y

= 2 x 2y

= f(x)f(y)

CA QUESTION 7 ON FUNCTIONS AND RELATIONS:

IF f : RR , f(x) = 2X + 7 ,THEN THE INVERSE OF f IS :


SOLUTION :

f(x) = 2X + 7

y = 2x + 7

y – 7 = 2x

x=(y—7)/2

so the inverse of f is (x—7)/2 .

CA QUESTION 8 ON FUNCTIONS AND RELATIONS:

IN A TOWN OF 20,000 FAMILIES IT WAS FOUND THAT 40% FAMILIES BUY NEWSPAPER A , 20%
FAMILIES BUY NEWSPAPER B AND 10% FAMILIES BUY NEWSPAPER C , 5% FAMILIES BUY A AND
B , 3% BUY B AND C AND 4% BUY A AND C .IF 2% FAMILIES BUY ALL THE THREE NEWSPAPER ,
THEN THE NUMBER OF FAMILIES WHICH BUY A ONLY IS :

SOLUTION :

THE NUMBER OF FAMILIES WHICH BUY A ONLY IS 6600 FAMILIES .

CA QUESTION 9 ON FUNCTIONS AND RELATIONS:

OUT OF TOTAL 150 STUDENTS , 45 PASSED IN ACCOUNTS , 30 IN ECONOMICS AND 50 IN


MATHS ,30 IN BOTH ACCOUNTS AND MATHS , 32 IN MATHS AND ECONOMICS , 35 IN BOTH
ACCOUNTS AND ECONOMICS , 25 PASSED IN ALL THE THREE SUBJECTS ,FIND THE NUMBER WHO
PASSED ATLEAST IN ANY ONE OF THE SUBJECTS .

n(AUBUC)= n(A) + n(B) + n(C) -- n(A∩B) -- n(B∩C) -- n(A∩C) + n(A∩B∩C)

n(AUBUC )= 45+30+50—30—32—35+25

n (AUBUC )=53.

CA QUESTION 10 ON FUNCTIONS AND RELATIONS:

IF f(x) = (2+x)/(2—x) then f—1(x) is

Solution :

f(x) = (2+x)/(2—x)

y = (2+x)/(2—x)
y(2--x) = (2+x)

2y—xy—2 = x

2y—2=xy+x

2(y—1)=x(y+1)

X=2(y—1)/(y+1)

Therefore f—1 (x)=2(x—1)/(x+1) .

CA QUESTION 11 ON FUNCTIONS AND RELATIONS:

If A={1,2,3,4} B={2,4,6,8} f(1)=2 f(2)=4 f(3)=6 f(4)= 8 and f : AB THEN f—1 is ?

If f : AB then f = {(1,2)(2,4)(3,6)(4,8)}

So if f--1 : BA then f = {(2,1)(4,2)(6,3)(8,4)} .

CA QUESTION 12 ON FUNCTIONS AND RELATIONS:

If f(x)=x2 + x – 1 and 4(f(x))= f(2x) , then find x .

Solution :

f(x)=x2 + x – 1

4(f(x))= f(2x)

4(x2 + x – 1)= (2x)2 + 2x – 1

4x2 + 4x – 4 = 4x2 + 2x –1

4x – 2x = 4—1

2x = 3

x=3/2 .

CA QUESTION 13 ON FUNCTIONS AND RELATIONS:

If A ={p,q,r,s} B = {q,s,t } C = {m,q,n } . FIND C – (A∩B)


SOLUTION :

(A∩B) = ={p,q,r,s} ∩ {q,s,t }

= {q,s}

C – (A∩B) = {m,q,n } -- {q,s}

= {m,n} .

CA QUESTION 14 ON FUNCTIONS AND RELATIONS:

Given the function f(x) = 2x + 3 , then the value of f(2x) – 2f(x) + 3 will be

Solution :

f(x) = 2x + 3

f(2x) – 2f(x) + 3

(2(2x) + 3) – [ 2(2x + 3)] + 3

4x + 3 – 4x – 6 + 3

0.

CA QUESTION 15 ON FUNCTIONS AND RELATIONS:

IF f(x)= 2X +h THEN FIND f (x+ h) –2 f(x).

Solution :

f(x)= 2X +h

f (x+ h) –2 f(x).

((2x+2h)+ h) –2 ( 2x + h)

(2x+2h+h –4x --2h)

h—2x
CA QUESTION 16 ON FUNCTIONS AND RELATIONS:

IF A ={x : x2—3x + 2 = 0 } B={ x : x2+4x –12 = 0 }

Then B – A = ?

A ={x : x2—3x + 2 = 0 }

x2—3x + 2 = 0

(X—1)(X—2)=0

A = {1,2}

B={ x : x2+4x –12 = 0 }

x2+4x –12 = 0

(X—2)(X+6)=0

B = {2,--6}

B—A= {2,--6} – {1, 2}

={--6}

CA QUESTION 17 ON FUNCTIONS AND RELATIONS:

IF f : R  R , f(x) = x + 1 , g : R  R , g (x) = x2 + 1 , then fog(--2) = ?

fog(--2) = f(g(--2))

= g (--2) = (--2)2 + 1 = 5

= f (5)

=5+1=6.

CA QUESTION 18 ON FUNCTIONS AND RELATIONS:

IF f(x—1 ) = x2—4x + 8 , then f(x+1)= ?

Solution :
f(x—1)= x2—4x + 8

= (x2—2x +1)—2x+7

= (x—1)2—2x+2+7—2

= (x—1)2—2(x--1)+5

So f(x+1)= (x+1)2—2(x+1)+5

=(x2+2x +1)—2x—2+5

= x2+4 .

CA QUESTION 19 ON FUNCTIONS AND RELATIONS:

There are 40 students , 30 of them passed in English , 25 of them passed in maths and 15 of
them passed in both. Assuming that every student had passed atleast in one subject . how
many student passed in English only but not in maths .

Solution :

Total no of students passed in English = 30

Total no of students passed in maths = 25

Total no of students passed in both = 15

Total no of student passed in English only = 30 – 15 = 15

CA QUESTION 20 ON FUNCTIONS AND RELATIONS:

If f(x)= x / √ 1+ x2 and g(x) = x / √ 1−x 2 , then find (fog)

Solution :

(fog) = (f(g(x))

=f {x / √ 1−x 2 }


x / √ 1+ x2 / 1−(x / √1+ x 2) }
2
=

=x.

CA QUESTION 21 ON FUNCTIONS AND RELATIONS:

If f(x)= 3 + x for –3<x<0 and 3—2x for 0<x<3 , then value of f(2) will be :

Solution :

f(x)= 3 + x if –3<x<0

f(x)= 3—2x if 0<x<3

2 lies in 0<x<3

f(x)= 3—2x

f(2)= 3—2(2)

= --1 .

CA QUESTION 22 ON FUNCTIONS AND RELATIONS:

If A = {1,2,3,4,5} and B= {2,4} and C = {1,3,5} then (A—C) x B IS

SOLUTION :

A—C= {1,2,3,4,5} – {1,3,5}

= {2,4}

(A—C) X B = {2,4} X {2,4}

= {(2,2)(2,4)(4,2)(4,4)}

CA QUESTION 23 ON FUNCTIONS AND RELATIONS:

The no of proper subsets of the set {3,4,5,6,7} is ?

Solution :

No of proper subsets = 2n – 1 where n=5

= 25 -- 1

= 32 – 1 = 31

CA QUESTION 24 ON FUNCTIONS AND RELATIONS:


The range of the function f : N N ; f(x) = (--1)x—1 is ?

Solution :

If x=1 f(1) = (--1)1—1 = 1

If x=2 f(2) = (--1)2—1 = --1

If x=3 f(3) = (--1)3—1 = 1

If x=4 f(4) = (--1)4—1 = --1

So the range of the function is {1,--1}

CA QUESTION 25 ON FUNCTIONS AND RELATIONS:

For a group of 200 persons , 100 are interested in music , 70 in photography and 40 in swimming
, furthermore 40 are interested in both music and photography , 30 in both music and
swimming , 20 in photography and swimming and 10 in all the three . How many are interested
in photography but not in music and swimming ?

Solution :

Persons interested in photography but not in music and swimming = 20

CA QUESTION 26 ON FUNCTIONS AND RELATIONS:

If f : R  R is a function defined by f(x) = 10x – 7 , if g(x) = f —1(x) , then g(x) = ?

Solution

f—1(x) = 10x – 7

y = 10x – 7

y + 7 = 10x

(y + 7)/10 = x

(x + 7)/10 = f—1(x)

CA QUESTION 27 ON FUNCTIONS AND RELATIONS:

If f(x) = x + 2 , g(x) = 7x , then gof(x)

Solution :

g(f(x)) = g(x + 2)

= 7x + 2
= 7x 72

= 49 (7x)

CA QUESTION 28 ON FUNCTIONS AND RELATIONS:

If f(x) = log {(1 + x)/(1—x)} , then f{2x/(1+x 2)} = ?

Solution :

f{2x/(1+x2)} = log { [ 1 + 2x/(1+x2)] /[ 1 -- 2x/(1+x2) ] }

= log { [ (1 + x2 + 2x)/(1+x2)] /[ (1 + x2 -- 2x)/(1+x2) ] }

= log { (1 + x2 + 2x) / (1 + x2 -- 2x) }

= log { (1 + x )2 / (1 – x ) 2 }

= log { (1 + x ) / (1 – x ) } 2

= 2 log { (1 + x ) / (1 – x ) }

= 2 f (x)

CA QUESTION 29 ON FUNCTIONS AND RELATIONS:

If f(x) = (a—xn)1/n , a > 0 and n is a positive integer , then f(f(x))

Solution :

f(x) = (a—xn)1/n , a > 0

f(f(x)) = f((a—xn)1/n)

= (a—((a—xn)1/n)n)1/n)

= (a – a + xn )1/n

=x.

CA QUESTION 30 ON FUNCTIONS AND RELATIONS:

Of the 200 candidates , who were interviewed for a position at a call center , 100 had a two
wheeler , 70 had a credit card and 140 had a mobile phone , 40 of them had both two wheeler
and a credit card , 30 had both credit card and mobile phone , 60 had both two wheeler and
mobile phone and 10 had all the three . how many candidates had none of these ?

Solution :
n(A) = 100 n(B) = 70 n(C) = 140 n(A∩B) = 40 n(B∩C) = 30 n(A∩C) = 60 n(A∩B∩C) = 10

n(AUBUC)= n(A) + n(B) + n(C) -- n(A∩B) -- n(B∩C) -- n(A∩C) + n(A∩B∩C)

n(AUBUC)=100+70+140—40—30—60+10

n(AUBUC)=190

THE NO OF CANDIDATES USING NONE OF THESE

= 200 -- n(AUBUC)

=200 -- 190

= 10 .

CA QUESTION 31 ON FUNCTIONS AND RELATIONS:

If f(x) = X2 – 25 /(x—5) then f(5) is ?

Solution :

f(x) = X2 – 25 /(x—5)

f(5) = (5)2 – 25 /(5—5)

= 25 – 25 / 5 – 5

= not defined .

CA QUESTION 32 ON FUNCTIONS AND RELATIONS:

In a class of 50 students , 35 choose mathematics and 37 choose commerce , the no of students


who choose both are ?

n(AUB) = 50

n(A) = 35

n(B) = 37

n(A∩B) = ?

n(AUB) = n(A) + n(B) -- n(A∩B)

50 = 35 + 37 -- n(A∩B)

n(A∩B) = 72 – 55

= 22 .
CA QUESTION 33 ON FUNCTIONS AND RELATIONS:

If A = {2,3} B = {4,5} C = {5,6} then A x (B∩C) = ?

SOLUTION:

B = {4,5} C = {5,6}

(B∩C) = {5}

A x (B∩C) = {2,3} x {5}

= {(2,5) (3,5)} .

CA QUESTION 34 ON FUNCTIONS AND RELATIONS:

If f(x) = x / (x – 1) , then f(x/y) / f(y/x) is ?

Solution :

f(x) = x / (x – 1)

f(x/y) = (x/y) / ((x/y) – 1)

= (x/y) / ((x--y) / y)

= x / (x—y)

f(y/x) = (y/x) / ((y/x) – 1)

= (y/x) / ((y--x) / x)

= y / (y—x)

f(x/y) / f(y/x)
= x / (x—y) / y / (y—x)

= [x / (x—y)] / [y /-- (x—y)]

= -- x/y .

CA QUESTION 35 ON FUNCTIONS AND RELATIONS:

In a class of 80 students , 35% students can play only cricket , 45% students can play only table
tennis and the remaining students can play both the games . In all how many students can play
cricket ?

Solution :
n(A) = 35% OF 80 = 28 + x

n(B) = 45% OF 80 = 36 + x

n(A∩B) = x

n(AUB)= 80

n(AUB) = n(A) + n(B) -- n(A∩B)

80 = 28 + x + 36 + x – x

80 = 64 + x

x = 16 .

no of students can play cricket = 28 + x

= 28 + 16 = 44.

CA QUESTION 36 ON FUNCTIONS AND RELATIONS:

If f(x) = 2x + 2 and g(x) = x2 , then the value of fog(4) ?

Solution :

fog(x) = f(g(x))

= f(x2)

= 2x2 + 2

fog(4) = 2(4)2 + 2

= 32 + 2 = 34.

CA QUESTION 37 ON FUNCTIONS AND RELATIONS:

If SET A = {x : (x/2) belongs to Z , 0≤x≤10}

SET B = { x : x is one digit prime number }

SET C = { x : (x/3) belongs to N , x≤12 }

Then A∩(B∩C) = ?

SOLUTION :

SET A = { 0,1,2,3,4,5 }
SET B = { 2,3,5,7 }

SET C = { 1,2,3,4 }

B∩C = { 2,3 }

A∩(B∩C) = { 2,3 } .

CA QUESTION 38 ON FUNCTIONS AND RELATIONS:

THE DOMAIN (D) AND RANGE (R) OF THE FUNCTION f(x) = 2 -- | x + 1 | is?

Solution :

Domain = real numbers

f(x) = 2 -- | x + 1 |

y = 2 -- | x + 1 |

| x + 1 | = 2 -- y

±( x + 1 ) = 2 -- y

if

+(x+1) = 2 – y

x=1–y

if

-- (x+1) = 2 – y

x = y -- 3

so the range is {-- ∞ , 2}


PERMUTATIONS AND COMBINATIONS

CPT QUESTION 1 ON PERMUTATION

DEFINE PERMUTATION.

SOLUTION

PERMUTATION MEANS AN ARRANGEMENT THAT CAN BE MADE BY TAKING SOME OR ALL


NUMBER OF THINGS.

CPT QUESTION 2 ON PERMUTATION

YOU HAVE 3 HOLDAYS . BUT YOU HAVE 4 FILMS . IF A FILM A DAY, IN HOW MANY WAYS CAN
YOU PLAN YOUR FILM SEEING WITHOUT REPEAT OF SAME FILM ?

SOLUTION:
npr

= n! / (n—r)!

4p3.

=4! / (4—3)!

=4!/1!

=4X3X2X1/ 1

=24 WAYS.

234
243
324
432
423
342

134
143
314
341
431
413

124
142
214
241
421
412

123
132
213
231
312
321

CPT QUESTION 3 ON PERMUTATION

YOU HAVE 3 HOLDAYS . BUT YOU HAVE 5 FILMS . IF A FILM A DAY, IN HOW MANY WAYS CAN
YOU PLAN YOUR FILM SEEING WITHOUT REPEAT OF SAME FILM ?

SOLUTION:
5p3.

=5! / (5—3)!

=5!/2!

=5X4X3X2X1/ 2

=60 WAYS.

Explanation:

12 --6

13-6

146

156

23 --6

24-6

256

346

356

456

CPT QUESTION 4 ON PERMUTATION

YOU HAVE 9 DIGITS 1 TO 9 . YOU HAVE TO FIGURE OUT A 4 DIGIT CODE WITHOUT DIGIT
REPETITION IN THE SAME CODE . HOW MANY DIFFERENT CODES CAN BE GENERATED?

SOLUTION:

9p4.

=9! / (9—4)!

=9!/5!

=9X8X7X6X5X4X3X2X1/ 5X4X3X2X1

=9X8X7X6=3024
CPT QUESTION 5 ON PERMUTATION

YOU HAVE 9 DIGITS 1 TO 9 . YOU HAVE TO FIGURE OUT A 4 DIGIT CODE .IF REPETITION IN OF
DIGIT IN THE SAME CODE IS ALLOWED, HOW MANY DIFFERENT CODES CAN BE GENERATED?

ANSWER :

Nn = 94 =6561.

CPT QUESTION 6 ON PERMUTATION

WHETHER npr = np(n—r) ?

SOLUTION:

NO.

PROOF.

5P4.

n=5

r=4

n—r=1

npr = 5p4=120

np(n—r)= 5p(5—4)= 5p1=5

=5! / (5—4)!

=5!/1!

=5X4X3X2X1/ 1

=120

SO IT IS PROVED THAT

npr = n p(n—r)
CPT QUESTION 7 ON PERMUTATION

If 6p2 = 6p(6—r) ,find r.

Solution

Given that 6p2 = 6p(6—r)

2=6—r

r=6—2

r=4.

--------------------------------------------------------------------------

CPT QUESTION 8 ON PERMUTATION

np3 = 24

Find n.

Solution :

np3 can be expressed as

n!

---------

(n—3)!

n x (n-1) x (n-1-1)x(n-1-1-1)….

--------------------------------------

(n-3) x(n-3-1)x(n-3-1-1)……

n x (n-1) x (n-2)x(n-3)….

--------------------------------------

(n-3) x(n-4)x(n-5)……

= n x (n-1) x (n-2)
=24

n x (n-1) x (n-2)=24

(n2—n)(n—2)=24

n3—2n2 –n2 +2n=24

n3 –3n2 +2n--24=0

n3 –3n2 +2n=24

n=4

CPT QUESTION 9 ON PERMUTATION

np4 = 10 [ np3]

Find n.

Solution :

np4 =

n!

------

(n—4) !

np3 =

n!

------

(n—3) !

np4 = 10 [ np3]

n! 10 n!

------ = ----------------

(n—4) ! (n—3) !
1 10 (1)

------ = ----------------

(n—4) ! (n—3) !

1 10

------ = ----------------

(n—4) ! (n—3) !

1 10

------ = ----------------

(n—4)x(n—5)x(n—6)… (n—3)(n—4)(n—5)(n—6)…

1 10

------ = ----------------

1 (n—3)

n—3=10

n=13

CPT QUESTION 10 ON PERMUTATION

npr can be expressed as P( n,r) say true or false.

Solution : true.

CPT QUESTION 11 ON PERMUTATION

10p6 = P( 10,r) .Find r

Solution :

10p6= 10pr

So r= 6
CPT QUESTION 12 ON PERMUTATION

10p3 = P( 10,r) .Find r

[A]4

[B]3

[C]0

[D]NONE OF THESE

Solution : r= 3

CPT QUESTION 13 ON PERMUTATION

P [ 10 , (r+1)] : P[ 11 , r]= 30 : 11

FIND r.

SOLUTION:

10p(r+1) : 11pr = 30 : 11

11x10 ! 30x11!

-------------- = ----------------------

(10—r—1 )! (11—r) !

11x1 30x11

-------------- = ----------------------

(10—r—1 )! (11—r) !

1 30

-------------- = ----------------------
(9—r)! (11—r) !

30(9—r)! = (11—r) !

30(9—r)(8—r)(7—r)… = (11—r)(10—r)(9—r)(8—r)…

30= (11—r)(10—r)

30=110—11r—10r+r2

110—11r—10r+r2—30 =0

r2—21r+80 =0

solving the above quadratic equation

a=1

b= --21

c=80

−b ± √ b 2−4 ac
2a

21± √ 441−320
2

21± √ 121
2

21± 11

---------

21
r=16.

Or

r=5

but r=5 fulfils permutation.

So r= 5.

----------------------------------------------------------------------------

CPT QUESTION 14 ON PERMUTATION

P[ 11 , r] = P[ 12 , (r—1)]

FIND r.

SOLUTION:

11 ! 12!

-------------- = ----------------------

(11—r )! (13—r) !

1 12

-------------- = ----------------------

1 (13—r)(12—r)

(13—r)(12—r)=12

r=16 or 9

but for permutation r=9.

CPT QUESTION 15 ON PERMUTATION

np4= 20 [np2].

Find n.

Solution :
n! [20] n!

----- = ----------------

(n—4)! [n—2]!

1 [20] 1

----- = ----------------

(n—4)! [n—2]!

1 20

----- = ----------------

(n—4)(n—5)(n—6)… (n—2)(n—3)….

1 20

----- = ----------------

1 (n—2)(n—3)

20=(n—2)(n—3)

n=7.

CPT QUESTION 16 ON PERMUTATION

The letters of the word “VIOLENT” are arranged so that the vowels occupy even places only.
How many permutations [arrangements] are possible ?

Solution :

V I O L E N T

1 2 3 4 5 6 7

4 WAYS 3 WAYS 3 WAYS 2 WAYS 2 WAYS 1 WAY 1 WAY


VOWELS ARE I,O,E

SO PERMUTATIONS ARE
4WAYSX3WAYSX3WAYSX2WAYSX2WAYSX1WAY=144 WAYS.

CPT QUESTION 17 ON PERMUTATION

The letters of the word “REGULATION” are arranged so that the vowels occupy odd places
only. How many permutations [arrangements] are possible ?

Solution :

R E G U L A T I O N

1 2 3 4 5 6 7 8 9 10
5WAYS 5WAY 4 WAYS 4WAY 3WAYS 3WAY 2 WAYS 2WAY 1 1WAY
WAY

VOWELS ARE E,U,A,I,O=5

SO PERMUTATIONS ARE

5WAYSX5WAYSX4WAYSX4WAYSX3WAYSX3WAYSX2WAYSX2WAYSX1WAYX1WAY=

14400 WAYS.

CPT QUESTION 18 ON PERMUTATION

Find the number of arrangements in which the letters of the word “FAILURE” are arranged so
that consonents occupy odd places.

Solution :

F A I L U R E

1 2 3 4 5 6 7

3WAYS 4 WAYS 2 WAYS 3 WAYS 1 WAY 2 WAYS 1 WAY


CONSONENTS =F,L,R3ODD PLACES4

VOVELS =A,I,U,E4EVEN PLACE/ODD PLACE4

 4 ARE ODD PLACES AND 3 RESERVED FOR CONSONENTS WHICH ARE F,L AND R
4P3 WAYS

 BALANCE 4 PLACES ARE THERE AND 4 LETTERS ARE THERE.


4P4
SO PERMUATION = 4P3 X4P4=576

CPT QUESTION 19 ON PERMUTATION

Find the number of arrangements in which the letters of the word “ORIENTAL” are arranged so
that A and E OCCUPY odd places.

Solution :

O R I E N T A L

1 2 3 4 5 6 7 8

 4 ARE ODD PLACES AND 2 RESERVED FOR VOWELS WHICH ARE A AND E

4P2 WAYS

 BALANCE 6 PLACES ARE THERE AND 6 LETTERS ARE THERE.


6P6
SO PERMUATION = 4P2 X6P6=8640

CPT QUESTION 20 ON PERMUTATION

THE NUMBER OF WAYS OF ARRANGING 6 BOYS AND 4 GIRLS IN A ROW SO THAT ALL 4 GIRLS
ARE TOGETHER =?

SOLUTION:

7 ! X 4! =120960

CPT QUESTION 21 ON PERMUTATION


THE NUMBER OF WAYS OF DEVELOPING A 6 DIGIT TELEPHONE NUMBER USING 10 DISTINCT
DIGITS =?

SOLUTION:

10P6=151200

OR

10 X9X8X7X6X5=151200

CPT QUESTION 22 ON PERMUTATION

HOW MANY NUMBERS NOT EXCEEDING 1000 CAN BE MADE FROM THE DIGITS 1,2,3,4,5,6,7,8
AND 9 IF REPETITION IS NOT ALLOWED?

SOLUTION:

ONE DIGIT NUMBERS =9P1=9

TWO DIGIT NUMBERS=9P2=72

THREE DIGIT NUMBERS=9P3=504

SO TOTAL PERMUTATION=9 + 72 + 504=585

9 + 9X8 + 9X8X7= 585

CPT QUESTION 23 ON PERMUTATION

A GARDEN IS HAVING 6 TALL TREES IN A ROW.IN HOW MANY WAYS 5 CHILDREN STAND, ONE
IN A GAP BETWEEN THE TREES IN ORDER TO POSE FOR A PHOTOGRAPH?

SOLUTION:
T1 G1 T2 G2 T3 G3 T4 G4 T5 G5 T6
5!=120

CPT QUESTION 24 ON PERMUTATION

THERE ARE 12 QUESTIONS TO BE ANSWERED “TRUE” OR “FALSE” TYPE. HOW MANY


DIFFERENT ARRANGEMENTS IN THE ANSWER SHEET OF A STUDENT WHO TRIES HIS LUCK
WITHOUT KNOWLEDGE CAN BE THERE ?

SOLUTION:

2X2X2X2X2X2X2X2X2X2X2X2=4096

CPT QUESTION 25 ON PERMUTATION

CONSIDER THE NAME “ RAMALAKSHMI”. HOW MANY PERMUATATIONS ARE POSSIBLE FOR A
11 LETTER CODE WORD USING THE LETTERS OF THE ABOVE NAME.

SOLUTION :

OBSERVE THAT THERE ARE 11 LETTERS IN THIS NAME

 R appears for 1 time


 A appears for 3 times
 M appears for 2 times
 L appears for 1 time
 K appears for 1 time
 S appears for 1 time
 H appears for 1 time
 I appears for 1 time.
--------------
11 times
--------------
An analyst wants to coin a 11 letter PASSWORD from the above 11 letters.
How many such different passwords can the analyst form such that in the same
password letter repetition is not allowed ?

Answer

11 !
----------------
3 ! x 2!

=11x10x9x8x7x6x5x4x3x2x1

------------------------------------

3x2x1 x2x1

=399,16,800

------------------

12

=33,26,400 different pass words can be formed.

-------------------------------------------------------------------------

CPT QUESTION 26 ON PERMUTATION

CONSIDER THE word “ LIBERTY”. HOW MANY PERMUATATIONS ARE POSSIBLE FOR A 7 LETTER
CODE WORD USING THE LETTERS OF THE ABOVE WORD.

SOLUTION :

OBSERVE THAT THERE ARE 7 LETTERS IN THIS NAME

 L appears for 1 time


 I appears for 1 time
 B appears for 1 time
 E appears for 1 time
 R appears for 1 time
 T appears for 1 time
 Y appears for 1 time
----------
7 times
--------------
An analyst wants to coin a 7 letter PASSWORD from the above 7 letters.
How many such different passwords can the analyst form ?

Answer

7!
=5,040 different pass words can be formed.
CPT QUESTION 27 ON PERMUTATION

CONSIDER THE name “ VIOLENI”. HOW MANY PERMUATATIONS ARE POSSIBLE FOR A 7 LETTER
CODE WORD USING THE LETTERS OF THE ABOVE WORD.

SOLUTION :

7!
---- = 2520.
2!

CPT QUESTION 28 ON PERMUTATION

How many words can be formed with all the letters of the word “ARTICLE ” such that vowels
have to occupy EVEN places ?

Solution :

OBSERVE THAT THERE ARE 7 LETTERS IN THIS WORD

 A appears for 1 time VOWEL


 R appears for 1 time CONSONANT
 T appears for 1 time CONSONANT
 I appears for 1 time VOWEL
 C appears for 1 time CONSONANT
 L appears for 1 time CONSONENT
 E appears for 1 time VOWEL

---------- ------------
7 times 3 VOWEL & 4 CONSONENTS
--------------
An analyst wants to coin a 7 letter PASSWORD from the above 7 letters such that in all
the pass words, VOWELS have to appear in the EVEN PLACES only.
How many such different passwords can the analyst form ?

Answer
Vowels are A,E,I,O,U
Consonents are B,C,D,F,G,H,J,K,L,M,N,P,Q,R,S,T,V,W,X,Y,Z
1 2 3 4 5 6 7
THERE ARE 3 EVEN PLACES AND THERE ARE 3 VOWELS .
PERMUTATIONS ARE 3! = 3X2X1 =6

THERE ARE 4 OTHER PLACES AND THERE ARE 4 OTHERS .


PERMUTATIONS ARE 4! =4x3X2X1=24

ANSWER
6 X24 =144.

CPT QUESTION 29 ON PERMUTATION

How many words can be formed with all the letters of the word “MONDAY ” such that the
beginning letter is M but the ending letter should not be N ?

Solution :

OBSERVE THAT THERE ARE 7 LETTERS IN THIS WORD

 M appears for 1 time


 O appears for 1 time
 N appears for 1 time
 D appears for 1 time
 A appears for 1 time
 Y appears for 1 time

----------
6 times
--------------
An analyst wants to coin a 6 letter PASSWORD from the above 6 letters such that in all
the pass words, M is the beginning letter and N should not be the ending letter.
How many such different passwords can the analyst form ?

Answer
1 2 3 4 5 6
1 way 4 WAYS 3 WAYS 2 WAYS 1 WAY 4 ways
reserved de reservation
for M for N
M N AND 3 LETTERS=4 LETTERS O,D,A,Y
ANSWER =1WAYX4WAYSC3WAYSX2WAYSX1WAYX4WAYS=
96 WAYS

CPT QUESTION 30 ON PERMUTATION

How many words can be formed with all the letters of the word “TUESDAY ” such that the
beginning letter is D but the ending letter should not be U ?

1 2 3 4 5 6 7
1 way 5 WAYS 4 WAYS 3 WAYS 2 WAYS 1 WAY 5 ways
reserved de reservation
for D for U
D U AND 4 LETTERS=5 LETTERS T,E,S,A,Y
ANSWER =1WAY X 5WAYS X 4 WAYS X 3WAYS X 2WAYS X1WAY X 5WAYS= 600 WAYS

CPT QUESTION 31 ON PERMUTATION

How many permutations can be formed from all the letters of the word “DRAUGHT”, if
both the vowels are not separated ?

SOLUTION
D appears for 1 time Consonant
R appears for 1 time Consonant
A appears for 1 time Vowel
U appears for 1 time Vowel
G appears for 1 time Consonant
H appears for 1 time Consonant
T appears for 1 time Consonant
---
7 times 2 vowels.
-----
6 letter word
DRAUGHT
1 2 3 4 5 6
6! X2!
=6X5X4X3X2X1X2X1 =1440 WAYS

CPT QUESTION 32 ON PERMUTATION

How many permutations can be formed from all the letters of the word “DRAUOGHT”, if
the three vowels are not separated ?

SOLUTION
D appears for 1 time Consonant
R appears for 1 time Consonant
A appears for 1 time Vowel
U appears for 1 time Vowel
O appears for 1 time Vowel
G appears for 1 time Consonant
H appears for 1 time Consonant
T appears for 1 time Consonant
---
8 times 3 vowels.

-----
6 letter word
DRAUOGHT
1 2 3 4 5 6

6! X3!
=6X5X4X3X2X1X3X2X1 =4320 WAYS

CPT QUESTION 34 ON PERMUTATION

There are five students who wish to dine in a circular dining table. In how many
different ways can they sit and dine.
Solution :
[5—1]!
=4!
=4x3x2x1=24

CPT QUESTION 35 ON PERMUTATION

5 oranges and 4 mangoes have to be arranged in a row for fruit display by a stall owner.
Mangoes have to occupy even places only. How many shows can the stall owner create
which are different in whole look.
Solution :
Oranges= 5
Mangoes=4
1 2 3 4 5 6 7 8 9
Even places=4
Mangoes=4
4 mangoes can be placed in 4 even places in 4! Ways=4x3x2x1=24 ways.
5 odd places are there and five oranges are there and these can be placed in 5! Ways=
5x4x3x2x1=120 ways
Answer
24x120=2880

CPT QUESTION 36 ON PERMUTATION

There are 3 nights and 4 days in a tour. 3 hot places and 4 cool places are to be visited
from your lodge and returned. In how many ways can you visit all places such that
during nights you visit hot places ?

ANSWER
3!X4!=144 WAYS.

CPT QUESTION 37 ON PERMUTATION

There are 5 books in Malayalam, 4 books in Telugu and 3 books in Hindi which you have
won in various competitions. You have a shelf for knowledge display in your reception
hall. You need to properly arrange these 12 books in a row in such a way that the
language of the books need to be kept along.

Solution :
Malayalam books=5
Telugu books=4
Sindhi books=3
Answer
5!x4!x3!X3!=1,03,680

CPT QUESTION 38 ON PERMUTATION


There are 4 boys and 5 girls to stand for a group photograph on a row in such a that all
girls have to be together and all the boys have to be together . The photographer can
make how many different snaps which gives different whole looks ?

Solution :
Girls=5
Boys=4
Answer
2! X 5!x4!= 2x1x5x4x3x2x1x4x3x2x1=5,760.

CPT QUESTION 39 ON PERMUTATION

In a centre there are 3 teachers. In the year 2017, the centre management plans to give
leave to the teachers with pay on their birth day. Find the number of ways such that
birthday leave falls on different dates for different teachers.
Solution :
Number of days in the year 2017=365 days
Number of teachers=3
1st teacher’s birth day can fall in any one of the 365 days =365 ways.
2nd teacher’s birth day can fall in any one of the 364 days =364 ways.
3rd teacher’s birth day can fall in any one of the 363 days =363 ways.

So the number of ways in which each teacher can celebrate his /her birth day on
different dates=

365 x364x363=482,28,180 ways.

CPT QUESTION 40 ON PERMUTATION

In a centre there are 3 teachers. In the year 2020, the centre management plans to give
leave to the teachers with pay on their birth day. Find the number of ways such that
birthday leave falls on different dates for different teachers.

ANSWER =366 WAYS X 365 WAYSX364 WAYS=486,26,760 WAYS.

ncr= n!
------------------
[n—r]! x r!

CPT QUESTION 41 ON PERMUTATION


There are 10 persons. They wish to say namaskaram to each other.How many
namaskarams will be there ?

Solution :
Persons=10
To complete a namaskaram we need two persons.
So answer 10c2 = 10!/ [( 10—2)! X 2! ]
=
10!/ [ 8! X 2!]
=
10x9
-------
2x1
=45 namaskarams.

ncr =

n!
----------------
[n—r]! x r !

CPT QUESTION 42 ON PERMUTATION


In a shape there are n corners [or points]. How many diagonals can you form in this
shape ?
Answer :
nc2--n

CPT QUESTION 43 ON PERMUTATION


A square has 4 sides and 4 points. How many diagonals can you draw in a square ?
Answer
nc2--n
4c2—4
6—4 =2.

CPT QUESTION 44 ON PERMUTATION


A triangle has 3 sides and 3 points. How many diagonals can you draw in a triangle ?
Answer
nc2--n
3c2—3
3—3 =0.

CPT QUESTION 45 ON PERMUTATION


A shape has 5 sides and 5 points. How many diagonals can you draw in the shape?
Answer
nc2--n
5c2—5
10—5 =5.

CPT QUESTION 46 ON PERMUTATION


A shape has 10 sides and 10 points. How many diagonals can you draw in the shape ?
Answer
nc2--n
10c2—10
45—10 =35.

CPT QUESTION 47 ON PERMUTATION


A shape has 14 diagonals. How many sides does this shape have?

Answer
nc2--n
nc2—n =14

n!
---------------- --n =14
[n—2]! X 2 !

n x [n—1] x[n—2]x[n—3]…
--------------------------------------- --n =14
[n—2]x[n—3]… X 2

n x [n—1] x[n—2]x[n—3]…
--------------------------------------- --n =14
[n—2]x[n—3]… X 2

n x [n—1]
--------------- -- n =14
2

n[n—1]—2n
------------------ =14
2
n[n—1]—2n=28
n2—n—2n—28=0
n2—3n—28=0
n=7

CPT QUESTION 48 ON PERMUTATION


A polygon [shape] has 44 diagonals. How many
Sides it has?

Answer
nc2--n
nc2—n =44

n!
---------------- --n =44
[n—2]! X 2 !

n x [n—1] x[n—2]x[n—3]…
--------------------------------------- --n =44
[n—2]x[n—3]… X 2

n x [n—1] x[n—2]x[n—3]…
--------------------------------------- --n =44
[n—2]x[n—3]… X 2

n x [n—1]
--------------------------------------- --n =44
2

n[n—1]—2n
------------------ =44
2
n[n—1]—2n=88
n2—n—2n—88=0
n2—3n—88=0
n=11.
---------------------------------------------------------------

Important equation on combination.


ncr + nc(r—1)= (n+1) c r
proof with imaginary numbers.
4c3 + 4c(3—1) = (4+1) c 3
4c3 + 4c2 = 5c3
4 +6 =10
-----------------------------------------------------------
CPT QUESTION 49 ON COMBINATION
Find x if
10c3 + 10c2 = xc 3
Solution :
Let us use the equation
ncr + nc(r—1)= (n+1) c r
10c3 + 10c(3—1) = (10+1)c 3
10c3 + 10c2 = 11c 3
So x=11
--------------------------------------------------------------------
Another equation
ncr = n c(n—r)
proof with imaginary numbers.
4c1 = 4c(4—1)
4c1=4c3
----------------------------------------------------------------------
CPT QUESTION 50 ON PERMUTATION

Find x if
1000c98= 999c97 + xc901
Solution :
Let us try to use the equation
ncr + nc(r—1) = (n+1)cr
 999c97 can be written as
999c(999—97) =999c902 because
ncr= = nc(n—r)
1000c98= 999c97 + xc901
1000c98= 999c902 + xc901
999c902 + xc901 =1000C98
999c902 + xc901 =1000C902
THIS IS OF THE FORM
ncr + nc(r—1) = (n+1)cr
so x= 999
CPT QUESTION 51 ON PERMUTATION
Find x if
13c6 + 2 [ 13c5] + 13c4 = 15cx

Solution :
13c6 + 13c5+ 13c5 + 13c4 = 15cx
(13c6 + 13c5 ) + ( 13c5 + 13c4 ) = 15cx
 ncr + nc(r—1) = (n+1) c r
 (13c6 + 13c5 ) + ( 13c5 + 13c4 ) = 15cx
14c6 + 14c 5 = 15c x
ncr + nc(r—1)= (n+1) c r
14c6 + 14c 5 = 15c x
X=6

----------------------------------------------------------------------
CPT QUESTION 52 ON PERMUTATION
Find n and r if ncr + nc(r—1) = 12 c 5
Solution :
we know that
ncr + nc(r-1) = (n+1) c r
ncr + nc(r—1) = 12 c 5 can be written as
ncr + nc(r—1) = (11+1) c 5
so n= 11
r=5
CPT QUESTION 53 ON PERMUTATION
Find n and r if
ncr + 15c(r—1) = 16 c7

solution : n=15 , r=7


CPT QUESTION 54 ON PERMUTATION
If 10c7=nc(10—r) find n and r.
solution :
n=10
r=7 or 3
----------------------------------------------------------------------
CPT QUESTION 55 ON PERMUTATION
15c3 + 15 c13 =
[a] 16 c3
[b] 30c16
[c] 15c16
[d]15c15

Solution :
15c3 + 15 c13 can be written as
15c3 +15c2
Because 15c13 can be written as 15c(15—13)=15c2
Remember the equation
ncr = nc(n—r)
We know that
ncr + nc(r—1) =(n+1) cr
15c3 +15c(3-1)= (15+1) c 3
15c3 +15c(2)= 16 c 3
----------------------------------------------------------------------
CPT QUESTION 56 ON PERMUTATION
If p(7,k) = [60] p (7, k—3) , find k
(A)9
(B)8
(C)5
(D)0
Solution :
We know that npr can be written as p(n,r)
= n!
------------
(n-r)!
So
p(7,k) = 60 p (7, k—3)
LHS
p(7,k) = 7pk = 7!
-----
(7—k) !

7! 7x6x5x4x3x2x1
----- = ---------------------
(7—k) ! (7—k) (7—k—1) (7—k—2)….

7x6x5x4x3x2x1
= ---------------------------------------------------------
(7—k) (7—k—1) (7—k—2)….

5040
= ---------------------------------------------------------
(7—k) (7—k—1) (7—k—2)….

5040
= ---------------------------------------------------------
(7—k) (6—k) (5—k)….

RHS
60 p (7, k—3)
KEEP 60 SEPARATELY

p (7, k—3)
CAN BE WRITTEN AS
7 p (k—3)
7!
--------------
(7—K+3)!

7!
--------------
(10—K)!

7X6X5X4X3X2X1
---------------------
(10—K)!

7X6X5X4X3X2X1
---------------------
(10—K)(10—K—1)(10—K—2)(10—K—3)…

5040
----------------------------------------------------
(10—K)(9—K)(8—K)(7—K)(6—K)(5—K)…

RHS =

60 X5040
----------------------------------------------------
(10—K)(9—K)(8—K)(7—K)(6—K)(5—K)…

LHS =RHS

5040 60X5040
--------------------------- = ---------------------------------------------------------
(7—k) (6—k) (5—k)…. (10—K)(9—K)(8—K)(7—K)(6—K)(5—K)…

60
1 = ---------------------------------------------------------
(10—K)(9—K)(8—K)
(10—K)(9—K)(8—K)=60
K=5
---------------------------------------------------------------------------------------------
TUTORIAL NOTE TO BE MEMORISED
npr can be written as p(n,r)

--------------------------------------------------------------------------------------------
CPT QUESTION 57 ON PERMUTATION
Assume 12 points on a graph sheet where, 7 of such 12 points are collinear.How many
triangles can you form from the above 12 points ?

Solution :
If 12 non collinear points are there in a graph, we can form 12c3 number of triangles
=220 triangles.

If 7 collinear points are there in a graph, we can not form 7c3 number of triangles =35
triangles.

So we can form
220—35 =185 triangles.
----------------------------------------------------------------------
CPT QUESTION 58 ON PERMUTATION
There are 20 points on the plane(graph), 15 of which are collinear. How many triangles
can you draw from this situation ?

Answer :

[20c3]—[15c3] =685 .

CPT QUESTION 59 ON PERMUTATION


Jaishree likes 26 alphabets and digits 1 to 9. Her boss asks her to coin a secret pass code
consisting of 2 distinct alphabets first and two distinct digits next. How many choices
are before the boss ?

Answer :
26x25x9x8 =46800

Explanation :
Alphabets are A B C D E F G H I J K L M N O P Q R S T U V W X YZ
Digits are 123456789

alphabet alphabet digit digit

First alphabet can be filled up in 26 ways


Second alphabet can be filled up in 25 ways
First digit can be filled up in 9 ways
Second digit can be filled up in 8 ways

So the answer
26x25x9x8=46,800 ways.

CPT QUESTION 60 ON PERMUTATION


You have to complete one income tax file per day . You have 3 working days only
before you go on CA final leave. But there are 8 income tax files. Your principal auditor
asks you to take any three cases such that Rajesh file if taken, Suresh file case has also
to be taken.
In how many ways can you complete three tax cases?

Solution :
 If Rajesh file is taken

6c1

 If Rajesh file is not taken

7c3

Answer = 6c1 + 7c3 = 6 +35=41 ways.

[1c1x1c1x6c1=6ways ] + [7c3ways=35 ways]= 41 ways.

CPT QUESTION 61 ON PERMUTATION


You have to complete one income tax file per day . You have 3 working days only
before you go for CA final leave. But there are 8 income tax files. Your principal auditor
asks you to take any three cases such that if Rajesh file if taken, suresh file has also to
be taken and if Suresh file is taken, Rajesh file has also to be taken.
In how many ways can you complete three tax cases?
Answer
Rajesh Suresh 3 4 5 6 7 8
1 Y Y 6c1
2 n n 6c3

Answer = 6c1 +6c3 =26 ways

CPT QUESTION 62 ON PERMUTATION


You have to select 8 students for a cleaning camp. You have 7 girl students and 5 boy
students ready for cleaning camp. The professor says that there should be atleast 3 girls
and atleast 3 boys in the cleaning team. In how many ways can you form up the
cleaning team ?
Solution :
Girls =7
Boys=5
Total=12.
1 3b 5g 8 5c3 x7c5=210
2 4b 4g 8 5c4 x7c4=175
3 5b 3g 8 5c5x7c3=035
420 ways

[7c3x5c5] + [7c4x5c4]+[7c5x5c3] = 420 ways.

1st approach
3 girls and 5 boys
7c3 x 5c5

2nd approach
5 girls and 3 boys
7c5 x 5c3

3rd approach
4 girls and 4 boys
7c4 x 5c4

Answer
1st approach + 2nd approach + 3rd approach
35 + 175 + 210 =420.
CPT QUESTION 63 ON PERMUTATION
You are an articled clerk in C.GOVINDARAJ & Co. Principal C.Govindaraj says that you
have to complete a total of 4 cases in this month choosing atleast 1 tax case and 1 audit
case. There are 4 tax cases and 2 audit cases. In how many ways can you frame your
work plan?

Solution :
tax =4
audit=2
Total=6.
1 t2 a2 4 4c2 x2c2=6
2 t3 a1 4 4c3x2c1=8
3
14 ways

Answer : 14
[4C2X2C2]+[4C3X2C1]= 14 WAYS.

CPT QUESTION 64 ON PERMUTATION


IN HOW MANY WAYS CAN A PARTY OF 4 MEN AND 4 WOMEN CAN BE SEATED AT A
ROUND TABLE SO THAT NO TWO WOMEN ARE ADJACENT ?

[A] 164 [B]174 [C] 144 [D] 480


ANSWER :
SOLUTION :

MEN WOMEN
CHAIR CHAIR
WOMEN MEN
CHAIR CHAIR

MEN WOMEN
CHAIR CHAIR
WOMEN MEN CHAIR
CHAIR

(n—1)! X 4!
3! X4!
=144

CPT QUESTION 65 ON PERMUTATION


IN HOW MANY WAYS CAN A PARTY OF 4 MEN AND 4 WOMEN CAN BE SEATED AT A
ROUND TABLE SO THAT TWO WOMEN ARE ADJACENT ?
[A] 164 [B]174 [C] 144 [D] 480

ANSWER :

5! X2!X2!=480
CPT QUESTION 66 ON PERMUTATION

5
∑ 5cr = ?
r=1
[a] 29 [b] 31 [c] 35 [d] 26

SOLUTION :
5C1 + 5C2 +5C3 +5C4 +5C5=31

CPT QUESTION 67 ON PERMUTATION


3
∑ 5pr = ?
r=1

Answer =85
5P1 + 5P2 + 5P3 =85

CPT QUESTION 68 ON PERMUTATION

If 6pr = 24 [ 6cr ] find r.

[a] 4 [b] 6 [c] 2 [d] 1

ANSWER =4

CPT QUESTION 69 ON PERMUTATION

7p3 can be written as


[a] 7p0 [b] 7p4 [c] 3p7 [d] none of these

ANSWER : NONE OF THESE

CPT QUESTION 70 ON PERMUTATION

7c3 can be written as

[a] 7c0 [b] 7c4 [c] 3c7 [d] none of these

ANSWER =7C4
CPT QUESTION 71 ON PERMUTATION

If 500 c98 = 499 c97 + x c 401 , find

[a] 500 [b] 497 [c] 498 [d] 499

SOLUTION
RECALL
ncr + nc(r—1) =(n+1)cr
499c98+499c97=500c98
500c98 can be written as 500c402 because ncr=nc(n—r)
499c97 can be written as 499c402
ncr + nc(r—1) =(n+1)cr
499c402 +xc401 =500c402
So x=499
CPT QUESTION 72 ON PERMUTATION

How many numbers greater than a million can be formed with the digits 4,5,5,0,4,5 and 3

[a] 260 [b] 360 [c] 280 [d] 380


Answer :
Million=10,00,000
4 appears for 2 times
5 appears for 3 times
0 appears for 1 time
3 appears for 1 time
----
7 times
----------
6 6 5 4 3 2 1

6x6x5x4x3x2x1
-----------------------
2! X 3!
=360

CPT QUESTION 73 ON PERMUTATION

If C(n,r) : C (n, r+1) = 1:2 and


If C(n, r+1) : C (n, r+2) =2:3
n and r respectively are
[a] 14 & 4
[b] 12 & 4
[c] 14 & 6
[d] none of these.

Solution :
Given that
C(n,r) : C (n, r+1) = 1:2
ncr : n c (r+1) =1:2

n! n!
-------- : --------------- = 1:2
(n—r)! r! (n—r—1)! (r+1)!
1 1
-------- : --------------- = 1:2
(n—r)! r! (n—r—1)! (r+1)!

We know that
(n—r)! = (n—r) (n—r—1) (n—r—1—1) …

1 1
-------- : --------------- = 1:2
(n—r)! r! (n—r—1)! (r+1)!

Product of extremes= product of means

2 1
-------- = ---------------
(n—r)! r! (n—r—1)! (r+1)!

2 (n—r—1)! (r+1)! = (n—r)! r!

****************************
2(n—r—1)(n—r—2)(n—r—3)…(r+1) (r)(r—1)(r—2)…
=
(n—r) (n—r—1) ( n—r—2) (n—r—3)… r(r—1)(r—2)(r—3)…

2(r+1)
=
(n—r)

2r+2=n—r
3r=n—2 ------------eqn 1

C(n, r+1) : C (n, r+2) =2:3

3 C(n, r+1) = 2 C (n, r+2)

3 [ nc(r+1)] = 2 [nc(r+2)]
We land in equation

2n—5r=8 = eqn [2]

solving equation [1] n—3r=2


and equation [2] 2n—5r=8

we get n=14

and r=4

so option [A]

CPT QUESTION 74 ON PERMUTATION

5 men and 4 women to sit in a row in such a manner that the woman always occupy the even
places. The number of such arrangements will be

[A] 2080 [B]2880 [C]126 [D]1056

Solution :

1 2 3 4 5 6 7 8 9
odd even odd even odd even odd even odd

There are 4 women

There are 4 even places

So women can sit in

4p4 ways.

There are 5 men

There are 5 odd places

So men can sit in

5p5 ways.

So the permutation for the arrangement

= 4p4 x5p5
=2880.

Answer : option[B]

CPT QUESTION 75 ON PERMUTATION

There are 5 books in English, 4 books in Tamil and 3 books in Hindi. In how many ways can these
books be placed on a shelf if the books on the same language are to be together.

[A] 183600 [B]136800 [C]163800 [D]103680

Solution :

5p4 x 4p4 x3p3 x 3p3 = 103680

Answer : option[D]

CPT QUESTION 76 ON PERMUTATION

Given that

6pr= 360,

find r

[A] 3 [B]4 [C]5 [D]none of these

Solution :

6pr=360

6!

-------------- =360

[6—r]!

6x5x4x3x2x1

------------------- =360

[6—r]!

720
------------------- =360

[6—r]!

720

------------------- =[6—r]!

360

2 =[6—r]!

2 can be written as 2!.

So

2! =[6—r]!

2=6—r

r=6—2

r=4.

So r= 4.

Answer : option[B]

CPT QUESTION 76 ON PERMUTATION

If six times the number of permutations of “n” items taken 3 at a time is equal to seven times the
number of permutations of (n—1) items taken 3 at a time , then value of n will be

[A] 9 [B]21 [C]7 [D]13

Solution :

6 [np3] = 7[(n—1)p3]

6[ n!/(n—3)!] = 7[(n—1)!/(n—1—3)!

6[ n!/(n—3)!] = 7[(n—1)!/(n—4)!

6[ n(n—1)!/(n—3)(n—4)!] = 7[(n—1)!/(n—4)!

6[ n(n—1)!/(n—3)(n—4)!] = 7[(n—1)!/(n—4)!
6[n/(n—3)] = 7

6n= 7(n—3)

6n=7n—21

n=21

Answer : option[B]

CPT QUESTION 77 ON PERMUTATION

In how many ways can a family consisting of three members with different birth days
celebrate their birth days in a leap year ?
[a] 365c3
[b]366c3
[c]366c3—3
[d]366x365x364

Answer :[d]

CPT QUESTION 78 ON PERMUTATION

If 15c3r = 15c(r+3)
find r
[a] 5
[b] 4
[c] 3
[d] 2
Solution :
We know that
ncr=nc(n—r)
15c3r =15c(15—3r)
15c(15—3r)= 15c(r+3)
So
15—3r=r+3
15—3=r+3r
12=4r
r=3
so the answer is option [c]
CPT QUESTION 79 ON PERMUTATION
Number of ways of shaking hands in a group of 10 persons shaking hands to each other
is
[a] 10
[b] 90
[c] 54
[d] 45

Solution :
This question resembles the number of ways of selecting any two persons from and out
of 10 persons .

So the answer is 10c2


=
10!
------------------
[10—2]! X 2!
=
10!
--------
8! X 2 !
=
10X9
------
2X1
=45
SO THE ANSWER IS OPTION[d]

CPT QUESTION 80 ON PERMUTATION

A man has 3 children and 6 schools at his area.In how many ways can the 3 children join
schools such that they study at different schools
[a] 36
[b]63
[c]6p3
[d]6c4
SOLUTION
Answer [c]

CPT QUESTION 81 ON PERMUTATION

A person has 3 children and he has to choose a dress model to each child for Diwali
2018 in such a way that out of 7 different designs available at Chennai Silks, he has to
select a design to each child differently for each child. In how many ways can he select
dress for his 3 children for Diwali shopping 2018 ?
SOLUTION
7p3.

CPT QUESTION 82 ON PERMUTATION

A team of 5 has to be chosen from 8 boys and 3 girls. How many combinations will have
two particular girls in the team ?
Answer :9c3

CPT QUESTION 83 ON PERMUTATION

Aravind does not know answers for 12 questions of true or false options only.He
decided to guess. In how many ways can he complete exam?
[a]4096
[b]2048
[c]1024
[d]500
Answer :
4096.

CPT QUESTION 84 ON PERMUTATION

In how many ways 3 prizes out of 5 prizes can be distributed amongst 3 students
equally?
[a]120
[b]60
[c]10
[d]45
Answer :
5c3
=10.

CPT QUESTION 85 ON PERMUTATION

Find the number of arrangements of 5 things taken out of 12 things, in which one
particular thing must always be included?
11p4=7920
CPT QUESTION 86 ON PERMUTATION

How many ways a team of 11 players can be selected out of 15 players if one particular
player is not to be selected in the team?
[a] 1,234
[b] 364
[c] 1001
[d] 728
Answer :
14c11=364
CPT QUESTION 87 ON PERMUTATION

Six points are on a circle. The number of quadrilaterals that can be formed are
[a]30
[b]15
[c]360
[d] none of these
SOLUTION:

6c4
=6!
-----
2! X4 !
=15.

CPT QUESTION 88 ON PERMUTATION

Out of 4 gents and 6 ladies, a committee is to be formed. Find the number of ways that
the committee can be formed such that it comprises of atleast 2 gents and atleast the
number of ladies should be double of gents?
[a]104
[b]136
[c] 94
[d] 132
SOLUTION:

[4c2 x 6c4] + [4c2x6c5]+ [4c2x6c6]+ [4c3x6c6]=136


So option[b]
CPT QUESTION 89 ON PERMUTATION

If [n+1]! =20[n—1]!, what is n ?


[a]10
[b]4
[c] 5
[d] 6
SOLUTION:

[n+1]! =20[n—1]!,
[n+1] x [n+1—1]x[n+1—1—1]… = 20[n—1]x[n—1—1]x[n—1—1—1]…
[n+1] x [n]x[n—1]… = 20[n—1]x[n—1—1]x[n—1—1—1]…
[n+1] x [n]x[n—1]… = 20[n—1]x[n—2]x[n—3]…
[n+1] [n] =20
n2 +n—20=0
−b ± √b 2−4 ac
n=
2a

−1± √ 1+ 80
n=
2

−1± √ 81
n=
2

−1± 9
n=
2
n= [--1+9]/2 =4
CPT QUESTION 90 ON PERMUTATION

In how many ways can 17 billiards balls be arranged if 7 of them are black,6 red and 4
white?

[a]8048040
[b]1
[c] 4084080
[d] 6

SOLUTION:

17!
---------------
7! X6!x4!

=4084080 = option [c]

CPT QUESTION 91 ON PERMUTATION

7 books are to be arranged in such a way that 2 particular books are always in first and
last place. Find the number of arrangements.
[a]480
[b]60
[c] 120
d] 240
Answer =5! X 2!=240.

CPT QUESTION 92 ON PERMUTATION

If
18cr= 18c(r+2),find the value of rc5

[a]60
[b]56
[c] 50
[d] 55

solution :
We know that
ncr = nc(n--r)
18cr can be written as 18c(18—r)
So 18c(18—r)= 18c(r+2)
So
18—r=r+2
18—2=2r
16=2r
r=8
rc5 =8c5 =56 so the option is[b]
-----------------------------------------------------------
CPT QUESTION 93 ON PERMUTATION

Number of ways of painting a face of a cube by 6 colours is


[a]1
[b]24
[c] 6
d] 36
solution :
answer [c] =6
CPT QUESTION 94 ON PERMUTATION

If npr=np(r+1) and ncr=nc(r—1) , find the value of n


[a]5
[b]4
[c] 3
d] 2

solution :
npr= n!
--------
[n—r]!

np(r+1) = n!
--------
[n—r—1]!
n! n!
-------- = --------------
[n—r]! [n—r—1]!

[n—r]! = [n—r—1]!

[n—r][n—r—1][n—r—2]…= [n—r—1][n—r—2]…

[n—r][n—r—1][n—r—2]…= [n—r—1][n—r—2]…

n—r =1--------------equation 1

ncr = nc(r—1)

ncr= n!
-------------
[n—r]! r!

nc(r—1) = n!
----------------
[n—r+1]! x [r—1]!

[n—r]! r! = [n—r+1]! x [r—1]!

[n—r][n—r—1][n—r—2] ..[r][r—1][r—2]..
=
[n—r+1][n—r+1—1][n—r+1—2]..[r—1][r—2][r—3]..

[n—r][n—r—1][n—r—2]..[r][r—1][r—2]..
=
[n—r+1][n—r][n—r—1]..[r—1][r—2][r—3]..

r= n—r+1
r=1+1
r=2
n—r=1
n—2=1
n=3
so the option [c]
-----------------------------------------------------------------
CPT QUESTION 95 ON PERMUTATION
Six persons A,B,C,D,E and F are to be seated in a circular table. In how many ways can
this be done if A must always have either B or C to his right and B must always have
either C or D to his right.
[a]18
[b]12
[c] 6
[d] 3
solution :
AB or AC
BC or BD
ABC, D,E,F=3!
ABD, C,E,F=3!
AC, BD, E,F =3!
3! + 3!+3!=18
CPT QUESTION 96 ON PERMUTATION

Six seats of articled clerks are vacant in a CA firm. How many different batches of
candidates can be chosen out of ten candidates?
[a]220
[b]216
[c] 210
d] 123
solution :
10c6 =210

CPT QUESTION 97 ON PERMUTATION

There are 3 blue balls, 4 red balls and 5 green balls. In how many ways can they be arranged in a
row?

[A] 26620

[B] 27820

[C] 27720
[D]26720
Solution :
12!
------------
3! X 4!x 5!
=27720 so option [c]

CPT QUESTION 98 ON PERMUTATION

For 3 helpers post 10 people applied. In how many ways can there be a selection completion ?

[A] 120
[B] 150

[C] 15
[D]36
Solution :
10c3 =120 = option [a]
====================================
CPT QUESTION 99 ON PERMUTATION

The maximum number of points of intersection of 10 circles will be ?

Solution :

np2.

10p2 =90.

CPT QUESTION 100 ON PERMUTATION

There are 10 students in a class including 3 girls. The number of ways to arrange them in a row
when any two girls out of 3 never comes together?

Solution :

Boys=7

Girls=3

Answer :

8p3 x 7!

CPT QUESTION 101 ON PERMUTATION

In how many ways can a selection of 6 out of 4 teachers and 8 students be done so as to include
atleast two teachers ?

Solution :

4c2 x 8c4 + 4c3 x 8c3 + 4c4 x 8c2

= 672 ways.

CPT QUESTION 102 ON PERMUTATION


There are 6 men and 4 women in a group. The number of ways in which a committee of 5
persons can be formed of them if the committee is to include atleast 2 women are ?

Solution :

4c2 x 6c3 + 4c3 x 6c2 + 4c4 x 6c1 =

186 ways.

CPT QUESTION 103 ON PERMUTATION

If npr = 720 and ncr=120, then the value of r is ?

Solution :

n! 720

------ = ------

[n—r]! 1

6 n! n!

------ = ------

[n—r]! r! [n—r]!

----- =1

r!

r!=6

r=3

CPT QUESTION 104 ON PERMUTATION

An examination paper with 10 questions consist of 6 questions in maths and 4 questions in


stat . Atleast 1 question from each part is to be attempted . In how many ways can this be
done ?
Solution :

m s
6c1 4c4
6c1 4c3
6c1 4c2
6c1 4c1
6c2 4c4
6c2 4c3
6c2 4c2
6c2 4c1
6c3 4c4
6c3 4c3
6c3 4c2
6c3 4c1
6c4 4c4
6c4 4c3
6c4 4c2
6c4 4c1
6c5 4c4
6c5 4c3
6c5 4c2
6c5 4c1
6c6 4c4
6c6 4c3
6c6 4c2
6c6 4c1
2n—1 = 26—1=63

2n—1 = 24—1=15

Answer 63 x15 =945

CPT QUESTION 105 ON PERMUTATION

A student has 3 books on computer, 3 books on economics and 5 books on commerce . If these
books are to be arranged subjectwise, then this can be placed on a shelf in how many number
of ways?

Solution :

3! X 3!x 5!x 3!=25920.


CPT QUESTION 106 ON PERMUTATION

A person has 10 friends of whom 6 are relatives. If he invites 5 guests such that 3 of them are
relatives. Then the total number of ways in which he can invite them are?

Solution :

Non relative friends=4

Relative friends= 6

6c3 x 4c2=120.

CPT QUESTION 107 ON PERMUTATION

The 4 digit numbers that can be formed out of 7 digits 1,2,3,5,7,8,9 such that no digit is
repeated in any number and are greater than 3000 are ?

Solution :

5ways 6ways 5ways 4ways


5ways x 6 ways x 5 ways x 4 ways=600 ways.

CPT QUESTION 108 ON PERMUTATION

5 men and 4 women to sit in a row in such a manner that the women always occupy the even
places. The number of such arrangements will be ?

Solution :

1 2 3 4 5 6 7 8 9

4! X 5!=2880

CPT QUESTION 109 ON PERMUTATION

If 6pr=360 then the value of r is ?

Solution :

6pr=360

6!

----- =360

[6—r ] !

6x5x4x3x2x1 360
-------------------- = -------

[6—r ] ! 1

720 360

-------------------- = -------

[6—r ] ! 1

2 1

-------------------- = -------

[6—r ] ! 1

[6—r]! =2

By trial and error method , r= 4.

CPT QUESTION 110 ON PERMUTATION

Six seats of articled clerks are vacant in a Chartered Accountant firm. How many different
batches of candidates can be chosen out of 10 candidates.

Solution :

10c6 =210.

CPT QUESTION 111 ON PERMUTATION

A building contractor needs three helpers out of 10 men supply. In how many ways can the
selection can place ? Solution :

10c3 =120.

CPT QUESTION 112 ON PERMUTATION

Find the number of combinations of the letter of the word COLLEGE taken 4 together .

Solution :
7c4 =35

CPT QUESTION 112 ON PERMUTATION

Find the number of combinations of the letter of the word COLLEGE taken 4 together .

Solution :

7c4 =35

c o l e g 4
1 1 2 2 1 7
1 1 1 1 0 4
1 1 2 0 0 4
1 1 1 0 1 4
1 1 0 1 1 4
1 1 0 2 0 4
0 1 1 1 1 4
0 1 2 1 0 4
0 1 0 2 1 4
0 0 2 2 0 4
0 0 1 2 1 4
0 0 2 1 1 4
0

CPT QUESTION 113 ON PERMUTATION

5 bulbs of which 3 are defective are to be tried in 2 light points in a dark room. In how many
trials the room can be lighted?

Solution :

Good=2

Bad =3

good1 good2 bad1 bad2 bad3 light


1 * 5c1
2 * 5c1
10 ways

CPT QUESTION 114 ON PERMUTATION

The number of ways in which n books can be arranged on a shelf so that 2 particular books are
not together.

Solution :

n! -- [n—1]! X 2!

n X (n—1)! -- [n—1]! X 2!

n X (n—1)! -- [n—1]! X 2

(n—1)! {n—2}

CPT QUESTION 115 ON PERMUTATION

A supreme court bench consist of 5 judges. In how many ways the bench can give a majority
decision .

Solution :

5c3 + 5c4 +5c5 =10+5+1=16 ways.

CPT QUESTION 116 ON PERMUTATION

An examination paper consist of 12 questions divided into two parts A and B. Part A contains 7
questions and part B contains 5 questions. A candidate is required to attempt 8 questions
selecting atleast 3 from each part. In how many maximum ways can the candidate select the
questions?

Solution :

Part A = 7

Part B = 5

7c3 x 5c5 + 7c4 x 5c4 + 7c5 x 5c3 =420 ways.

CPT QUESTION 117 ON PERMUTATION


A boy has 3 library tickets and 8 books of his interest in the library. Of these 8, he does not want
to borrow mathematics part 2 unless mathematics part 1 is also borrowed. In how many ways
can he chose the 3 books to be borrowed ?

Solution :

Maths 2

Others 6

2c2 x 6c1 + 2c0 x6c3 = 32.

CPT QUESTION 118 ON PERMUTATION

The number of triangles that can be formed by choosing the vertices from a set of 12 points , 7
of which are collinear is ?

Solution :

12c3—7c3 =185

EQUATIONS:

WHAT IS THE MEANING OF VARIABLE ?

VARIABLE MEANS A VALUE OF A PARTICULAR PHENOMENON

[ SAY INCOME OF PEOPLE / POPULATION OF DISTRICTS ] WHICH


MAY VARY FROM ONE OBSERVATION TO ANOTHER OBSERVATION.

IF THERE IS ONLY VARIBALE SAY “X” THEN IT IS CALLED AN EQUATION

WITH ONLY ONE VARIABLE WHICH MAY BE A LINEAR EQUATION IN

WHICH THE INDEX VALUE(POWER VALUE) OF VARIABLE X WILL BE NOT

EXCEEDING “1”

IF THE INDEX VALUE (POWER VALUE) OF VARIABLE X IS “2”, THEN

SUCH AN EQUATION IS KNOWN AS “QUADRATIC EQUATION”

IF THE INDEX VALUE (POWER VALUE) OF VARIABLE X IS”3”, THEN

SUCH AN EQUATION IS KNOWN AS “CUBIC EQUATION”

IF THE INDEX VALUE (POWER VALUE) OF VARIABLE X IS”MULTIPLE

AND HAS “X”, THEN SUCH AN EQUATION IS KNOWN AS

“EXPONENTIAL EQUATION”

VARIABLES AND CONSTANTS JOIN TOGETHER IN A MEANINGFUL WAY,

THEN THE EFFECT OF SUCH COMBINATION IS CALLED AN EQUATION.

EXAMPLE :

PRASAD NEEDS Rs50

Rs20 is already with him. How many Rs10 notes he has to earn?

Rs10(x) +Rs20=Rs50

10(x) +20=Rs50
10 x=50—20

10x=30

X=30/10

X=3

EXAMPLE :

PRASAD NEEDS Rs50

Rs20 is already with him. How many Rs5 notes he has to earn?

Solution:

Rs5(x) +Rs20=Rs50

5(x) +20=Rs50

5 x=50—20

5x=30

X=30/5

X=6

Example: The sum of two numbers ,one of which is 2/3 times the other is 50. Find the two
numbers.

Solution :

Understanding the problem

 Sum of two numbers=50


 Let the first number be taken as X
 Let the second number be taken as [2/3] X
 So the two numbers are X and (2/3) X
 Sum of these numbers =50
 Now we can formulate the following equation

X + (2/3) X = 50

X+ 2X 50

---- =

3X+ 2X 50

--------- =

5X 50

----- =

5X= 150

X= 30

SO THE TWO NUMBERS ARE…

30 , 30X[2/3]

30 , 20

X + (0.6666666) X = 50

1.6666666 X = 50
X= 50/1.66666666

X =30

One number X = 30

Another number (2/3) X = (2/3) 30 =20

So the two numbers are 30 & 20.

Solution : let the two numbers be x and 2/3x

X + [2/3[ x=50

X + 2x

--- =50

3x +2x

-------- =50

3x +2x=150

5x=150

X=150/5

X=30
So the numbers are 30 & 2/3 (30)

30 & 20.

Example:

The length of a rectangle is 4 cm more than the breadth of the Rectangle.

The perimeter of the rectangle is 11 cm more than the breadth of the Rectangle. Find the

 The length of the rectangle.


 The breadth of the rectangle.

SOLUTION:

Let the length of the rectangle be “l”

Let the breadth of the rectangle be “b”

Since it is given in the problem that the length of the rectangle is 4 cm more than the breadth
of the rectangle, we can express it as an equation

l= b+4

It is further given in the problem that the perimeter of the rectangle is 11 cm more than the
breadth of the rectangle.

We know that the formula for perimeter of a rectangle = 2( l+b)

So 2(l+b) = b+11

2l + 2b= b+11

2l= --b+ 11

So we have two equations now…

l=b+4
2l=--b+11

============

Adding we get

3l= 15

l= 15/3 =5

Let us substitute the value of l =5 in any equation

l =b+4

5 =b+4

b= 5—4 =1

LET “b” BE TAKEN AS THE BREADTH OF THE RECTANGLE.

THE LENGTH OF THE RECTANGLE =(b+4)

FORMULA FOR PERIMETER OF THE RECTANGLE

2(l+b) =b + 11

ANALYSIS

l= b+4 -----------------equation 1

2l+2b =b + 11

2l =b—2b + 11

2l = --b + 11--------------------equation 2

Let us substitute l=b+ 4 in equation 2

2l = --b + 11

2(b+4) = --b + 11
2b+8 = --b + 11

2b+b = 11--8

3b = 3

b=3/3

b=1

l=b+4

l=1+4

l=5

Example:

A number consists of two digits. The digit at ten’s place is two times the digit at unit’s place. The
number formed by reversing the digits is 27 less than the original number. Find the original
number.

Solution:

21—12=09

42—24=18

63—36=27

84—48=36

Solution :

21

42

63

84

====================

12

24
36

48

original number reversed number original number—


reversed number
21 12 21—12=9
42 24 42—24=18
63 36 63—36=27
84 48 84—48=36

So the original number is 63.

Solution :

21

42

63

84

Let the digit at unit’s place be X

Then, the digit at ten’s place will be 2X

The original number will be

[ ten’s place ] [unit’s place ]

[2X] [ X]

Original number =

10[2X] + 1[x]

=20X+X

=21 X

The number formed by reversing the digits of the original number


[X] [2X]

10[x] +1[2x]

=10X+2X

=12 X

It is given in the problem that the number formed by reversing

the digits of the original number is 27 less than the original number

10X+ 2X+27 =20X + X

12X+27=21X

27=21X—12X

27=9X

X=3

The original number is

[ ten’s place ] [unit’s place ]

[ 2 (3)] [ 3 ]

[ 6] [ 3 ]

63

Proof:

Original number =63

Reversed number =36

Difference =27

Exercise:

A number consists of two digits. The digit at ten’s place is thrice the

digit at unit’s place. The number formed by reversing the digits is 54

less than the original number. Find the original number.


Solution:

31—13=18

62—26=36

93—39=54

Solution

original number reversed number original number—


reversed number
31 13 31—13=18
62 26 62—26=36
93 39 93—39=54

Let the digit at unit’s place be X

Then, the digit at ten’s place will be 3X

The original number will be

[ ten’s place ] [unit’s place ]

[3X] [ X]

Original number =

10[3X] + 1[x]

=30X+X

=31 X

The number formed by reversing the digits of the original number

[X] [3X]

10[x] +1[3x]
=10X+3X

=13 X

It is given in the problem that the number formed by reversing

the digits of the original number is 54 less than the original number

31 X –13X=54

18 X =54

X=54/18

X=3

So

The original number is

[ ten’s place ] [unit’s place ]

[ 3 (3)] [ 3 ]

[ 9] [ 3 ]

93

Proof:

Original number =93

Reversed number =39

Difference =54

Example:

Divide 300 into two parts so that half of the one part may be less than the other by 48.

Solution :

X+Y=300
0.50 X=Y—48

===================

X+Y =300

0.50X—Y =--48

=====================

1.50 X =252

X =252/1.5

X= 168

Y=132

Illustration to understand the situation…

Rs300 is there with the father.

He has one son and one daughter in law.

Let the big share go to daughter in law

and small share go to son.

Let D be the share of daughter in law

Let S be the share of son.

½ of D= S—48

0.5 D=S—48

D+S=300

S=300—D

Let us substitute S=300—D in

0.5 D=S—48

0.5D=300—D—48

0.5D=252—D
1.50D=252

D=252/1.5

D=168

S=300—D

S=300-168=132

The two parts are 168 & 132

Proof:

½ of 168=84

84 is less than 132 by 48.

==================================

Exercise :

Guru has two sons named Ram and Mohan.

Guru has Rs5 lakhs with him. Ram has to get twice as Mohan gets.

Divide Guru’s Rs5,00,000 in between Ram and Mohan.

Solution :

R + M =5,00,000

R = 2M

=============================

2M + M =5,00,000

3M =5,00,000

M= 5,00,000/3

M= 1,66,666.666666

R= 2 x 1,66,666.666666 = 3,33,333.33
Ram = Rs3,33,333.33

Mohan=Rs1,66,666.67

==================================

Exercise :

Divide Rs450 in to two parts such that three fourths of one part is less than the other part by
Rs10.

Solution

X + Y= 450

================

(¾) X--10 = Y

0.75 X –10 = Y

0.75 X—Y =10

=================

X + Y= 450

0.75 X—Y =10

====================

1.75 X = 460

X =460/1.75

X=262.8

=====================

X + Y= 450

262.8 + Y= 450

Y= 450—262.8

Y= 187.2
=================================

Exercise :

Divide 850 into two parts such that one part is half the other part.

Answer :

283.33 and 566.67

--------------------------------------------------------------------------------

LINEAR EQUATION IN TWO VARIABLES

The general form of linear equation in two variables is given by

aX + bY + c =0

where “a” is the coefficient of variable X,

“b” is the coefficient of variable Y and

“c” is a constant.

Example

3X + 4Y =20 is a linear equation in two variables X and Y.

SIMULTANEOUS LINEAR EQUATION IN TWO VARIABLES:

Another example:

----------------------------------------------------------------------------------

SOLVE :

3 x + 4y = 100

4x + 3 y = 110

SOLUTION :
3 x + 4y=100---------------EQUATION 1

4x + 3y=110--------------EQUATION 2

 LET US MULTIPLY THROUGH OUT THE EQUATION 1

WITH THE COEFFICIENT OF VARIABLE X SOURCED FROM

EQUATION 2 AND LET THE NEW EQUATION BE NAMED AS

EQUATION 3

4[3 x + 4y]=4[100]---------------EQUATION 3

12 x + 16y = 400---------------EQUATION 3

 COEFFICIENT OF VARIABLE X SOURCED FROM

EQUATION 2 =4

*********************************************

 LET US MULTIPLY THROUGH OUT THE EQUATION 2

WITH THE COEFFICIENT OF VARIABLE X SOURCED FROM

EQUATION 1 AND LET THE NEW EQUATION BE NAMED AS

EQUATION 4

3[4 x + 3y]=3[110]---------------EQUATION 4

12 x + 9y = 330---------------EQUATION 4

 COEFFICIENT OF VARIABLE X SOURCED FROM

EQUATION 2 =3

NOW WE HAVE TWO NEW EQUATIONS 3 & 4


12 x + 16y = 400---------------EQUATION 3

12 x + 9y = 330---------------EQUATION 4

LET US DEDUCT EQUATION 4 FROM THE EQUATION 3

12 x + 16y = 400---------------EQUATION 3

(--) (--) (--)

12 x + 9y = 330---------------EQUATION 4

7y =70

y =70/7

y =10 first derivation

we can substitute y=10 in any one of the equations 1 or 2 or 3 or 4.

Then we can derive the coefficient of variable X.

Let us choose equation 1

3 x + 4y=100---------------EQUATION 1

3 x + 4(10)=100---------------EQUATION 1

3 x + 40=100

3x =100--40

3x =60

x =60 /3

x =20.

**************************************************

EXAMPLE:

SOLVE
2x + 3y =13

&

5x --2y = 4

Ans

X=2

Y=3

************************************************

Exercise :

MAHESH IS A TRADER IN PRODUCT X AND PRODUCT Y.

 ON 18-10-2013, MAHESH EARNED A TOTAL PROFIT OF

RS 630 BY WAY OF SELLING 4 UNITS OF PRODUCT X PLUS 3 UNITS OF PRODUCT Y.

 ON 19-10-2013, MAHESH EARNED A TOTAL PROFIT OF RS 640 BY WAY OF SELLING 2


UNITS OF PRODUCT X PLUS 8 UNITS OF PRODUCT Y.

 QUESTION 1 :FORM THE LINEAR EQUATION IN TWO VARIABLES

SOLUTION :

 PROFIT OF 18-10-2013 IS A CONSTANT FIGURE WHICH IS 630

COMPOSED OF 4X AND 3Y

4X +3Y=630-----------------EQUATION 1

 PROFIT OF 19-10-2013 IS A CONSTANT FIGURE WHICH IS 630


COMPOSED OF 2X AND 8Y

2X +8Y=640-----------------EQUATION 2

SO THE TWO LINEAR EQUATIONS IN TWO VARIABLES

X (PROFIT PER UNIT OF PRODUCT X) AND

Y(PROFIT PER UNIT OF PRODUCT Y) CAN BE

PRESENTED AS BELOW…

4X+3Y=630

&

2X +8Y=640

EXAMPLE:

MAHESH IS A TRADER IN PRODUCT X AND PRODUCT Y.

ON 18-10-2013, MAHESH EARNED A TOTAL PROFIT OF RS 630 BY WAY OF SELLING 4 UNITS OF


PRODUCT X PLUS 3 UNITS OF PRODUCT Y.

ON 19-10-2013, MAHESH EARNED A TOTAL PROFIT OF RS 640 BY WAY OF SELLING 2 UNITS OF


PRODUCT X PLUS 8 UNITS OF PRODUCT Y.

 QUESTION :FIND THE PROFIT PER PRODUCT X AND THE PROFIT PER
PRODUCT Y.

Solution
X=Rs120
Y=Rs50
EXERCISE :

SURESH IS A TRADER IN TWO PRODUCTS X AND Y.


THE PROFIT PER PRODUCT X=RS480 &
THE PROFIT PER PRODUCT Y=RS200
QUESTION: ON 20-10-2013 HE PLANS FOR A TOTAL PROFIT OF RS 4,400. GIVEN THAT HE HAS
DECIDED TO SELL 5UNITS OF X, HOW MANY UNITS OF Y HAVE TO BE SOLD?
SOLUTION :

5(480)+(Y)200=4,400

2,400+200Y=4,400

200Y=4,400—2,400

200Y=2,000

Y=2,000/200

Y=10.

EXERCISE

SIVA IS THE FATHER OF MURUGA.NOW SIVA’S AGE IN YEARS IS THRICE AS THAT OF THE AGE
OF MURUGA. AFTER 5 YEARS FROM TODAY, SIVA’S AGE WILL BE 2 ½ TIMES OF THE AGE OF
MURUGA .FIND THE PRESENT AGE OF SIVA AND MURUGA IN YEARS.

SOLUTION :

NOW THE AGE IN YEARS OF SIVA= S

NOW THE AGE IN YEARS OF MURUGA= M

S= 3M

AFTER 5 YEARS FROM NOW , THE AGE IN YEARS OF SIVA= S+5

AFTER 5 YEARS FROM NOW ,THE AGE IN YEARS OF MURUGA= M+5

S+5 = (M+5) x 2.5

 S= 3M
 S+5=(M+5) X2.5

LET US PUT S=3M IN S+5=(M+5) X2.5

3M+5=(M+5) X2.5

3M+5=2.5M+12.5

3M—2.5M=12.5--5

0.5M=7.5

M=7.5/0.5

M=15

PRESENT AGE OF MURUGA =15 YEARS

PRESENT AGE OF SIVA= 45 YEARS

================================

AFTER 5 YEAR MURUGA’S AGE= 20 YEARS

AFTER 5 YEAR SIVA’S AGE= 50 YEARS

SOLUTION :

 LET THE PRESENT AGE OF MURUGA BE X

THEN , THE PRESENT AGE OF SIVA WILL BE 3X

 AFTER 5 YEARS THE AGE OF MURUGA WILL BE (X+5)

AFTER 5 YEARS THE AGE OF SIVA WILL BE 2.5(X+5)

2.5 X +12.5

3X +5 = 2.5 X +12.5

3X --2.5 X =12.5--5

0.5 X =7.5

X =7.5 /0.5
X =15

PRESENT AGE OF MURUGA=15

PRESENT AGE OF SIVA =3X=3(15)=45

PROOF:

AFTER 5 YEARS THE AGE OF MURUGA= 20

AFTER 5 YEAR THE AGE OF SIVA=50

50/20 =2.5

Example :

Five years ago father’s age in years was thrice as the age of his daughter. Ten years from now,
his age will be twice as the age of her. How old are they now ?

SOLUTION :

IN 2011 F AND D

F=3D-------------EQN 1

IN 2026 F AND D

F+15=(D+15)x2

F+15=2D+30

F= 2D+15------EQN 2

solution :

F=3D
F+15= (D+15)2

PUT F=3D IN F+15= (D+15)2

3D+15= (D+15)2

3D+15= 2D+30

D= 15

F= 45

SO BEFORE 5 YEARS FATHER’S AGE WAS 45

SO BEFORE 5 YEARS DAUGHER’S AGE WAS 15

SO NOW FATHER’S AGE IS 45+5=50

SO NOW DAUGHER’S AGE IS 15+5=20

SO AFTER 10 YEARS FATHER’S AGE WILL BE 60

SO AFTER 10 YEARS DAUGHER’S AGE WILL BE 30

==================================================

let the present age of the father in the year 2013 be X years

The age of the father in the year 2008 be ( X—5) years

 The age of the daughter in the year 2008 be (X—5)/3 years

The age of the father in the year 2023 will be (x+10) years

 The age of the daughter in the year 2023 be (X+10)/2 years

2008 2013 2023

(x—5) /3 (x—5) /3 + 15

Daughter’s
age
[ (X—5)/3 ] + 15 = (X+10)/2

X 5 15 x 10

--- (-- ) --- (+) ---- == ----- (+) ------

3 3 1 2 2

X x 5 10 15

--- (-- ) --- == ----- (+) ------ (--) ------

3 2 3 2 1

2X—3x 5 10 15

-------- == ----- (+) ------ (--) ------

6 3 2 1

2X—3x 10 +30--90

-------- == --------------

6 6

2X—3x --50

-------- == --------------

6 6
2X—3x = --50

--x = --50

X= 20

So present age of father in 2013=50

Age of father 5 years ago in 2008=45

Age of daughter 5 years ago in 2008 =45/3 =15

Present age of daughter in 2013 =20

2008 2013 2023


Father’s age 45 50 60
Daughter’s age 15 20 30

Alternative solution:

Simple terminology

2008 2013 2023


Father’s age X--5 X X+10
in years
Daughter’s age in years [X—5] [X+10]
3 2

[X—5] [x+10]

------- +15 = --------

3 2
=============REVISE UPTO THIS ON 3.6.2016=================

Example : Arun and Balaji are CPT students at CCC. Their ages differ by 2 years. Arun’s father
Mr Dayal’s age is twice of Arun’s age. Balaji is twice the age of his sister Chithra. The ages of
Dayal and Chithra differ by 40 years. Find the ages of Arun and Balaji.

Solution :

A = B+2

B= 2C OR C = B/2

D =2A= 2(B+2)

D—C=40

2(B+2)—B/2 =40

2B +4 –B/2 =40

4B+8—B=80

3B=72

B=24
A = B+2

A = 24+2=26

A=26

C = B/2

C = 24/2

C=12

D =2A

D =2(26)

D=52

ARUN’S AGE= A= 26

BALAJI’S AGE=B=24

CHITHRA’S AGE=C=12

DAYAL’S AGE=D=52

Person Arun Balaji Dayal Chithra


Age X X—2 2X [X—2]
--------
2

26 26—2 2x26 26—2


=24 =52 2
=12

[X—2]
--------
2 + 40 =2x

x—2 40 2x
------ + ----- = ------

2 1 1

x 2 40 2x

------- (---) ----- ( +) ----- = ------

2 2 1 1

x 2x -- 40 1

------- (---) ----- == ----- + ------

2 1 1 1

x—4x -- 40 1

------- = ------ + ------

2 1 1

—3x -- 40+1

------- =

—3x -- 39

------- =

2
—3x = -- 78

X=26

Example :

A fraction becomes 4/5 if 1 is added to both numerator and denominator. If, however,5 is
subtracted from both numerator and denominator, the fraction becomes ½ . Find that fraction.

Solution:

LET THE FRACTION BE X/Y

NOW ADD 1 TO BOTH NUMERATOR AS WELL AS DENOMINATOR

(X+1)/(Y+1) =4/5

X+1 4

----- = ---------

Y+1 5

CROSS MULTIPLY

5(X+1) = 4(Y+1)

5X+5=4Y+4

5X—4Y=--1 ----------------- EQN 1

LET THE FRACTION BE X/Y

NOW 5 IS SUBTRACTED FROM BOTH THE NUMERATOR AS WELL AS DENOMINATOR


(X—5) / (Y—5) =1/2

X—5 1

------- = ----

Y—5 2

CROSS MULTIPLY

2(X—5) = 1(Y—5)

2X—10=Y—5

2X—Y=--5+10

2X—Y=5-------------- EQN 2

SO THE TWO EQUATIONS ARE

5X—4Y=--1 AND

2X—Y=5

IF WE SOLVE THE ABOVE TWO EQUATIONS WE WILL GET X AND Y

LET THE FRACTION BE X

---

X IS THE NUMERATOR IN THIS FRACTION

Y IS THE DENOMINATOR IN THIS FRACTION

1 IS ADDED TO X ---------- X+1

1 IS ADDED TO Y ---------- Y+1

NOW
X+1 4

----- === ----

Y+1 5

5(X+1)===4(Y+1)

5X+5=4Y+4

5X—4Y=4—5

5X—4Y=--1 -------------------EQN 1

LET THE FRACTION BE X

---

X IS THE NUMERATOR IN THIS FRACTION

Y IS THE DENOMINATOR IN THIS FRACTION

5 IS SUBTRACTED FROM X ---------- X--5

5 IS SUBTRACTED FROM Y ---------- Y--5

NOW

X--5 1

----- === ----

Y--5 2

2(X--5)===1(Y--5)

2X--10=Y--5

2X—Y=—5+10

2X--Y=5 -------------------EQN 2
LET US SOLVE

5X—4Y=--1 -------------------EQN 1

&

2X--Y=5 -------------------EQN 2

LET US MULTIPLY EQN 2 WITH 4, WE GET

8X—4Y=20-------------EQN3

EQN3—EQN1

8X—4Y=20

5X—4Y=--1

---------------

3X =21

---------------

X=7

PUT X=7 IN EQN 1

5X—4Y=--1

5(7)—4Y=--1

35—4Y=--1

—4Y=--1--35

—4Y=--36
Y= 9

THEREFORE, THE FRACTION IS 7/9

PROOF

(7+1)/(9+1)=8/10=4/5

PROOF

(7--5)/(9--5)=2/4=1/2

EXAMPLE:

SUM OF THE DIGITS OF A TWO DIGIT NUMBER IS 9.THE NUMBER OBTAINED BY REVERSING
THE ORDER OF THE DIGITS OF THE GIVEN NUMBER EXCEEDS THE GIVEN NUMBER BY 27.FIND
THAT NUMBER.

SOLUTION :

18

27

36

45

54

63

72

81
ORIGINAL NUMBER REVERSED NUMBER REVERSED NUMBER—
ORIGINAL NUMBER
18 81 81—18=63
27 72 72—27=45
36 63 63—36=27
45 54
54 45
63 36
72 27
81 18

LET THE DIGIT IN THE UNIT’S PLACE BE X

LET THE DIGIT IN THE TEN’S PLACE BE Y

DIGIT IN THE TEN’S PLACE SHALL BE ANY ONE OF THE FOLLOWING=Y

1 OR 2 OR 3 OR 4 OR 5 OR 6 OR 7 OR 8 OR 9

IT HAS TO BE MULTIPLIED WITH TEN.

DIGIT IN THE UNIT’S PLACE SHALL BE ANY ONE OF THE FOLLOWING=X

0 OR 1 OR 2 OR 3 OR 4 OR 5 OR 6 OR 7 OR 8 OR 9

IT HAS TO BE MULTIPLIED WITH ONE.

SO THE ORIGINAL NUMBER WILL BE

10Y + 1X=10Y +X

NOW THE DIGITS ARE REVERSED

DIGIT IN THE TEN’S PLACE SHALL BE ANY ONE OF THE FOLLOWING=Y

1 OR 2 OR 3 OR 4 OR 5 OR 6 OR 7 OR 8 OR 9
IT HAS TO BE MULTIPLIED WITH TEN.

DIGIT IN THE UNIT’S PLACE SHALL BE ANY ONE OF THE FOLLOWING=X

0 OR 1 OR 2 OR 3 OR 4 OR 5 OR 6 OR 7 OR 8 OR 9

IT HAS TO BE MULTIPLIED WITH ONE.

SO THE REVERSED NUMBER WILL BE

10X + 1Y=10X +Y

X+Y=9

X=9—Y

PUT X=9—Y IN ORIGINAL DIGIT

10Y +X

10Y +(9-Y)=10Y+9—Y=9Y+9=9(Y+1)=9Y+9

PUT X=9—Y IN REVERSED DIGIT

10X +Y

10(9—Y) +Y=90—10Y+Y=90—9Y

REVERSED DIGIT—ORIGINAL DIGIT =27

90—9Y—(9Y+9)=27

90—9Y—9Y—9=27

--18Y =27—90+9

--18Y=--54

Y=3
X=6

ORIGINAL DIGIT =36

REVERSED DIGIT=63

EXAMPLE :

SEVEN TIMES A TWO DIGIT NUMBER IS EQUAL TO FOUR TIMES THE NUMBER OBTAINED BY
REVERSING THE ORDER OF THE DIGITS.THE SUM OF THE DIGITS OF THE NUMBER IS 3.FIND THE
NUMBER.

SOLUTION:

original number reversed number


12 21 7x12 = 4x21
84 equal to 84
21 12 7x21=4x12
147 not equal to 48

THE TWO DIGIT NUMBER WILL HAVE A DIGIT IN TEN’S PLACE WHICH MAY BE 1 OR 2 OR 3 OR 4
OR 5 OR 6 OR 7 OR 8 OR 9.

THE TWO DIGIT NUMBER WILL HAVE A DIGIT IN UNITS ’S PLACE WHICH MAY BE O OR 1 OR 2
OR 3 OR 4 OR 5 OR 6 OR 7 OR 8 OR 9.
LET US ASSUME THAT THE DIGIT WHICH OCCUPIES TEN’S PLACE BE “Y” AND LET US FURTHER
ASSUME THAT THE DIGIT WHICH OCCUPIES UNIT’S PLACE BE “X”.

LET THE NUMBER BE 10(Y) + 01(X)=10Y+X

 ORIGINAL NUMBER=10Y+X
IF WE REVERSE THE ORDER OF THE DIGITS
OF THE ORIGINAL NUMBER,THEN THE DIGIT
WHICH OCCUPIES TEN’S PLACE =X
AND THE DIGIT WHICH OCCUPIES UNIT’S PLACE=Y
 REVERSED NUMBER=10X+Y

AS PER THE PROBLEM

(7XORIGINAL NUMBER)=(4 X REVERSED NUMBER)

7[10Y+1X] =4[10X+1Y]

7[10Y+X] =4[10X+Y]

70Y+7X =40X+4Y

70Y—4Y =40X—7X

66Y =33X

2Y =X

X—2Y =0------------------EQN 1

IT IS FURTHER GIVEN IN THE PROBLEM THAT

X+Y=3-----------------------EQN 2

EQN 1—EQN2

X—2Y =0

X+Y =3

--3Y==--3

Y=1

SUBSTITUTE Y=1 IN EQN 2

X+1=3
X=3—1=2

SO THE ORIGINAL NUMBER IS 10Y+X=10(1)+2=12

SO THE REVERSED NUMBER IS 10X+Y=10(2)+1=21

PROOF:

1+2=3

2+1=3

12 X7= 21X4

84 =84

EXAMPLE :

SOLVE THE FOLLOWING SYSTEM OF EQUATIONS

X+Y+Z=2,

2X—Y=3 AND

--X+2Y+3=0

SOLUTION :

LET US SOLVE

2X—Y=3 AND

--X+2Y+3=0

2X—Y=3 ----------- EQN 1

--X+2Y=--3 ---------- EQN 2

==================

LET US MULTIPLY EQN 2 WITH 2

--2X+4Y=--6 ----------EQN 3

EQN 1 + EQN 3

2X—Y=3 ----------- EQN 1

--2X+4Y=--6 ----------EQN 3
3Y=--3

SO

Y= --3/3=--1

PUT Y=--1 IN EQN 1

2X—Y=3 ----------- EQN 1

2X—(--1)=3

2X+1=3

2X= 3—1

2X=2

X= 2/2=1

LET US PUT X= 1, Y=--1 IN ORIGINAL EQUATION

X+Y+Z=2,

1+(--1) +Z= 2

1—1+Z=2

0+Z=2

Z=2

SO THE SOLUTIONS ARE

X=1

Y=--1

Z=2.

2X—Y=3 AND

--X +2Y+3=0

FOR X & Y

2X—Y =3----------------EQN1

--X+2Y=--3--------------EQN2
LET US MULTIPLY EQN2 WITH 2

--2X+4Y=--6-----------EQN 3

EQN1 + EQN 3

3Y=--3

Y=--1

LET US SUBSTITUTE Y=--1 IN EQN1

2X—Y =3----------------EQN1

2X—(-1) =3

2X+1 =3

2X =3--1

2X =2

X=1

LET US SUBSTITUTE X=1 AND Y=--1 IN

X+Y+Z=2

1+(--1)+Z=2

0+Z=2

Z=2

SOLUTION

{ X=1, Y=-1 AND Z=2}

PROOF:

X+Y+Z=2,-------------1+(--1) +2=2

2X—Y=3 AND-----2(1)—(-1) =3

--X+2Y+3=0-------- --1+2(-1)+3=0
EXERCISE:

SOLVE

X+2Y+3Z=14

3X+Y+2Z=11

2X+3Y+Z=11

SOLUTION:

X+2Y+3Z=14------------------EQN 1

3X+Y+2Z=11------------------EQN2

2X+3Y+Z=11-------------------EQN3

SOLUTION :

X+2Y+3Z=14------------------EQN 1

3X+Y+2Z=11------------------EQN2

LET US ELIMINATE X

X+2Y+3Z=14------------------EQN 1 X3 =EQN 4

3X+Y+2Z=11------------------EQN2X 1=EQN 5

-------------------------------------

3X+6Y+9Z=42---------EQN 4

3X+Y+2Z=11----------EQN 5
------------------------------------

EQN 4—EQN5

5Y+7Z=31--------------EQN 6

LET US TAKE EQUATIONS 1 AND 3

X+2Y+3Z=14------------------EQN 1

2X+3Y+Z=11-------------------EQN3

WE WISH TO ELIMINATE X SO

MULTIPLY EQN 1 WITH 2

2X+4Y+6Z=28 ------------------ EQN 7

=================================

EQN 7—EQN 3

2X+4Y+6Z=28 ------------------ EQN 7

2X+3Y+Z=11-------------------EQN3

---------------------------------------------------

Y +5Z=17----------------EQN 8

5Y+7Z=31--------------EQN 6

LET US ELIMINATE Y

LET US MULTIPLY EQN 8 WITH 5

5Y+25Z=85 --- EQN9

----------------------------------------

EQN 9—EQN 6

5Y+25Z=85

5Y+7Z=31

18Z= 54
Z= 54/18 =3

PUT Z= 3 IN EQN 9

5Y+25Z=85

5Y+25(3)=85

5Y+75=85

5Y= 85—75

5Y=10

Y=10/5 =2

PUT Y=2 AND Z=3 IN EQN 1

X+2Y+3Z=14------------------EQN 1

X+2(2)+3(3)=14

X+4+9=14

X+13=14

X= 14—13=1

SO THE SOLUTION IS

X=1

Y=2

Z=3

-----------------------------------------

NOW WE HAVE TWO EQUATIONS IN Y AND Z WHICH ARE EQUATION 8 AND EQN 6

LET US WRITE THEM

Y +5Z=17----------------EQN 8
LET US MULTIPLY EQN 1 WITH 3

3X+6Y+9Z=42---------------------EQN4

EQN2—EQN4

3X+Y+2Z=11------------------EQN2

3X+6Y+9Z=42---------------------EQN4

---------------------------------------------------

--5Y-7Z=--31-----------------------EQN5

EQN 2 X2= 2[3X+Y+2Z=11]

6X+2Y+4Z=22-----------------------EQN6

EQN 3 X3= 3[2X+3Y+Z=11]

6X+9Y+3Z=33---------------------EQN7

EQN6—EQN7

6X+2Y+4Z=22

6X+9Y+3Z=33

-----------------------

--7Y+Z=--11-----------------------EQN 8

WE CAN SOLVE EQUATIONS 5 & 8

--5Y-7Z=--31--------------EQN 5

--7Y+Z=--11----------------EQN 8

LET US MULTIPLY EQN 8 WITH 7

--49Y +7Z=--77----------------EQN9

EQN5 +EQN 9
--5Y-7Z=--31

--49Y +7Z=--77

--------------------

--54 Y =--108

Y=2

LET US SUBSTITUTE Y=2 IN EQN 9

--49Y +7Z=--77

--49(2) +7Z=--77

--98 +7Z=--77

7Z=--77+98

7Z=21

Z=3

LET US SUBSTITUTE Y=2 AND Z=3 IN EQUATION 1

X+2Y+3Z=14------------------EQN 1

X+2(2)+3(3)=14

X+4+9=14

X+13=14

X=14--13

X=1

SO THE SOLUTION IS { X=1, Y=2 AND Z=3 }

------------------------------------------------------------

1ST QUADRANT OF A PLANE = X POSITIVE & Y POSITIVE

2ND QUADRANT OF A PLANE = X NEGATIVE & Y POSITIVE

3RD QUADRANT OF A PLANE = X NEGATIVE & Y NEGATIVE


4TH QUADRANT OF A PLANE = X POSITIVE & Y NEGATIVE

X,Y COMBINATION WE ASSUME THAT X IS INDEPENDENT AND Y IS DEPENDENT

Y=MX+C

EXAMPLE :

DRAW THE GRAPH OF X=0

(+)

X AXIS 0 X AXIS

(-)SIDE (+) SIDE

(-)

THERFORE X=0 WILL REPRESENT A PURE Y AXIS

WHICH IS AS BELOW

Y AXIS
EXAMPLE :

DRAW THE GRAPH OF Y=0

SOLUTION

X AXIS

EXAMPLE :

DRAW THE GRAPH OF X=5

SOLUTION Y AXIS CURVE X=5

1 2 3 4 5

EXAMPLE :

DRAW THE GRAPH OF Y=4

SOLUTION: 4 Y=4

X AXIS
CPT QUESTIONS:

 CHOOSE THE CORRECT ANSWER

X=3 WILL BE

A) PARALLEL TO X AXIS
B) PARALLEL TO Y AXIS
C) PERPENDICULAR TO Y AXIS
D) NONE

Answer : [B]

 CHOOSE THE CORRECT ANSWER

Y=3 WILL BE

A) PARALLEL TO X AXIS
B) PARALLEL TO Y AXIS
C) PERPENDICULAR TO X AXIS
D) NONE

Answer : [A]

EXAMPLE : DRAW THE GRAPH

OF THE EQUATION Y=4

Y=4

X AXIS

Y AXIS

EXAMPLE:
DRAW THE GRAPH OF Y=0

X AXIS
IS Y=0

Y AXIS

EXAMPLE:

DRAW THE GRAPH OF X=0

X AXIS

Y AXIS IS X=0

EXAMPLE:

DRAW THE GRAPH OF X=5

X AXIS

X=5

Y AXIS

EXAMPLE:

DRAW THE GRAPH OF Y=2X

SOLUTION:
WE WANT TO DRAW THE STRAIGHT LINE OF THE EQUATION Y=2X

X ASSUME X AS 2 ASSUME X AS –3
Y Y WILL BE 4 Y WILL BE –6

FROM THE POINTS IMAGINED BY US WE ARE ABLE

TO FORM TWO POINTS ON THE PLANE OR ORBIT OR

GRAPH.

(X1 , Y1) & (X2 , Y2)

(2 ,4) & (--3 , --6)

Y=2X

*(2,4)

*(--3,--6)

EXAMPLE : DRAW THE GRAPH OF THE EQUATION

2X+Y=6

SOLUTION :

THE GIVEN EQUATION IS 2X+Y=6

LET US COVERT OR MODIFY IT INTO


Y=mX + c form

Or

X=m1Y +c1 form

Let us choose the first version.

Y=mX + c form

2X+Y=6

Y=6—2X

Y=—2X+6

--2 IS THE SLOPE

6 IS THE CONSTANT

Y=—2X+6

To form an equation two points are enough.

X 1 --4
Y --2(1)+6=4 --2(-4)+6=14

THE TWO POINTS ON THE GRAPH ARE

(X1,Y1) AND (X2,Y2) WHICH ARE

(1,4) AND (--4, 14)

*
CONTINUATION EXAMPLE: IN THE ABOVE EXAMPLE

CHECK WHETHER X=2 AND Y=2 IS A SOLUTION.

SOLUTION:

WHEN Y=2 AND X=2

2=—2(2)+6

2=—4+6

2=2

THE EQUATION GETS SATISFIED.

THEREFORE

X=2 AND Y=2 IS A SOLUTION FOR THIS EQUATION.

IN OTHER WORDS,

X=2 AND Y=2 IS A POINT WHICH LIES IN THE TRAVEL

PATH OF THIS NEVER ENDING STRAIGHT LINE.


EXAMPLE :SOLVE GRAPHICALLY THE FOLLOWING SYSTEM OF LINEAR EQUATIONS :

2X—Y=2 AND

4X—Y=8

SOLUTION :

CONSIDER 2X—Y=2

PUT X=0, Y--2 SO THE POINT IS [ 0,--2]

PUT Y=0, X1 SO THE POINT IS [ 1,0]

CONSIDER 4X—Y=8

PUT X=0, Y--8 SO THE POINT IS [ 0,--8]

PUT Y=0, X2 SO THE POINT IS [ 2,0]

EQUATION 1
2X—Y=2
X VALUE Y VALUE
PUT X=--2 2(--2)—Y=2
--4—Y=2
--4—2=Y
Y=--6
PUT X=0 2(0)—Y=2
0—Y=2
Y=--2
PUT X=2 2(2)—Y=2
4—Y=2
--Y=2—4
--Y=--2
Y=2

EQUATION 2
4X—Y=8
X VALUE Y VALUE
PUT X=--2 4(--2)—Y=8
--8—Y=8
--8—8=Y
Y=--16
PUT X=0 4(0)—Y=8
0—Y=8
Y=--8
PUT X=2 4(2)—Y=8
8—Y=8
--Y=0
Y=0

SO (--2, --6 ) , (0,--2), (2,2) FIRST EQUATION

SO(--2,--16),(0,--8),(2,0) SECOND EQUATION.


5

0
-2 0 2

-5
Y
X
Y
-10

-15

-20
CONSIDER THE EQUATION 2X—Y=2

CASUALLY ASSIGN A VALUE TO X SAY 2

2(2)—Y=2

4—Y=2

4—2=Y

Y=2

SO WE GET A POINT (X=2, Y=2)

CONSIDER THE EQUATION 4X—Y=8

CASUALLY ASSIGN SOME VALUE TO X SAY 1

4(1)—Y=8

4—Y=8

4—8=Y

Y= --4

SO WE GET A POINT (X=1, Y=--4)

EXAMPLE :

AT WHICH POINT 2X—Y=2 WILL INTERSECT X AXIS ?

SOLUTION:

LET US CONVERT 2X—Y=2 INTO Y=mX+c form

2X—Y=2
—Y=2--2x

—Y=--2x+2

Y=2x--2

THE EQUATION OF X AXIS IS NOTHING BUT

Y=0

NOW WE HAVE 2 EQUATIONS BEFORE US

Y=2X—2 -----------EQN 1

Y=0 -----------EQN 2

THE POINT OF INTERSECTION OF THESE 2 LINEAR EQUATIONS ARE THE SOLUTION TO THESE
TWO EQUATIONS

LET US FIND X=? AND Y=? THAT CAN SATISFY BOTH OF THE ABOVE 2 LINEAR EQUATIONS

IT IS GIVEN THAT Y=0

THEREFORE 2X—2 IS ALSO =0

2X-2=0

2X=2

X=1

THEREFORE (X,Y)=(1,0)

SO THE LINEAR EQUATION2X—Y=2 WILL CUT OR INTERSECT X AXIS AT THE POINT(1,0)


EXAMPLE :

AT WHICH POINT 4X—Y=8 WILL INTERSECT Y AXIS ?

SOLUTION

4X—Y=8

4(O)—Y=8

0—Y=8

Y=--8

SO IN THE FOLLOWING POINT THE EQUATION OR STRAIGHT LINE 4X—Y=8 WILL CUT OR
INTERCEPT OR INTERSECT Y AXIS

(0,--8)

Example:

The sum of the ages in years of father and a son is 35 and the product of their ages is 150. Find
their age in years.

SOLUTION

F+S=35

FS=150

F=35—S

PUT F=35—S IN FS=150


(35—S) S =150

35S—S2 =150

--S2 + 35S—150 =0

S2 -- 35S+150 =0

IT IS IN QUADRATIC FORM

WHERE a= COEFFICIENT OF S2 =1

b= COEFFICIENT OF S= --35

c= CONSTANT

SOLUTION TO THE QUDRATIC EQUATION MEANS FINDING THE VALUE OF X FROM THE
QUADRATIC EQUATION WHICH IS OF THE FORM

a X2 + b X + C= 0

THERE IS A READY MADE FORMULA FOR DERIVING THE VALUE OF X FROM THE QUADRATIC
EQUATION WHICH IS OF THE FORM

−b ± √ b −4 ac
2

2a

a X2 + b X + C= 0

X2 -- 35 X +150= 0

a= 1

b=--35

c=150

−b ± √ b −4 ac
2
X=
2a
−(−35)± √(−35)2−4 (1 ) (150)
X=
2(1)

¿
X= 35 ± √ 1225−600 ¿ 2

¿
X= 35 ± √ 625 ¿
2

35± 25
X=
2

X= (35 +25)/2 is a solution

X= (35 --25)/2 is a solution

X = 30 is a solution

X =5 is a solution

Age of father =30

Age of son

F=35—S

30=35—S

S = 35—30

S= 5

Father’s age=30
Son’s age=5

================================================

LET THE AGE IN YEARS OF THE FATHER BE X

LET THE AGE IN YEARS OF THE SON BE Y

X+Y=35

XY =150

X=150/Y

LET US SUBSTITUTE X=150/Y IN THE EQUATION

X+Y=35

[150/Y] + Y =35

[150 +Y2] /Y =35

[150 +Y2] =35Y

[150 +Y2] =35Y

150+ Y2=35Y

150+ Y2—35Y=0

Y2—35Y+150=0

THE ABOVE IS A QUADRATIC EQUATION WHICH IS OF THE FORM

aY2 +bY + c=0

where

a=1

b=--35

c=150

what is the standard formula for solving a quadratic equation ?

−b ± √ b 2−4 ac
2a
Let us substitute the values of a,b and c into the above formula so that we can solve the
quadratic equation and find the value of Y.

35± √ 1225−( 4 ) ( 1 ) (150)


2(1)

35± √ 1225−600
2(1)

35± √ 625
2

35± 25
2

[35+25]/ 2 or [35—25]/2

30 or 5

Let us consider y=30

So x=5

Let us consider y=5

So x=30

So the age of father=30.

The age of son=5.

Exercise : The addition of two different numbers amounts to 29 and their product amounts to
154. Find the two different numbers.
Answer :

22 & 7

=============revise upto this on 4.6.2016=================

Example: The ratio of the income of two persons is 9:7 and the ratio of their expenditure is
4:3.Each of them saves Rs200 . Find the income of each person.

Solution :

Let the income of two persons be 9x : 7x

Let their expenditure be in the ratio 4y: 3y

Income – Expenditure= Saving

Income Ratio—Expense Ratio = Saving ratio

9x—4y=200

7x—3y=200

2x—y=0

Y=2x

Put y=2x in any one ratio

9x—4y=200

9x—4(2x) =200

9x—8x=200

X=200
Income of first man=9(200)=1800

Income of second man=7(200)=1400

Solution:

Let the income of first person be 9X

Let the income of the second person be 7X

Let the expense of first person be 4y

Let the expense of the second person be 3y

Using economics concept, we know that

INCOME—EXPENSE=SAVINGS

Now, using the above economic concept, we can formulate the following two linear equations
involving two variables which are variable x and variable y.

Variable x stands for the income coefficient and variable y stands for expense coefficient.

9x—4y=200----------equation 1

7x—3y=200----------equation 2

We have to solve these two linear equations

Let us multiply equation 1 with 3 and we get

27x—12y=600----------equation 3

Let us multiply equation 2 with 4 and we get

28x—12y=800----------equation 4

Equation 4—equation 3, we get

X=200

So the income of first person=9x=9x200=1800

So the income of second person=7x=7x200=1400


Exercise home work of 5TH JUNE 2016 :

Solve the following three equations…

2x—y+z=3,

X+3y-2Z=11

3x—2y+4z=1

Solution:

2x—y+z=3---------equation1

X+3y-2Z=11-------equation 2

3x—2y+4z=1------equation3

Let us multiply equation 2 with 2 and we get

2X +6Y-4Z=22 -----------EQUATION 4

LET US DO THE FOLLOWING…

EQUATION 4—EQUATION 1

7Y—5Z=19-----------EQAUTION 5

Let us multiply equation 2 with 3 and we get

3X +9Y-6Z=33 -----------EQUATION 6

LET US DO THE FOLLOWING…

EQUATION 6—EQUATION 3
11Y—10Z=32-----------EQUATION 7.

NOW WE CAN EASILY SOLVE EQUATIONS 5 AND 7

AS BELOW…

7Y—5Z=19-----------EQAUTION 5

11Y—10Z=32-----------EQUATION 7.

LET US MULTIPLY EQUATION 5 WITH 2 AND WE GET

14Y—10Z=38-----------EQUATION 8

LET US DO THE FOLLOWING

EQUATION 8—EQUATION 7

3Y=6

Y=6/3=2

Y=2

LET US SUBSTITUTE Y=2 IN EQUATION 7

11Y—10Z=32-----------EQUATION 7.

11[2]—10Z=32

22—10Z=32

--10Z=32—22

--10Z=10

Z=10/[--10]=--1

Z=--1

NOW WE CAN SUBSTITUTE Y=2 AND Z=--1 IN THE EQUATION 1

2x—y+z=3---------equation1
2x—2+(--1)=3

2x—2--1=3

2x—3=3

2x=3+3

2x=6

X=6/2

X=3

X=3

SO THE ULTIMATE SOLUTION IS

X=3
Y=2
Z=--1

EXAMPLE : SOLVE

XY

---- = 70

X+Y

YZ

---- = 140

Y+Z

XZ

---- = 84

X+Z
Solution :

COMPARING EQUATIONS 1 &2

2 {XY/ (X+Y) } = YZ/(Y+Z)

BECAUSE FIRST EQUATION INTO 2 WILL GIVE US THE SECOND EQUATION

2 {XY/ (X+Y) } = YZ/(Y+Z)

ELIMINATE Y IN THE NUMERATOR IN LHS AND RHS

2 {X/ (X+Y) } = Z/(Y+Z)

2X Z

----- = ---------

X+Y Y+Z

CROSS MULTIPLY

2X(Y+Z) = Z(X+Y)

2XY+2XZ=ZX+ZY

2XY+2XZ—XZ—YZ =0

2XY+ZX—YZ=0-----------------------EQUATION A

COMPARING EQUATIONS 1 &3

XY/(X+Y) =70 ----EQN 1

&

XZ/ (X+Z) =84----EQN 3

EQN 1 INTO 1.2 = EQN 3

1.2XY/(X+Y) = XZ/(X+Z)
ELIMINATE X IN THE NUMERATOR OF LHS AND RHS

1.2 Y/(X+Y) = Z/(X+Z)

1.2 Y Z

-------- = -------

(X+Y) ( X+Z)

CROSS MULTIPLY

1.2Y(X+Z) = Z(X+Y)

1.2XY+1.2YZ = ZX +ZY

1.2XY+1.2YZ—ZX—ZY =0

1.2XY+0.2ZY—ZX=0---EQN B

LET US WRITE DOWN EQN A & EQN B

2XY+ZX—ZY=0 ----------EQN A

1.2XY+0.2ZY—ZX=0 ----------EQN B

EQN A + EQN B

3.2XY—0.8ZY =0

3.2XY=0.8ZY

CANCEL Y ON LHS AND RHS

3.2X=0.8Z

X 0.8

--- = -----

Z 3.2

X
---- = 0.25

X= 0.25Z

PUT X= 0.25 Z IN EQUATION 3

XZ

--- = 84

(X+Z)

0.25Z(Z)

----------- = 84

(0.25Z+Z)

0.25Z2
-------------- = 84

1.25 Z

0.25Z

------- =84

1.25

0.25Z = 105

Z= 105/ 0.25
Z=420

PUT Z=420 IN

X= 0.25 Z

X= 0.25(420)

X= 105

PUT X= 105 , Z= 420 IN EQN 1

XY /(X+Y) =70

105Y/(105+Y)=70

105Y

------- =70

105 +Y

105Y =70(105+Y)

105Y=7350+70Y

105Y—70Y=7350

35Y=7350

Y= 7350/35=210

SO

X=105,

Y=210 &

Z=420
IS THE SOLUTION

===========================================

YZ/(Y+Z) =140

XZ/(X+Z) =84

LET US CONSIDER XY/ (X+Y) =70

XY 70

---- = ------

X+Y 1

LET US CROSS MULTIPLY

XY(1) = {X+Y}(70)

XY= 70X+70Y

70X+70Y—XY=0--------------EQUATION 1

LET US CONSIDER YZ/ (Y+Z) =140

YZ 140

---- = ------

Y+Z 1

LET US CROSS MULTIPLY

YZ(1) = {Y+Z}(140)

YZ= 140Y+7140Z
140Y+140Z—YZ=0--------------EQUATION 2

LET US CONSIDER YZ/ (Y+Z) =140

ZX 84

---- = ------

Z+X 1

LET US CROSS MULTIPLY

ZX(1) = {Z+X}(84)

ZX= 84Z+84X

84Z+84X—ZX=0--------------EQUATION 3

SO THE THREE EQUATIONS ARE

70X+70Y—XY=0 ------------EQN1

140Y+140Z—YZ=0 -------------EQN2

84Z+84X—ZX=0 ---------------EQN3

FORMATTING THE FIRST EQUATION WE GET,

X+Y 1

---- = ----

XY 70
X Y 1

--- + --- = ---

XY XY 70

1 1 1

--- + --- = ---

Y X 70

FORMATTING THE SECOND EQUATION WE GET,

Y+Z 1

---- = ----

YZ 140

Y Z 1

--- + --- = ------

YZ YZ 140

1 1 1

--- + --- = ---

Z Y 140

FORMATTING THE THIRD EQUATION WE GET,

X+Z 1
---- = ----

XZ 84

X Z 1

--- + --- = ------

XZ XZ 84

1 1 1

--- + --- = -----

Z X 84

SO THE NEWLY FORMATTED THREE EQUATIONS ARE…

1 1 1

--- + --- = --- -----------------EQN 1

Y X 70

1 1 1

--- + --- = ----- --------------EQN2

Z Y 140

1 1 1

--- + --- = ----- ----------EQN 3

Z X 84

EQN 1—EQN3
1 1 1 1

-- -- --- = --- --- ---

Y Z 70 84

1 1 84 --70

-- -- --- = ----------

Y Z 5880

1 1 14

-- -- --- = ----------

Y Z 5880

1 1 2

-- -- --- = ----------

Y Z 840

1 1 1

-- -- --- = 420

WE CAN DO THE FOLLOWING

EQN 2 +EQN4
(2) 1 1 1

--- = ----- + -----

Y 140 420

2 420+140

--- = --------------

Y 58800

2 560

--- = --------------

Y 58800

560 Y =58800{2}

560 Y =117600

Y=117600/560

Y=210.

LET US SUBSTITUTE Y=210 IN EQUATION 4

1 1 1

-- -- --- = ---------- --EQN 4

Y Z 420

1 1 1

-- -- --- = ----------

210 Z 420
1 1 1

-- -- --- = ----------

210 420 Z

420--210 1

----------- = ----------

88200 Z

210 1

----------- = ----------

88200 Z

210 Z=88200

Z=420

LET US SUBSTITUTE Z=420 IN THE EQUATION 3

1 1 1

-- + --- = ---------- --EQN 3

Z X 84

1 1 1

-- + --- = ----------

420 X 84
1 1 -- 1

-- -- ---- --- = ----------

420 84 X

84--420 -- 1

----------- = ----------

35280 X

--336 -- 1

----------- = ----------

35280 X

--336X=--35280

X=--35280/336

X=105

SO THE SOLUTIONS ARE

X=105
Y=210
Z=420

EXAMPLE :

IN CAR, IT TAKES 3 HOURS LESS TIME AS COMPARED TO BUS TO REACH CHENNAI FROM
MADURAI WHICH IS 420 KILOMETRES AWAY.THE SPEED OF CAR IS 10 KMs MORE AS THAT OF
BUS.

Q.1 FIND THE SPEED OF BUS PER HOUR


Q.1 FIND THE SPEED OF CAR PER HOUR.

SOLUTION :

SAY FOR EXAMPLE

IF A VEHICLE TRAVELS AT A SPEED OF 60 KM PER HOUR , IN 7 HOURS IN CAN REACH CHENNAI


FROM MADURAI

SPEED PER HOUR X HOURS = DISTANCE

 LET B BE THE SPEED PER HOUR FOR BUS


 LET (B+10) IS THE SPEED PER HOUR FOR CAR

 LET H BE THE HOURS TAKEN IN BUS TRAVEL


 LET (H—3) IS THE HOURS TAKEN IN CAR TRAVEL

DISTANCE = SPEED X HOURS

CAR : (B+10) (H—3) =420

BUS : (B) (H) =420

H= 420/B

(B+10) (H—3) =BH

BH—3B+10H—30 =BH

--3B +10H=30

--3B+10H—30 =0

3B—10H+30=0

3B—10H=-- 30----------- EQN 1

3B—10(420/B) =--30
3B—4200/B =--30

3B2 –4200

---------------- =--30

3B2 –4200=--30B

3B2 +30 B –4200=0

B2 +10 B –1400=0

USING QUADRATIC EQUATION SOLUTION METHOD

−b ± √ b 2−4 ac
2a

HERE a=1

b=10

c=--1400

B=
−b ± √ b 2−4 ac
2a

−10 ± √ 100+5600
2

−10 ± √ 5700
2

(--10+75.4983443527 )/2

=32.7491721763

SPEED OF BUS PER HOUR =32.749172163 KM PER HOUR

SPEED OF CAR PER HOUR=42.749172163 KM PER HOUR


TIME CONSUMED IN BUS TRAVEL =420/32.749172163 KM PER HOUR

=12.824427 HOURS

TIME CONSUMED IN CAR TRAVEL =420/42.749172163 KM PER HOUR

=9.824427 HOURS

TIME REDUCTION

-------------------------

SPEED ADDITION

3 HOURS

--------------

10 KM PER HOUR

0.333333333 HOURS

SOLUTION:

LET THE SPEED OF BUS PER HOUR IS X KM PER HOUR.

LET THE SPEED OF CAR PER HOUR IS [X+10] KM PER HOUR.

FORMULA = SPEED x TIME = DISTANCE


BUS X x Y =420

CAR [X+10] x [ Y—3] =420

XY =420

(X+10)(Y—3)=420

XY =(X+10)(Y—3)

XY =XY—3X+10Y—30

30=--3X +10Y

--3X+10Y=30

X=420/Y

--3[420/Y] +10Y=30

--1260/Y +10Y=30

--126/Y +Y=3

--126+Y2
---------------- =3

--126+Y2 =3Y

Y2--3Y—126=0

Y=12.8247516

SO X = 420/12.8247516=32.7491721

SPEED OF BUS PER HOUR= 32.7491721 KMs

SPEED OF CAR PER HOUR= 42.7491721 KMs


PROOF : BUS

SPEED X TIME =DISTANCE

32.7491721 KMs X 12.8247516 HOURS=420

PROOF : CAR

SPEED X TIME =DISTANCE

42.7491721 KMs X 9.8247516 HOURS=420

EXERCISE:

A BIKE TAKES 4 HOURS LESS TIME TO REACH A DESTINATION OF 250 KMs AS COMPARED TO A
SCOOTER WHICH IS SLOW BY 15 KMs PER HOUR AS COMPARED TO BIKE.

 FIND THE SPEED OF BIKE


 FIND THE SPEED OF SCOOTER
 FIND THE HOURS SPENT IN BIKE
 FIND THE HOURS SPENT IN SCOOTER

LET THE SPEED OF SCCOTER PER HOUR IS X KM PER HOUR.

LET THE SPEED OF BIKE PER HOUR IS [X+15] KM PER HOUR.

FORMULA = SPEED X TIME = DISTANCE

SCOOTER X X Y =250

BIKE X+15 X Y—4 =250

XY =250
(X+15)(Y—4)=250

XY =(X+15)(Y—4)

XY =XY—4X+15Y—60

60=--4X +15Y

--4X+15Y=60

X=250/Y

--4[250/Y] +15Y=60

--1000/Y +15Y=60

--200/Y +3Y=15

--200+3Y2
---------------- =15

--200+3Y2 =15Y

3Y2--15—200=0

Y=23.50781059

SO X = 250/23.50781059=10.63476324

SPEED OF BIKE PER HOUR=38.50781059 KMs

SPEED OF SCOOTER PER HOUR= 23.50781059 KMs

HOURS SPENT IN BIKE =06.63476324

HOURS SPENT IN SCOOTER =10.63476324


PROOF : SCOOTER

SPEED X TIME =DISTANCE

23.50781059 KMs X 10.63476324

HOURS=250

PROOF : BIKE

SPEED X TIME =DISTANCE

38.50781059 KMs KMs X 06.63476324=250

-------------------------------------------------------------------------

EXAMPLE:

RAM PAID TO A CALL TAXI A SUM OF RUPEES 89 FOR TRAVELLING 12 KILOMETRES.

KUMAR PAID TO THE SAME CALL TAXI A SUM OF RUPEES 145 FOR TRAVELLING 20
KILOMETRES.

HOW MUCH GEETH WILL PAY IF HE TRAVELS 15 KILOMETRES.

SOLUTION :

Y= mX +C
RAM 89= m[12] +C
KUMAR 145= m[20] +C

GEETH Y= m[15] +C
12m +C=89 ----------1

20m+C=145 ----------2

------------------

2-1

8m=56

m=7

PUTTING m=7 IN 1

12m +C=89

12(7) +C=89

84 +C=89

C=5

GEETH Y= 7[15] +5
Y=110 105 +5

EXERCISE:

TOTAL COST OF PRODUCING 11 ARTICLES IN A MONTH WAS Rs91.

TOTAL COST OF PRODUCING 3 ARTICLES IN ANOTHER MONTH WAS Rs37.

FIND THE TOTAL COST IN ANOTHER MONTH IF 4 ARTICLES ARE PRODUCED.

ANSWER : 27

Example :

FIND A FRACTION WHICH BECOMES ½ WHEN 1 IS SUBTRACTED FROM THE NUMERATOR AND 2
IS ADDED TO THE DENOMINATOR AND BECOMES 1/3 WHEN 7 IS SUBTRACTED FROM THE
NUMERATOR AND 2 IS SUBTRACTED FROM THE DENOMINATOR.
SOLUTION :

LET THE ORIGINAL FRACTION BE X

---

FRACTION WHICH BECOMES ½ WHEN 1 IS SUBTRACTED FROM THE NUMERATOR AND 2 IS


ADDED TO THE DENOMINATOR

X—1 1

------- = -----

Y+2 2

NOW WE DO CROSS MULTIPLICATION AND WE GET

2[X—1] =1[Y+2]

2X—2 =1Y+2

2X—2 =Y+2

2X—Y =2+2

2X—Y =4 -----------LINEAR EQUATION 1

THE ORIGINAL FRACTION BECOMES 1/3 WHEN 7 IS SUBTRACTED FROM THE NUMERATOR AND
2 FROM THE DENOMINATOR

X—7 1

------- = -----
Y –2 3

NOW WE DO CROSS MULTIPLICATION AND WE GET

3[X—7] =1[Y—2}

3X—21 =1Y—2

3X—21 =Y—2

3X—Y = -- 2 + 21

3X—Y =19 -----------LINEAR EQUATION 2

NOW WE NEED TO SOLVE BOTH OF THE ABOVE TWO LINEAR EQUATIONS SO THAT WE DERIVE
THE VALUES FOR X AND Y.

2X—Y =4 -----------LINEAR EQUATION 1

3X—Y =19 -----------LINEAR EQUATION 2

EQUATION 2—EQUATION 1 , WE GET

X =15

NOW LET US SUBSTITUTE X=15 IN ANY ONE OF THE TWO EQUATIONS FOR THE PURPOSE OF
FINDING THE VALUE OF ANOTHER VARIABLE Y. LET US TAKE EQUATION 1 FOR THIS PURPOSE

2X—Y =4 -----------LINEAR EQUATION 1

2[15]—Y =4

30—Y =4

30—4 =Y

26=Y

Y=26

SO

X 15
Y 26
SO THE ORIGINAL FRACTION IS 15

----

26

------------------------------------------------------------------

EXAMPLE : A MAN WENT TO THE CURRENCY COUNTER OF A PRIVATE BANK AND TENDERED 2
FIVE HUNDRED RUPEE NOTES REQUESTING THE CASHIER TO GIVE HIM RS 5 NOTES AND RS 10
NOTES AS CHANGE FOR THE THOUSAND RUPEE NOTE.THE CASHIER GAVE HIM 175 NOTES IN
TOTAL. YOU HAVE TO FIND THE NUMBER OF 5 RUPEE NOTES AND THE NUMBER OF 10 RUPEE
NOTES WHICH THE MAN HAD GOT FROM THE CASHIER.

SOLUTION :

LET X BE THE NUMBER OF FIVE RUPEE NOTES AND

LET Y BE THE NUMBER OF TEN RUPEE NOTES

IT IS GIVEN IN THE PROBLEM THAT THE TOTAL NUMBER OF NOTES IS 175

THEREFORE

X+Y=175----------------LINEAR EQUATION 1

WE KNOW THAT THE TOTAL AMOUNT IS RUPEES THOUSAND.

THEREFORE,

5X +10Y=1000---------------------LINEAR EQUATION 2

NOW WE NEED TO SOLVE BOTH THE ABOVE TWO LINEAR EQUATIONS

X+Y=175----------------LINEAR EQUATION 1

5X +10Y=1000---------------------LINEAR EQUATION 2

LET US MULTIPLY EQUATION 1 WITH 5 WE GET


5X +5Y=875-------------------LINEAR EQUATION 3

LET US DEDUCT EQUATION 3 FOM EQUATION 2 WE GET,

5Y=125

Y=125/5=25

Y=25.

LET US SUBSTITUTE Y=25 IN EQUATION 1 OR EQUATION 2. LET US TAKE EQUATION 1

X+Y=175

X+25=175

X=175—25=150

SO

X FIVE RUPEE NOTES 150


Y TEN RUPEE NOTES 25

EXAMPLE :

A PERSON INVESTED SOME AMOUNT AT THE RATE OF 12% SIMPLE INTEREST AND SOME OTHER
AMOUNT AT THE RATE OF 10% SIMPLE INTEREST. HE RECEIVED YEARLY INTEREST RS130. BUT IF
HE HAD INTERCHANGED THE AMOUNTS INVESTED, HE WOULD HAVE RECEIVED RS 4 MORE AS
INTEREST. HOW MUCH AMOUNT DID HE INVEST AT THE RATE OF 10% PER YEAR?

SOLUTION:

LET X BE THE RUPEES INVESTED IN 10% YEARLY INTEREST RATE AND LET Y BE THE RUPEES
INVESTED IN 12% YEARLY INTEREST RATE.

THE RUPEES OF TOTAL INTEREST IS 130

SO WITH THE ABOVE DATA , WE CAN FORMULATE A LINEAR EQUATION

X[10/100] + Y[12/100] =130


X[0.10] + Y[0.12] =130

0.10X + 0.12Y =130

10X + 12 Y =13000--------------EQUATION 1

INTERCHANGING:

LET Y BE THE RUPEES INVESTED IN 12% YEARLY INTEREST RATE AND LET X BE THE RUPEES
INVESTED IN 10% YEARLY INTEREST RATE.

THE RUPEES OF TOTAL INTEREST IS 134

SO WITH THE ABOVE DATA , WE CAN FORMULATE A LINEAR EQUATION

Y[10/100] + X[12/100] =134

Y[0.10] + X[0.12] =134

0.10Y + 0.12X =134

10Y + 12 X =13400

12X + 10 Y=13400------------LINEAR EQUATION 2

NOW, WE NEED TO SOLVE BOTH THE ABOVE EQUATIONS AND DERIVE THE VALUES OF X AND Y

10X + 12 Y =13000--------------EQUATION 1

12X + 10 Y=13400------------ EQUATION 2

MULTIPLY EQUATION 1 WITH 1.2 WE GET THE FOLLOWING EQUATION

12X +14.4Y=15600--------------EQUATION3

WE DEDUCT EQUATION 2 FROM EQUATION 1

WE GET

4.4 Y=2200
Y=2200/4.4=500

PUT Y=500 IN EQUATION 1

10X + 12 Y =13000

10X +12[500]=13000

10X+6000=13000

10X=13000—6000

10X=7000

X=700

ANSWER :700

EXAMPLE :

IF TWICE THE SON’S AGE IN YEARS IS ADDED TO THE FATHER’S AGE, THE SUM IS 70,BUT IF
TWICE THE FATHER’S AGE IS ADDED TO THE SON’S AGE, THE SUM IS 95. THE AGE OF FATHER
AND SON IN YEARS=?

SOLUTION:

LET X BE THE AGE OF SON IN YEARS AND

LET Y BE THE AGE OF FATHER IN YEARS

TWICE THE SON’S AGE IN YEARS IS ADDED TO THE FATHER’S AGE, THE SUM IS 70,

Y+2X=70

2X+Y=70-----------EQUATION 1

TWICE THE FATHER’S AGE IS ADDED TO THE SON’S AGE, THE SUM IS 95.

X+2Y=95--------------EQUATION 2

LET US SOLVE BOTH OF THE ABOVE 2 EQUATION..

2X+Y=70-----------EQUATION 1
X+2Y=95------------EQUATION 2

EQUATION 2 X2 WE GET,

2X +4Y=190----------EQUATION 3

EQUATION 3—EQUATION 1

WE GET

3Y=120

Y=40

PUT Y=40 IN THE EQUATION 1

2X+Y=70-----------EQUATION 1

2X+Y=70

2X+40=70

2X=30

X=15

SON’S AGE X 15
FATHER’S AGE Y 40

EXAMPLE : A NUMBER CONSISTING OF 2 DIGITS IS 7 TIMES THE SUM OF ITS DIGITS. WHEN 27 IS
SUBTRACTED FROM THE NUMBER, THE DIGITS ARE REVERSED. THE NUMBER IS ?

SOLUTION:

LET THE DIGIT IN 10’S PLACE BE X

AND LET THE DIGIT IN 1’S PLACE BE Y

SO THE ORIGINAL DIGIT IS

10X+Y

SUM OF THE DIGITS=X+Y

7 TIMES THE SUM OF THE DIGITS= 7[X+Y]

10X+Y=7[X+Y]
10X+Y=7X+7Y

3X=6Y

X=2Y------------EQN 1

WHEN 27 IS SUBTRACTED FROM THE NUMBER, THE DIGITS ARE REVERSED. THE NUMBER IS ?

IF WE REVERSE THE DIGITS Y WILL GO TO 10’S PLACE AND X WILL GO TO THE UNIT’S PLACE. SO
THE REVERSED DNUMBER IS

10Y+X

ORIGINAL NUMBER –27 =REVERSED NUMBER

10X+Y—27=X+10Y

9X=9Y+27

X=Y+3

PUT X=Y+3 IN EQN 1

X=2Y------------EQN 1

X=2Y

Y+3=2Y

3=Y

Y=3

PUT Y=3 IN 1

X=2Y

X=2[3]

X=6

ORIGINAL DIGIT IS

10X+Y
10{6} +3

63

Positive value of k has to be found out, when the quadratic equation

12x2 + kx +5=0 have 2 two roots α and β

in the ratio 3:2.

a) 5/12 b)12/5 c)[5√ 10 /2] d) 5√ 10

Solution:

12x2 + k x +5=0 IS A QUADRATIC EQUATION OF THE FORM

ax2 + b x +c =0

here a =12, b= k and c=5

we know that for any quadratic equation , the following axiom holds good “ sum of the two
solutions to a quadratic equation will be equal to

--b/a of the quadratic equation”

α + β = --b/a

α + β = --k/12

we know that for any quadratic equation , the following axiom holds good “ product of the
two solutions to a quadratic equation will be equal to c/a of the quadratic equation”

α β = c/a

α β = 5/12
given that α : β = 3:2

product of extremes = product of means

2α =3β

α =3β/2

α + β = --k/12

( 3β/2)+ β = --k/12

[3β+ 2β]/2 = --k/12

12[3β+ 2β]/2 = --k

6[3β+ 2β] = --k

6[5β] = --k

30β = --k

We know that

α β = 5/12

α =3β/2

α β = 5/12

[3β/2] β = 5/12

[3β2]/2 = 5/12

β2 = [5x2]/[12x3]

β2 = 10/36

β = √ 10/6

30β = --k

30[√ 10/6 ¿ = --k

5√ 10 ¿ = --k
Solve by factorization method:

8 x2 --14 x=15

8 x2 --14 x—15 =0

8 x2 --20 x+6x—15 =0

4x(2x—5) +3(2x—5 )=0

(2x—5)(4x+3) =0

(2x—5)=0 means

2x=5

X=5/2

(4x+3)=0 means

4x = --3

X=--3/4

α = 5/2

β = --3/4

Solve by formula substitution method:

8 x2 --14 x=15

8 x2 --14 x—15 =0

a=8

b= --14

c= --15

−b ± √b −4 ac
2
x=
2a
14 ± √196 −4 [ 8 ] [−−15 ]

x=
2[8]

14 ± √196+ 480
x=
16

14 ± √ 676
x=
16

14 ±26
x=
16

14+26
x=
16

x=40/16 =5/2

14−26
x=
16

−12
x=
16

x=--12/16 =-3/4

----------------------------------

EXAMPLE :

A TRADER SELLS 6 RADIOS AND 4 TELEVISIONS FOR Rs18,480.

IF 14 RADIOS AND 2 TELEVISIONS ARE SOLD FOR THE SAME AMOUNT, WHAT IS THE PRICE
OF A TELEVISION ?
Solution :

6R + 4T=18,480

14R+2T=18,480

EXAMPLE :

A TRADER SELLS 3 A’s AND 4 B’s FOR Rs3,900.

IF 4 A’s AND 3 B’s ARE SOLD FOR Rs3,800, WHAT IS THE PRICE OF A & B ?

----------------------------------------------------------------------------------

========================================================

YOU ARE GIVEN TWO DIFFERENT POINTS ON THE PLANE or GRAPH NAMELY

(X1 , Y1) AND (X2,Y2)

YOU ARE ASKED TO FIND OUT THE EQUATION OF THE STRAIGHT LINE WHICH IS PASSING
THROUGH THE ABOVE 2 POINTS.

METHOD :

THE EQUATION FORMAT

Y—Y1 X—X1

-------- = ----------

Y2—Y1 X2—X1

IF WE SIMPLIFY THIS WE CAN GET AN

EQUATION WHICH WILL BE OF THE FORM


Y=mX +C

WHERE Y= DEPENDENT VARIABLE

X=INDEPENDENT VARIABLE

m= COEFFICIENT THAT GETS

MULTIPLIED TO X=SLOPE

OF THE EQUATION

C = CONSTANT VALUE OF Y

BEFORE GETTING ADDED

TO THE X VARIABLE.

EXAMPLE :

FIND THE EQUATION OF A STARIGHT LINE PASSING THROUGH THE TWO POINTS ( 5,3) and
( 12,2)

ANSWER:

Y—3 X--5

------ = -------

2—3 12—5

Y—3 X--5

------ = -------

—1 7

7(Y—3) = --1 (X—5)

7Y—21=--X+5

7Y =--X+5+21
7Y= --X+26

Y= --X 26

----- + ----

7 7

Y= (--1/7) X + (26/7)

HOW TO FIND OUT THE SLOPE OF THE EQUATION OF A STRAIGHT LINE PASSING THROUGH
TWO DIFFERENT POINTS ?

METHOD:

Y2—Y1

----------

X2—X1

FIND THE SLOPE OF THE EQUATION OF A STARIGHT LINE PASSING THROUGH THE TWO
POINTS ( 5,3) and ( 12,2)

-----------------------------------------------

IF TWO POINTS ARE GIVEN , THE EQUATION IS


(Y—Y1) = Y2—Y1 ( X—X1)

----------

X2—X1

FIND THE EQUATION OF A STARIGHT LINE PASSING THROUGH THE TWO POINTS ( 5,3) and
( 12,2)

------------------------------------------

FIND THE SLOPE OF THE FOLLOWING EQUATION

X+7Y—26=0

SOLUTION

--

[ COEFFICIENT OF X/ COEFFICIENT OF Y]

=--[1/7]

=--1/7

( X1 ,Y1)=( 29/5 , --2/5)

( X2 ,Y2)=( 1 , 0 )

WE HAVE TO FIND THE EQUATION OF THE STRAIGHT LINE PASSING THROUGH THE ABOVE
TWO POINTS…

Y—Y1 X—X1

-------- = ----------

Y2—Y1 X2—X1
Y—[--2/5] X—29/5

-------------- = ------------

0—[--2/5] 1—29/5

Y+2/5 X—29/5

-------------- = ------------

2/5 1—29/5

[5Y+2]/5 [5X—29]/5

-------------- = --------------

2/5 [5—29]/5

[5Y+2] [5X—29]

-------------- = --------------

2 [5—29]

[5Y+2] [5X—29]

-------------- = --------------

2 [—24]

[5Y+2] [5X—29]

-------------- = --------------

1 [—12]

[--12][5Y+2] = 5X—29
--60Y—24=5X—29

--60Y=5X—29+24

--60Y=5X—5

--12Y=X—1

X+12Y=1

-----------------------------------------------

5X—2Y=7

LET US FIND ITS SLOPE

--[5/(--2)]

=--5/(--2)

=5/2

WHAT IS THE SLOPE OF A STRAIGHT LINE WHICH IS PERPENDICULAR TO THE ABOVE


STRAIGHT LINE ?

=--2/5

-------------------------------------------------------------------------------------------

EQUATION OF A STRAIGHT LINE WHICH PASSES THROUGH A POINT (X1, Y1) AND THE SLOPE
OF THE STRAIGHT LINE IS “m”

ANSWER

(Y—Y1)= m (X—X1)

MID POINT OF [X1,Y1] AND [X2,Y2]


= X1+X2 Y1+Y2

-------- AND ---------

2 2

= 2+(--4) 7+1

-------- AND ---------

2 2

= 2--4 7+1

-------- AND ---------

2 2

= --2 8

-------- AND ---------

2 2

= --1 AND 4

WHAT IS THE EQUATION OF A STRAIGHT LINE PASSING THROUGH

(X,Y) AND HAVING ITS SLOPE m

IS

Y—Y1= m(X—X1)
Y—4 =--2/5(X—(-1))

Y—4 =--2/5(X—(-1))

Y—4 =--2/5(X+1)

Y—4 =--2/5X—2/5

Y =--2/5X—2/5+4

Y =(--2/5)X—2/5+4

Y +(2/5)X=—2/5+4

Y +(2/5)X=(—2+20)/5

5Y +2X=(—2+20)

5Y +2X=18

2X+5Y—18=0

============REVISE UPTO THIS ON 5.6.2016==============

EXERCISE:

FIND THE EQUATION OF A STRAIGHT LINE WHICH IS PERPENDICULAR TO THE LINE 3X—
4Y=10

AND PASSING THROUGH THE MID POINT OF THE LINE JOINING

(4,2) AND(6,8)

ANSWER

4X+3Y=35

SOLUTION :

3X—4Y=10

--4Y= --3X+10

4Y=3X—10

Y= (3/4)X—10/4

SLOPE =3/4.
SLOPE OF THE STRAIGHT LINE WHICH IS PERPERNDICULAR TO THE ABOVE STRAIGHT LINE
=--4/3.

MID POINT OF (4,2) AND(6,8) = (5,5)

EQUATION OF A STRAIGHT LINE PASSING THROUGH (5,5) AND HAVING ITS SLOPE –4/3 IS
GIVEN BY

(Y—Y1)= M(X—X1)

(Y—5)= --4/3(X—5)

3(Y—5)= --4(X—5)

3Y—15=--4X+20

3Y+4X—35=0

TUTORIAL NOTE:

IF A STRAIGHT LINE IS PERPENDICULAR TO ANOTHER STRAIGHT LINE, THEY WILL CUT EACH
OTHER AT RIGHT ANGLES AT 90 DEGREES.

IF WE MULTIPLY THE SLOPES OF TWO PERPENDICULAR STRAIGHT LINES WE WILL GET –1

IF THE SLOPE OF A STRAIGHT LINE IS m THEN THE SLOPE OF A STRAIGHT LINE WHICH IS
PERPENDICULAR TO IT WILL BE
–1

----

A STRAIGHT LINE MAY HAVE A NUMBER OF PERPENDICULAR STRAIGHT LINES.

---------------------------------------------------------------------------------------------

TWO PARALLEL STRAIGHT LINES WILL HAVE THE SAME SLOPE.

FORMULA FOR FINDING THE DISTANCE UNITS IN BETWEEN TWO POINTS ON THE PLANE

[X1 , Y1] and [X2 ,Y2]

WHOLE SQUARE ROOT OF

[X2—X1]2 + [Y2—Y1]2

FIND OUT THE DISTANCE UNITS IN BETWEEN THE POINTS

[4,3] AND [6,10]

SOLUTION :

WHOLE SQUARE ROOT OF

[X2—X1]2 + [Y2—Y1]2

WHOLE SQUARE ROOT OF

[6—4]2 + [10—3]2

WHOLE SQUARE ROOT OF

[2]2 + [7]2
=

WHOLE SQUARE ROOT OF

53

=7.280 UNITS.

TUTORIAL NOTE

WHAT IS THE EQUATION OF A STRAIGHT LINE WHICH IS PERPENDICULAR TO THE STRAIGHT


LINE ax + by + c1=0 ?

ANSWER

bx –ay + c2 =0

WHAT IS THE EQUATION OF A STRAIGHT LINE WHICH IS PERPENDICULAR TO THE STRAIGHT


LINE 2x + 1y --6=0 ?

ANSWER

1x –2y + c =0

x –2y + c =0

------------------------------------------------

Formula for calculating the area of a triangle whose vertices are

(x1,y1) , (x2,y2) and (x3,y3)

½ {x1(y2—y3) + x2(y3—y1)

+ x3(y1—y2)}

-------------------------------------

FORMULA FOR INTERNAL DIVISION


x= mx2 + nx1

----------------

m+n

y= my2 + ny1

----------------

m+n

FORMULA FOR EXTERNAL DIVISION

x= mx2 -- nx1

----------------

m--n

y= my2 -- ny1

----------------

m--n

WHAT IS THE MEANING OF INCENTRE OF A TRIANGLE ?

ANSWER:

THE INTERNAL BISECTORS OF ANGLES OF A TRIANGLE ARE CONCURRENT AT A POINT AND


SUCH A POINT IS NAMED AS INCENTRE OF A TRIANGLE.

EXAMPLE :

FIND THE INCENTRE OF A TRIANGLE WHOSE VERTICES ARE (0,0),(--3,4 ) AND (8,6)

SOLUTION :
FORMULA FOR FINDING THE x CORDINATE OF THE INCENTRE OF A TRIANGLE =

ax1 +bx2 +cx3

--------------------

a+b+c

we know that

x1=0

x2=--3

x3=8

how to find out a ?

consider second vertex and third vertex.

(--3,4) and (8,6)

Whole Sq rt of

[8—(--3)]2 + [6—4]2

Whole Sq rt of

[8+3]2 + [2]2

Whole Sq rt of

[11]2 + [2]2

Whole Sq rt of
[121] + [4]

Whole Sq rt of

125

Whole Sq rt of

25 x5

5 x √5

how to find out b ?

consider third vertex and first vertex.

(8,6) and (0,0)

Whole Sq rt of

[0—8)]2 + [0—6]2

Whole Sq rt of

[64] + [36]

Whole Sq rt of

100

=10

how to find out c ?

consider first vertex and second vertex.

(0,0) and (--3,4)


Whole Sq rt of

[--3—(0)]2 + [4—0]2

Whole Sq rt of

[9] + [16]

Whole Sq rt of

25

5.

x CO ORDINATE OF THE INCENTRE OF A TRIANGLE =

ax1 +bx2 +cx3

--------------------

a+b+c

we know that

x1=0

x2=--3

x3=8

a=5√ 5
b=10

c=5

x CORDINATE OF THE INCENTRE OF A TRIANGLE =

ax1 +bx2 +cx3

--------------------

a +b+c

5√ 5 {0 } + 10{--3} + 5{8}

--------------------------------

5√ 5 +10 +5

0 + --30} + 40

--------------------------------

5√ 5 +15

10

--------------------------------

5{√ 5 +3}

--------------------------------

{√ 5 +3}

y COORDINATE OF THE INCENTRE OF THE TRIANGLE =


ay1 +by2 +cy3

--------------------

a+b+c

5√ 5 {0 } + 10{4} + 5{6}

--------------------------------

5√ 5 +10 +5

0 + 40 + 30

--------------------------------

5√ 5 +15

70

--------------------------------

5{√ 5 +3}

14

--------------------------------

{√ 5 +3}

The incentre of a the triangle is

2 14

---- , -------

{√ 5 +3} {√ 5 +3}
Example :

A MAN JOINS A JOB WITH A CERTAIN MONTHLY SALARY AND EARNS A FIXED INCREMENT
EVERY YEAR.IF HIS SALARY WAS Rs15,000 PER MONTH AFTER 4 YEARS OF JOINING AND
Rs18,000 PER MONTH AFTER 10 YEARS OF SERVICE,FIND HIS JOB JOINING STARTING PAY.
ALSO FIND HIS ANNUAL INCREMENT.

SOLUTION :

LET THE STARTING PAY BE c

LET THE ANNUAL INCREMENT BE a

LET THE NUMBER OF YEARS BE x

SO THE EQUATION FORM

IS

Y= ax + c

5TH YEAR MONTHLY SALARY=Rs15,000

15,000= a(4) +c

4a+c=15,000----------EQN 1

11TH YEAR MONTHLY SALARY=Rs18,000

18,000= a(10) +c

10a+c=18,000----------EQN 2

EQN 2—EQN1

6a=3,000

a=500
Substituting a=500 in equation 1, we get

4a+c=15,000----------EQN 1

4[500]+c=15,000

2000+c=15,000

C=13,000.

Starting pay Rs13,000

ANNUAL INCREMENT=Rs500.

Solution to the cubic equation

X3 + X2 –X—1 =0

[X3 –1] + [X2 –X] =0

[X—1 ][X2 +X+1] +[ X2 –X]=0

[X—1 ][X2 +X+1] +X[ X –1]=0

[X—1 ]{[X2 +X+1] +X]=0

[X—1 ]{[X2 +X+1 +X]=0

[X—1 ][X2 +2X+1 ]=0

[X—1 ][X2 +2X+1 ]=0

[X—1 ][X+1]2=0

IF[X—1 ]=0

X=1

IF [X+1]2 =0

X=--1
SO

THE VALUE OF X MAY BE 1 OR –1

ANOTHER METHOD FOR SOLVING A CUBIC EQUATION

THE CUBIC EQUATION IS

X3 + X2 –X—1 =0

ON TRIAL BASIS LET US PUT X=1

1+1—1—1=0

SO X=1 IS A SOLUTION TO THE ABOVE CUBIC EQUATION.

LET US TRY TO EXPLORE WHETHER OTHER SOLUTIONS EXIST FOR THE ABOVE CUBIC
EQUATION.

LET US WRITE DOWN THE ORIGINAL CUBIC EQUATION…

X3 + X2 –X—1 =0

SINCE X=1

X—1=0

SO X—1 IS A FACTOR.

LET US DIVIDE THE ORIGINAL CUBIC EQUATION THROUGH OUT BY X—1

X 3 + X2 – X — 1 = 0

-----------------------
X--1

INEQUALITIES:

3 X1 + 1.5 X2 ≤ 225

OR

3X + 1.5 Y ≤ 225

--------------------------------------------

LET US ASSUME THAT BOTH THE ABOVE INEQUALITIES ARE PRESENT FOR THE
BUSINESSMAN.

HE CAN DRAW A GRAPH OF INEQUALITIES FOR FINDING A FEASIBLE REGION USING THE
FOLLOWING POINTS ON THE GRAPH

X1 X2 CONTRAINT
1ST POINT 75 0 SPACE=225
1ST POINT 0 150 SPACE=225
2ND POINT 130 0 LABOUR HOURS=520
2ND POINT 0 173.33 LABOUR HOURS=520
KNOW THAT

[A3—B3] CAN BE WRITTEN AS

[A—B][A2 +AB +B2]

THE SOLUTION OF THE INEQUALITY

5—2X X

--------≤ { ------- --5 }

3 6

5—2X X--30

--------≤ -------

3 6

6[5—2X] ≤ 3[ X—30]

30—12X ≤ 3X—90

30+90 ≤ 3X+12X
120 ≤ 15X

8 ≤ X

-----------------------------------------------

IF MODULUS OF 3X—4 5

-------- ≤ ----

4 12

IF PLUS IS CONSIDERED

12[3X—4] ≤ 20

36X—48 ≤ 20

36X ≤ 20+48

36X ≤ 68

X ≤ 68

----

36

X ≤ 17

----

IF MINUS IS CONSIDERED

-- 12[3X—4] ≤ 20

--36X+48 ≤ 20

-- 36X ≤ 20--48

--36X ≤ 68
--X ≤ 68

----

36

-- X ≤ 17

----

=======================================

EXAMPLE

2X+3Y=18

 FIND ITS SLOPE AND CONSTANT


 CHECK WHETHER (30 AND --14) LIE ON THIS EQUATION ?

3Y=--2X+18

Y= (--2/3) X + 18/3

Y= (--2/3) X + 6

SLOPE OF THE ABOVE EQUATION IS THE COEFFICIENT OF X =--2/3

CONSTANT VALUE OF THE ABOBE EQUATION =6

=====================================

2(30) + 3(--14) =18

SO (30,--14) LIE IN THIS EQUATION.

HOME WORK :

EXAMPLE

X+Y=10

 FIND ITS SLOPE AND CONSTANT


 CHECK WHETHER (2, 4) LIE ON THIS EQUATION ?
 CHECK WHETHER (2,8) LIE ON THIS EQUATION ?
 CHECK WHETHER (1,9) LIE ON THIS EQUATION ?

EXAMPLE:

RAM PAID TO A CALL TAXI A SUM OF RUPEES 89 FOR TRAVELLING 12 KILOMETRES.

KUMAR PAID TO THE SAME CALL TAXI A SUM OF RUPEES 145 FOR TRAVELLING 20
KILOMETRES.

HOW MUCH GEETH WILL PAY IF HE TRAVELS 15 KILOMETRES.

SOLUTION

FIXED COST + VARIABLE COST = MIXED COST

=============================

12 KM’s MIXED COST = Rs 89

20 KM’s MIXED COST = Rs 145

============================

Variable cost per km = DIFFERENCE IN MIXED COST/ DIFFERENCE IN KM

= ( Rs145—Rs 89) / ( 20 km—12km)

= Rs56/8

= Rs 7.

mixed cost – variable cost=fixed cost

Rs89—(12kmxRs7) =Rs 5

Rs145—(20kmxRs7)=Rs5

========================================

Geeth’s fare

(15 km x Rs7)+ Rs5 = Rs110.


Exercise :

Anjay spent Rs1600 to produce 10 articles while sanjay spent Rs1310 to produce 8 articles.
Assuming same cost pattern how much manjay will spend to produce 6 articles?

Solution :

10 articles = Rs1600

8 articles = Rs 1310

Fixed cost +variable cost = mixed cost

Variable cost per article= difference in mixed cost/ difference in units

= (Rs1600—Rs1310)/ ( 10 articles—8articles)

=Rs 290 /2

= Rs145

Mixed cost—variable cost = fixed cost

Rs1,600—( Rs145x10 articles) = Rs150

Rs1310—( Rs145x8 articles) = Rs150

============================================

Manjay’s mixed cost

=(Rs145 x 6 articles) + Rs150

= Rs1020
--Example :

FIND A FRACTION WHICH BECOMES ½ WHEN 1 IS SUBTRACTED FROM THE NUMERATOR AND 2
IS ADDED TO THE DENOMINATOR AND BECOMES 1/3 WHEN 7 IS SUBTRACTED FROM THE
NUMERATOR AND 2 FROM THE DENOMINATOR.

Solution :

Let the fraction be X/Y

(X—1)/ ( Y+2) =1/2

X—1 1

------- = ----

Y+2 2

CROSS MULTIPLY

2(X—1) = 1(Y+2)

2X—2 =Y+2

2X—Y=4 ------------EQN 1

Let the fraction be X/Y

X—7 1

-------- = ----

Y—2 3

CROSS MULTIPLY
3(X—7)= 1(Y—2)

3X—21 =Y—2

3X—Y=19--------EQN 2

NOW WE HAVE TWO EQUATIONS WITH US

2X—Y=4 ------------EQN 1

3X—Y=19--------EQN 2

UPON SOLVING

X=15 AND Y=26

SO THE FRACTION IS 15/26

SOLUTION :

Class work :

Anu asked a trader to give change for a Rs500 note.The trader gave change in Rs20 and Rs 50
denomination. Totally 19 currency notes were tendered by trader to Anu. The pack contained
how many Rs20 notes and how many Rs50 notes?

ANSWER :

50 X4 + 20 X15

EXAMPLE :

A PERSON INVESTED SOME AMOUNT AT THE RATE OF 12% SIMPLE INTEREST AND SOME OTHER
AMOUNT AT THE RATE OF 10% SIMPLE INTEREST. HE RECEIVED YEARLY INTEREST RS130. BUT IF
HE HAD INTERCHANGED THE AMOUNTS INVESTED, HE WOULD HAVE RECEIVED RS 4 MORE AS
INTEREST. HOW MUCH AMOUNT DID HE INVEST AT THE RATE OF 10% PER YEAR?

SOLUTION :

LET X BE THE AMOUNT INVESTED AT 12% SIMPLE INTEREST PER ANNUM

SO THE INTEREST PER ANNUM = (12/100) X =0.12 X


LET Y BE THE AMOUNT INVESTED AT 10% SIMPLE INTEREST PER ANNUM

SO THE INTEREST PER ANNUM = (10/100) Y =0.10 Y

0.12 X + 0.10 Y= 130---- EQN 1.

LET Y BE THE AMOUNT INVESTED AT 12% SIMPLE INTEREST PER ANNUM

SO THE INTEREST PER ANNUM =(12/100)Y= 0.12 Y

LET X BE TE AMOUNT INVESTED AT 10% SIMPLE INTEREST PER ANNUM

SO THE INTEREST PER ANNUM =(10/100) X= 0.10 X

0.12 Y+ 0.10 X=134---- EQN 2

WE HAVE TO SOLVE THE EQUATIONS

0.12 X + 0.10 Y= 130---- EQN 1.

0.12 Y+ 0.10 X=134---- EQN 2

------------------------------------------

0.12 X +0.10 Y=130--------EQN 1

0.10X+0.12Y=134----------EQN 2

------------------------------------------------

LET US MULTIPLY EQN 1 WITH 10

1.2 X+ 1Y=1300-----------EQN 3

LET US MULTIPLY EQN 2 WITH 12

1.2X+1.44Y=1608-----------EQN 24

---------------------------------------------

EQN4—EQN3
1.2X+1.44Y=1608

1.2X+1Y=1300

-------------------------

0.44Y=308

Y= 308/0.44 =700

SO Y=700

PUT Y=700

IN EQN 4

1.2X+1.44Y=1608

1.2X+1.44(700)=1608

1.2X+1008=1608

1.2X=1608—1008

1.2X=600

X=500

-----------------------------------------

AMOUNT INVESTED IN 12% =X= Rs500

AMOUNT INVESTED IN 10%=Y= Rs 700

EXAMPLE :

IF TWICE THE SON’S AGE IN YEARS IS ADDED TO THE FATHER’S AGE, THE SUM IS 70,BUT IF
TWICE THE FATHER’S AGE IS ADDED TO THE SON’S AGE, THE SUM IS 95. THE AGE OF FATHER
AND SON IN YEARS=?
Solution :

LET THE AGE OF FATHER BE F

sLET THE AGE OF SON BE S

IF TWICE THE SON’S AGE IN YEARS IS ADDED TO THE FATHER’S AGE, THE SUM IS 70

2S +F =70-----------EQN 1

IF TWICE THE FATHER’S AGE IS ADDED TO THE SON’S AGE, THE SUM IS 95

2F+S=95----------EQN 2

WE HAVE TO SOLVE

2S+F= 70------EQN 1

S+2F=95---EQN 2

-----------------------------

EQN 2 X 2 WE GET

2S+4F=190-------EQN 3

-------------------------------------

EQN 3—EQN 1

2S+4F=190

2S+F=70

--------------------

3F=120

-----------------------

F= 120/3 =40

PUT F=40 IN EQN 1

2S+F= 70------EQN 1
2S+40=70

2S= 70—40

2S=30

S=15

EXAMPLE : A NUMBER CONSISTS OF 2 DIGITS IS 7 TIMES THE SUM OF ITS DIGITS. WHEN 27 IS
SUBTRACTED FROM THE NUMBER, THE DIGITS ARE REVERSED. THE NUMBER IS ?

Solution :

LET THE TWO DIGIT NUMBER BE “XY”

A NUMBER CONSISTS OF 2 DIGITS IS 7 TIMES THE SUM OF ITS DIGITS.

THE NUMBER IN THE UNITS PLACE= Y

THE NUMBER IN THE TEN’S PLACE=X

(X x10)+ (Yx1) = 7(X+Y)

10X + Y =7X+7Y

10X—7X=7Y—Y

3X=6Y

X=2Y ---------EQN 1

WHEN 27 IS SUBTRACTED FROM THE NUMBER, THE DIGITS ARE REVERSED.

ORIGINAL NUMBER =10X+Y

REVERSED NUMBER=10Y+X

10X+Y—27=10Y+X

10X—X—27=10Y—Y

9X—27=9Y============EQN 2

PUT X= 2Y IN EQN 2
9X—27=9Y============EQN 2

9(2Y)—27=9Y

18Y—27=9Y

18Y—9Y=27

9Y=27

Y=27/9

Y=3

X= 2Y

X=2(3)=6

SO THE NUMBERS ARE 63 AND 36

Example :

Find the positive value of k to be found out, when the quadratic equation

12x2 + kx +5=0 have 2 two roots x and y

in the ratio 3:2.

a) 5/12 b)12/5 c)[5√ 10 /2] d) 5√ 10

Solution

x+y= --k/12 -------eqn 1

[ axiom is sum of solutions =--b/a]

xy = 5/12 -----------eqn 2

[axiom is product of solutions=c/a]


Given that x and y are in the ratio 3:2

x 3

--- = ----

y 2

2x= 3y

x= (3/2)y

put x= (3/2) y on eqn 1

x+y= --k/12 -------eqn 1

(3/2)y +y=--k/12

1.5y+y=--k/12

2.5y=--k/12

2.5y x12=--k

30y=--k

Y= (--k/30)

xy = 5/12 -----------eqn 2

put x= (3/2)y in eqn 2

(3/2)yy = 5/12 -----------eqn 2

(3/2)y 2 = 5/12

y 2 = (5/12) x (2/3)

y 2 = (10/36)

put y= --k/30 in

y 2 = (10/36)

(--k/30)2 = 10/36

K2 10
------ = ----------

900 36

K2 = (10/36) x900

K2 = 250

K2 = 25x10

K= 5√ 10

EXAMPLE :

A TRADER SELLS 6 RADIOS AND 4 TELEVISIONS FOR Rs18,480.

IF 14 RADIOS AND 2 TELEVISIONS ARE SOLD FOR THE SAME AMOUNT, WHAT IS THE PRICE
OF A TELEVISION ?

SOLUTION :

LET R BE THE THE SELLING PRICE PER RADIO

LET T BE THE SELLING PRICE PER TELEVISION

A TRADER SELLS 6 RADIOS AND 4 TELEVISIONS FOR Rs18,480.

6R + 4T =18,480----------EQUATION 1

IF 14 RADIOS AND 2 TELEVISIONS ARE SOLD FOR THE SAME AMOUNT

14R +2T=18,480-------EQUATION 2

WE HAVE TO SOLVE THE ABOVE TWO EQUATIONS….


6R + 4T =18,480----------EQUATION 1

14R +2T=18,480-------EQUATION 2

------------------------------------------------

EQN 2 X2

28R +4T=36,960----------------EQN 3

EQN3—EQN1

28R +4T=36960

6R+4T=18480

---------------------

22R =18480

----------------------

R= 18480/22 =840

PUT R=840 IN EQN 1

6R+4T=18480

6(840) +4T=18480

5040 +4T=18480

4T=18480—5040

4T=13440

T=13440/4 =3,360

===========================================================

EXAMPLE : RAVI JOINED A COMPANY AS MANAGER ON 01-04-2001 AT A MONTHLY SALARY OF


Rs5,000. EVERY YEAR HE WAS GRANTED AN INCREMENT IN THE MONTHLY SALARY. ASSUME
THAT THE INCREMENT IS DONE ON EVERY 1 ST APRIL. THE SAME SALARY WILL BE PAID FOR THAT
YEAR FROM 1ST APRIL TO NEXT 31ST MARCH. ON 01—04—2010, HIS MONTHLY SALARY IS
Rs6,800. FIND THE INCREMENT PER YEAR IN THE MONTHLY SALARY .

ANSWER :

y = mx +c

6,800=m(9) + 5,000

6,800 =9m +5,000

6,800—5,000=9m

9m=1,800

m=1,800/9

m= 200

Example :

A MAN JOINS A JOB WITH A CERTAIN MONTHLY SALARY AND EARNS A FIXED INCREMENT
EVERY YEAR.IF HIS SALARY WAS Rs15,000 PER MONTH AFTER 4 YEARS OF JOINING AND
Rs18,000 PER MONTH AFTER 10 YEARS OF SERVICE,FIND HIS JOB JOINING STARTING PAY. ALSO
FIND HIS ANNUAL INCREMENT.
Solution :

y= m x +c

15,000= m(4) +c

4m + c =15,000 ----------eqn 1

y= m x +c

15,000= m(10) +c

10m + c =18,000-------------eqn 2

When we solve the above two equations, we get c=13,000 and m=500

His initial salary was Rs13,000 per month

His year increment in the monthly salary = Rs500.

==========================================================

ANALYTICAL GEOMETRY :

EXAMPLE

IN THE YEAR 5 , A VARIABLE’S VALUE= Rs1,000 AND IN THE YEAR

7, THE VARIABLE’S VALUE= 1,400. WHAT WILL BE THE VARIABLE’S VALUE IN THE YEAR 3 IF THE
VARIABLE FOLLOWS A TIME SERIES WITH VALUE 0 AT YEAR 0?

SOLUTION :

SLOPE OF A STRAIGHT LINE WHICH HAS TWO POINTS ON THE GRAPH

(X1, Y1) AND (X2,Y2)

(5, 1000) AND ( 7,1400)


y—1000 x—5

---------------- = -----------------

1400—1000 7--5

y—1000 x—5

---------------- = -----------------

400 2

2y—2000 =400x—2000

2y=400x

Y=200x

Y=200(3)=600

= DIFFERENCE IN Y / DIFFERENCE IN X

= 1400—1000 / 7—5

=400/2

=200

(X3, Y3)

3X200 =600

=====================================================

Example :

Consider a point ( 10,20) and another point ( 3,7).

 Join these two points and express the equation of the straight line passing through
these two points in the graph.

Solution :
Here the point (10,20) is taken as (x1,y1)

Here the second point (3,7) is taken as ( x2,y2)

The following pattern is given

y—y1 x –x1

-------- = ---------

Y2—y1 x2—x1

y—20 x –10

-------- = ---------

7—20 3—10

y—20 x –10

-------- = ---------

—13 —7

--7(y—20) =--13(x—10)

--7y+140 =--13x+130

--7y+13x=130—140

--7y+13x=--10

--7y+13x+10 =0

13x—7y+10 =0

Exercise

Find the equation of the straight line which passes through the two points
(3,--8) and (--2 ,11)

Answer

19x+5y—17=0

EXAMPLE :

AN EQUATION 19X+5Y—17=0 IS GIVEN. FIND ITS SLOPE.

SOLUTION:

WE HAVE TO CONVERT 19X+5Y—17=0 IN TO

y= mx + c form

where

y= value of dependent variable


m= regression coefficient of y on x or slope of y on x

c= constant value of the dependent variable independent of x

19X+5Y—17=0

5Y = --19X +17

Y= =( --19/5)X + (17/5)

Y=--3.8X + 3.4

SO THE SLOPE OF Y ON X= --3.8

SO THIS EQUATION WILL SLOPE DOWNWARDS FROM Y TO X

AS X INCREASES Y WILL DECREASE.

RESEMBLES DEMAND CURVE. AS PRICE INCREASES, DEMAND FALLS DOWN.

====================================================

EXAMPLE : FIND THE SLOPE OF

3X+ 10Y=100

ANSWER : --0.3

-------------------------------------------------------------------------------------
EXAMPLE :

FIND THE EQUATION OF A STRIGHT LINE PASSING THROUGH A POINT (4,10) GIVEN THAT THE
SLOPE OF THE EQUATION IS 3.

SOLUTION :

(y—y1) = m(x—x1)

(y—10) =3(x—4)

y—10 =3x—12

y—3x= --12+10

y—3x=--2

y= 3x—2

Home work

FIND THE EQUATION OF A STRIGHT LINE PASSING THROUGH A POINT

(--4,2) GIVEN THAT THE SLOPE OF THE EQUATION IS 2.

==========================================================
Example :

A line passes through the point ( 2,8). If its slope is 1

find the constant value of the equation.

Answer 6

Example :

A line passes through the point ( 7,8). Its slope is –1.

Find y if x= 9

Answer 6

Example :

A line passes through the point ( 7,8). Its slope is –1.

Find x if y= 4
Answer 11

Example :

Find the distance between the two points on the plane

(4,5) and ( 25,1)

Solution :

What is the formula for computing the distance in between two points ?

Square root of

(X2—X1)2 + (Y2—Y1)2
Let us record the values first

(4,5) and ( 25,1)

X1,Y1 and X2,Y2

Let us substitute the values

Square root of

(25—4)2 + (1—5)2

Square root of

(21)2 + (—4)2

Square root of

441 + 16

Square root of

457

=21.38 units

Example :

Two parallel lines will have same slope


Proof :

EXAMPLE

10 Y—4X—30=0 IS THE EQUATION OF A STRAIGHT LINE.

5Y—2X+40 =0 IS THE EQUATION OF ANOTHER STRAIGHT LINE.

ARE THESE TWO STRAIGHT LINES PARALLEL TO EACH OTHER?

SOLUTION :

10 Y—4X—30=0

SLOPE OF 10 Y—4X—30=0

10 Y =4X+30

Y = (4/10)X + (30/10)

Y = 0.4X + 3

SLOPE =0.4

5Y—2X+40 =0

SLOPE OF 5Y—2X+40 =0

5Y—2X+40=0
5Y=2X—40

Y= (2/5)X—(40/5)

Y= 0.4X –8

SLOPE =0.4

=======================================================

Class of 6TH JUNE 2016.

Two perpendicular lines will intersect each other at an angle of 90 degrees.

If we multiply the slope of two perpendicular lines the result will be –1 .

Example :

Consider a straight line 2x+y=10. Consider its perpendicular line which is

3x + ( W )y=5.

FIND W.

Solution :

Slope of 2x+y=10

2x+y=10

y= --2x + 10

slope =--2

====================================

The perpendicular line will have slope ( ½)

So

3x + Wy=5. Will have slope as ½


W y = -- 3x +5

Y= --3 X +5

---

--3 1

---- = ----------

W 2

W= --6

======================================================

Class work

Two lines on a plane are given below…

10y +20x=50

100y =50 x + 1800

[a] They are parallel to each other

[b] They are perpendicular to each other

[c] Their slopes are equal

[d] They are not lines

=======================================================

Example :

A boat travels downstream of a flowing water body and reached the destination in 4 hours.
While returning 5 hours was spent. Stream speed is 2 km per hour. What is the boat’s speed
per hour. Also compute the distance in between destinations?
Solution

Speed per hour x hours spent = distance covered

 LET X REPRESENT THE SPEED OF THE BOAT PER HOUR IN THE STILL WATER
 LET[ X+2 ] REPRESENT THE SPEED OF THE BOAT PER HOUR IN THE DOWN FLOW
WATER
 LET[ X—2] REPRESENT THE SPEED OF THE BOAT PER HOUR IN THE UPFLOW WATER

[X+2] 4 =[X—2]5

Down stream

(X +2) x 4 = D-----eqn 1

Upstream

(X—2) x 5 =D------eqn 2

===============================

4x+ 8 = 5x—10

X=18

Boat’s still water speed =18km per hour

Boat’s upstream speed = 16 km per hour

Boat’s down stream speed = 20 km per hour

===========================================

Upstream time = 5 hours

Upstream distance = 16 km per hour x5 hours= 80 km

Down stream distance = 20 km per hour x4 hours= 80 km

=======================================================
6TH JUNE 2016

EXAMPLE :

FIND THE AREA OF THE TRIANGLE FORMED BY THE THREE POINTS

(5,2 ) , ( --4,--10) AND ( 6,--1)

SOLUTION :

AREA OF A TRIANGLE FORMED BY THREE POINTS

(X1,Y1) , (X2,Y2) AND ( X3,Y3) IS GIVEN BY

(5,2 ) ( --4,--10) ( 6,--1)

X1(Y2—Y3) + X2(Y3—Y1) + X3(Y1—Y2)

------------------------------------------------------

5(--10+1) --4(--1—2) + 6(2+10)

------------------------------------------------------

5(--9) --4(--3) + 6(12)

------------------------------------------------------

--45 +12 + 72

------------------------------------------------------

2
--45 +84

------------------------------------------------------

39

-------- =19.5 SQUARE UNITS.

=====================================================

Meaning of centroid of a triangle:

Centroid of a triangle means the point of concurrence of its medians.

Centroid of a triangle is symbolized as G.

The Centroid G will divide each median in the ratio 2:1 internally.

=======================================================

All the 3 medians of the triangle will pass through the centroid.

Formula for computing the centroid of a triangle

Whose vertices are (x1,y1), (x2,y2) and (x3,y3)

G= Centroid =[ (x1+x2+x3)/ 3 , (y1+y2+y3)/3]

Example :

Find the centroid of a triangle whose vertices are

(1,--2) ,(--5,3) and (7,2)

Solution :

G = {( 1—5+7)/3 , ( --2+3+2)/3 }
G = {( 3)/3 , (3)/3 }

G = {1,1 }

Home work

The vertices of a triangle are

(4,3) , (10,y) and (x,18). Its centroid is ( 2,4). Find x and y.

Answer :

x= --8 and

y=--9

=====================================================

INCENTRE OF A TRIANGLE

Incentre is a point at which the internal bisectors of angles of a triangle are concurrent.

Incentre of a triangle is symbolized as I

Formula for Incentre of a triangle

I = ax1+bx2+cx3 ay1+by2+cy3

-------------------- , ------------------

a+b+c a +b+c

where

A(x1,y1) , B(x2,y2) and C(x3,y3) are the three veritices of the triangle and

a= distance between B and C

b= distance between C and A

c= distance between A and B

Home work:

Find the incentre of a triangle whose vertices are


A(0,0),

B(--3,4)

and

C (8,6)

Formula for Incentre of a triangle

I = ax1+bx2+cx3 ay1+by2+cy3

-------------------- , ------------------

a+b+c a +b+c

Answer : [5/13 , 35/13]

WHEN THE DIFFERENT POINTS OF [X,Y] ARE SAID TO BE COLLENEAR ON A GRAPH ?

ANSWER : THE POINTS[ X1,Y1],[ X2,Y2], [X3,Y3,] ETC ARE SAID TO BE COLLENEAR WHEN ALL
THESE POINTS LIE ON A STRAIGHT LINE. IN OTHER WORDS IF WE SUBSTITUTE X AND Y VALUES,
THE EQUATION WILL GET SATISFIED.

EXAMPLE: CONSIDER A STRIGHT LINE

2X+3Y=40 CHECK WHETHER

(5,10) AND ( 20,0) ARE COLLINEAR TO THE ABOVE LINE ?


ANSWER :

IN THE STARAIGHT LINE IF WE SUBSTITUTE(5,10) IT IS SATISFIED.

IN THE SAME STRIGHT LINE IF WE SUBSTITUTE (20,0) IT IS SATISFIED.

SO THE POINTS ARE COLLENEAR.

EXAMPLE :

PROVE THAT THE POINTS (5,10), (20,0) AND (2,12) CAN NEVER BE THE VERTICES OF A
TRIANGLE.

ANSWER :

SUBSTITUTE THE ABOVE 3 POINTS IN THE AREA FORMULA FOR A TRIANGLE. THE ANSWER WILL
BE 0.

-------------------------------------------------------------------------------

Formulate the equation of the straight line which passing through the points (--4,2) and (2,--4)

Answer :

Y=--x—2

or

X+y+2=0

-------------------------------------------------------------------------------------------
Consider two points on the plane

(10,20) and (100,200).

What is the slope of the straight line which passes through these two points ?

----------------------------------------------------------------------------------------------

Example :

Find the positive value of k to be found out, when the quadratic equation

12x2 + kx +5=0 have 2 two roots x and y

in the ratio 3:2.

a) 5/12 b)12/5 c)[5√ 10 /2] d) 5√ 10

Solution :A quadratic equation will have two solutions. The solutions of a quadratic equation
are also named as roots of the quadratic equation.

Normally the roots are named as α and β .

12x2 + kx +5=0  ax2 + bx +c=0

Here a= 12

b=k

c=5

α + β = --k/12 -------eqn 1

[ axiom is sum of solutions =--b/a]

α β = 5/12 -----------eqn 2

[axiom is product of solutions=c/a]

Given that α and β are in the ratio 3:2

α 3

--- = ----

β 2
2α=3β

α= (3/2) β

put α = (3/2) β on eqn 1

α + β = --k/12 -------eqn 1

(3/2) β+ β =--k/12

1.5 β+ β =--k/12

2.5 β=--k/12

2.5 β x12=--k

30 β =--k

β = (--k/30)

α β = 5/12 -----------eqn 2

put α = (3/2) β in eqn 2

(3/2) β β = 5/12 -----------eqn 2

(3/2) β 2 = 5/12

β 2 = (5/12) x (2/3)

β 2 = (10/36)

put β = --k/30 in β 2 = (10/36)

(--k/30)2 = 10/36

K2 10

------ = ----------

900 36

K2 = (10/36) x900

K2 = 250

K2 = 25x10
K= 5√ 10

CA QUESTION 1 ON EQUATIONS :

ON SOLVING √ X /(1−X ) + √(1−X )/ X = 13/6 FIND X

SOLUTION :

LET √ X /(1−X ) be y and √ (1−X )/ X be 1/y

So y + 1/y = 13/6

(y2 + 1)/ y = 13/6

6(y2 + 1) = 13y

6y2 + 6 = 13y

6y2 –13y + 6 = 0

(2y--3)(3y--2)=0

(2y--3)=0

2y=3

Y=3/2

(3y--2)=0

3y=2

Y=2/3

If y=3/2 ,

So √ X /( 1−X ) =3/2

Squaring on both sides

x/(1--x) = 9/4

4x = 9(1--x)

4x = 9 – 9x

4x + 9x = 9

13x = 9
X = 9/13

If y=2/3 ,

So √ X /( 1−X ) =2/3

Squaring on both sides

x/(1--x) = 4/9

9x = 4(1--x)

9x = 4 –4x

9x + 4x = 4

13x = 4

X = 4/13

CA QUESTION 2 ON EQUATIONS :

Find the positive value of k, for which the equations x2 + kx + 64 = 0 and

x2 -- 8x + k = 0 will have real roots.

Solution:

For real roots , b2 – 4ac ≥ 0

x2 + kx + 64

here a=1 , b = k , c=64

k2 – 4(1)(64) =0

k2 – 256 =0

k2 =256

k = 16.

CA QUESTION 3 ON EQUATIONS :

The equation of the straight line through the point of intersection of x+2y—5 =0 and x—3y—7
=0 and passing through the point (1,0) is:

Solution :

To get the point of intersection , solve


x+2y—5 =0 and x—3y—7 = 0

x+2y—5 =0

x—3y—7 = 0

(-) (+) (+)

-----------------

5y = -2

Y = --2/5

Substitute Y = --2/5 in x+2y—5 =0

x+2(--2/5)—5 =0

x--4/5—5 =0

x = 5 + 4/5

x = 29 / 5 .

equation of a line passing through the point(29/5 , --2/5 ) and (1 , 0 )

y – y1 / y2 –y1 = x – x1 / x2 – x1

y – (--2/5) / 0 – (--2/5) = x – (29/5) / 1 – (29/5)

y + 2/5 / 2/5 = x – 29/5 / --24/5

(--24/5)(y + 2/5) = (2/5)(x – 29/5)

(--24/5)y – 48/25 = (2/5)x – 58/25

(--24/5)y + 10/25 = (2/5)x

10/25 = (2/5)x + (24/5)y

10/25 = (10/25)x + (120/25)y

10x + 120y = 10

X + 12y = 1.

CA QUESTION 4 ON EQUATIONS :

If one root of a equation is 2 + √ 5 ,then the quadratic equation is :


Solution:

If one root of a equation is 2 + √ 5 , then the other root will be 2 -- √ 5

So the equation will be

(2 +√ 5) = α (2 --√ 5) = β

α + β = (2 +√ 5) + (2 --√ 5)

α + β = (2) + (2)

α+β=4

αβ = (2 +√ 5) (2 --√ 5)

= 4 – 5 = --1

The quadratic equation is,

X2 – (α + β)x + αβ = 0

X2 – 4x -- 1 = 0.

CA QUESTION 5 ON EQUATIONS:

THE EQUATION OF A LINE WHICH IS PERPENDICULAR TO 5X –2Y =7 AND PASSES THROUGH MID
POINT OF THE LINE JOINING (2,7) AND (--4,1) IS :

SOLUTION:

MID POINT = ((X1+X2)/2,(Y1+Y2)/2)

= (2+ (--4))/2, (7+1)/2

= (--2/2 , 8/2)

= (--1,4)

FIND SLOPE FROM THE EQUATION 5X –2Y =7

--2Y = --5X +7

Y = (--5/--2)X + (7/--2)

SINCE EQUATION IS PERPENDICULUR , m = --2/5

One point (--1,4) and one slope –2/5

Equation of a straight line = (y – y1) = m (x – x1)


= (y – 4) = --2/5 (x + 1)

= 5(y – 4) = --2 (x + 1)

= 5y – 20 = --2 x –2

5y + 2x – 18 = 0 .

CA QUESTION 6 ON EQUATIONS:

The area of the triangle with vertices (1,3) , (5,6) , (--3 , 4) in terms

Of square units is ?

Solution :

area of the triangle = ½ {x1(y2—y3) +x2 (y3—y1) + x3 (y1—y2)}

= ½ {1(6--4) +5 (4--3) – 3 (3—6)}

= ½ {2 +5 + 9}

= ½ x 16

= 8 sq.units.

CA QUESTION 7 ON EQUATIONS:

The line joining (--1,1) and (2,--2) and the line joining (1,2) and (2,k) are perpendicular each
other for the following value of k :

Solution :

slope of (--1,1) and (2,--2)

= ((y2—y1)/(x2—x1))

= ((--2—1)/(2+1))

= ((--3)/(3)) = --1 = m1

slope of (1,2) and (2,k)

= ((y2—y1)/(x2—x1))

= ((k—2)/(2—1))

= ( k--2)/(1)) = k—2 =m2


Since it is perpendicular ,

M1 x M2 = --1

--1 X (K—2) = --1

--K +2 =--1

--K = --2--1

--K = --3

K=3.

CA QUESTION 8 ON EQUATIONS:

THE SIDES OF AN EQUAILATERAL TRIANGLE ARE SHORTENED BY 12 UNITS,13 UNITS AND 14


UNITS RESPECTIVELY AND A RIGHT ANGLED TRIANGLE IS FORMED . THE SIDE OF THE
EQUILATERAL TRIANGLE IS ?

SOLUTION :

LET THE SIDE OF THE EQUILATERAL TRIANGLE BE x

THEN , SIDE 1 = (x – 12) units , SIDE 2 = (x – 13) units , SIDE 3 = (x – 14) units .

Since , the new triangle formed is a right angled triangle , therefore by Pythagoras theorem :

(SIDE 1)2 = (SIDE 2)2 + (SIDE 3)2

(x – 12)2 = (x – 13)2 + (x – 14)2

(x2 – 24x +144) = (x2 – 26x +169) + (x2 – 28x +196)

(x2 – 24x +144 --x2 + 26x –169 -- x2 + 28x --196) =0

X2 – 30x + 221 =0

(X – 17) (x -- 13 ) = 0

X = 17 , x =13 (not possible).

CA QUESTION 9 ON EQUATIONS:


The value of 6+ √ 6+ √ 6+ … … … … ∞ is ?

Solution :


Let 6+ √ 6+ √6+ … … … … ∞ = y
Squaring on both sides


6 + 6+ √ 6+ √ 6+ … … … … ∞ = y2


6+ y = y2 (since 6+ √ 6+ √6+ … … … … ∞ =y )

Y2 –y—6= 0

(y+2)(y--3)=0

Y=--2(not possible) , y=3 .

CA QUESTION 10 ON EQUATIONS:

Area of a rectangular garden is 8000 square metres.Ratio in length and breadth is 5:4 . A path of
uniform width , runs all round the outside of the garden. If the path occupies 3200 m 2 ,what is
its width ?

Solution :

Let length and breadth is 5x:4x

Area = length x breadth 100

8000 = (5x)(4x) P Q

8000 = 20x2              80 AAAaaaaaaaaaaaaaaaaaAAAAAAA 80

X2 = 400

X = 20 R S

Length = 5x = 5(20) = 100 100

breadth = 4x = 4(20) = 80

let width of the path be y metre.

Then length = (100 – 2y )

Breadth = (80 – 2y )

Area of width = Area of PQRS – Area of inner rectangle

3200 = 8000 – [(100—2y)(80—2y)]

3200 = 8000 – [8000 – 160y – 200y + 4y 2]

3200 = 8000 – 8000 + 160y + 200y – 4y 2


3200 = 360y – 4y2

4y2 – 360y + 3200 = 0

y2 – 90y + 800 = 0

(y--10)(y--80) =0

y=80 (not possible) or 10

so y = 10 .

CA QUESTION 11 ON EQUATIONS:

The value k which the points (k,1) (5,5) and (10,7) may be collinear is :

Solution:

area of the triangle = ½ {x1(y2—y3) +x2 (y3—y1) + x3 (y1—y2)} = 0

½{k(5—7)+5(7—1)+10(1—5)} = 0

½{5k — 7k +35—5+10—50} = 0

½{--2k--10} = 0

(½)2{--k—5}=0

k=--5

CA QUESTION 12 ON EQUATIONS:

The centroid of the triangle ABC is at the point (2,3) .A and B are the points (5,6) and (-1,4)
respectively. The coordinates of C are?

Solution:

Let the coordinates of C be (x,y)

Centroid =((x1+x2+x3)/3 , (y1+y2+y3)/3)

= ((5+(--1)+x)/3 ,(6+4+y)/3)

= 4+x=6 ; 10+y=9

= x=2 ; y= --1.

The coordinates of C are (2,--1).

CA QUESTION 13 ON EQUATIONS:
A man rowing at the rate of 5 km in an hour in still water takes thrice as much time in going 40
km up the river as in going 40 km down .Find the rate at which the river flows :

Solution :

Let the rate be x

In upstream In downstream

5+x 5—x

3(40/(5+x)) = (40/(5--x))

3/(5+x) = 1/(5--x)

(5+x) = (15--3x)

4x = 10

X =2.5 km per hr

CA QUESTION 14 ON EQUATIONS:

The value of 2 + 1

----------

2+ 1

----------

2 + ………..∞

Solution :

Let 2 + 1 be y

----------

2+ 1

----------

2 + ………..∞

2 + (1/y) = y
2y + 1 = y2

Y2 –2Y--1 =0

−b ± √b −4 ac
2
Y=
2a

−(−2) ± √ (4)−4 (1)(−1)


Y=
2(1)

(2)± √ ( 4 ) +(4)
Y=
2

(2)± √ 8
Y=
2

(2)± 2 √2
Y=
2

2{(1)± √ 2}
Y=
2

Y = {(1)± √ 2 }

So y = 1± √2 .

CA QUESTION 15 ON EQUATIONS:

If x3—6x2+11x—6 =0, then find the value of (3x—4).

Solution:

By trial and error method x = 1,2,3

So the answer is (--1, 2 ,5)

CA QUESTION 16 ON EQUATIONS:

If area and perimeter of a rectangle is 6000cm2 and 340 cm respectively, then the length of
rectangle is :

Solution :

Area = 6000cm2

Perimeter = 340 cm
(l)(b)=6000

B=6000/l ----------eqn 1
2(l + b) = 340

Substitute B=6000/l in 2(l + b) = 340

2(l + (6000/l)) = 340

2((l2 + 6000)/l) = 340

((l2 + 6000)= 340l/2

L2 – 170l +6000=0

(l--120)(l--50)=0

So l= 120 or 50(not possible).

CA QUESTION 17 ON EQUATIONS:

A straight line passes through a point (3, 2). Find the equation of a straight line.

Solution :

By trial and error method, point (3,2) satisfies the equation x+y=5 .

CA QUESTION 18 ON EQUATIONS:

One root of the equation x2—2(5+m)x +3(7+m) = 0 is reciprocal of the other. Find the value of m
.

Solution :

x2—2(5+m)x +3(7+m) = 0

x2—(10+2m)x +21+3m = 0

since the root of the equation is reciprocal of the other,therefore product of the roots= 1.

So c/a =1

21+3m/1=1

21+3m=1

3m=--20

M=--6.667 (or) 7 .

CA QUESTION 19 ON EQUATIONS:
If the length of a rectangle is 5 cm more than the breadth , and if the perimeter of the rectangle
is 40 cm , then the length and breadth of a rectangle will be?

Solution :

Let the breadth be b

So length = b+5

perimeter of the rectangle = 2( l + b )

40 = 2 ( (b+5) + b )

20 = 2b + 5

15 = 2b

B=7.5 .

length = b+5 = 7.5 +5 = 12.5 .

CA QUESTION 20 ON EQUATIONS:

The point of intersection of a line 2x –5y = 6 and x + y = 3 is ?

Solution :

2x –5y = 6

2x +2y = 6

-------------

--7y = 0

So y = 0

Substitute y =0 in x + y = 3

X+0=3

X=3

The point of intersection is ( 3 , 0 )

CA QUESTION 21 ON EQUATIONS:

Find the equation of a line passing through the point (1,1) and parallel to the line 3x+5y+17=0 .

Solution :
3x+5y+17=0

5y=--3x--17 .

Y=(--3/5)x –(17/5)

Slope = --3/5

Equation of a straight line = (y – y1) =m(x – x1)

(y – 1)=--3/5(x –1)

5y – 5 = --3x + 3

5y + 3x – 8 =0 .

CA QUESTION 22 ON EQUATIONS:

Find the equation of the line joining the point ( 3 , 5 ) with the point of intersection 2x +3y—5= 0
and 3x+5y—7 =0.

Solution :

2x +3y—5= 0 ---eqn 1 and 3x+5y—7 =0 --- eqn 2

Solve

Eqn1 x 3 = 6x + 9y –15 =0

Eqn 2 x 2 = 6x +10y –14=0

----------------------

--y—1=0

Y= --1

Substitute Y= --1 in 2x +3y—5= 0

2x +3(--1)—5= 0

2x --3—5= 0

2x—8=0

2x=8

X=4

The point is (4 ,--1)


Equation of a straight line = (y – y1) /( y2 –y1) = (x – x1) / (x2 – x1)

y – 5 / --1--5 = x – 3 / 4--3

y – 5 / --6 = x – 3 / 1

y – 5 = --6x + 18

6x + y – 23 =0 .

CA QUESTION 23 ON EQUATIONS:

The roots of the equation 3x2 – 14x +k = 0 will be reciprocal of each other find k ?

Solution :

A=3 , b=--14 , c = k

Product of the roots = c/a =1

k/3 =1

k =3.

CA QUESTION 24 ON EQUATIONS:

The lines 3x+4y +10 = 0 and 4x—3y +5 = 0 are ?

Solution :

3x+4y +10 = 0

4y=--3x—10

Y= (--3/4)x –(10/4)

M1=--3/4

4x—3y +5 =0

--3y=--4x—5

Y=(4/3)x—(5/3)

M2=4/3

M1 ≠ m2 so it is not parallel

M1 x M2 = --1
--3/4 x 4/3 = --1

--1 =--1

So the equations are perpendicular

CA QUESTION 25 ON EQUATIONS:

If one root of the equation x2 – 3x + k = 0 is 2 , then find k .

Solution :

Here one root = 2 .

So put x = 2 in x2 – 3x + k = 0

(2)2 – 3(2) + k = 0

4–6+k=0

--2 + k = 0

K=2.

CA QUESTION 26 ON EQUATIONS:

If the ratio of 5x—3y and 5y—3x is 3:4 ,then the value of x:y is

Solution :

5x—3y : 5y—3x = 3:4

4(5x—3y ) = 3( 5y—3x)

20x—12y = 15y—9x

20x + 9x = 15y + 12y

29x = 27y

x/y = 27/29

so x:y = 27:29 .

CA QUESTION 27 ON EQUATIONS:

If roots of the equation x2 + x + r = 0 are α and β and α3 + β3 = --6 . find r .

Solution :
x2 + x + r = 0

Here a=1 , b=1 , c=r

Sum of the roots (α + β) = --b/a = --1/1 = 1

Product of the roots (αβ)= c/a = r/1 = r

α3 + β3 = (α + β) 3 -- 3 αβ (α+ β)

--6 = (1)3 – 3(r)(1)

--6 = 1 – 3r

--6 +1 = 3r

--5 = 3r

r=--5/3 .

CA QUESTION 28 ON EQUATIONS:

For all λ belongs to R , the line (2+λ)x + (3—λ)y + 5 =0 passing through a fixed point , then the
fixed point is ?

Solution :

By trail and error method ,

x=--1 , y=--1

(2+λ)--1 + (3—λ)--1 + 5 =0

--2--λ –3+λ+ 5 =0

--2—3+5 = 0

0=0.

So the required point is (--1,--1)

CA QUESTION 29 ON EQUATIONS:

A straight line “L” is perpendicular to the line 2x +y --6 = 0 and cuts the axis at (3,0). Find the
distance from the point (2,--3) to the line “L”.

Solution :

The equation of line perpendicular to the given line 2x +y --6 = 0 is


x—2y = λ -----eqn 1

it passes through the point (3,0)

so x=3 y=0

x—2y = λ -----eqn 1

3—2(0)= λ

λ=3

put the value of λ = 3 in eqn 1

x—2y = λ

x—2y = 3

x—2y—3=0

we get a=1 b=--2 c=--3 and the point (2,--3)

P = modulus of a(x1) + b(y1) + c / √ a2 +b 2

1(2) + --2(--3) + (--3) / √ (1)2+(−2)2

2 + (6) --3 / √ 1❑ + 4❑

5/√5

√5 x √5 /√5

So p = √5

CA QUESTION 30 ON EQUATIONS:

If one root of the equation px2+qx + r = 0 is r then other root of the equation will be ?

Solution :

The root of the equation px2+qx + r = 0 are α and β

Given that α = r

Sum of the roots (α + β) = --b/a

(r + β) = --q/p ----eqn 1

Product of the roots (αβ)= c/a


(rβ)= r/p

β= 1/p

so the other root β= 1/p .

CA QUESTION 31 ON EQUATIONS:

If the ratio of the roots of the equation 4x2—6x + p = 0 is 1:2 then the value of p is ?

Solution :

Let the roots of the equation 4x2—6x + p = 0 be α , β

Here a=4 b=--6 and c=p

α : β = 1:2

α=x β=2x

Sum of the roots (α + β) = --b/a

x +2x = --(--6)/4

3x = 3/2

x=1/2 .

Product of the roots (αβ)= c/a

(x)(2x)= p/4

2x2 = p/4

Substitute x=1/2 in 2x2 = p/4

2(1/2)2 = p/4

2(1/4) = p/4

2/4 = p/4

p=2.

CA QUESTION 32 ON EQUATIONS:

If p and q are the roots of the equation x2—bx + c = 0, then what is the equation whose roots
are (pq + p + q) and (pq –p –q ) ?

Solution :
Let α = p and β = q

Sum of the roots (α + β) = --b/a

p + q = --(--b)/1

p+q=b

product of the roots (αβ) =c/a

pq=c/1

pq=c

new roots are (pq + p + q)=α and (pq –p –q )=β

sum of the roots (α + β)

= (pq + p + q) + (pq –p –q )

= pq + p + q + pq – p – q

= pq + pq

= 2pq ( since pq=c)

=2c

Product of the roots = αβ

= (pq + p + q)(pq –p –q )

= (pq)2 – (p+q)2 (since pq=c , p+q=b)

= c2 – b2

x2 – (α+β )x + αβ = 0

so x2 – 2cx + c2 – b2 = 0

CA QUESTION 33 ON EQUATIONS:

If arithmetic mean between roots of a quadratic equation is 8 and the geometric mean between
them is 5 , the equation is ?

Solution :

arithmetic mean between roots of a quadratic equation is 8

so (α+β) / 2 = 8
(α+β) = 16 ---eqn 1

Geometric mean between roots of a quadratic equation is 5

So √ αβ = 5

αβ = 25

the equation is x2 – (α+β )x + αβ = 0

so x2 – 16x + 25 = 0 .

CA QUESTION 34 ON EQUATIONS:

The equation of the straight line passing through the intersection of 4x—3y—1 = 0 and 2x—5y
+3 = 0 and parallel to 4x +5y = 6 is

Solution :

4x—3y—1 = 0 -----eqn 1

2x—5y +3 = 0 ---- eqn 2

Solving these equation (1) and (2) , we get

x=1 y=1

so the intersection point is (1,1)

the equation to the line parallel to 4x +5y = 6 passing through the point

(1,1)

We can say that 4x + 5y = λ

4(1) + 5(1) = λ

4+5= λ

λ=9

so 4x + 5y = 9

4x + 5y – 9 = 0 .

CA QUESTION 35 ON EQUATIONS:

The minimum value of the function x2—6x + 10 is

Solution :
Let x2—6x + 10= y

x2—6x + 10 – y = 0

a=1 b=--6 c=(10—y)

for real roots ,

b2 – 4ac ≥ 0

(--6)2 – 4(1)(10—y) ≥ 0

36 – 40 + 4y ≥ 0

--4 + 4y ≥ 0

4y ≥ 4

y≥1

so y = {1,2,3,………∞}.

CA QUESTION 36 ON EQUATIONS:

Find the points which divides the line joining the points (2,--2) and (--4 ,1) in the ratio 5:2
externally.

Solution :

The coordinates of the point P(x,y)

x=(m(x2) – n(x1))/m—n

m=5 , n=2

x=(5(--4) – 2(2))/5—2

x=(--20 – 4)/3

x=--24/3 = --8

y=(m(y2) – n(y1))/m—n

m=5 , n=2

y=(5(1) – 2(--2))/5—2

y=(5 + 4)/3
y=9/3 = 3

The coordinates of the point P(x,y) = (--8,3).

CA QUESTION 37 ON EQUATIONS:

If one of the roots of the equation x2+px + q is √ 3 +2 ,then the value of p and q are ?

Solution :

one of the roots of the equation is 2 + √ 3

then the other root will be 2 -- √ 3

sum of the roots = (2 + √ 3) + (2 -- √ 3)

=2+2=4

Product of the roots = (2 + √ 3)(2 -- √ 3)

= (2)2 – (√ 3)2 = 4 – 3 = 1

the equation is x2 – (α+β )x + αβ = 0

so x2 – 4x + 1= 0

comparing with x2+px + q = 0

then p=--4 q=1

CA QUESTION 38 ON EQUATIONS:

If log105 + log10(5x+1) = log10(x+5) + 1 find x

Solution :

log105 + log10(5x+1) = log10(x+5) + 1

log10 (5(5x+1))= log10(x+5) + log1010

log10 (5(5x+1))= log10 ((x+5)10)

25x + 5 = 10x +50

15x = 45

X = 3.

CA QUESTION 39 ON EQUATIONS:
If |x—2 | + |x—3 | = 7 , then x will be equal to

Solution :

If x—2 ≥ 0 and x—3 ≥ 0

x –2 + x -- 3 = 7

2x – 5 = 7

2x = 12

X=6

If x—2 ≤ 0 and x—3 ≤ 0

--(x –2) – (x – 3) = 7

--2x + 5 = 7

--2x = 2

X = --1.

CA QUESTION 40 ON EQUATIONS:

Roots of equation 2x2 +3x + 7 = 0 are α and β .the value of αβ—1 + βα—1 is

Solution :

2x2 +3x + 7 = 0

a=2 b=3 c=7

α+β= --b/a

=--3/2

αβ=c/a

=7/2

αβ—1 + βα—1

α/β + β/α

(α2+β2)/αβ

((α+β)2-- 2αβ) /αβ


((--3/2)2—2(7/2))/(7/2)

((9/4)—(7))/(7/2)

(-19/4)(2/7)

--19/14

CA QUESTION 41 ON EQUATIONS:

If the point (k,3) is at a distance of √ 5 units from the points (2,k) the value of ‘k’ is ?

Solution :

Distance of two points = whole square root of (x1 – x2) 2 + (y1 – y2)2

√5 = whole square root of (k – 2)2 + (3 – k)2

5 = (k – 2)2 + (3 – k)2

5 = k2 – 4k + 4 + 9 – 6k + k2

2k2 – 10k + 8 = 0

k2 – 5k + 4 =0

(k – 1 )(k – 4)=0

k = 1 or 4 .

CA QUESTION 42 ON EQUATIONS:

The quadratic equation x2--2kx + 16 = 0 will have equal roots when the value of ‘k’ is

Solution :

x2--2kx + 16

a=1 b=2k c=16

for equal roots ,

b2 – 4ac = 0

(2k)2 – 4(1)(16) = 0

4k2 – 64 =0

4k2 = 64
k2 =16

k = ±4 .

CA QUESTION 43 ON EQUATIONS:

If α and β are the roots of the equation x 2 + 7x + 12 = 0 , then the equation whose roots (α + β) 2
and (α – β)2 will be :

Solution :

Solve x2 + 7x + 12 = 0

(x + 3)(x + 4) = 0

x = --3 or –4

so α = --3 β = --4

(α + β)2 = (--3—4)2 = (--7)2 = 49

(α -- β)2 = (--3+4)2 = (1)2 = 1

Here

(α + β)2 = α (α – β)2 = β

Sum of the roots = (α + β)2 + (α – β)2

= 49+1 = 50

Product of the roots = (α + β)2 (α – β)2

= 49 x 1 = 49

x2 – (α+β )x + αβ = 0

x2 – 50x + 49 = 0 .

CA QUESTION 44 ON EQUATIONS:

State the type of quadrilateral formed by the vertices (1 ,1),(4, 4), (4, 8), (1,5) :

Solution :

Let the points be A(1 ,1),B(4, 4), C(4, 8),D (1,5) :

MID POINT OF AC = ((X1 +X2)/2 , (Y1 +Y2)/2)

= ((1+4)/2 , (1+8)/2)
= ( 5/2 , 9/2 )

MID POINT OF BD = ((X1 +X2)/2 , (Y1 +Y2)/2)

= ((4+1)/2 , (4+5)/2)

= ( 5/2 , 9/2 )

SO IT IS A PARALLELOGRAM.

CA QUESTION 45 ON EQUATIONS:

IF THREE POINTS (1, 3) ,(--2 , 1) AND (K , --1 ) ARE COLLINEAR THEN THE VALUE OF ‘K’ IS :

SOLUTION

area of the triangle = ½ {x1(y2—y3) +x2 (y3—y1) + x3 (y1—y2)} = 0

= ½ {1(1+1) – 2 (--1--3) + k (3--1)} = 0

½ {2 + 8 + 2K } = 0

(10 + 2K) = 0

2K = --10

K = 5.

CA QUESTION 46 ON EQUATIONS:

A SELLER MAKES AN OFFER SELLING CERTAIN ARTICLES THAT CAN BE DESCRIBED THE
EQUATION X = 25 –2Y WHERE ‘X’ IS THE PRICE PER UNIT AND ‘Y’ DENOTES THE NUMBER OF
UNIT . THE COST PRICE OF THE ARTICLE IS Rs 10 PER UNIT . THE MAXIMUM QUANTITY THAT
CAN BE OFFERED IN A SINGLE DEAL TO AVOID LOSSES .

SOLUTION :

X = 25 –2Y WHERE X = 10

10 = 25 – 2Y

15 = 2Y

Y = 7.5 (APROXIMATELY )

Y=7.

CA QUESTION 47 ON EQUATIONS:

IF KX – 4 =( K – 1 )X FIND X
SOLUTION :

KX – 4 =( K – 1 )X

KX – 4 = KX – X

X = 4.

CA QUESTION 47 ON EQUATIONS:

THE VALUE OF K FOR WHICH THE SYSTEM OF EQUATIONS KX + 2Y = 5 AND 3X +Y =1 HAS NO


SOLUTION IS ?

SOLUTION :

CA QUESTION 48 ON EQUATIONS:

The lines 3x—4y +5=0 , 7x—8y +5= 0 , 4x + 5y—45 =0 are

Solution :

3x—4y +5=0 ----eqn 1

7x—8y +5= 0 ----eqn 2

4x + 5y—45 =0 ---- eqn 3

Solve eqn1 and eqn2 we get x=5 y=5

Substitute x=5 y=5 in eqn 3, we get 0

So the lines are concurrent.

CA QUESTION 49 ON EQUATIONS:

If the roots of the equation y3 + y2 – y –1 = 0 are

Solution :

y3 + y2 – y –1 = 0

y2 (y + 1) –1(y + 1) = 0

(y + 1)(y2 – 1)=0

(y + 1)(y + 1)(y -- 1) = 0

y = --1,--1,1.

CA QUESTION 50 ON EQUATIONS:
The distance from the origin to the point of intersection of two straight lines having equations
3x—2y = 6 and 3x +2y = 18 is

Solution :

Solve 3x—2y = 6 and 3x +2y = 18

We get x=4 and y=3

So the point is (4,3)

Distance between (0,0) and (4,3)

= whole square root of (x1 – x2)2 + (y1 – y2)2

= whole square root of (0 – 4 )2 + (0 – 3)2

= whole square root of (--4)2 + (--3)2

= whole square root of 16+9

= whole square root of 25

=5.

CA QUESTION 51 ON EQUATIONS:

The equation x + 5y = 33 , (x + y )/ (x—y ) = 13/3 has the solution (x,y) is

Solution :

(x + y )/ (x—y ) = 13/3

3(x + y ) = 13 (x—y )

3x + 3y = 13x – 13y

13x—3x + 13y – 3y =0

10x—16y =0 -----eqn 1

x + 5y = 33 -----eqn 2

Eqn 2 x 10 = 10x + 50y = 330 ----eqn 3

Solve eqn 1 and eqn 3

We get x=8, y=5.

CA QUESTION 52 ON EQUATIONS:
The equation of the line parallel to the line joining the points (7,5) and (2,9) which passes
through the point (3,--4) is

Solution :

equation of the line joining the points (7,5) and (2,9).

y – y1 = { (y2 –y1) / (x2 – x1) } x –x1

y – 5 = { (9 – 5) / (2 – 7) } x – 7

y – 5 = --4/5 (x – 7)

5y – 25 = --4x + 28

5y + 4x – 53 =0

So slope m = --4/5

Equation of a straight line = y – y1 = m( x –x1 )

y + 4 = --4/5 ( x – 3)

5y + 20 = --4x +12

5y + 4x + 8 = 0.

CA QUESTION 53 ON EQUATIONS:

The number of students in each section of school is 36 .After admitting 12 new students , four
new sections were started. If total number of students in each section now is 30 , then the
number of sections initially were :

Solution :

Let the number of sections initially = x

No of students in each section = 36

Total no of students in x section = 36x

After admitting 12 new students

No of section = x + 4

no of students in each section = 30

total no of students in (x + 4 ) section = 30(x + 4)

so
36x + 12 = 30(x + 4)

36x + 12 = 30x + 120

36x –30x = 120 – 12

6x = 108

x = 18.

CA QUESTION 54 ON EQUATIONS:

If α+β = --2 and αβ= --3 , then α and β are the roots of the equation which is ?

Solution :

X2 – (α+β)x + αβ = 0

So X2 – (--2)x + (--3) = 0

x2 + 2x –3 = 0 .

CA QUESTION 55 ON EQUATIONS:

if α and β are the roots of the quadratic equation 2x2 – 4x = 1 , the value of α2/ β + β2/ α is ?

solution :

2x2 – 4x = 1

2x2 – 4x -- 1 = 0

Here a=2 b=--4 c=--1


sum of the roots = α + β = --b/a

= 4/2 = 2
product of the roots = αβ = c/a

=--1/2 .

α2/ β + β2/ α

= α3 + β3 / βα
= (α + β)3 -- 3 αβ(α + β) / αβ

= (2)3 -- 3 (-1/2)(2) / (--1/2)


= 8 + 3 (--2)

= 11(--2)
= --22 .

CA QUESTION 56 ON EQUATIONS:

A person on a tour Rs. 9600 for his expenses .If his tour is extended by 16 days , he has to cut
down his daily expenses by Rs. 20 , his original duration of tour had been .

Solution :

Let his original duration of the tour = x

Daily expense = 9600 / x

Tour extended by 16 days

Now daily expense = 9600 / ( x + 16 )

So

{ 9600 / x } -- {9600 / ( x + 16 )} = 20

{ 9600(x+16) -- 9600x } / x( x + 16 ) = 20

{ 9600x+1,53,600 -- 9600x } / x2 + 16x = 20

1,53,600 = 20(x2 + 16x)

1,53,600 = 20x2 + 320x

20x2 + 320x – 1,53,600 = 0

x2 + 16x – 7680 = 0

(x—80)(x+96)=0

x=80 or –96(not possible)

so x=80.

CA QUESTION 57 ON EQUATIONS :

The present age of a man is 8 years more than thrice the sum of the ages of his grandsons who
are twins. After 8 years , his age will be 10 years more than twice the sum of the ages of his
grandsons .The age of a man when his grandsons were born was :

Solution :

Let the present age of grandfather =x

Let the present age of grandson =y


1st condition

X = 3y + 8

After 8 years

age of grandfather =x + 8

present age of grandson =y + 16

2nd condition

( x + 8 ) = 2( y + 16 ) + 10

( 3y + 8 + 8 ) = 2y + 32 + 10

3y +16 = 2y + 42

3y – 2y = -- 16 + 42

y = 26

X = 3y + 8

X = 3(26) + 8

X = 78 + 8

X = 86

Both grandson’s are twins so their age = 26/2 = 13

So the grandfather age when his grandson born = 86 – 13 = 73.

CA QUESTION 57 ON EQUATIONS :

The roots of the cubic equation x3—7x + 6 = 0 are :

Solution :

x3—7x + 6 = 0

x3 – x2+ x2—7x + 6 = 0

x3 – x2+ x2—x – 6x + 6 = 0

x3 – x2+ x2—x – 6x + 6 = 0

x2(x—1) + x(x—1) –6(x—1)=0


(x—1)( x2+ x – 6)=0

(x—1)(x—2)(x +3)=0

So x= 1,2,--3.

CA QUESTION 58 ON EQUATIONS :

The roots of the equation 4x2--12x + k = 0 are equal , then the value of k is

Solution :

4x2--12x + k = 0

Here a=4 b=--12 c=k

Since the roots are equal ,

b2 – 4ac = 0

(--12)2 – 4(4)(k) = 0

144 – 16k =0

16k = 144

k =9.

CA QUESTION 59 ON EQUATIONS:

If 3x –y = 2 , 5x + ay = 3 and 2x + y = 3 are concurrent lines , then the value of ‘a’ is :

Solution :

3x –y = 2 ---eqn1

5x + ay = 3 ---eqn2

2x + y = 3 ---eqn3

By solving eqn1 and eqn3

We get x=1 , y=1

Substitute x=1 , y=1 in eqn2

5x + ay = 3

5(1) + a(1) = 3
5+a=3

a = --2.

CA QUESTION 60 ON EQUATIONS:

The equation of the line passing through the point of intersection of the lines y = 3 and x + y = 0
and parallel to 2x –y = 4.

Solution :

y = 3 --- eqn 1

x+y=0

solving eqn1 and eqn2 we get

x= --3 , y=3

find slope for 2x –y = 4.

--y = --2x +4

y = 2x – 4

so slope m = 2

equation of a straight line = y – y1 = m( x – x1 )

y – 3 = 2 ( x + 3)

y – 3 = 2x + 6

2x – y + 9 = 0

CA QUESTION 61 ON EQUATIONS:

Let E1 and E2 are two linear equations in two variables x and y . (0 ,1 ) is a solution of both
equations E1 and E2 . (2 ,--1 ) is a solution of equation E1 only and (--2 , --1 ) is solution of E2
only then E1 and E2 are ,

(a) x=0 , y=1


(b) 2x –y =--1 , 4x +y =1
(c) x+y=1 , x—y = --1
(d) x +2y=2, x+y=1

Solution :

By trial and error method , option (c) is correct


(2 ,--1 ) is a solution of equation E1

So E1 be x+y=1

The solution satisfies the equation

(--2 ,--1 ) is a solution of equation E2

So E2 be x--y=--1

The solution satisfies the equation

CA QUESTION 62 ON EQUATIONS:

If a line passes through the mid point of the line segment joining the points (--3 , --4 ) and (--
5 ,6 ) its slope is 4/5 , then the equation of the line is :

Solution :

Midpoint = {(x1+x2)/2 , (y1+y2)/2}

= {(--3—5)/2 , (--4+6)/2}

= {--8/2 , 2/2}

= {--4 , 1}

Slope=4/5

equation of a straight line = y – y1 = m( x – x1 )

y -- 1 = 4/5 ( x + 4)

5(y – 1) = 4(x + 4)

5y – 5 = 4x + 16

4x – 5y +21 =0.

CA QUESTION 62 ON EQUATIONS:

If difference between the roots of the equation x2—kx + 8 = 0 is 4 , then the value of k is:

Solution :

x2—kx + 8 = 0

here a=1 b=--k c=8

sum of the roots (α+β)= --b/a


= --(--k)/1 = k

Product of the roots (αβ) =c/a

= 8/1 = 8

Given α—β=4

α+β=k

α—β=4

-----------

2 α = k+4

α=(k+4)/2

α+β=k

{(k+4)/2 }+β=k

β = k – (k+4)/2

β = {2k – k—4}/2

β = (k--4)/2

substitute in αβ = 8

{(k+4)/2}{(k--4)/2} = 8

(k2—16 )/4 = 8

(k2—16 ) = 32

k2 = 48

k =√ 4 x 12

k =2 √ 12

k =2 √ 4 x 3

k = 4√ 3 .

CA QUESTION 63 ON EQUATIONS:

If 2x+y = 22x—y = √ 8 . find x and y


Solution :

2x+y = 22x—y = 81/2

2x+y = 22x—y = 23/2

2x+y = 23/2

22x—y = 23/2

x+y=3/2

2x+2y=3 ---eqn1

2x—y=3/2

4x—2y=3 ---eqn2

By solving eqn1 and eqn2 we get

x=1 , y=1/2.
CALCULUS

DIFFERENTIATION AND INTEGRATION

WHAT IS THE MEANING OF THE TERM CALCULUS ?

ANSWER:

CALCULUS IS THE MATHEMATICS OF MOTION AND CHANGE.

ORIGINATORS OF CALCULUS:

NAME OF THE SCIENTIST COUNTRY OF THE SCIENTIST SPECIALISATION


OF THE SCIENTIST
SIR ISACC NEWTON ENGLAND PHYSICS AND ASTRONOMY
GOTTFRIED WILLHELM GERMANY GEOMETRY
LEIBNITZ

“CALCULUS” IS A LATIN WORD.IT MEANS A “PEBBLE” OR A “SMALL STONE”.

THERE ARE 9 RULES OF DIFFERENTIATION OF A FUNCTION Y[DEPENDENT VARIABLE] WITH


RESPECT TO X [INDEPENDENT VARIABLE].

Y CAN ALSO BE CALLED f(X).

“Y” IS A CARDINAL VARIABLE WHICH CAN BE MEASURED IN TERMS OF UNITS OF


MEASUREMENT SUCH AS METRES OR KILOGRAMS OR LITRES OR MARKS OR SALES VOLUME OR
SALES VALUE OR HEIGHT IN CENTIMETRES OR ANY MEASUREMENT FOR THIS SAKE.

S.No NAME NATURE OF DIFFERNTIATION WAY


OF THE THE FUNCTION =d
RULE OF OF --- [y]
DIFFERENTIATION DEPENDENT dx
VARIABALE, THE VALUE OR
OF dy
WHICH DEPENDS UPON ---
THE VALUE dx
ASSIGNED OR
TO Differential coefficient of y
THE INDEPENDENT with respect to x
VARIABLE
1. CONSTANT Y=C dy
RULE OR --- = 0
Y=10 Dx
2. POWER FUNCTION Y=Xn Dy
RULE --- =
dx

Example (n)(X) (n—1)


Y=X4 dy
--- =
dx

(4)(X) (4—1)
=
(4)(X) (3)
=
4X3
3 SUM RULE Y=U+V dy
---
dx
=
du dv
--- + ----
dx dx

Example :
Y=
10x4 +2x9 dy
---
dx
=
40x 3 +18x8
4. DIFFERENCE RULE Y=U --V dy
---
dx
=
du dv
--- -- ----
dx dx

Example :
Y=
10x4 --2x9 dy
---
dx
=
40x 3 --18x8
5 PRODUCT RULE Y=UV dy
---
dx
=U [dv/dx]
+
V[du/dx]

Example : dy
Find dy/dx ---
If dx
Y= =(4X2 +2)
(4X2 +2) [d/dx]
( 3X2 –3) [ 3X2 –3)
+
[ 3X2 –3)
[d/dx]
[ 4X2 +2)
=
(4X2 +2)
[ 6X]
+
[ 3X2 –3)
[ 8X ]
=
(24X3 +12x
+24 X3—24x)
=
48X3 --12x

6. QUOTIENT RULE Y=U/V dy


Example : ---
V du –U dv Find dy/dx dx
-------------- If =
V2 Y= {V[dU/dX]
(4X2 +2) ---
------------ U[dV/dx] }
( 3X2 –3) --------------
V2

=
{(3X2 --3)
[d/dx]
[ 4X2 +2)
--
[ 4X2 +2)
[d/dx]
[ 3X2 --3) }
-----------
(3X2 --3)2
=
{(3X2 --3)

[ 8X ]
--
[ 4X2 +2)
[ 6X)
-----------
(9X4 --18X2+9)

=
(3X2 --3)
( 8X )
--
( 24X3 +12X)

-----------
(9X4 --18X2+9)
=
(24X3 –24X)
--
( 24X3 +12X)
-----------
(9X4 --18X2+9)
=
=
–36X)
--------
9X4 --18X2+9

7. FUNCTION OF A FUNCTION CONSIDER A FUNCTION


RULE Y=f(x)
x IS A FUNCTION
OF ANOTHER VARIABLE
“u”.

dy
----
dx
=
dy du
---- x ----
du dx

8. LOGARITHMIC FUNCTION y = log x dy 1


--- = ----
dx x
9. EXPONENTIAL FUNCTION y= eX dy
--- = eX [e]
dx

Some explanations on the differential coefficient:

Let y= f(x) be a continuous function of x. let $x be a small increment in x.Corresponding to this


small increment $x, let $y be the increment in y. Then $y/$x as $x-- 0 through positive or
negative values,if it exists and is finite is called the differential coefficient or derivative of y
with respect to x and is denoted by dy/dx.

Consider the function y=f(x)----eqn1

Let $x be a small increment in x and the corresponding increment in y be $y, so that we have,

(y+$y) = f(x+$x)------eqn 2

Y = f(x)-----------eqn1

y+$y = f(x+$x)------eqn 2

eqn2 –eqn 1

$y = f(x+$x)-- f(x)

Let us divide LHS and RHS by $x

$y = f(x+$x)-- f(x)

--- -----------------

$x $x
(4X2 +2)( 3X2 –3)

=(4X2) (3X2)-- (4X2) (3)+(2) (3X2)—(2)(3)

=(12X4) -- (12X2) + (6X2)—(6)

Y=12X4 -- 12X2 + 6X2—6

dy

---=48X3 -- 24X1 + 12X—0

dx

dy

---=48X3 -- 24X + 12X—0

dx

dy

---=48X3 --12X

dx

------------------------------------------------

Exercise

f(x1)=3x3+3

f(x2)=2x3--8

d/dx[ f(x1) + f(x1)] =?

-------------------------------------------

Illustration for the function of a function rule:

y= f(x)=[ x2 +8x—8]16
find dy

---

dx

solution :

y= f(x)=[ x2 +8x—8]16

put u=[ x2 +8x—8]

y= f(x)=[ u]16

dy d d

--- = ----[ u]16 x ------


[ x2 +8x—8]

dx du dx

dy

--- =16u15 x [2x+8]

dx

dy

--- =16[x2 +8x—8]15 x [2x+8]

dx

[ or simply ]

dy

---=16[x2 +8x—8]16—1[d/dx][ x2 +8x—8]

dx
dy

---=16[x2 +8x—8]15[ 2x +8]

dx

dy

---=16[x2 +8x—8]15[ 2x +8]

dx

Exercise for the function of a function rule:

y= f(x)=[ x3 +8x2—3x+4]19

find dy

---

dx

-------------------------------------------------

QUESTION 1 ON DIFFERENTIATION

If y= 5x2 ,find dy

---

dx

answer : 10x

QUESTION 2 ON DIFFERENTIATION

Find d/dx [ √ x ]

Answer :

Y= x1/2
dy
/dx = ½ [x] ½--1
dy
/dx = ½ [x] --½

QUESTION 3 ON DIFFERENTIATION

Find d/dx [ √
3
x]

Solution :

[√
3
x ] can be written as x1/3

d/dx [ x1/3]

= 1/3[x (1/3—1) ]

= 1/3[x (—2/3) ]

= 1/3[ 1/x (2/3) ]


=
1 1

--- x -----

3 x2/3

QUESTION 4 ON DIFFERENTIATION

If y= 1

------

x√ x

find dy

---

Dx

QUESTION 5 ON DIFFERENTIATION

If y= 3
----

√x
find dy

---

dx

QUESTION 6 ON DIFFERENTIATION

If y= x3/2 ,

find

dy

---

dx

QUESTION 7 ON DIFFERENTIATION

Find d

--- [ 1/x3 ]

dx

QUESTION 8 ON DIFFERENTIATION

If y=75 , find dy /dx

Answer 0

QUESTION 9 ON DIFFERENTIATION

If y=ex , find dy /dx

Answer = ex

QUESTION 10 ON DIFFERENTIATION
If y= x2 + x3 , find dy/dx

Answer : 2x + 3x2

QUESTION 11 ON DIFFERENTIATION

If y= x7 + x3/2 , find dy/dx

Answer : 7x6 + 3/2 [x]3/2—1

7x6 + 3/2 [x]1/2

QUESTION 12 ON DIFFERENTIATION

Find the differential coefficient of

x –7 + x2—x1

answer : --7x—8 + 2x—x0

--7x—8 + 2x—1

QUESTION 13 ON DIFFERENTIATION

If y= (1—x2)2

---------- ,

x2

find dy

----

dx

answer :

U/V

Where U= (1—x2)2
and
V= x2

QUESTION 14 ON DIFFERENTIATION

If y= x5 + x2 +x

-----------
x2 ,

find dy

----

dx

QUESTION 15 ON DIFFERENTIATION

If y= 3x4 – 1

----

√3 x
Find dy

----

dx

QUESTION 16 ON DIFFERENTIATION

Find d

---[ 1/x + √ x + ex ]

dx

QUESTION 17 ON DIFFERENTIATION

y= x7 + x 3/2 +5 +ex + log x,

find dy

----

dx

QUESTION 18 ON DIFFERENTIATION

y= x3 –x2—x
------------

√x
Find dy/dx

QUESTION 19 ON DIFFERENTIATION

If y= x5/2 -- x3/2 + x1/2


--------------------------------

x1/2

find dy/dx

QUESTION 20 ON DIFFERENTIATION

If y= x2 +1 log x
----------------- +

x3

find dy/dx

QUESTION 21 ON DIFFERENTIATION

Find the differential coefficient of

9x4 -- 7x3 + 8x2—8 +10

--- ----

x x3

QUESTION 22 ON DIFFERENTIATION

If y= 2 x

---

Find dy/dx.

QUESTION 23 ON DIFFERENTIATION

If y= √ 4 x find dy/dx.
QUESTION 24 ON DIFFERENTIATION

Find the differential coefficient of

the function x5 ex

QUESTION 25 ON DIFFERENTIATION

Find dy

---- [ x2+2] e3x

dx

QUESTION 26 ON DIFFERENTIATION

Differentiate (x2—1)(x2+2) with respect to x.

QUESTION 27 ON DIFFERENTIATION

Find the differential coefficient of

x2 ex logx

solution hint

y= uvw

dy/dx= uv d/dx(w) + uw d/dx(v) +

vw d/dx(u)

QUESTION 28 ON DIFFERENTIATION

Find the differential coefficient of

(x2+7x+2) (ex—logx)

QUESTION 29 ON DIFFERENTIATION

Find the differential coefficient of


x2 logx

QUESTION 30 ON DIFFERENTIATION

Find the differential coefficient of

(2x2+3x3+ex)(7x5—10+logx)

QUESTION 31 ON DIFFERENTIATION

Find the differential coefficient of

(--2x3+7x--9) (5x2—7x--12√ x ) (x2 +3)

QUESTION 32 ON DIFFERENTIATION

Find the differential coefficient of

(ax2 +bx+c) (bx3 + mx2)

QUESTION 33 ON DIFFERENTIATION

Find the differential coefficient of

x2 –1

------

x2 +1

QUESTION 34 ON DIFFERENTIATION

Find the differential coefficient of

2 + 3logx
------------

x2 + 5

QUESTION 35 ON DIFFERENTIATION

Find the differential coefficient of

7x + 6

---------

8x2 + 4

QUESTION 36 ON DIFFERENTIATION

Find the differential coefficient of

x3

-------

x2 + 1

QUESTION 36 ON DIFFERENTIATION

Differentiate with respect to x

x4--9

-------

x2 + 3

QUESTION 37 ON DIFFERENTIATION

If y=

4x3 + 3x2 +11x

-------------------

7x2 – 5x + 9
find dy

----

dx

QUESTION 38 ON DIFFERENTIATION

If y=

logx

-------

x2

find dy

----

dx

QUESTION 39 ON DIFFERENTIATION

If y=

2x +3

-------

e x +5

find dy

----

dx

QUESTION 40 ON DIFFERENTIATION

Differentiate with respect to x


1

-------

X2 –1

QUESTION 41 ON DIFFERENTIATION

Differentiate with respect to x

eax

where “a” is a constant.

QUESTION 42 ON DIFFERENTIATION

Differentiate with respect to x

y=e2x

QUESTION 43 ON DIFFERENTIATION

Differentiate with respect to x

y= (x2 +5x+7)

QUESTION 44 ON DIFFERENTIATION

Differentiate (2x+5)3 with respect to x.

QUESTION 45 ON DIFFERENTIATION

Differentiate log (5x+7) with respect to x.

QUESTION 46 ON DIFFERENTIATION
Differentiate 4 x with respect to x.

Solution:

Let y=4 x

Taking log on both sides

log y= log 4 x

log y= x log 4

differentiating both sides with respect to x

d/dx[logy] =d/dx{[x][log4]}

[1/y] [d/dx]y=[1] [log4]+x[0]

1 dy

--- ---- = log 4 + 0

y dx

1 dy

--- ---- = log 4

y dx

dy

---- = log 4[y]

dx
dy log4[4 x ]

---- =

dx

standard form

if y= ax means

dy

--- = ax[log a]

dx

QUESTION 47 ON DIFFERENTIATION

Differentiate 6 x with respect to x.

QUESTION 48 ON DIFFERENTIATION

Differentiate (x 2 +5)3/2 with respect to x

QUESTION 49 ON DIFFERENTIATION

Differentiate (2x 2 +5x--7)3/2 with respect to x

QUESTION 49 ON DIFFERENTIATION

Differentiate

(x3 +3x+2) with respect to x

y= 2x5
find the second derivative

first derivative=

dy

--- =10x4

dx

second derivative=

d 2y

--- =40x3

dx2

INTEGRAL CALCULUS OR INTEGRATION:

INTEGRATION IS THE REVERSE PROCESS OF DIFFERENTIATION.

WE KNOW THAT THE DIFFERENTIAL COEFFICIENT OF A FUNCTION IS FOUND OUT ONCE, THE
ORIGINAL FUNCTION WHICH IS GIVEN TO US IS DIFFERENTIATED WITH RESPECT TO X.SUCH A
DIFFERENTIAL COEFFICIENT IS KNOWN AS FIRST DERIVATIVE OR DERIVATIVE OF THE FIRST
ORDER WHICH IS NOTHING BUT THE RATE OF CHANGE IN THE ORIGINAL FUNCTION’S VALUE
FOR A GIVEN SMALLEST INCREMENT.

IN INTEGRATION , WE TRY TO REACH THE ORIGINAL FUNCTION IF THE FIRST DERIVATIVE OF THE
ORIGINAL FUNCTION IS GIVEN TO US.

SO WE SAY THAT INTEGRATION IS THE REVERSE PROCESS OF DIFFERENTIATION.


EXAMPLE

INTEGRATE THE DIFFERENTIAL COEFFICIENT 5X4

SOLUTION

LOGICALLY IF WE DIFFERENTIATE

X5 WE WOULD GET THE ANSWER

5X4

SO THE ORIGINAL FUNCTION IS

y=f(x)=x5 + c

example : integrate 3x2 dx

example : integrate ½ x—1/2 dx

example : integrate 8 x dx

example :evaluate ∫ 10x9 dx

example :evaluate ∫ 11x10 dx

example :evaluate ∫ 1/(x+5) dx

example :evaluate ∫ e 2x dx

answer= e 2x
------- + c

2
example : Evaluate ∫ e 4x dx

example : Evaluate ∫ e 4x dx

example : Evaluate ∫ x 1/2 dx

example : Evaluate ∫ 7x2 dx

=∫ 7x2 dx can be written as

7∫ x2 dx

=[7][x3]

---------- + c

= [7/3] [x3] +c

example : Evaluate ∫ 3x4 dx

example : Evaluate ∫ (3—2x—x4) dx

solution:

∫ (3—2x—x4) dx can be written as

∫ 3dx—∫ 2xdx --∫ x4 dx

=3∫ dx— 2∫x dx -- ∫x4 dx

=[3][x]—[2][x]2 -- [x]5 c
------------ -------
+

2 5
=3x—2x2 -- [x]5 c
------ -------
+

2 5

=3x—x2 -- x5 c
-------
+

example :

Evaluate ∫ (4x 3 + 3x2—2x+5) dx

example :

Evaluate ∫ ( x 3 + 4x2—5x—6 ) dx

ANALYTICAL GEOMETRY:

ASSUME A GRAPH SHEET.

A GRAPH SHEET HAS 4 QUADRANTS SPREAD ACROSS TWO AXES.


THE PARALLEL AXIS IS NAMED AS X AXIS(ABSSISCA).

THE PERPENDICULAR AXIS IS NAMED AS Y AXIS(ORDINATE)

THE RIGHT UPPER QUARTER OF THE GRAPH SHEET IS KNOWN AS FIRST QUADRANT

WHERE X COORDINATE OR X COEFFICEINT IS POSITIVE AND Y COORDINATE OR Y COEFFICEINT IS


POSITIVE. EXAMPLES FOR FIRST QUADRANT POINTS ON THE PLATE OR PLANE OR GRAPH (X,Y)=

(1, 4),(4,7),(2,2),(3,2)

EXAMPLE. ASSUME THAT MADURAI IS AT (4,3) AND CHENNAI IS AT (7,6). FIND THE DISTANCE IN

BETWEEN MADURAI AND CHENNAI.

SOLUTION :

WHAT IS THE FORMULA FOR COMPUTING THE DISTANCE IN BETWEEN TWO POINTS ON THE GRAPH?

LET THE TWO POINTS BE(X1 ,Y1) AND (X2,Y2)

DISTANCE FORMULA

SQUARE ROOT OF

{ (X2—X1)2 + (Y2—Y1)2 }

THE TWO POINTS BE(X1 ,Y1) AND (X2,Y2) ARE (4 ,3) AND (7,6)

DISTANCE IN BETWEEN THE TWO POINTS=

SQUARE ROOT OF

{ (7—4)2 + (6—3)2 }

SQUARE ROOT OF

{ (3)2 + (3)2 }

SQUARE ROOT OF
{9 +9 }

SQUARE ROOT OF

18

=4.24.

EXERCISE

DISTANCE BETWEEN (5,6) AND (4,X)=7.

FIND X.

Solution :

Square root of [x2—x1]2 + [y2—y1]2 =7

Square root of [4—5]2 + [x—6]2 =7

Square root of [—1]2 + [x—6]2 =7

Square root of 1 + [x—6]2 =7

Square root of 1 + [x2—12x+36] =7

Squaring both sides

1 + [x2—12x+36] =49

1 + x2—12x+36—49=0

x2—12x --12=0

solving the quadratic equation ,

x = 12.93 units

TUTORIAL NOTE: THE SECOND QUADRANT HAS X NEGATIVE AND Y POSITIVE .


EXAMPLE (X,Y)=(--4, 7)

THE THIRD QUADRANT HAS X NEGATIVE AND Y NEGATIVE

EXAMPLE (X,Y)=(-7,-6)

THE FOURTH QUADRANT HAS X POSITIVE AND Y NEGATIVE.

EXAMPLE (X,Y)=(7,-6)

EXAMPLE: FIND THE DISTANCE BETWEEN (-3,7) & (6,-7)

ANSWER :16.64 UNITS.

==========LESSON OF 8.12.2016 IS OVER==================

EXAMPLE :

FIND THE EQUATION OF A STRAIGHT LINE WHICH PASSES THROUGH TWO POINTS (4,2) AND (7,6)

SOLUTION:

LET THE TWO PONTS BE (X1,Y1) AND (X2, Y2).

THESE TWO POINTS REGRESS WITH EACH OTHER .

THEN WE NEED TO FIND OUT THE SLOPE THAT EXISTS IN BETWEEN THESE TWO POINTS. WHAT IS THE
MEANING OF SLOPE IN BETWEEN THESE TWO PONTS ?

ANS : NOTHING BUT THE REGRESSION COEFFICEIENT THAT EXISTS BETWEEN Y AND X, THE

ASSUMPTION BEING THAT THE EQUATION HAS THE FOLLOWING FORM….

Y= mx + C

Where Y= Value of dependent variable.

m=slope or regression coefficient of Y on X.

X= Value of independent variable.

C=constant value of dependent variable.


First step in equation formation is to find the slope of the line or equation that is formed in between the
two points.

Formula for computing slope in between

(X1,Y1) and (X2, Y2)

(4,2) AND (7,6)

m=(Y2—Y1) /(X2—X1)

m=(6—2) /(7—4)

m=(4) /(3)

m=4/3

EQUATION IN BETWEEN

(X1,Y1) and (X2, Y2)

(4,2) AND (7,6)

WITH SLOPE m=4/3 WILL BE

(Y—Y1) =m(X—X1)

(Y—2) =4/3(X—4)

(Y—2) =(4X/3—16/3)

3(Y—2)=4X—16

3Y—6=4X—16

3Y=4X—16+6

3Y=4X—10

Y=(4/3)X—10/3

WITHOUT FINDING “m” EQUATION CAN BE FORMED

(Y—Y1) / (Y2—Y1) =
(X—X1) / (X2—X1)

EQUATION IN BETWEEN

(X1,Y1) and (X2, Y2)=(4,2)& (7,6)

(Y—2) / (6—2) =

(X—4) / (7—4)

3(Y—2) = 4 (X—4)

3Y – 6 = 4X -- 16

3Y = 4X – 10

Y = (4X/3) – (10/3)

-------------------------------------

Example

Check whether the point (3,2) lie in the line 2x+3y=21.

Solution :

The equation 2x+3y=21 is given to us. We want to check whether this equation or this straight line

is passing through the point on the plane, the point being (3,2). If we substitute X = 3 and Y = 2, in

the above equation and if it agrees then we can say that this point lies in the straight line. Otherwise

it slips out of the straight line.

Substituting X=3 and Y=2 in 2x+3y=21.

2(3) + 3(2) = 21 don’t agree.

Therefore this point is out of the line.

EXAMPLE :

FIND THE MID POINT OF THE POINTS (7,2) AND (9,11)

SOLUTION:

FORMULA FOR COMPUTING THE MID POINT IN BETWEEN TWO POINTS (X1, Y1) AND ( X2,Y2)

(7,2) AND (9,11)


= (X1+X2) / 2 , (Y1+Y2)/2

= (7+9)/2 ,(2+11)/2

= (16)/2 ,(13)/2

= (8 ,6.5 )

EXERCISE:

(3,4) IS THE MIDPOINT OF (-7,2) AND (X,Y) .FIND X and Y.

Solution : x=13 and y=6

Method :

[--7+x] /2 = 3

--7+x =6

X=6+7=13

[2+y]/2 = 4

2+y= 8

Y= 6

So the two points are

X=13 and y= 6.

EXAMPLE: FORM THE EQUATION OF A STRAIGHT LINE PASSING THROUGH THE POINT (8,-3) AND

HAVING ITS SLOPE 3.

SOLUTION:

USE THE FORM

Y—Y1 = m (X—X1)

HERE m=3

Y1=--3

X1=8
Y—(--3)=m(X—8)

Y+3=3(X—8)

Y+3=3X--24

Y=3X—24—3

Y=3X—27

EXERCISE: FORM THE EQUATION OF A STRAIGHT LINE PASSING THROUGH THE POINT (--7,--3) AND

HAVING ITS SLOPE 2.

TUTORIAL NOTE

 X AXIS IS POPULARLY REPRESENTED AS Y=0

 Y AXIS IS POPULARLY REPRESENTED AS X=0

 THE LINE Y=8 IS WHAT TO X AXIS?

ANS: PARALLEL TO X AXIS.

 THE LINE X=8 IS WHAT TO Y AXIS?

ANS: PARALLEL TO Y AXIS.

ANOTHER TUTORIAL NOTE :

IF TWO DIFFERENT LINES ARE PERPENDICULAR TO EACH OTHER , THEN THE PRODUCT OF THEIR

SLOPES WILL BE –1

EXAMPLE:

CHECK WHETHER Y=4X—82 AND Y=[--1/4] X+120 ARE PERPENDICULAR TO EACH OTHER?

ANOTHER TUTORIAL NOTE:


TWO PERPENDICULAR LINES BISECT OR INTERSECT OR CUT EACH OTHER AT 90 DEGREES.

EXAMPLE X AXIS AND Y AXIS CUT EACH OTHER AT 90 DEGREES.

ANOTHER TUTORIAL NOTE:

TWO PARALLEL LINES HAVE EQUAL SLOPES. BEST EXAMPLE “RAILWAY TRACK”.

EXAMPLE: CHECK WHETHER

2X+3Y=8

AND 3X+2Y=8 ARE PARALLEL TO EACH OTHER.

TUTORIAL NOTE:

TWO PARALLEL LINES WILL BE EQUIVI DISTANT TO EACH OTHER.

Y=8X+3 AND

Y=8X-15 ARE TWO PARALLEL LINES .

EXERCISE:

LINE FORMED BY (2,3) WITH SLOPE 8 IS PERPENDICULAR TO THE LINE PASSING THROUGH

(1,--8)

FIND THE TWO LINES.

========================

Example [1]

Find the equation of the straight line passing through the two points.

(10,5) and (20,20)

Here

X1=10
Y1=5

X2=20

Y2=20

Slope m = y2—y1 / x2—x1 = 20—5 / 20—10 = 15/10 = 3/2

Let us form the equation of the straight line joining the above two points

(y—y1) = m (x—x1)

(y—5) = 3/2 (x—10)

2(y—5) = 3 (x—10)

2y—10 =3x—30

2y =3x—30+10

2y =3x—20

Y= [3/2] x --20/2

Y= [3/2] x --10

Example [2]

Find the equation of the straight line passing through the two points.

(10,5) and (20,20)

Solution :

Here

X1=10

Y1=5

X2=20

Y2=20

Equation is

y—y1 x—x1
-------- = -----------

y2—y1 x2—x1

y—5 x—10

-------- = -----------

20—5 20—10

y—5 x—10

-------- = -----------

15 10

10(y—5) = 15(x—10)

10y—50 =15x—150

10y= 15x—150+50

10y= 15x—100

Y= [15/10] x—100/10

Y= [3/2]x—10

You might also like